You are on page 1of 283

HI TON HC H NI S GD & T HNG YN

**************** *******************

NGUYN VN MU, NGUYN VN PH


(C H BIN)

HI THO KHOA HC

C C CHUYN CHN LC
BI DNG HC SINH GII TON

HNG YN 25-26/02/2017
LI M U
Nguyn Vn Ph
Tnh y vin - Gim c S GD& T Hng Yn

Thc hin Ngh quyt 29-NQ/TW Hi ngh ln th 8, Ban Chp hnh Trung ng kha
XI v i mi cn bn, ton din gio dc v o to; Ngh quyt s 09-NQ/TU ngy
4/10/2016 ca Ban Chp hnh ng b tnh kha XVIII v Chng trnh pht trin GD
& T tnh Hng Yn giai on 2016-2020, mt s nh hng n nm 2025; K hoch
321/KH_UBND ngy 29/12/2016 ca UBND tnh Hng Yn v thc hin Ngh quyt s 09-
NQ/TU ngy 4/10/2016 ca Ban Chp hnh ng b tnh kha XVIII, S GD & T Hng
Yn phi hp vi Hi Ton hc H Ni t chc Hi tho Cc chuyn chn lc bi dng hc
sinh gii Ton THPT tnh Hng Yn ln th Nht nm 2017.
Trong nhng nm qua, cng tc pht hin v bi dng hc sinh gii ca tnh lun c
quan tm ch trng. Hng nm, S GD & DT t chc k thi hc sinh gii cp tnh cc mn
vn ha cho hc sinh lp 9 v hc sinh THPT, lm tt cng tc tuyn chn v bi dng cho
hc sinh tham gia k thi Hc sinh gii cp quc gia THPT. Kt qu k thi chn hc sinh gii
quc gia hng nm c gi n nh, trong c nhiu em t gii cao. Trong 2 nm tr li
y, c 3 hc sinh c tham gia thi Olympic Quc t Chu - Thi Bnh Dng. Song song
vi vic bi dng hc sinh, cng tc o to, bi dng i ng gio vin c quan tm,
nh mt khu quan trng to nn cht lng cng tc bi dng hc sinh gii. Nhiu hi
tho, cc lp tp hun chuyn mn c t chc hng nm theo quy m cp tnh vi mc
tiu nng cao kin thc, k nng, nghip v s phm cho gio vin.
Hi tho Cc chuyn Ton hc bi dng hc sinh gii tnh Hng Yn ln th Nht nm
2017 c t chc nhm nh gi khch quan thc trng cng tc o to, bi dng hc
sinh gii, t ra nhng gii php hu hiu nhm o to hc sinh gii trong nhng
nm tip theo.
Hi tho c vinh d n tip nhiu nh khoa hc, nh gio lo thnh, cc chuyn gia
Ton hc u ngnh, cc chuyn gia gio dc v vi t Hng Yn ngn nm Vn Hin. Ban
t chc Hi tho hy vng y s l c hi cc gio vin b mn Ton ca tnh Hng Yn
v cc tnh bn c trao i, chia s, rt ra nhng bi hc kinh nghim qu bu trong cng
tc bi dng hc sinh gii ni chung, b mn Ton cp THPT ni ring. Hi tho Ton hc
cng s l tin chng ta t chc nhng hi tho tip theo i vi cc b mn khoa hc
khc.
Nhn dp ny, S GD& T Hng Yn chn thnh cm n Hi Ton hc H Ni, cc nh
khoa hc, nh gio lo thnh, cc chuyn gia Ton hc u ngnh. S GD & T tnh ghi
nhn s ng gp ca cc chuyn gia gio dc, lnh o, cn b ph trch chuyn mn ca
S GD & T, cc thy, c gio b mn Ton vit bi v tham gia Hi tho. S ng gp
ca cc ng ch gp phn lm nn thnh cng ca Hi tho.
Hi tho khoa hc, Hng Yn 25-26/02/2017

TRNG THPT C HUYN HNG YN -


NI THP SNG NHNG TI NNG TON HC

Nguyn Th Hng Thy


Hiu trng trng THPT Chuyn Hng Yn

Trng Trung hc ph thng Chuyn Hng Yn c thnh lp ngy 23 thng 5 nm


1997. Ban u, trng c tn l trng Ph thng Nng khiu Hng Yn. T nm 2003 n
nay, trng mang tn trng THPT Chuyn Hng Yn. Trng ta lc ti s 1, ng Chu
Vn An, phng An To, thnh ph Hng Yn. Mc tiu ca nh trng l xy dng mt
ngi trng cht lng cao, trong hc sinh c bi dng nng khiu, ng thi c
pht trin ton din v tr tu, nng lc t duy, nghin cu v th cht.
Tri qua gn 20 nm k t khi thnh lp n nay, trng THPT Chuyn Hng Yn khng
ngng pht trin ln mnh v mi mt. Trong , cng tc bi dng hc sinh gii, hc sinh
nng khiu c coi l nhim v trng tm hng u. Tnh n nay, trng c tng s
714 hc sinh ot gii hc sinh gii Quc gia, trong c 12 gii nht, 85 gii nh, 328 gii ba,
290 gii khuyn khch; c 1 hc sinh t Huy chng ng quc t, 4 hc sinh c tham
gia d thi Olympic Quc t Chu -Thi Bnh Dng. i vi b mn Ton, nh trng c
92 gii, trong c 1 gii nht, 6 gii Nh, 53 gii Ba, 32 gii khuyn khch; 1 hc sinh c
tham gia d thi Olympic Quc t Chu -Thi Bnh Dng.
Trong khng kh ca nhng ngy u xun mi inh Du 2017, trng THPT Chuyn
Hng Yn c vinh d chn l n v ng cai t chc Hi tho Cc chuyn chn lc bi
dng hc sinh gii Ton THPT ca ngnh GD& T Hng Yn. y l dp sinh hot chuyn
mn b ch gp phn nng cao cht lng i ng gio vin, nng cao cht lng bi dng
hc sinh gii mn Ton, ng thi cng l hot ng ngha hng ti k nim 20 nm
thnh lp trng THPT Chuyn Hng Yn (1997-2017).
Tham gia vit bi cho Hi tho c 25 tc gi, nhm tc gi l cc chuyn gia u ngnh
b mn Ton; hi vin Hi Ton hc H Ni; Bo Ton hc v Tui tr; ging vin trng
i hc Thng Long, hc vin An Ninh nhn dn; gio vin b mn Ton thuc cc trng
THPT ca thnh ph Hi Phng, tnh Bc Giang, Bnh nh; gio vin Ton cc trng THPT
Chuyn HSP H Ni, Vnh Phc, Tuyn Quang v THPT Chuyn Hng Yn. Cc chuyn
tp trung vo cc lnh vc khc nhau ca Ton hc nh: S hc, Hnh hc, T hp, Dy s,
Bt ng thc, Phng trnh, Bt phng trnh. . . . Nhiu chuyn c cht lng chuyn
mn su, th hin s u t ca tc gi trong nghin cu, tng hp. Nhiu chuyn c
tnh ng dng cao v mi, c th p dng trong cng tc bi dng hc sinh gii quc gia.
Hi tho nhn c s quan tm su sc ca Lnh o S GD& T, Hi Ton hc H Ni,
cc trng THPT, THPT Chuyn trong v ngoi tnh. Nhn dp ny, trng THPT Chuyn
Hng Yn xin chn thnh cm n s quan tm ch o ca Ban Gim c S GD& T, cc

2
Hi tho khoa hc, Hng Yn 25-26/02/2017

phng chuyn mn thuc S GD& T; xin by t tnh cm bit n su sc n GS-TSKH


Nguyn Vn Mu v Hi Ton hc H Ni phi hp gip v chuyn mn v cng tc
t chc Hi tho; xin cm n cc cn b qun l, cc thy c gio tham gia v vit bi gp
phn to nn thnh cng ca Hi tho.

C HNG TRNH HI THO


Sng ngy 25.02.2017
10h00-11h00 Hp Ban T chc v Ban Chng trnh
11h30-12h30 n tra
Chiu ngy 25.02.2017
13h30-13h45 Vn ngh cho mng
13h45-14h15 Khai mc
Pht biu cho mng: Nguyn Th Hng Thy, HT trng THPT Chuyn Hng Yn
Pht biu khai mc: Nguyn Vn Ph, Gim c s GD & T Hng Yn
Pht biu dn: GS.TSKH Nguyn Vn Mu, Ch tch hi THHN

14h15-15h15 CC BO CO MI PHIN HP TON TH


iu khin: PGS.TS Trn Huy H, PGS.TS Nguyn Thy Thanh
1. PGS.TS. m Vn Nh, Mt vi kt qu v ng bc ba
2. TS. Nguyn Vn Ngc, Mt s bt ng thc ca cc hm hyperbolic
3. TS V Tin Vit, Xung quanh mt bi ton thi IMO
4. TS Nguyn Vit Hi, S ly tha
5. ThS Nguyn B ang, T gic iu ha

15h15-15h45 Ngh gii lao

15h45-17h45 CC BO CO CHUYN PHIN HP TON TH


iu khin: PGS.TS. Nguyn Minh Tun, PGS.TS. H Tin Ngon
6. Hong Minh Qun, Mt s m rng v p dng ca bt ng thc Klamkin
7. ng Th Mn, Mt s phng php gii phng trnh hm trn tp s nguyn
8. Trn Th Lan Hng, Phng trnh v h phng trnh qua cc k thi hc sinh gii
9. Hunh Duy Thy, Tip cn cc phng php tm cc tr ca mt biu thc a bin
10. L Th Mai, Mt s dng ton v bt ng thc i vi hm phn thc
11. V Vn Thng, S dng bt ng thc Karamata kho st bt ng thc trong tam gic
12. Phan Ngc Ton, S dng tnh cht gii tch trong cc bi ton s hc
13. Phm Vn Dng, Mt s ng dng ca nh l Lagrange
14. Mng Thanh Hng, Phng php lng gic gii phng trnh a thc bc cao

17h45-18h00 Tng kt hi tho


iu khin: GS.TSKH. Nguyn Vn Mu
18h30-20h30 n ti v giao lu vn ngh

3
Hi tho khoa hc, Hng Yn 25-26/02/2017

MC LC
Chng trnh 2
2. m Vn Nh, Mt vi kt qu v ng bc ba 5
3. Nguyn Vit Hi S ly tha 12
4. Nguyn Vn Ngc, Mt s bt ng thc ca cc hm hyperbolic 23
5. Nguyn B ang, T gic iu ha 39
6.V Tin Vit, Xung quanh mt bi ton thi IMO 49
7. Nguyn Duy Lin, nh l thng d Trung hoa v mt s ng dng 55
8. Hong Phng Anh, L thuyt th v ng dng 79
9. Phm Vn Dng - Hong Th Nhung,
Phng php o hm trong chng minh bt ng thc 90
10. Phm Vn Dng - Hong Th Minh Thy, Mt s ng dng ca nh l Lagrange 105
11. Mng Thanh Hng, Phng php lng gic gii phng trnh a thc bc cao 116
12. Lng Th Hng, Phng php hm sinh xc nh dy s 130
13. Trn Th Lan Hng,
Phng trnh, bt phng trnh, h phng trnh qua cc k thi hc sinh gii 143
14. L Th Mai,
Mt s dng ton v bt ng thc i vi hm phn thc bc hai trn bc nht 166
15. ng Th Mn, Mt s phng php gii phng trnh hm trn tp s nguyn 172
16. ng Th Mn, Cp ca mt s nguyn v ng dng gii mt s bi ton s hc 184
17. Hong Minh Qun - Ngy Phan Tin,
Mt s m rng v p dng ca bt ng thc Klamkin 193
18. Nguyn Th Tm - Hong Th Nhung - Trn Thi Hng, nh l Helly v ng dng 205
19. V Th Thun - Nguyn Th an Qu, S chnh phng theo modul bc ty 214
20. V Th Thun, S dng i lng bt bin v n bin trong ton t hp 226
21. Hunh Duy Thy, Tip cn cc phng php tm cc tr ca mt biu thc a bin 241
22. Phan Ngc Ton, S dng tnh cht gii tch trong cc bi ton s hc 254
23. ng Th Mn, Mt s ng dng ca s phc trong i s v ton t hp 259
24. V Vn Thng, S dng bt ng thc Karamata kho st bt ng thc trong tam gic 270
25. Quch Th Tuyt Nhung, Mt s bi ton v dy s truy hi tuyn tnh cp hai 279
26. Hong Tun Doanh, Mt s tnh cht ca hm s hc c bn v p dng 290

4
Hi tho khoa hc, Hng Yn 25-26/02/2017

MT VI KT QU V NG BC BA

m Vn Nh
Trng THPT Chuyn HSP H Ni

Tm tt ni dung
Mc ch ca bo co ny l trnh by mt phn ni dung ca cun gio trnh (xem
[2]) nhm l gii s hnh thnh, pht trin ca mt s kt qu trong Hnh hc S cp v
xu chui cc vn lin quan li thnh mt th logic. tc gi mong mun a n cho
ngi c mt ci nhn ton cc v mn hc ny v s ra i ca H tin Hilbert, H
tin Wayne v H tin Pogorelov ca Hnh hc Euclid, gii thiu M hnh Carte
kim tra h tin do Pogorelov a ra tha mn ba yu cu: c lp-Phi mu thun-
y . Tc gi tp trung trnh by nh l Pascal cho ng cong bc hai v ch ra nh
l Newton, Pappus, Brianchon v m rng bi ton con bm. Vic s dng phng
php a thc v m rng trng cho ta hnh dung c s pht trin ca mn Hnh hc
S cp t a thc bc 0, a thc bc 1 ri n a thc bc 2 v tip tc phi xt a thc
bc 3. Bc cao hn na ca hnh hc s c tip tc nghin cu trong Hnh hc i s.
Mc tiu ca chuyn nhm trang b mt cch su sc cc kin thc, rn luyn t
duy v k nng gii ton hnh cho cc em hc sinh kh, gii; cho cc em sinh vin, cc
hc vin cao hc. Vi mt vi bi ton m cc em tp dt nghin cu, xy dng kt
qu mi.

1 nh l Pascal
nh l 1 (Pascal). Gi s 6 im A1 , A2 , . . . , A6 nm trn ng trn (C ). Khi , im giao
A = ( A1 A5 ) ( A2 A4 ), B = ( A3 A4 ) ( A6 A1 ), C = ( A2 A6 ) ( A3 A5 ) thng hng.

Chng minh. Gi I = A1 A5 A2 A6 , J = A1 A5 A3 A4 v K = A3 A4 A2 A6 . Theo


nh l Menelaus, vi tam gic I JK v cc b ba im ( A, A2 , A4 ), ( B, A1 , A6 ), (C, A3 , A5 )
thuc ba cnh tng ng thng hng ta nhn c ba h thc sau:
AI A4 J A2 K A6 K A1 I BJ CK A5 I A3 J
. . = 1, . . = 1, . . = 1.
AJ A4 K A2 I A6 I A1 J BK CI A5 J A3 K
Nhn cc kt qu ny li v bin i theo phng tch
AI A4 J A2 K A6 K A1 I BJ CK A5 I A3 J
1 = . . . . . . . .
AJ A4 K A2 I A6 I A1 J BK CI A5 J A3 K
AI BJ CK A4 J.A3 J A2 K.A6 K A1 I.A5 I AI BJ CK
= . . . . . = . . .
AJ BK CI A1 J.A5 J A3 K.A4 K A2 I.A6 I AJ BK CI
AI BJ CK
Do . . = 1 nn A, B, C thng hng theo nh l Menelaus.
AJ BK CI

5
Hi tho khoa hc, Hng Yn 25-26/02/2017

V d 1. Gi s t gic li ABCD ni tip trong ng trn ( T ). Gi s giao I = AC BD,


J = AB CD v K l giao hai tip tuyn Bb v Cc ca ng trn ti B v C. Khi ba im
I, J, K thng hng.

Bi gii. Kt qu ny c suy ra t nh l Pascal vi 6 im A, B, B, C, C, D.

V d 2. Gi s 5 im A1 , A2 , A3 , A4 , A6 nm trn ng trn (C ). Khi ba im giao


A = ( A1 A3 ) ( A2 A4 ), B = ( A3 A4 ) ( A6 A1 ) v C = ( A2 A6 ) ( A3 t), A3 t l tip
tuyn vi ng trn (C ) ti A3 , thng hng.

Bi gii. Kt qu ny c suy ra t nh l Pascal v A5 A3 .

H qu 1 (Newton). Gi s ng trn ni tip trong t gic li ABCD tip xc vi cc cnh


AB, BC, CD, DA ti E, F, G, H, tng ng. Khi , bn ng thng AC, BD, EG, FH ng
quy ti mt im.

Chng minh. Gi O = EG FH, X = EH FG. V D l giao im ca hai tip tuyn


GD, HD nn, theo nh l Pascal, nh l 1, cho 6 im E, G, G, F, H, H c ba im
O, D, X thng hng. Tng t, vi 6 im E, E, H, F, F, G ta cng c B, X, O thng hng.
Ta c : BD, EG, FH ng quy ti O. Hon ton tng t, AC, EG, FH cng ng quy
ti O. Nh vy, AC, BD, EG, FH ng quy ti mt im.

nh l 2. Gi s 6 im A1 , A2 , . . . , A6 nm trn ng cong bc hai khng suy bin (). Khi


ba im giao A = ( A1 A5 ) ( A2 A4 ), B = ( A3 A4 ) ( A6 A1 ) v C = ( A2 A6 ) ( A3 A5 )
thng hng.

Chng minh. Qua php bin i tuyn tnh c th coi () : x = y2 . K hiu dij ( x, y) = 0
l phng trnh ng thng Ai A j . t

p( x, y) = d24 ( x, y)d16 ( x, y)d35 ( x, y) d34 ( x, y)d26 ( x, y)d15 ( x, y).

y l mt a thc bc ba i vi x v y. Ta 6 im A1 , . . . , A6 tha mn phng


trnh p( x, y) = 0. Ly mt im bt k A thuc (), khc 6 im trn. Ta chn ta

6
Hi tho khoa hc, Hng Yn 25-26/02/2017

im A tha mn p( x, y) = 0. Nu th x qua y2 ta c a thc p(y2 , y) bc 6 6.


Phng trnh p(y2 , y) = 0 cho tung ca 7 im khc nhau, c ngha: p(y2 , y) = 0 c
7 nghim y khc nhau. Vy p(y2 , y) 0 hay mi im thuc () u c ta tha mn
p( x, y) = 0. Coi p( x, y) nh l mt a thc ca x. Vi php chia a thc, ta nhn c

p( x, y) = ( x y2 )q( x, y) + r(y),

trong q( x, y) l thng v r(y) l phn d. V mi im thuc () : x = y2 u tha


mn p( x, y) = 0 nn r(y) 0. Vy p( x, y) = ( x y2 )q( x, y). V p( x, y) l a thc bc
3 nn q( x, y) phi l a thc bc nht hay p( x, y) = ( x y2 )( ax + by + c). Do 9 im
A1 , A2 , . . . , A6 v A, B, C u thuc ng cong phng p( x, y) = 0, nhng A, B, C khng
thuc () : x = y2 nn A, B, C thuc ng thng (d) : ax + by + c = 0 hay A, B, C thng
hng.

H qu 2 (Pappus). Gi s 3 im A1 , A2 , A3 nm trn ng thng (d1 ) v 3 im A4 , A5 , A6


nm trn ng thng (d2 ) khc (d1 ). Khi ba im giao A = ( A1 A5 ) ( A2 A4 ), B =
( A1 A6 ) ( A3 A4 ) v C = ( A2 A6 ) ( A3 A5 ) thng hng.
Chng minh. Kt qu ny c suy ra t nh l 2.

H qu 3 (Brianchon). Gi s lc gic li ABCDEF ngoi tip mt ng trn. Khi , ba


ng thng AD, BE, CF ng quy ti mt im.

Chng minh. Gi s ng trn ni tip trong lc gic li ABCDEF tip xc vi cc


cnh AB, BC, CD, DE, EF, FA ti G, H, I, J, K, L, tng ng. t M = AB CD, N =
DE FA. Theo H qu 1, vi t gic AMDN ta c cc ng thng AD, IL, GJ ng
quy ti A . Tng t, cc ng thng BE, HK, GJ ng quy ti B ; cc ng thng
CF, HK, IL ng quy ti C . Ch rng, hai ng thng IL v A C l mt. Theo nh
l Pascal, nh l 1, vi cc im G, G, I, L, L, H c ba im A, O, P thng hng,
O = GI HL, P = IL GH. Cng theo nh l Pascal, vi cc im H, H, L, I, I, G c ba
im C, O, P thng hng. Vy A, C, P thng hng. Xt G = AB A B , H = BC B C ,
P = CA C A . Theo phn o ca nh l ??, nh l Desargues, i vi ABC v
A B C c AA = AD, BB = BE, CC = CF ng quy.

7
Hi tho khoa hc, Hng Yn 25-26/02/2017

2 M rng bi ton con bm


Mnh 1 (Bi ton con bm). Qua trung im I ca dy cung MN ca mt ng trn ta
k hai dy cung bt k AB v CD, u khc MN. Gi giao im gia AC v BD vi MN l P
v Q tng ng. Ta lun c IP = IQ.
Chng minh. Gi O l tm ng trn v J, K l trung im on AC, BD, tng ng.
V I AC IDB nn I AJ IDK v suy ra I J A = IKD. Do t gic IOJP v
IOKQ u ni tip trong ng trn tng ng nn IOP = I JP = IKQ = IOQ.
T suy ra IP = IQ.
B 1. Cho t gic li ABCD vi giao hai ng cho I = AC BD. Ly E, H, F, G thuc
cnh AB, BC, CD, DA, tng ng sao cho EF v GH cng i qua I. Gi s EG, FH ct AC ti
M, N, tng ng. Chng minh rng
1 1 1 1
+ = + .
IA IN IC IM
AM IN IA
Bi gii. Trc tin ta ch ra h thc P = . = . Bin i
I M CN IC
S AEG S IFH S S S S
P = . = IFH . CBD . ABD . AEG
S IEG SCFH S IEG SCFH SCBD S ABD
IF.IH CB.CD I A AE.AG
= . . .
IE.IG CF.CH IC AB.AD
S FAC S HAC SDAC SBAC I A SEAC SGAC IA
= . . . . . . = .
SEAC SGAC S FAC S HAC IC SBAC SDAC IC
am n a 1
t I A = a, IC = c, I M = m, IN = n. Ta c . = v suy ra h thc +
m cn c IA
1 1 1
= + .
IN IC IM

Mnh 2. Cho dy cung MN ca mt ng trn, (ng bc hai), v im I thuc on


MN. Gi s hai dy cung bt k AB v CD, u khc MN, i qua im I. Gi giao im gia
AC v BD vi MN l P v Q tng ng. Ta c ng nht thc
1 1 1 1
+ = + .
IM IP IN IQ

8
Hi tho khoa hc, Hng Yn 25-26/02/2017

Chng minh. Gi s MA ct ND ti T v MC ct NB ti H. Do 6 im A, M, C, B, N, D
cng thuc mt ng trn, (ng bc hai), nn ba im T, I, H thng hng theo nh
1 1 1 1
l Pascal. Ta c ng nht thc + = + theo B 1.
IM IP IN IQ

3. th phng 21-im K3
Mc ny dnh trnh by Mnh 3. y l li gii cho Bi tp 16 trang 56 trong "Bi
tp v cc th phng bc ba" ca Smgrdevski v Stlva (bn ting Nga).
Gi s tam gic ABC c di ba cnh a = BC, b = CA, c = AB. K hiu a l ng
thng ct ng thng AC ti im cch u hai im A v B, ct ng thng AB ti
im cch u hai im A v C. K hiu b l ng thng ct ng thng BA ti im
cch u hai im B v C, ct ng thng BC ti im cch u hai im B v A. K
hiu c l ng thng ct ng thng CB ti im cch u hai im C v A, ct ng
thng CA ti im cch u hai im C v B.
Gi A0 , B0 , C0 l nhng im i xng vi A, B, C qua BC, CA, AB, tng ng. Gi
A1 , B1 , C1 l nhng im i xng vi A, B, C qua a, b, c, tng ng. Gi 2 im E l
nhng im tha mn aEA = bEB = cEC, gi 2 im F l nhng im nhn cc cnh
2
tam gic ABC di gc hay . Gi H l trc tm v O l tm ng trn ngoi tip
3 3
ABC.
Gi A2 , A3 l cc nh ca cc tam gic u BCA2 , BCA3 , gi B2 , B3 l cc nh ca cc
tam gic u CAB2, CAB3 v gi C2 , C3 l cc nh ca cc tam gic u ABC2, ABC3 .

V d 3. Xc nh phng trnh a khi bit ta A, B, C.

Bi gii. Dng h ta Oxy sao cho A(0; 0), B( x2 ; kx2 ), C ( x3 ; hx3 ). im P0 ( p; hp) cch
u A v B khi v ch khi

p2 + h2 p2 = ( p x2 )2 + (hp kx2 )2 .

x2 ( 1 + k2 ) x2 h ( 1 + k2 ) 
Vy P0 ; . im N0 (n; kn) cch u A v C khi
2(1 + hk) 2(1 + hk)

9
Hi tho khoa hc, Hng Yn 25-26/02/2017

x3 ( 1 + h2 ) x3 k ( 1 + h2 ) 
v ch khi N0 ; . ng thng a l ng thng
2(1 + hk) 2(1 + hk)
x y 1
x y 1


x2 ( 1 + k2 ) 2

x2 h ( 1 + k )
2(1 + hk)

1

x2 x2 h
= 0 hay

1 + k2 = 0.
2(1 + hk) 2(1 + hk)


x3 ( 1 + h2 ) x3 k ( 1 + h2 ) 2(1 + hk)
x3 x3 k

1
1 + h2
2(1 + hk)
2(1 + hk)

Mnh 3. Gi s ABC c di cnh a, b, c. Khi 21 im nu trn u thuc tp K3 gm


tt c cc im M tha mn
1 a2 + MA2 a2 MA2


1 b2 + MB2 b2 MB2 = 0.

1 c2 + MC2 c2 MC2

Chng minh. Khi M A th MA = 0, MB = AB = c.MC = AC = b. Khi


1 a2 + MA2 a2 MA2 1 a2 0
1 b + MB b MB = 1 b + c b c2 = 0. Vy A, A0 K3 . Tng t
2 2 2 2
2 2 2

1 c2 + MC2 c2 MC2 1 c2 + b2 c2 b2
B, B0 , C, C0 K3 .
1 a2 + H A 2 a2 H A 2

V a2 + H A2 = b2 + HB2 = C2 + HC2 = 4R2 nn 1 b2 + HB2 b2 HB2 =



1 c2 + HC2 c2 HC2
1 4R2 a2 H A2

1 4R2 b2 HB2 = 0. Vy H K3 .


1 4R2 c2 HC2
1 a2 + OA2 a2 OA2 1 a2 + R2 a2 R2

V 1 b2 + OB2 b2 OB2 = 1 b2 + R2 b2 R2 = 0 nn O K3 .

1 c2 + OC2 c2 OC2 1 c2 + R2 c2 R2
1 a2 + EA2 a2 EA2 1 a2 + EA2 u2

t aEA = u. V 1 b2 + EB2 b2 EB2 = 1 b2 + EB2 u2 = 0 nn hai im E



1 c2 + EC2 c2 EC2 1 c2 + EC2 u2
a2 + b 2 + c 2
thuc K3 . t v = AA22 = + 2 3S. Do
2
1 a2 + A 2 A 2 a2 A 2 A 2 1 a2 + v a2 v


T = 1 b2 + A2 B2 b2 A2 B2 = 1 b2 + a2 b2 a2
1 c 2 + A 2 C 2 c 2 A 2 C 2 1 c 2 + a2 c 2 a2

a2 + v v 1 a2 v

1
nn T = 1 b2 + a2 b2 a2 = 1 a2 b2 a2 = 0. Vy A2 K3 . Tng t

c 2 + a2 c 2
1 1 a2 c 2
A3 , B2 , B3 , C2 , C3
K3 .
2
Gi s F nhn ba cnh di cng mt gc . t x = FA, y = FB, z = FC.
3
Khi ta c a = y + z + yz, b = z + x + zx, c2 = x2 + y2 + xy. V F :=
2 2 2 2 2 2

10
Hi tho khoa hc, Hng Yn 25-26/02/2017

1 a2 + FA2 a2 FA2 1 x2 + y2 + z2 + xy a2 x2

1 b2 + FB2 b2 FB2 = 1 x2 + y2 + z2 + yz b2 y2 nn


1 c2 + FC2 c2 FC2 1 x2 + y2 + z2 + zx c2 z2

1 xy a2 x2

F = 1 yz b2 y2 = c2 xz3 b2 xy3 + a2 x3 y c2 yz3 + b2 y3 z a2 x3 z



1 zx c2 z2

v d dng thy F = 0 qua vic thay a2 = y2 + yz + z2 , b2 = z2 + zx + x2 v c2 =


x2 + xy + y2 . Nh vy hai im F cng thuc K3 . Vic kim tra ba im cn li A1 , B1 , C1
cng thuc K3 c coi l mt bi tp.

Mnh 4. Nu ABC khng u th bc ca K3 bng 3.

Chng minh. t u = MA2 , v = MB2 , t = MC2 ; = a2 , = b2 , = c2 . Vy



1 + u u

0 = 1 + v v
1 + t t
= vt vt + ut ut + uv uv
+ u u + v v + t t.

Biu din u = x2 + y2 + a1 x + b1 y + c1 , v = x2 + y2 + a2 x + b2 y + c2 v t = x2 + y2 +
a3 x + b3 y + c3 , Khi h s ca ( x2 + y2 )2 ng bng + + = 0. Vy K3
l th bc 6 3. D dng kim tra bc ca K3 bng 3 khi tam gic ABC khng u.

Nhn xt 1. Phng trnh cho K3 khng qua nh thc rt phc tp

( a2 b2 ) MA2.MB2 + (b2 c2 ) MB2 .MC2 + (c2 a2 ) MC2.MA2 =

a2 c2 ( MA2 MC2 ) + c2 b2 ( MC2 MB2 ) + b2 a2 ( MB2 MA2 ).

Ti liu
[1] N. V. Mu v . V. Nh (2015), ng nht thc v phng php ta trong Hnh hc,
Nh xut bn HQG H Ni.
[2] T. T. Nam, . V. Nh, T. T. Tnh, N. A. Tun (2016), Gio trnh HNH HC S CP,
Nh xut bn HSP Thnh ph HCM.

11
Hi tho khoa hc, Hng Yn 25-26/02/2017

S LY THA
Nguyn Vit Hi
Tp ch Ton hc & Tui tr

Cc bi ton v s ly tha, ni ring l s chnh phng, thng khng cn nhiu vn


kin thc, nhng i hi s phn tch v tng hp gi thit mt cch thng minh, phng
php bin i kho lo, kh nng suy lun cht ch, bin lun y . Chnh v th m cc
bi ton v s ly tha thng gp trong cc k thi chn hc sinh gii cp 2, cp 3, thi ton
quc t v cc cuc thi tuyn vo lp 10.
Cc bi ton v s ly tha kh phong ph, y ch trnh by mt s kin thc c bn
dng xt xem mt s c l s chnh phng, s ly tha hay khng; ng thi nu mt
s bi ton lin quan n cc dng ca s ly tha.
Cn nhiu bi ton v s ly tha trong h thp phn cha c nu ra do s trang c
hn.
Mt s kin thc c bn v c s chung ln nht, cc s nguyn t cng nhau v phng
trnh v nh bc nht hai n c s dng, coi nh bn c bit.

1 Mt s tnh cht ca s ly tha


nh ngha 1.
a) Ta gi ly tha bc n, (n 2) ca mt s t nhin a, tc l s an , l s ly tha.
b) Ta gi bnh phng ca mt s t nhin a, tc l s a2 , l s chnh phng, nh th s
chnh phng l s ly tha bc hai.
c) S nguyn ln hn 1 m khng chia ht cho s chnh phng ln hn 1 no c gi l
s phi chnh phng.
Chng hn, cc s sau l s phi chnh phng: 2; 3; 5; 7; 6 = 2.3; 30 = 2.3.5.
Cc s sau khng l s chnh phng v cng khng l s phi chnh phng:
12 = 22.3; 60 = 22.3.5.

Nhn xt 1. S 0, s 1 l s chnh phng v l s ly tha bc ty . Cc tn gi s ly


tha, s chnh phng, s phi chnh phng ch s dng cho cc s nguyn khng m.
nh l 1. a) S phi chnh phng hoc l mt s nguyn t ln hn 1, hoc l tch cc s
nguyn t phn bit vi s m u bng 1.
b) Mi s nguyn dng a u biu din duy nht c trong dng tch ca mt s chnh
phng v mt s phi chnh phng, tc l c dng a = b2 c.
Chng minh.

12
Hi tho khoa hc, Hng Yn 25-26/02/2017

a) Gi p l c s nguyn t bt k ca s phi chnh phng c vi s m l s. Nu s 2


th c chia ht cho p2 , tri vi nh ngha s phi chnh phng, vy s = 1.
b) Gi b l s nguyn dng ln nht tha mn b2 l c ca a th a = b2 c. Nu s c khng
l s phi chnh phng th n phi chia ht cho mt s chnh phng e2 vi e > 1, lc
c = e2 d nn a = b2 c = b2 e2 d = (be)2 d m be > b, tri vi s xc nh s b.
Gi s s a c hai cch biu din a = b2 c = e2 d, trong c, d l cc s phi chnh phng. t
(b, e) = n th b = nb1 , e = ne1 v (b1 , e1 ) = 1. Lc b2 c = e2 d n2 b12 c = n2 e12 d b12 c =
e12 d. T v (b1 , e1 ) = 1 th (b12 , e12 ) = 1 nn e12 l c s ca c. Do c l s phi chnh phng
th e1 = 1. Xt tng t th b1 = 1. Vy cch biu din a = b2 c l duy nht
nh l 2. a) Nu s ly tha bc n chia ht cho s nguyn t p th s chia ht cho pn .
b) Nu s chnh phng chia ht cho s nguyn t p th s chia ht cho p2 .

Chng minh.
a) Gi s cn chia ht cho s nguyn t p vi n 2. Nu (c, p) = 1 th (cn , p) = 1, iu ny
khng xy ra nn (c, p) = p, tc l c chia ht cho p, do cn chia ht cho pn .
Khi n = 2 th c cu b).
nh l 3. a) Nu s ly tha bc n l tch ca hai s nguyn t cng nhau, tc l cn = a.b
vi ( a, b) = 1, th mi tha s a, b l s ly tha bc n. b) Nu s chnh phng l tch ca hai
s nguyn t cng nhau, tc l c2 = a.b vi ( a, b) = 1, th mi tha s a, b l s chnh phng.

Chng minh. a) t ( a, c) = e th a = ed v c = em vi (d, m) = 1. T cn = a.b vi n 2 c


en mn = edb en1 mn = db. V ( a, b) = 1 th (e, b) = 1, ng thi c en1 mn = db nn b l
c ca mn . T (d, m) = 1 th (d, mn ) = 1, ng thi c en1 mn = db th mn l c ca b. Suy
ra b = mn v d = en1 . T c a = ed = dn .
Khi n = 2 th c cu b).
nh l 4. Cn bc n ca mt s nguyn dng hoc l s nguyn dng, hoc l s v t.
Ni cch khc, nu an = d vi d l s nguyn dng m a l s hu t th a l s nguyn.

n r
Chng minh. Gi s d = a an = d vi d l s nguyn dng. Gi s a = l phn s
s
ti gin vi r, s l cc s nguyn dng, tc l (r, s) = 1, suy ra (s, r n ) = 1. T iu gi s c
r n = an sn = dsn , suy ra s l c s ca r n nn s = (s, r n ) = 1. Vy nu a l s hu t th a = r
l s nguyn dng.
nh l 5. Gi s a, b, m, n l cc s nguyn dng. a) Nu an l c ca bn th a l c
ca b. b) Nu aAm = bn v (m, n) = 1 th tn ti s nguyn dng c sao cho a = cn v b = cm .

Chng minh.
a) t ( a, b) = d th a = de v b = dk vi (e, k ) = 1. T (en , kn ) = 1. Theo gi thit
b = an c nn dn kn = dn en c, suy ra kn = en c m (en , kn ) = 1 nn en = 1, tc l e = 1, do
n

b = ak.
b) Theo iu kin c nghim ca phng trnh v nh bc nht, nu (m, n) = 1 th tn
ti cc s nguyn x, y sao cho mx + ny = 1. (Bn c t chng minh rng hai s x, y khng

13
Hi tho khoa hc, Hng Yn 25-26/02/2017

th cng du). Gi s mx ny = 1 vi x, y u dng (nu nx my = 1 th xt tng t),


b n
 x
nx mx ny bx
hay l mx = ny + 1. T b = a = a a , hay l a = . Theo nh l 4 th = c l
ay ay
s nguyn dng, suy ra a = cn , thay vo am = bn c b = cm .

nh l 6. Cho s nguyn s 2 th chn c s nguyn ns sao cho vi mi s nguyn


m ns th tn ti s ly tha as tha mn m < as < 2m.

Chng minh. Gi s c m < as < 2m hay l s m < a < s 2m. tn ti s a nguyn th cn
1
c s m + 1 < s 2m s 2m s m > 1 s m( s 2 1) > 1 m > s
. Nu chn s
( 2 1) s
1 1 s
nguyn ns 1 + s
th vi m n s 1 +
s
s c s
m < a < 2m
( 2 1) s ( 2 1) s

nh l 7. Gi s a, b, n l cc s nguyn dng. a) Nu s b tha mn an < b < ( a + 1)n th


s b khng l s ly tha bc n.
b) Nu s b tha mn a2 < b < ( a + 1)2 th s b khng l s chnh phng.

Chng minh. a) Gi s b = cn th c an < cn < ( a + 1)n , suy ra a < c < ( a + 1), nhng khng
tn ti s nguyn c nh th.
Khi n = 2 th c cu b)

nh l 8 (nh l Liouville ). Vi s nguyn dng a v n 2 th ng thc ( a 1)! + 1 = an


ch xy ra khi a = 5.

Chng minh. Gi s c s a > 5 tha mn ( a 1)! + 1 = an (n > 1). Do ( a 1)! l s chn


nn an l s l, suy ra a l s l. Theo gi thit c

( a 2)!( a 1) = an 1 = ( a 1)( an1 + an2 + + a + 1) ( a 2)! = ( an1 1) + ( an2 1) +


(1)
a1 a1
Vi a > 5 th a 2 > > 2nn( a 2)! cha tch .2 = a 1. T v (1) suy ra
2 2
a 1 l c ca n, do n a 1. Thay vo (1) c
( a 2)! a a2 + a a3 + + a + 1 > a a2 , nhng iu ny khng xy ra nn khng tn ti
s a nh th. Vi a 5 th ch xy ra ng thc 4! + 1 = 52.

Ch 1. Ta bit mt s ng thc dng a! + 1 = b2 (n > 1) nh: 4! + 1 = 52 ; 5! + 1 =


112 ; 7! + 1 = 712 . Nh ton hc M. Kraitchik kim tra (nm 1924) vi a 1020 th khng
c s a no a! + 1 l s chnh phng. Ta khng bit vi a > 1020 th c s a no a! + 1
l s chnh phng hay khng ?

nh l 9. a) S chnh phng c ch s tn cng l mt trong cc ch s 0, 1, 4, 5, 6, 9 v


khng c ch s tn cng l 2, 3, 7, 8.
b) Nu s chnh phng c ch s tn cng l 5 th hai ch s cui cng l 25.
c) Nu s chnh phng c ch s tn cng l 6 th ch s hng chc l ch s l.
d) Nu s chnh phng c ch s tn cng l 4 hoc l ch s l 1, 5, 9 th ch s hng chc

14
Hi tho khoa hc, Hng Yn 25-26/02/2017

l ch s chn.

Chng minh. Xt s n = 10k + r vi k, r u l s nguyn v 0 r 9 th n2 = (10k + r )2 =


20k(5k + r ) + r2 .V r2 ch c th l 00, 01, 04, 09, 16, 25, 36, 49, 64, 81 nn ch s tn cng ca
s chnh phng ch c th l 0, 1, 4, 5, 6, 9. Hn na, do 20k (5k + r ) l s chn nn ch s
hng chc ca s chnh phng c cng tnh chn l vi r2 , t kt lun c hai ch s
cui cng ca n2

nh l 10. a) S chnh phng khi chia cho 3 c dng 3n hoc 3n + 1 v khng c dng
3n + 2.
b) S chnh phng khi chia cho 4 c dng 4n hoc 4n + 1 v khng c dng 4n + 2, 4n + 3.
c) S chnh phng khi chia cho 5 c dng 5n, hoc 5n + 1, hoc 5n + 4 v khng c dng
5n + 2, 5n + 3.
d) S chnh phng khi chia cho 6 c dng 6n, hoc 6n + 1, hoc 6n + 3 hoc 6n + 4 v
khng c dng 6n + 2, 6n+.
e) S chnh phng khi chia cho 8 c dng 8n hoc 8n + 1 hoc 8n + 4 v khng c dng
8n + r vi r bng 2, 3, 5, 6, 7.
g) S chnh phng khi chia cho 9 c dng 9n hoc 9n + 1 hoc 9n + 4 hoc9n + 7 v khng
c dng 9n + r vi r bng 2, 3, 5, 6, 8.

Chng minh. p dng lp lun nh chng minh nh l 9.


a) Xt n = 3k + r vi k, r u l s nguyn v 0 r 2 th n2 = (3k + r )2 = 3k (3k + 2r ) + r2
v r2 ch c th l 0, 1, 4, t rt ra kt lun. Chng minh tng t cho cc trng hp cn
li

nh l 11. a) S ly tha bc s 3 khi chia cho 4 khng c dng 4n + 2.


b) S ly tha bc s 3 khi chia cho 8 khng c dng 8n + r vi r bng 2, 4, 6.
c) S ly tha bc ba khi chia cho 9 khng c dng 9n + r vi r bng 2, 3, 4, 5, 6, 7.
d) S ly tha bc s > 3 khi chia cho 9 khng c dng 9n + r vi r bng 3, 6.
Chng minh. a) Vi r bng 0, 1, 2, 3 th s r s khng c dng 4n + 2 vi s 3 nn (4t + r )s =
4m + r s khng c dng 4n + 2. Chng minh tng t i vi b) c) v d).
chng minh mt s l s ly tha ta c th dng cc nh l 2, nh l 3, nh l 4, nh
l 5, hoc bin i s ang xt thnh ly tha bc n ca s nguyn.
chng minh mt s khng phi l s chnh phng hoc khng l s ly tha ta ch ra
rng s khng tha mn mt iu kin cn ca s chnh phng, s ly tha theo cc
nh l 2, nh l 7, nh l 9, nh l 10, nh l 11.

2 Xc nh v nhn dng s ly tha


Bi ton 1. Chng minh rng mi cp s nguyn dng (m, n) m tng v tch ca chng
u l s chnh phng th chng c dng m = ka2 , n = kb2 , trong a2 + b2 = kc2 vi k l
s phi chnh phng.
Li gii. Gi s m + n = q2 v mn = e2 . t d = (m, n) th m = dm1 , n = dn1 v (m1 , n1 ) = 1
theo tnh cht ca c chung ln nht. Thay vo mn = e2 c d2 m1 n1 = e2 . T d2 l c

15
Hi tho khoa hc, Hng Yn 25-26/02/2017

ca e2 , xt (d, e) suy ra d l c ca e, tc l e = dh, thay vo ng thc trn c m1 n1 = h2 .


T v (m1 , n1 ) = 1 th m1 = u2 v n1 = v2 theo nh l 3, tc l m = du2 , n = dv2 .
Theo nh l 1 vit c d = kg2 vi k l s phi chnh phng. t a = gu, b = gv th
vit c m = kg2 u2 = ka2 , n = kg2 v2 = kb2 , trong k l s phi chnh phng v
q2 = m + n = ka2 + kb2 = k( a2 + b2 ). t (k, q) = p th k = ps, q = pc v (s, c) = 1
theo tnh cht ca c chung ln nht. T ng thc trn k = ps l c ca q2 = p2 c2 ,
tc l p2 c2 = tps, suy ra pc2 = ts, m (s, c) = 1 nn s l c ca p, tc l p = rs, do
k = ps = rs2 , nhng k l s phi chnh phng nn s = 1, tc l k = p v q = pc = kc. Thay
vo ng thc q2 = k( a2 + b2 ) c k2 c2 = q2 = k ( a2 + b2 ) rt ra kc2 = a2 + b2 ..

Ngc li, d thy cc cp s (m, n) c dng trn th c tng v tch u l s chnh


phng.

Bi ton 2 (Cp s cng (nh thc bc nht) cha s chnh phng). a) Chng minh rng
nu mt cp s cng a + bn (vi cc s nguyn a, b cho, b > 0 v n = 1, 2, . . . ) cha mt
s chnh phng th cp s cha v hn s chnh phng.
b) Tm cc s nguyn n sao cho 27 + 22n l s chnh phng.
c) T cu trn hy ch ra cch tm cc s nguyn n sao cho a + bn l s chnh phng (vi
cc s nguyn a, b cho, b > 0).

Li gii. a) Gi s trong cp s cng a + bn vi n = 1, 2, . . . tn ti s a + bn0 = e2 th cc


s dng (bn + e)2 = b2 n2 + 2ben + e2 = b(bn2 + 2en + n0 ) + a cng l s chnh phng vi
n = 1, 2, . . .
b) Ta chuyn vic xt s chnh phng 27 + 22n = e2 v xt s chnh phng 5 + 22s = e2 ,
trong n = s 1 1. t e = 22t + rvi 10 r 10 th 22s + 5 = e2 = 22(22t2 + 2tr ) + r2
nn 22 l c s ca r2 5, suy ra 0 r2 = 22m + 5 100, dn n 0 m 4. T ta
tm c m = 2 nn r = 7 hoc r = 7. T 22n + 27 = e2 = 22(22t2 + 2tr ) + 49 suy ra hai
nghim l n1 = 22t2 + 14t + 1 v n2 = 22t2 14t + 1, trong t l s t nhin ty .
c) Gi s vi cc s nguyn a, b, n c s chnh phng a + bn = e2 . Ta t s a = bv + d vi
0 d b 1 th e2 = bn + a = bn + bv + d = b(n + v) + d.
Ta t s e = bt + r vi k r k , trong b = 2khocb = 2k + 1, th
bn + d = e2 = (bt + r )2 = b(bt2 + 2tr ) + r2 hay l bm + d = r2 vi m = n (bt2 + 2tr ).
Gi s b = 2k hoc b = 2k + 1. Ta chuyn vic xt s chnh phng a + bn = e2 v xt s
chnh phng d + bm = r2 vi 0 d b 1 v k r k (vi b = 2k th tha mt s),
dn n tm s m sao cho 1 b d bm = r2 d r2 k2 b2 , tc l 0 m b .
Nh vy s a + bn = e2 l s chnh phng khi v ch khi tn ti s m sao cho d + bm = r2
vi cc s d, m, r c xc nh nh trn.

Ch rng tn ti cp s cng a + bn khng cha s chnh phng no nh 2 + 22n.

Bi ton 3. Tam thc bc hai cha s chnh phng


a) Tm cc s nguyn n sao cho n2 + 4n + 25 l s chnh phng.
T hy ch ra cch tm cc s nguyn n sao cho n2 + 2kn + c l s chnh phng (vi c, k
l cc s nguyn cho, k > 0).
b) Tm cc s nguyn n sao cho n2 + 3n + 11 l s chnh phng.

16
Hi tho khoa hc, Hng Yn 25-26/02/2017

T hy ch ra cch tm cc s nguyn n sao cho n2 + (2k + 1)n + c l s chnh phng


(vi c, k l cc s nguyn cho, k > 0).

Li gii. a) Gi s n2 + 4n + 25 = e2 th c (n + 2)2 + 21 = e2 nn e2 (n + 2)2 = 21, hay l


(e
( n 2)(e + n + 2) = (1.3.7. Xt cc trng hp sau y.
e + n + 2 = 21 e = 11
1)
en2 = 1 n=8

Lc 2 2
( c 8 + 4.8 + 25 =
( 121 = 11
e+n+2 = 7 e=5
2)
en2 = 3 n=0

Lc c 02 + 3.0 + 25 = 52

Gi s n2 + 2kn + c = e2 th c (n + k )2 + c k2 = e2 nn e2 (n + k )2 = c k2 , hay l
(e n k)(e + n + k) = c k2 . V e (n + k)ve + (n + k) c cng tnh chn l nn cn phn
tch s c k2 thnh tch hai s cng tnh chn l.
Nu c k2 = u.v 0 vi 0 u vvu + v chn th gii h phng trnh e n k = u v
e + n + k = v s tm c nghim (e, n).
Nu k2 c = u.v 0 vi 0 u v v u + v chn th gii h phng trnh n + k e = u v
e + n + k = v s tm c nghim (e, n).
b) Gi s n2 + 3n + 11 = e2 hay l 4n2 + 12n + 44 = 4e2 th c (2n + 3)2 + 35 = 4e2 nn
4e2( (2n + 3)2 = 35, hay l((2e 2n 3)(2e + 2n + 3) = 1.5.7. Xt cc trng hp sau y.
2e + 2n + 3 = 35 e=9
1)
2e 2n 3 = 1 n=7
2 2
(Lc c 7 + 3.7 + 11(= 81 = 9
2e + 2n + 3 = 7 e=3
2)
2e 2n 3 = 5 n = 1

Lc c 12 3.1 + 11 = 9 = 32
c) Gi s n2 + (2k + 1)n + c = e2 hay l 4n2 + 4(2k + 1)n + 4c = 4e2 th c
(2n + 2k + 1)2 + 4c (2k + 1)2 = 4e2 nn 4e2 (2n + 2k + 1)2 = 4c (2k + 1)2 , hay
l (2e 2n 2k 1)(2e + 2n + 2k + 1) = 4c (2k + 1)2 .
V 2e (2n + 2k + 1) v e + (2n + 2k + 1) c cng tnh chn l nn cn phn tch s
4c (2k + 1)2 thnh tch hai s cng tnh chn l.
Nu 4c (2k + 1)2 = u.v 0 vi 0 u v v u + v chn th gii h phng trnh
2e 2n 2k 1 = u v 2e + 2n + 2k + 1 = v s tm c nghim (e, n).
Nu (2k + 1)2 4c = u.v 0 vi 0 u v v u + v chn th gii h phng trnh
2n + 2k + 1 2e = u v 2e + 2n + 2k + 1 = v s tm c nghim (e, n).

17
Hi tho khoa hc, Hng Yn 25-26/02/2017

3 S ly tha di dng tch cc s nguyn dng


Bi ton 4. Chng minh rng mi s sau khng l s chnh phng:
a) Tch hai s nguyn dng chn lin tip;
b) Tch bn s nguyn dng lin tip.

Li gii. a) Gi s 2a(2a + 2) = b2 th s b phi chn, tc l b = 2c. Thay vo ng thc trn


c a( a + 1) = b2 . V ( a, a + 1) = 1 nn theo nh l 3 phi c a = c2 , a + 1 = e2 , trong
c 1. D thy rng c2 < c2 + 1 < c2 + 2c + 1 = (c + 1)2 nn theo nh l 7 th khng tn ti
s chnh phng a + 1 = c2 + 1 = e2 , tri vi iu gi s. b) Xt tch a( a + 1)( a + 2)( a + 3) =
a( a + 3)( a + 1)( a + 2) = ( a2 + 3a)( a2 + 3a + 2). S a2 + 3a = a( a + 3) = 2b l s chn v hai
s a v a + 3 c tnh chn l khc nhau. Lc tch ban u tr thnh 2b(2b + 2), s dng
kt qu cu a).

Bi ton 5. Chng minh rng mi s sau khng l s ly tha bc n


a) Tch hai s nguyn dng lin tip;
b) Tch hai s nguyn dng l lin tip;
c) Tch ba s nguyn dng lin tip.

Li gii. a) Gi s a( a + 1) = bn . V ( a, a + 1) = 1 nn theo nh l 3 phi c a = cn , a + 1 = en ,


trong c 1. Vi n 2 ta s ch ra rng cn < cn + 1 < (c + 1)n , tc l c
a = cn < a + 1 = en = cn + 1 < (c + 1)n , nh th theo nh l 7 th khng tn ti
s a + 1 = en .
Ta s chng minh quy np theo n rng cn < cn + 1 < (c + 1)n . Vi n = 2 th
c2 < c2 + 1 < c2 + 2c + 1 = (c + 1)2 , khng nh ng. Gi s khng nh ng
n n, xt s m n + 1 c cn+1 + 1 < c.cn + 1 (cn + 1)c < (c + 1)n (c + 1) < (c + 1)n+1 ,
khng nh ng vi n + 1. Vy khng nh ng vi s nguyn dng n bt k nn khng
tn ti s a + 1 = en .
b) Xt s l a v gi s a( a + 2) = bn . t d = ( a, a + 2) th d l c ca ( a + 2) a = 2,
nhng do a l nn d = 1. Theo nh l 3 phi c a = cn , a + 2 = en , trong c 1. Vi n 2
ta s ch ra rng cn < cn + 2 < (c + 1)n , tc l c a = cn < a + 2 = en = cn + 2 < (c + 1)n ,
nh th theo nh l 7 th khng tn ti s a + 2 = en .
Ta s chng minh quy np theo n rng cn < cn + 2 < (c + 1)n . Vi n = 2 th
c2 < c2 + 2 < c2 + 2c + 1 = (c + 1)2 , khng nh ng. Gi s khng nh ng n n, xt s
m n + 1 c cn + 1 + 2 < c.cn + cn + 2c + 2 < (cn + 2)(c + 1) < (c + 1)n (c + 1) < (c + 1)n+1 ,
khng nh ng vi n + 1. Vy khng nh ng vi s nguyn dng n bt k nn khng
tn ti s a + 1 = en .
c) Gi s a( a + 1)( a + 2) = bn . Do ( a + 1, a( a + 2)) = 1 nn Theo nh l 3 phi c
a + 1 = cn , a( a + 2) = en , trong n 2 v c 2. T 1 = ( a + 1)2 a( a + 2) =
c2n en = (c2 e)(c2n2 + c2n4 e + + c2 en2 + en1 ), nhng v phi ca ng thc trn
ln hn 1 khi n 2

Nhn xt 2. 1) Cc nh ton hc P. Erdos


v J.L.Selfridge chng minh c rng
Tch ca n(n > 1) s nguyn dng lin tip khng l s ly tha. ([2] tr. 86).

18
Hi tho khoa hc, Hng Yn 25-26/02/2017

2) Nh ton hc P. Erdos
chng minh c rng:
Tch ca n(n > 1) s nguyn dng l lin tip khng l s ly tha. ([2] tr. 87).
3) Xt tch hai s nguyn dng chn lin tip 2a(2a + 2) = b3 (2a + 1)2 = b3 + 1. Ta bit
c ng thc 32 = 23 + 1. Nm 1844 nh ton hc ngi B C. E.Catalan nu gi thuyt:
Hai s nguyn dng lin tip khc 8 v 9 th khng th l nhng s ly tha.
Nhiu nh ton hc tm cch chng minh gi thuyt ny, mi n nm 2002 iu ny
mi c tin s Preda Mihailescu chng minh y (http://www.math.uni Paderborn
de/ preda/ papers/ caterelle.ps).

4 Tng cc s ly tha
Bi ton 6. Chng minh rng mi s sau khng l s chnh phng:
a) Tng cc bnh phng ca hai s l.
b) Tng cc ly tha bc chn ca hai s l.

Li gii. a) b) l chng c dng 4k + 2.


Bi ton 7. Chng minh rng tng cc bnh phng ca k s nguyn dng lin tip khng
l s chnh phng vi mi s k bng 3, 4, 5, 6, 7, 8, 9, 10.

Li gii. a) Vin 1 xt tng Sk = n2 + (n + 1)2 + + (n + k 1)2 ==


12 + 22 + + (n + k 1)2 (12 + 22 + + (n 1)2 ) == 12 + 22 + + (k 1)2 + k2 +
( k + 1 ) 2 12 + ( k + 2 ) 2 22 + + ( k + n 1 ) 2 ( n 1 ) 2 .
Ch rng 12 + 22 + + (k 1)2 = (k 1)k (2k 1).
Cn k2 + k (k + 2) + k(k + 4) + + k (k + 2n 2) = nk2 + 2k (1 + 2 + + (n 1)) =
nk2 + kn(n 1) = kn2 + k (k 1)n. T Sk = kn2 + k(k 1)n + (k 1)k (2k 1)
T s p dng nh l 2, nh l 10 cho mi trng hp sau y.
S3 = 3n2 + 6n + 5 = 3(n + 1)2 + 2, c dng 3m + 2.
S4 = 4n2 + 12n + 14 = 4(n2 + 3n + 3) + 2, c dng 4m + 2.
S5 = 5n2 + 20n + 30 = 5((n + 2)2 + 2). S c dng m2 + 2 khng chia ht cho 5 khi t
m = 5t + r vi r = 0, 1, 2, 1, 2.
Nu S6 = 6n2 + 30n + 55 = 6(n + 2)(n + 3) + 18 + 1 = m2 th s m l nn c
6(n + 2)(n + 3) + 18 = (m 1)(m + 1). V phi chia ht cho 4, nhng 18 = 2.9.
S7 = 7(n2 + 6n + 13) = 7(7n + 14 + n2 n 1). S n2 n 1 khng chia ht cho 7 khi t
n = 7t + rvir = 0, 1, 2, 3, 1, 2, 3.
S8 = 4(2n2 + 14n + 35). Nu 2n2 + 14n + 35 = m2 th s m l nn c 2(n + 3)(n + 4) + 10 =
(m 1)(m + 1). V phi chia ht cho 4, nhng 10 = 2.5.
S9 = 9(n + 4)2 + 9.6 + 6, c dng 9m + 6.
S1 0 = 5(2n2 + 18n + 57). S 2n2 + 18n + 57 = 2(n2 n + 1) + 20n + 55 khng chia ht cho
5 khi t n = 5t + r vi r = 0, 1, 2, 1, 2.

Nhn xt 3. 1) Vi k = 2 th phng trnh n2 + (n + 1)2 = m2 (2n + 1)2 2m2 = 1


t2 2m2 = 1 (phng trnh Pell ) vi t = 2n + 1, c v hn nghim nguyn dng, chng

19
Hi tho khoa hc, Hng Yn 25-26/02/2017

hn l 32 + 42 = 52 v 202 + 212 = 292 . D dng chng minh rng:


Nu phng trnh n2 + (n + 1)2 = m2 c nghim (n1 ; n1 + 1; m1 ) th n cng c nghim
(n2 ; n2 + 1; m2 ) vi n2 = 3n1 + 2m1 + 1; m2 = 4n1 + 3m1 + 2.
Mnh o cng ng, tc l h thc trn cng vi 32 + 42 = 52 l mi nghim ca phng
trnh n2 + (n + 1)2 = m2 ([2] tr. 43-44). Xem thm Bi tp 13, 15. 2) Vi k = 11 th mnh
trn khng ng. Chng hn:
182 + 192 + 202 + 212 + 222 + 232 + 242 + 252 + 262 + 272 + 282 = 772 .

Bi ton 8. Chng minh rng:


a) Tng cc ly tha bc chn ca ba s nguyn lin tip khng l s ly tha bc chn.
b) Tng cc ly tha bc chn bng nhau ca 9 s nguyn lin tip khng l s ly tha.

Li gii. a) Ba s nguyn lin tip khi chia cho 3 c cc s d khc nhau nn tng cc ly
tha bc chn ca chng c dng 3n + 2. p dng nh l 10.
b) Vit mi s trong dng 9t + r vi 4 r 4 th tng cc ly tha bc chn u bng 2n
ca 9 s nguyn lin tip c dng S = 9m + 2(1n + 4n + 16n ) = 9k + 2(1n + 4n + 7n ). t
n = 3v + s vi 0 s 2 th 1n + 4n + 7n c dng 9u + 3 nn S c dng 9x + 6. p dng
nh l 2.

Bi ton 9. Tm s t nhin n nh nht vi n > 1 sao cho tng cc bnh phng ca n s t


nhin lin tip t 1 n n l s chnh phng. Li gii. Tng cc bnh phng ca n s t
1
nhin lin tip t 1 n n bng S = 12 + 22 + + n2 = n(n + 1)(2n + 1). Gi s S = m2
6
th
n(n + 1)(2n + 1) = 6m2 . (2)
Xt cc trng hp sau.
1) Nu n = 6k th (2) c dng k (6k + 1)(12k + 1) = m2 . Cc tha s v tri nguyn t snh
i nn p dng lin tip nh l 3 i vi tch k (6k + 1) v tch (k (6k + 1))(12k + 1) th mi
tha s l s chnh phng. Vi k = 1 th v tri bng 7.13 nn khng l s chnh phng.
Vi k = 2 th v tri c dng 4k + 2 nn khng l s chnh phng. Vi k = 3 th v tri c
dng 9k + 3 nn khng l s chnh phng. Vi k = 4 th n = 24, v tri bng 4.25.49 nn c

12 + 22 + + 242 = 702 . (3)

2) Nu n = 6k + 1 th (2) c dng (6k + 1)(3k + 1)(2k + 1) = m2 . Lp lun tng t trn,


ch rng s 2k + 1 nh nht (ln hn 1) l s chnh phng khi k = 4, lc n = 25 > 24.
3) Nu n = 6k + 2 th (2) c dng (3k + 1)(2k + 1)(12k + 5) = m2 .
4) Nu n = 6k + 3 th (2) c dng (2k + 1)(3k + 2)(12k + 7) = m2 .
5) Nu n = 6k + 4 th (2) c dng (3k + 2)(6k + 5)(4k + 3) = m2 .
6) Nu n = 6k + 5 th (2) c dng (6k + 5)(k + 1)(12k + 1) = m2 .
C bn trng hp trn u khng xy ra ng thc khi k = 0.
Xt k > 0, lp lun nh trn th mi tha s v tri trong mi trng hp u l s chnh
phng.
Ch rng 12k + 5 = 3(4k + 1) + 2 v 6k + 5 = 3(2k + 1) + 2 nn mi tch v tri trong
mi trng hp u cha tha s 3t + 2, tha s ny khng l s chnh phng theo nh
l 10, tri vi nhn xt trn.

20
Hi tho khoa hc, Hng Yn 25-26/02/2017

Nh vy t ng thc (3) v lp lun trn rt ra s n = 24 l s nh nht tha mn bi.

Ch 2. 1) chng minh c s n = 24 l s duy nht tha mn bi. ([2] tr. 87).


2) Cng chng minh c tng 13 + 23 + + n3 khng l s ly tha bc ba. ([2] tr. 81).

5 Bi tp
Bi 1. Chng minh rng mi s sau khng l s chnh phng:
a) A = 12345678
b) B = 996699 + 20112012
c) C = 72012 + 52013
d) D = 3.512010 + 242

Bi 2. Chng minh rng mi s sau khng l s chnh phng vi cc s nguyn dng n,


m bt k:
a) E = 19n5 + 15n3 19n 2
b) F = n6 n4 + 2n3 + 2n2 (n > 1)
c) G = 3nm 1
d) H = 3n + 4

Bi 3. Chng minh rng mi s sau khng l s ly tha:


a) K = 1012 + 32
b) Tng cc bnh phng ca nm s nguyn lin tip.

Bi 4. Chng minh rng mi s sau khng l s chnh phng:


a) n3 + 1 vi s t nhin l n.
b) 2.13n + 5.7n + 26 vi s t nhin n.

Bi 5. Tm mi cp s (m, n), mi s nh hn 60, m tng m + n v tch mn u l s chnh


phng.

Bi 6. Chng minh rng vi x y 1 th s x2 + y khng l s chnh phng

Bi 7. Chng minh rng vi n 2 th s 2n 1 khng l s ly tha.

Bi 8. Chng minh rng vi n 4 th s 2n + 1 khng l s ly tha.

21
Hi tho khoa hc, Hng Yn 25-26/02/2017

Bi 9. Chng minh rng mi s sau l s chnh phng vi n l s t nhin:


a) Tch ca bn s nguyn lin tip cng vi 1.
b) 4S + 1 vi S = 1.2.3 + 2.3.4 + + n(n + 1)(n + 2).
c) Tng M = 13 + 23 + + n3

Bi 10. Tm cc s nguyn n sao cho 15n + 34 l s chnh phng.

Bi 11. Tm cc snguyn n sao cho n2 + 5n + 11 l s chnh phng.

Bi 12. a) Tm s nguyn n2 nh nht sao cho vi mi s nguyn m n2 th c s ly tha


a2 tha mn m < a2 < 2m.
b) Tm s nguyn n3 nh nht sao cho vi mi s nguyn m n3 th c s ly tha a3 tha
mn m < a3 < 2m.

Bi 13. Gii phng trnh nghim nguyn dng m2 + n2 = (n + 1)2 .

Bi 14. Cho phng trnh nghim nguyn dng x2 + y2 = z2 .


a) Chng minh rng hoc x hoc y phi chia ht cho 3.
b) Chng minh rng hoc x hoc y phi chia ht cho 4.
c) Chng minh rng hoc x hoc y hoc z phi chia ht cho 5.
d) Gii phng trnh x2 + y2 = z2 vi x, y, z tng cp nguyn t cng nhau.

Bi 15. a) Gii phng trnh nghim nguyn x2 + ( x + 1)2 = ( x + 2)2 .


b) Chng minh rng phng trnh x n + ( x + 1)n = ( x + 2)n khng c nghim s t nhin
vi n > 2.
c) Gii phng trnh nghim nguyn x3 + ( x + 1)3 + ( x + 2)3 = ( x + 2)3 .
d) Chng minh rng phng trnh x3 + ( x + 1)3 + ( x + 2)3 + ( x + 3)3 = ( x + 4)3 khng c
nghim nguyn.

Ti liu
[1] Sierpinski W. A (1964), Selection of Problems in the Theory of Number. The Macmillan Com-
pany. 1
[2] Sierpinski W.A (1998), Elementary Theory of Numbers, North-Holland. Amsterdam,
PWN- Polish. Scientific Publishers.Warszawa.(1ed : 1964).

1 Sch ny dch ba quyn ting Nga, c sa cha, b sung:


1) Ta bit g v khng bit g v cc s nguyn t ca W. Sierpinski (1961). 2) Mt trm vn n gin, ng
thi rt kh trong S hc ca W. Sierpinski (1961). 3) Trn ranh gii ca Hnh hc v S hc (ca A. Makowski,
in chung vi quyn 2).

22
Hi tho khoa hc, Hng Yn 25-26/02/2017

MT S BT NG THC CA CC HM
HYPERBOLIC

Nguyn Vn Ngc
i hc Thng Long

Tm tt ni dung
Cc hm hyperbolic c nhiu tnh cht tng t hoc ging cc tnh cht ca cc hm
lng gic, mc d chng c nh ngha nh l nhng hm m. V vy, nhiu ti liu
cn gi cc hm ny l hm lng gic hyperbolic.
Cc hm hyperbolic c nhiu ng dng trong gii tch Ton, c bit l trong cng
thc nghim ca cc phng trnh vi phn tuyn tnh h s hng, trong php tnh tch
phn, trong Vt l v.v..
Do nhu cu v so snh nh gi cc i lng cha cc hm hyperbolic, pht sinh cc
bt ng thc ca cc hm hyperbolic, vic nghin cu cc bt ng thc ca cc hm
hyperbolic c nhiu ngi quan tm. Hin nay c mt s lng ng k cc cng
trnh v bt ng thc ca cc hm hyperbolic. Tuy nhin, sch chuyn kho v bt ng
thc ca cc hm hyperbolic cha c nhiu, nht l bng ting Vit.
Bi vit ny gii thiu mt s bt ng thc ca cc hm hyperbolic, nhm gi v
lm c s cho nhng chuyn bi dng hc sinh gii.

1 B tr
1.1 Cc bt ng thc c bn ca dy s
Trong mc ny trnh by kin thc b tr v mt s bt ng thc ca dy s tin
s dng sau ny.
Trong lun vn ny xin trnh by li mt s bt ng thc i s c bn nht l
bt ng thc AM GM (Arithmetic Mean - Geometric Mean), bt ng thc Cauchy -
Schawrz, bt ng thc Chebyshev,...
nh l 1 (Bt ng thc AM - GM). Vi n s thc khng m bt k a1 , a2 , . . . , an , ta c
bt ng thc
a1 + a2 + . . . + a n
> n a1 .a2 . . . . .an .
n
ng thc xy ra khi v ch khi a1 = a2 = . . . = an .
nh l 2 (Bt ng thc Cauchy - Schawrz). Xt hai b s thc ty a1 , a2 , , an v
b1 , b2 , , bn . Khi ta c

( a1 b1 + a2 b2 + + an bn )2 6 ( a21 + a22 + + a2n )(b12 + b22 + + bn2 ).

23
Hi tho khoa hc, Hng Yn 25-26/02/2017
a1 a2 an
ng thc xy ra khi v ch khi = = = , (vi quy c nu mu bng 0 th t
b1 b2 bn
cng bng 0).
nh l 3 (Bt ng thc Holder). Cho a = ( a1 , a2 , ..., an ) v b = (b1 , b2 , ..., bn ) l hai b n
1 1
s thc dng v p > 1, + = 1. Khi y
p q
! 1p ! 1q
n n n
a i bi
p q
ai bi . (1.1)
i =1 i =1 i =1

p q
Du bng xy ra khi v ch khi cc vect a p v bq t l, ngha l ai = kbi vi mi i
{1, 2, , . . . , n}.
nh l 4 (Bt ng thc Minkovski cho dy s thc). Cho a = ( a1 , a2 , ..., an ) v b =
(b1 , b2 , ..., bn ) Rn v p > 1. Khi y
" # 1p ! 1p ! 1p
n n n
( a i + bi )
p p p
ai + bi . (1.2)
i =1 i =1 i =1

Du bng xy ra khi v ch khi cc vect a v b t l, ngha l ai = kbi vi mi i


{1, 2, ..., n}

1.2 Bt ng thc ca cc i lng trung bnh


Vi a, b l cc s dng, xt cc biu thc sau y, c gi l cc i lng trung
bnh (xem v d [2]).
Trung bnh s hc, hay trung bnh cng:

a+b
A( a, b) = .
2
Trung bnh hnh hc, hay trung bnh nhn:

G ( a, b) = ab.

Trung bnh logarit:


ab
L( a, b) = , L( a, a) = a.
ln a ln b
Trung bnh identric:
1  b a 1/ba
I ( a, b) = b /a , I ( a, a) = a.
e
Trung bnh ly tha:
1/r
ar + br

Ar ( a, b) = (r 6= 0), A0 ( a, b) = G ( a, b)(r = 0).
2
Trung bnh Seiffert:
ab
P( a, b) = , P( a, a) = a,
ab
2arcsin
a+b

24
Hi tho khoa hc, Hng Yn 25-26/02/2017

C cc bt ng thc sau y [2]

G ( a, b) L( a, b) P( a, b) I ( a, b) A( a, b) A2 ( a, b). (1.3)

Du ng thc xy ra khi v ch khi a=b.

1.3 Hm li v bt ng thc Jensen


nh ngha 1. Hm s thc f : ( a, b) R gi l hm li trn khong [ a, b] nu vi mi
x, y ( a, b) v mi [0, 1], ta c

f (x + (1 )y) 6 f ( x ) + (1 ) f (y). (1.4)

Nu trong (1.4) ta c bt ng thc nghim ngt (cht) th khi ta ni f l hm li


thc s. Cho hm f ta ni n l hm lm nu f l hm li.
Nu f c xc nh trn R, n c th xy ra trn mt vi khong hm ny l hm li,
nhng trn khong khc n l hm lm. V l do ny ta ch xt cc hm s xc nh trn
cc khong.

nh l 5. Cho hm s f(x) kh vi cp hai trong khong (a,b), khi f(x) l hm li trong


00
khong (a,b) khi v ch khi f ( x ) 0, x ( a; b).

Vn dng nh l 1.1, d dng thy rng cc hm s sau y l cc hm li trn cc


khong tng ng

V d 1.

a) coshn x, sinhn x, x R,
1 1
b) n , , x R+ .
cosh x sinhn x

nh l 6 (Bt ng thc Jensen 1906, Joham Ludwig Jensen 1859 - 1925)). Cho f :
( a, b) R l hm li trn khong ( a, b). Cho n N v 1 , 2 , ,
n (0, 1) l cc s thc tha mn 1 + 2 + + n = 1. Khi vi mi x1 , x2 , , xn
( a, b), ta c !
n n
f i xi 6 i f ( x i ),
i =1 i =1

ngha l

f ( 1 x1 + 2 x2 + + n x n ) 6 1 f ( x1 ) + 2 f ( x2 ) + + n f ( x n ). (1.5)

Bt ng thc (1.5) c chng minh bng quy np ton hc.

2 Tnh li ca cc hm hyperbolic
Phn ny trnh by nh ngha, cc tnh cht c bn ca cc hm hyperbolic v cc bt
ng thc dng Jensen ca cc hm hyperbolic.

25
Hi tho khoa hc, Hng Yn 25-26/02/2017

2.1 nh ngha
nh ngha 2. Cc hm hyperbolic, hay cn gi l cc hm lng gic hyperbolic, c
nh ngha theo cc cng thc sau
Hm sin hyperbolic :
e x e x
sinh x = ,
2
Hm cosin hyperbolic:
e x + e x
cosh x = ,
2
Hm tang hyperbolic :
sinh x
tanh x = ,
cosh x
Hm cotang hyperbolic:
cosh x
coth x = ,
sinh x
Hm sec hyperbolic :
1
sec hx = ,
cosh x
Hm cosec hyperbolic:
1
csc hx = ,
sinh x
trong x l s thc.

2.2 Cc tnh cht c bn


C th d dng chng minh cc tnh cht c bn sau bng cch suy ra trc tip t
nh ngha
Tnh chn l ca cc hm hyperbolic

sinh ( x ) = sinh x,
cosh ( x ) = cosh x,
tanh ( x ) = tanh x,
coth ( x ) = coth x,
sec h ( x ) = sec hx,
csc h ( x ) = csc hx.

Bnh phng ca cc hm hyperbolic

cosh2 x sinh2 x = 1,
1
1 tanh2 x = ,
cosh2 x
1
coth2 x 1 = .
sinh2 x

26
Hi tho khoa hc, Hng Yn 25-26/02/2017

2.3 Cng thc cng

sinh( x + y) = sinh x cosh y + cosh x sinh y,


cosh( x + y) = cosh x cosh y + sinh x sinh y,
sinh( x y) = sinh x cosh y cosh x sinh y,
cosh( x y) = cosh x cosh y sinh x sinh y,
tanh x tanh y
tanh( x y) = .
1 tanh x tanh y

2.4 Cng thc nhn hai v h bc

sinh 2x = 2 sinh x cosh x,


cosh 2x = cosh2 x + sinh2 x,
2 tanh x
tanh 2x = ,
1 + tanh2 x
cosh 2x 1
sinh2 x = cosh 2x cosh2 x = ,
2
cosh 2x + 1
cosh2 x = cosh 2x sinh2 x = .
2

2.5 Cng thc bin i tng thnh tch

x+y xy
sinh x + sinh y = 2 sinh cosh ,
2 2
x+y xy
sinh x sinh y = 2 cosh sinh ,
2 2
x+y xy
cosh x + cosh y = 2 cosh cosh ,
2 2
x+y xy
cosh x cosh y = 2 sinh sinh .
2 2

2.6 Cng thc bin i tch thnh tng

1
sinh x. cosh y = [sinh ( x + y) + sinh ( x y)] ,
2
1
sinh x. sinh y = [cosh ( x + y) cosh ( x y)] ,
2
1
cosh x. cosh y = [cosh ( x + y) + cosh ( x y)] .
2

27
Hi tho khoa hc, Hng Yn 25-26/02/2017

2.7 o hm
Vic tnh o hm ca mt hm hyperbolic hon ton d dng, vy nn ta c

d
sinh x = cosh x,
dx
d
cosh x = sinh x,
dx
d
tanh x = sec h2 x,
dx
d
coth x = csch2 x.
dx

2.8 Cc bi ton
Bi ton 1. Ta c cc ng thc sau y

a b c
a) cosh a + cosh b + cosh c = 3 + 2 sinh2 + 2 sinh2 + 2 sinh2 , (2.1)
2 2 2
 a a b b c c
b) sinh a + sinh b + sinh c = 2 sinh cosh + sinh cosh + sinh cosh , (2.2)
2 2 2 2 2 2
 a b
c) cosh a + cosh b = 2 1 + sinh2 + sinh2 . (2.3)
2 2
Li gii. Cc ng thc (2.1)-(2.3) dng c chng minh bng cch s dng cc
cng thc nhn i.

Bi ton 2. Vi ba s thc a, b, c bt k ta u c
 
a b c 3 a+b+c
sinh2 + sinh2 + sinh2 cosh 1 . (2.4)
2 2 2 2 3

Li gii. Theo ng thc (2.1) v s dng tnh li ca cosh x, ta c


 a b c
2 sinh2 + sinh2 + sinh2 = cosh a + cosh b + cosh c 3
2 2 2
1 1 1   a+b+c
= 3 cosh a + cosh b + cosh c 1 3 cosh 1).
3 3 3 3
T suy ra iu phi chng minh.

Bi ton 3. Vi mi tam gic ABC vi di cc cnh l a, b, c ta u c

c a b
cosh < cosh2 + cosh2 . (2.5)
2 2 2
Li gii. T tnh li ca hm cosh v t bt ng thc tam gic ta c
cosh a + cosh b a+b c
cosh > cosh .
2 2 2
T suy ra
 
c 2a 2b
2 cosh < cosh a + cosh b < 2 cosh + cosh
2 2 2

28
Hi tho khoa hc, Hng Yn 25-26/02/2017

v ta c iu phi chng minh.


Bi ton 4. Ta c bt ng thc
sinh x cosh x 2
< + , x > 0. (2.6)
x 3 3
Li gii. Bt ng thc (2.6) tng ng vi
f ( x ) := x cosh x + 2x 3 sinh x > 0, x > 0.
Tht vy, ta c
f 0 ( x ) = cosh x + x sinh x + 2 3 cosh x, f 0 (0) = 0,
f 00 ( x ) = sinh x + sinh x + x cosh x 3 sinh x = x cosh x sinh x,
f 00 (0) = 0,
f 000 ( x ) = cosh x + x sinh x cosh x = x sinh x > 0, x > 0.

Nh vy, f 00 ( x ), f 0 ( x ) v f ( x ) l nhng hm ng bin trm [0, +). T ta c iu


phi chng minh.
Bi ton 5. Vi cc s dng a, b ta c bt ng thc
 
sinh a sinh b 2 2 2a 2b
+ < + cosh + cosh . (2.7)
a b 3 3 2 2
Li gii. S dng bt ng thc trong cc bi ton trn, ta c
sinh a 2 1
< + cosh a, (2.8)
a 3 3
sinh b 2 1
< + cosh b. (2.9)
b 3 3
Dng cc bt ng thc (2.8) v (2.9), ta c
 
sinh a sinh b 4 1 2 2 a b
+ < + (cosh a + cosh b) + cosh2 + cosh2 .
a b 3 3 3 3 2 2
Ta c iu phi chng minh.
Bi ton 6. Ta c bt ng thc sau
 
sinh a sinh b sinh c 2 2a 2b 2c
+ + < 5 + sinh + sinh + sinh . (2.10)
a b c 3 2 2 2
Li gii. S dng ba ln bt ng thc(2.6) ta c
sinh a sinh b sinh c
+ +
a  b  c  (2.11)
6 1 2a 2b 2c
< + 4 + 2 sinh + sinh + sinh .
3 3 2 2 2
Do vy ta c
 
sinh a sinh b sinh c 2 2a 2b 2c
+ + < 5 + sinh + sinh + sinh .
a b c 3 2 2 2

29
Hi tho khoa hc, Hng Yn 25-26/02/2017

Bi ton 7. Ta c bt ng thc ng sau

a+b+c
6 sinh sinh a + sinh b + sinh c. (2.12)
6

Li gii. Chng ta s dng cc bt ng thc sau


x x x
sinh x = 2 sinh cosh 2 sinh , (2.13)
2 2 2
Suy ra
 
a b c
2 sinh + sinh + sinh sinh a + sinh b + sinh c. (2.14)
2 2 2

Mt khc s dng tnh li ca hm s sinh ta c

a b c a b c
a+b+c + + sinh + sinh + sinh
sinh = sinh 2 2 2 2 2 2. (2.15)
6 3 3
T (2.14) v (2.15) ta c iu phi chng minh.

Bi ton 8. Trong bt k tam gic hyperbolic no ta cng c

a+b+c
sinh
 6 
2 a b c a 2a b 2b c 2c
< sinh + sinh + sinh + sinh sinh + sinh sinh + sinh sinh .
3 2 2 2 2 4 2 4 2 4
(2.16)

Li gii. T (2.12) ta c

a+b+c
6 sinh
6

sinh
 a + sinh b + sinh c 
a a b b c c
2 sinh cosh + sinh cosh + sinh cosh
2 2 2 2 2 2
= .
6
Mt khc
a a a
cosh = 2sinh2 + 1 < 2 + 2sinh2 .
2 4 4
T ta c iu phi chng minh.

Bi ton 9. Trong bt k tam gic hyperbolic no ta cng c bt ng thc sau


 
a 2 a b c a 2a b 2b c 2c
sinh < sinh + sinh + sinh + sinh sinh + sinh sinh + sinh sinh .
3 3 2 2 2 2 4 2 4 2 4
(2.17)

30
Hi tho khoa hc, Hng Yn 25-26/02/2017

Li gii. S dng bt ng thc tam gic b + c > a, ta c a + b + c > 2a v do vy


a+b+c a a+b+c a
> . S dng tnh tng ca hm sinh ta c sinh > sinh .
6 3 6 3
Do bt ng thc (2.16) ta c iu phi chng minh.

Bi ton 10. Cho x, y, z (0; 1), ta c bt ng thc sau

1 1 1 1 1 1
x+y + y+z + z + x cosh x + cosh y + cosh z . (2.18)
cosh cosh cosh
2 2 2
1
Li gii. Xt hm li f : (0; 1) R, f ( x ) = . S dng bt ng thc Jensen cho
cosh x
f ( x ), ta c

1 1
+
1 cosh x cosh y
x+y 2
,
cosh
2
1 1
+
1 cosh y cosh z
y+z 2
,
cosh
2
1 1
1 +
cosh z cosh x .
z+x 2
cosh
2

Cng v theo v cc cng thc trn ta c bt ng thc (2.18).

3 Cc bt ng thc dng Jordan


Mc ny trnh by cc bt ng thc dng Jordan (c ngun gc l i vi cc hm
lng gic) i vi cc hm hyperbolic.
Cc bt ng thc c thit lp da trn hai phng php, l: phng php o
hm v phng php s dng bt ng thc gia cc i lng trung bnh. Phng
php cui cng cha c ph bin trong cc ti liu bng ting Vit.

3.1 Phng php o hm


Bi ton 11. Vi x (0; +), ta c

sin x
< cosh x. (3.1)
x

Li gii. Bt ng thc (3.1) ng nu f ( x ) = x cosh x sin x > 0 trn (0; +). Ta c

f 0 ( x ) = cosh x + x sinh x cos x, (3.2)


00
f ( x ) = sin x + 2 sinh x + x cosh x. (3.3)

31
Hi tho khoa hc, Hng Yn 25-26/02/2017

Ta c f 00 ( x ) > 0 vi x (0; +) v f 0 ( x ) tng trn (0; +).


Do

f 0 ( x ) = cosh x cos x + x sinh x > f 0 (0) = 0. (3.4)

v hm s f ( x ) tng (0; +). Do f ( x ) > f (0) = 0, x (0; +) .


 
Bi ton 12. Vi x 0; , ta c
2
sin x
< cosh x. (3.5)
x

gii. Bt ng thc (3.5) ng nu hm s f ( x ) = x cosh x sin x dng vi
Li 

b 0; . T
2
sinh x x
f 00 ( x ) = (cosh x x sinh x ) + cosh x + sin x. (3.6)
cohsx 2

Ta c    
f 00 ( x ) > 0 vi x 0; v f 0 ( x ) tng trn 0; . Do ta c
2 2
1  
f 0 (x) = 2 cosh x + x sinh x 2 cos x cosh x > f 0 (0) = 0, (3.7)
2 cosh x
   
v hm s f ( x ) tng trn 0; . Do vy f ( x ) > f (0) = 0 vi x 0; .
2 2
 
Bi ton 13. Vi x 0; ta c
4
cos x
cosh x < p . (3.8)
cos2 x sin2 x

Li gii. Bin trn ca cosh x ng nu hm s f ( x ) = cos2 x cosh2 x cos2 x sin2 x



 
dng trn 0; . T
4
f 00 ( x ) = 4 sin (2x ) sinh (2x ) > 0 (3.9)

Ta c

f 0 ( x ) = sin (2x ) sinh (2x ) cos (2x ) cosh (2x ) > f 0 (0) = 0 (3.10)

Do f ( x ) > f (0) = 0 v suy ra iu phi chng minh.

Bi ton 14. Vi x, k (0; +) ta c

sin x sinh kx
< . (3.11)
x kx

32
Hi tho khoa hc, Hng Yn 25-26/02/2017

Li gii. Bt ng thc (3.11) ng nu hm s f ( x ) = sinh kx k sin x dng trn


(0; +). T

f 0 ( x ) = k (cosh kx cos x ) . (3.12)

Ta c f 0 ( x ) > 0 vi x (0; +), do hm s f ( x ) tng trn (0; +).


Do vy f ( x ) > f (0) = 0 vi x (0; +) .

Bi ton 15.
tanh( x ) tanh(kx )
6 (3.13)
x kx
vi mi k (0; 1) v x (0; 1).
tanh( x )
Li gii. Tht vy, hm s f 1 ( x ) = c o hm bc nht
x
x sinh x cosh x
f 10 ( x ) = <0
x2 cosh2 x
do sinh x > x v cosh x > 1 vi x > 0. Nh vy, hm s f 1 ( x ) l nghch bin thc s,
dn n f 1 ( x ) < f 1 (kx ) do x > kx vi mi x > 0, k (0; 1). Ta lm mnh (ci tin)
(3.13) v mt iu kin (t x (0; 1) tr thnh x > 0). Hn th na, cn ch ra (3.13) l
bt ng thc thc s.

Bi ton 16. Vi x > 0 ta c bt ng thc

sinh x sinh (kx )


. (3.14)
x kx
sinh x
Li gii. Yu cu ca bi ton tng ng vi hm s f ( x ) = tng vi x > 0.
x
cosh x sinh x
T f 0 ( x ) = 0 v f 0 ( x ) 0 tng ng vi tanh x x, ta c iu
x x2
phi chng minh.

Bi ton 17.
sinh x 3 2 + cosh x  
< < vi mi x 0; . (3.15)
x 2 + cos x 3 2

Li gii. Bt ng thc th nht ca (3.15) c th vit li di dng


h 
f 3 ( x ) = 3x 2 sinh x sinh x. cos x > 0 trn 0; . (3.16)
2
sinh x
D dng c c f 300 ( x ) = 2 (cosh x sin x sinh x ) . Theo (3.27), ta c <
  sin x
cosh x, x 0; . T suy ra f 300 ( x ) > 0, f 30 ( x ) > f 30 (0) = 0, dn n f 3 ( x ) >
2  
f 3 (0) = 0 vi mi x 0; . Bt ng thc th hai ca (3.15) tng ng vi
2
f 4 ( x ) = 2 cos x + 2 cosh x + (cos x )(cosh x ) 5 > 0. (3.17)

33
Hi tho khoa hc, Hng Yn 25-26/02/2017

Ln lt ly o hm cc cp ta c

1 (4)
f ( x ) = cos x + cosh x 2(cos x )(cosh x. (3.18)
2 4
t f 5 ( x ) = (cos x )(cosh x ) 1. Ta c

f 50 ( x ) = (sin x )(cosh x ) + (cos x )(sinh x ),


 
f 500 ( x ) = 2(sin x )(sinh x ) < 0 vi mi x 0; .
2
T ta c f 50 ( x ) < f 50 (0) = 0, suy ra f 5 ( x ) 6 0, c ngha l
 
(cos x )(cosh x ) < 1 vi mi x 0; . (3.19)
2
Tip theo, t (3.18) v (3.19)ta c

1 (4)
f ( x ) > cos x + cosh x 2 := f 6 ( x ).
2 4
Bi f 60 ( x ) = sin x + sinh x > 0 do sinh x > x > sin x, ta c f 6 ( x ) > f 6 (0) = 0.
(4)
Cui cng, f 4 ( x ) > 0 vi x > 0, ko theo
(3) (2)
f 4 ( x ) > 0 ; f 4 ( x ) > 0 ; f 40 ( x ) > 0.

Nh vy f 4 ( x ) > 0 v nh l c chng minh. T (3.11) v (3.16) ta c th vit

sin x 3 x
< < . (3.20)
x 2 + cos x sin x

v ta thu c mt ci tin ca pha phi (3.15).

Bi ton 18.  x 3 1 tanh x sin x


< < < . (3.21)
sinh x cosh x x x
 
vi mi x 0; .
2
Li gii. Bt ng thc th nht ca (3.21) chnh l pha tri ca (3.25). Bt ng thc
sinh x
th hai c c do > 1, cn bt ng thc th ba tng ng vi sin x > tanh x,
x
chnh l bt ng thc (3.27).

Bi ton 19.
1 x 1 1
< < s <
4
(3.22)
cosh x sinh x cosh x + 1 2
 cosh x
3

2
 
vi mi x 0; .
2

34
Hi tho khoa hc, Hng Yn 25-26/02/2017

Li gii. Bt ng thc gia ca (3.22) chnh l bt ng thc (3.23). chng minh


pha tri ca (3.22) chng ta xt hm s
sinh x
f7 (x) = x.
cosh x
Bi
2 cosh x sinh2 x 2 cosh x
f 70 ( x ) =
2 cosh x
v nhn xt rng

sinh2 x 2 cosh x + 2 cosh x = cosh2 x 1 2 cosh x + 2 cosh x
 
= (cosh x 1)2 + 2 cosh x 1 > 0

(do cosh x > 1). Vy f 70 ( x ) < 0, ko theo f 7 ( x ) < f 7 (0) = 0.  


Nh vy, pha tri ca bt ng thc (3.22) thc s l ng vi mi x 0; .
 8 2
t+1
t t = cosh x, bt ng thc cn li tr thnh > t3 . Do x > 0 nn t > 1, ta
2
c
t+1 8
 
t+1
> t > t4 > t3 .
2 2

3.2 Phng php s dng cc i lng trung bnh


Bi ton 20. Chng minh bt ng thc
sinh x
1< < cosh x, x 6= 0.
x
Li gii. Ta c
sinh x
A(e x , e x ) = cosh x, G (e x , e x ) = 1, L(e x , e x ) = .
x
Vi x 6= 0, e x 6= e x . p dng bt ng thc G < L < A, ta c iu phi chng minh.
Bi ton 21. Chng minh bt ng thc
s
cosh x + 1 2
 
3 sinh x
< , x 6= 0. (3.23)
2 x
Li gii. Ta c
sinh x
A(e x , e x ) = cosh x, G (e x , e x ) = 1, L(e x , e x ) = .
x
p dng bt ng thc [2] s
 2
3 A+G
G L
2
ta c bt ng thc (3.23).

35
Hi tho khoa hc, Hng Yn 25-26/02/2017

Bi ton 22. Chng minh bt ng thc

sinh x cosh x + 3 cosh x/3


< , x 6= 0. (3.24)
x 4
Li gii. Ta c [2]

sinh x cosh x + 3 cosh x/3


L(e x , e x ) = , A1/3 (e x , e x ) = .
x 4
S dng bt ng thc [2]
L A1/3
ta c bt ng thc (3.24).

Bi ton 23. Chng minh bt ng thc



3 sinh x cosh x + 2
cosh x < < , x 6= 0. (3.25)
x 3
Li gii. Ta c

sinh x
A(e x , e x ) = cosh x, G (e x , e x ) = 1, L(e x , e x ) = . (3.26)
x
S dng bt ng thc [2]

3 A + 2G
AG2 L
3
v (3.26) d dng suy ra bt ng thc (3.25).

Bi ton 24. Chng minh bt ng thc

tanh x < sin x, x (0, /2). (3.27)

Chng minh. Ta c

sinh x sinh x
L(e x , e x ) = , P(e x , e x ) = .
x arcsin(tanhx )

S dng bt ng thc L P, ta c arcsin(tanhx ) < x, hay tanh x < sin x, x (0, /2).
Ta c iu phi chng minh.

Bi ton 25. Chng minh bt ng thc

sinh x 1 x cosh x sinh x


ln > , x>0 (3.28)
x 2 sinh x
Chng minh. chng minh bt ng thc (3.28) c th s dng bt ng thc ca cc
i lng trung bnh
L2 > GI. (3.29)
Ta c
sinh x
L(e x , e x ) = , G (e x , e x ), I (e x , e x ) = e x coth x1 . (3.30)
x
S dng (3.29) v (3.30), ta c (3.28).

36
Hi tho khoa hc, Hng Yn 25-26/02/2017

4 Bi tp ngh
4.1 Phng php hm li
Bi ton 26. Cho x, y, z (0; 1), chng minh bt ng thc sau

1 1 1 x 2 + y2 + z2
x+y + y+z + z+x < 3 . (4.1)
cosh cosh cosh 3
2 2 2

Bi ton 27. Cho x, y, z (0; 1), chng minh bt ng thc sau

1 1 1 1 1 1
x+y + y+z + z + x sinh x + sinh y + sinh z . (4.2)
sinh sinh sinh
2 2 2

Bi ton 28. Cho x, y, z (0; 1), chng minh bt ng thc

6
x+y+z
sinh x + sinh y + sinh z + sinh
3 (4.3)
1 1 1
+ + .
sinh x sinh y sinh z

4.2 Phng php o hm


Bi ton 29. Chng minh bt ng thc

sinh x x cosh x + 2
< cosh3 < vi mi x > 0.
x 3 3
 
Bi ton 30. Vi x 0; chng minh bt ng thc
2
1 sin x x
< < .
cosh x x sinh x
 
Bi ton 31. Vi x 0; chng minh
2
x2 sin x x
2
< < .
sinh x x sinh x

4.3 Phng php cc i lng trung bnh


Bi ton 32. Chng minh bt ng thc

sinh x
1< < e x coth x1 < cosh x, x > 0.
x

[S dng bt ng thc: 1 < L < I < A, a = e x , b = e x .]

37
Hi tho khoa hc, Hng Yn 25-26/02/2017

Bi ton 33. Chng minh bt ng thc

x coth x 1
sinh x x cosh x + 3 cosh x/3
e 2 < < , x > 0.
x 4

Chng minh bt ng thc [S dng bt ng thc: GI < I < A1/3 , a = ex , b =
e x .]

Bi ton 34. Chng minh bt ng thc

x coth x 1
sinh x
cosh x < e 2 < , x > 0.
x

[S dng bt ng thc: G2 A < GI < L < L( A, G ), a = e x , b = e x .]
3

Bi ton 35. Chng minh bt ng thc

sinh x cosh x + 2
< < e x coth x1 , x > 0.
x 3
A + 2G
a = e x , b = e x .]
3
[S dng bt ng thc: GA2 < IG < L < < I,
3

Ti liu
[1] Neumann Edward (2012), Inequalities Involving Hyperbolic Functions and Trigonometric
Functions, Bulletin of International Mathemqtics Institute ISSN 1840-4367, Volume 2
(2012), 87-92.
[2] Sandor J. et.al. (2011), Trigonometric and Hyperbolic Inequalities,
arxiv.org/pdf/,1105.0859 v1 [math.CA].
[3] Zhen Hang Yang (2015), Jordan Types Inequalities for Hyperbolic Funcions and Their Ap-
plicatipns, Hindaway Publishing Corporation, Journal of Function Spaces Volume
2015, Article ID 370997 (http://dx.doi.org./10.1155/2015 370 979).

38
Hi tho khoa hc, Hng Yn 25-26/02/2017

T GIC IU HA
Nguyn B ang
Hi ton hc H Ni

Tm tt ni dung

T gic iu ha a vo ni dung ging dy cc trng Trung hc ph thng chuyn.


gip cc thy c hiu r thm v t gic c bit ny xin gii thiu: khi nim, tnh
cht v cc bi ton p dng.

1 nh ngha
AB CB
nh ngha 1. T gic ABCD ni tip ng trn, ng thi tha mn iu kin =
AD CD
c gi l t gic iu ha.

V d 1. Cho ng trn (O), v im M. T M k tip tuyn MA, MB v ct tuyn


MCD ( MC < MD ) vi ng trn (O). Chng minh t gic ADBC l t gic iu ha.

Li gii. MA, MB l tip tuyn vi ng trn (O),

ADC = MAC MAC v MDA ng dng (g,g), tng t MBC, MDB ng


dng, MA = MB, suy ra t gic ADBC l t gic iu ha.

Nhn xt 1. T gic ABCD l t gic iu ha khi tip tuyn ti A v C v ng thng


BD ng quy, hoc tip tuyn ti A v C cng song vi ng thng BD. y l cch
dng t gic iu ha n gin mi khi ta gp bi ton xut hin t gic iu ha.

39
Hi tho khoa hc, Hng Yn 25-26/02/2017

2 Tnh cht
Tnh cht 1. T gic ABCD l t gic iu ha th AC.BD = 2AB CD = 2BC AD.

Chng minh. ABCD l t gic iu ha Mt khc ABCD l t gic ni tip theo nh l


Ptolemy AC BD = 2AB CD = 2BC AD.
Tnh cht 2. T gic ABCD l t gic iu ha ni tip ng trn (O), tip tuyn ti A
MD ED
v C ct nhau ti M, AC ct BD ti E th = .
MB EB
MD S CD sin DCM ED S AD sin DAE
Chng minh. Ta c = MCD = , = AED = ,
MB S MCB CB sin BCM EB S ABE AB sin BAE

ABCD ni tip ng trn (O) v MA, MC l tip tuyn nn sin BCM = sin BAC =
AD CD
sin BAE, sin DCM = sin DAE, ABCD l t gic iu ha. Suy ra = v
AB CB
MD ED
= .
MB EB
MD ED
t ( MEDB) = = .
MB EB
Tnh cht 3. T gic iu ha ABCD, ng phn gic cc gc BAD, BCD v ng
thng BD ng quy.

Chng minh.
Gi s ng phn gic gc BAD ct BD ti E theo tnh cht ng phn gic, suy
EB AB AB CB CB BE
ra = . ABCD l t gic iu ha nn = v = . Suy ra CE l
ED AD AD CD CD ED
phn gic ca gc BCD, (pcm).
Tnh cht 4. ABCD l t gic iu ha, M l giao im tip tuyn ti A v C, MO ct AC
ti I th AI l phn gic gc BID.

EB MB
Chng minh. AC, BD ct nhau ti E nn = suy ra I(MEBD) chm iu ha,
ED MD
I M AC, IA l phn gic gc BIC.
Tnh cht 5. Cho t gic ABCD, tip tuyn ti A v C ct nhau ti im M thuc BD th
tip tuyn ti B v D gp nhau trn AC.

40
Hi tho khoa hc, Hng Yn 25-26/02/2017

Chng minh. Vi gi thit cho t gic ABCD l t gic iu ha (pcm).

EB AB2
Tnh cht 6. ABCD l t gic iu ha, AC, BD ct nhau ti E th = .
ED AD2
Tnh cht 7. ABCD l t gic iu ha, M l trung im AC th ADB = MDC.

Chng minh. Hai tnh cht ny cho thy ABCD l t gic iu ha, AD l ng i
trung ca tam gic ABC, do ta c ngay pcm.
y ch l nhng tnh cht c bn v T gic iu ha.

41
Hi tho khoa hc, Hng Yn 25-26/02/2017

3 V d p dng
Bi ton 1. Cho ng trn (O) v dy AD, gi I l im i xng ca A qua D, k tip
tuyn IB vi ng trn (O), t A dng tip tuyn vi (O) ct IB ti C, CD ct ng (O)
ti E. Chng minh rng EB song song vi AD.

Li gii.

Vi gi thit suy ra CA, CB l tip tuyn t gic AEBD l t gic iu ha, suy ra
DA EA DI EA
= , mt khc AD = DI nn = , suy ra ADBE ni tip, nn BDI =
DB EB DB EB
BEA, suy ra BDI, BEA ng dng (c.g.c), do CBE = EAB = BID. Vy nn
BE k AD.
Bi ton 2. Cho tam gic ABC, M l AM, BM, CM ct BC, CA, AB ln lt ti D, E, F.
N l im BC sao AM vung gc vi MN, Gi P v Q l i xng ca M qua DE v DF.
Chng minh rng P, Q, N thng hng

Li gii. Theo gi thit P v Q i xng vi M qua DE v DF nn DP = DM = DQ, suy

ra Q, M, P nm trn ng trn tm D bn knh DM. Mt khc, AM MN nn NM l


tip tuyn ca ng trn tm (D).
Gi K l im i xng ca M qua BC th K thuc ng trn (D), suy ra NK l tip
tuyn ca (D) v MQKP l t gic iu ha N, P,Q thng hng.

42
Hi tho khoa hc, Hng Yn 25-26/02/2017

Bi ton 3. Cho t gic ABCD ni tip ng trn (O), AB v CD ct nhau ti P, AD ct


BC ti Q, AC ct BD ctnhau ti M. Chng minh rng O l trc tm tam gic MPQ (nh
l Brocard).
im Brocard c cng b bi Henri Brocard, mt s quan qun i Php, nm 1825.

Li gii. T P k tip tuyn PE, PF vi ng trn (O), gi I, K giao im ca IF vi AB,

CD, suy ra t gic BEAF l t gic iu ha nn (PIAB) = (PKDC) theo gi thit BC, AD,
EF ng quy. Vy Q thuc EF.
Mt khc cng t (PIAB) = (PKDC), suy ra AC, BD, IK ng quy. Vy M thuc EF do
PO EF nn PO MQ.
Chng minh tng t QO MP, suy ra O l trc tm MPQ.
Bi ton 4 (Moldova MO 2014). Cho t gic ABCD ni tip ng trn, ng phn
gic gc gp nhau trn cnh BD. ng thng qua C song song vi AD ct ng thng
i qua A v trung im BD ti P. Chng minh rng tam gic PCD l tam gic cn.

Li gii. Theo gi thit ng phn gic cc gc ct nhau ti E thuc BD, theo tnh cht

AB BE CB
ca ng phn gic, suy ra = = nn t gic ABCD l t gic iu ha.
AD DE CD
Theo nh ngha t gic iu ha, suy ra tip tuyn ti A v C ct nhau trn BD. Gi
M l trung im BD v T giao im ca hai tip tuyn l T. T suy ra AC l ng
i trung ca tam gic ABD. Suy ra CDT = CDB = CAB = MAD, CP k AD

43
Hi tho khoa hc, Hng Yn 25-26/02/2017

nn DAP = APC, PCD = ADC. TC l tip tuyn nn ACT = ADC (cng


CT CD
chn cung AC). Suy ra DCT v PCA ng dng (g.g) nn = suy ra TAC v
CA CP
DCP ng dng (c.g.c), suy ra TAC l tam gic cn nn DCP l tam gic cn.
Bi ton 5. Cho tam gic ABC ni tip ng trn (O), A c nh B v C thay i song
vi ng thng cho trc. Tip tuyn ti B v C vi ng trn (O) ct nhau ti D. Gi
M l trung im BC, ng thng AM ct ng trn (O) ti N. Chng minh rng DN
lun i qua im c nh.

Li gii. Tip tuyn ti B, C vi ng trn (O) nn AD l ng i trung ca ABC,

MB = MC nn BAD = CAM. ng thng DN ct ng trn (O) ti I nn AN


l ng i trung ca BIC, suy ra N IC = MIB, nhng N IC = CAN nn
N IC = DAB, gi AD ct BC ng trn (O) ti E, EN k BC, N I k BC, BC song vi
ng thng cho trc, A c nh, suy ra I c nh.
Ngoi ra, ta cn cch dng tnh cht: T gic ABEC, ANCI l t gic iu ha t
suy ra I c nh.
Bi ton 6 (STS VN 2001). Cho ng trn (O1 ) v (O2 ) ct nhau ti A v B mt tip
tuyn chung PQ (P thuc O1 v Q thuc O2 ). Tip tuyn ti P v Q vi ng trn ngoi
tip tam gic APQ ct nhau ti E, gi D l im i xng vi B qua PQ. Chng minh
rng A, D, E thng hng.

Li gii.
PQ l tip tuyn ca ng trn (O1 ) v (O2 ) nn PAB = BPQ, QAB = BQP,
suy ra BPQ + BQP = PAQ. Theo gi thit D i xng vi B qua PQ nn PDQ =
PBQ = 1800 BPQ BQP == 1800 PAQ, suy ra t gic APDQ ni tip. AB ct
BP MB
PQ ti M nn MP = MQ v PBM v APM ng dang (g.g), suy ra = . Tng
AP MP
BQ MB BP AP DP AP
t, = suy ra = , theo gi thit PB = PD, BQ = DQ vn = , suy
AQ MQ BQ AQ DQ AQ
ra t gic APDQ l t gic iu ha v E, D, A thng hng.
Bi ton 7. Cho tam gic ABC, ng trn ni tip tam gic tip xc vi cnh BC, CA,
AB ln lt ti D, E, F, ng thng AD ct ng trn (I) ti im th hai l M, BM,
CM ct ng trn (I) ti P v Q. Chng minh rng AD, BQ, CP ng quy.

44
Hi tho khoa hc, Hng Yn 25-26/02/2017

Li gii. BC, CC tip xc vi ng trn (I) ti D, E nn t gic MEQD l t gic iu ha.

Suy ra MEQD ni tip nn MQ DE = ME DQ + MD EQ, MQ DE = 2ME DQ,


MQ 2ME DQ
suy ra = .
QC DE QC
BP BP DF
Tng t MFPD l t gic iu ha nn = suy ra
PM 2MF PD
DC MQ BP DC 2ME DQ BP DF
= . (1)
DB QC PM DB DE QC 2MF PD

T gic MEDF cng l t gic iu ha nn

ME DE
= . (2)
MF DF
CQ CD
CDQ v CMD ng dng (g.g), nn = , tng t
DQ DM
BP DP
= . (3)
BD DM
DC MQ BP DC DQ BP DC MQ BP
T (1), (2) suy ra = . kt hp (3), ta c = 1.
DB QC PM DB DP CQ DB QC PM
Theo nh l Ceva th MD, BQ, CP ng quy.

45
Hi tho khoa hc, Hng Yn 25-26/02/2017

Bi ton 8. Cho tam gic nhn ABC ni tip ng trn (O), ng phn gic gc B, C
ct ng trn (O) ti M v N. G l im trn cung nh BC, I1 , I2 l tm ng trn ni
tip tam gic ABG v ACG, gi P l giao im th hai ca ng trn (O) vi ng trn
ngoi tip tam gic GI1 I2 , D l trung im MN. Chng minh rng P, I, D thng hng.

Bi ton 9. Cho t gic ABCD ngoi tip ng trn (I), gi M, N, P, Q l tip im ca


(I) vi cnh AB, BC, CD, DA. Chng minh rng AC, BD, MP, NQ ng quy.

Li gii.

V AC, BD i qua I nn AC, BD, MP, NQ ng quy. AC, BD khng qua I: Gi K, E l


giao im AC vi (I), KE khng qua I, tip tuyn ti K v E ct nhau ti S nn MKQE l
t gic iu ha, suy ra MQ, NP i qua S.
GM SM JN SN
Gi G, J l giao im MS, NP vi AC, suy ra = . = nn GE, MP, NQ
GQ SQ JP SP
ng quy. Suy ra AC, MP, NQ ng quy, tng t BD, MP, NQ ng quy nn AC, BD,
MP, NQ ng quy.

Bi ton 10. Cho t gic ABCD ngoi tip ng trn (I), gi M, N l tip im ca (I)
vi cnh AD, BC, ng thng BM, DN ct (I) ti E v F. Chng minh rng EF, MN, BD
ng quy.

Li gii.

46
Hi tho khoa hc, Hng Yn 25-26/02/2017

Gi P, Q l tip im ca (I) vi cnh AB, CD Theo v d trn th AC, BD, MN, PQ ng


MP MN QE MF
quy. Mt khc cc t gic MPEN, NQFM l t gic iu ha, = , =
EP EN QN MN
MP.EN.QF MP QF EN MN MF EN
nn = = = 1.
PE.NQ.FM EP QN FM EN MN FM
Do MN, PQ, EF ng quy EF, MN, BD ng quy.

Bi ton 11 (VN TST 2004). Cho hai ng trn (O1 ) v (O2 ) ct nhau ti A v B.Cc
tip tuyn ti A, B ca ng trn (O1 ) ct nhau ti K.Gi s M l im trn ng trn
(O1 ) nhng khng trng vi A, B, ng thng AM ct ng trn (O2 ) ti P, ng
thng KM ct ng trn (O1 ) ti C v ng thng AC ct ng trn (O2 ) ti Q.
Chng minh trung im PQ thuc MC v ng thng PQ lun i qua im c nh
khi M chuyn ng trn ng trn (O1 ).

Li gii. Theo gi thit KA, KB l tip tuyn vi ng trn (O1 ) ct nhau ti K, KM ct

(O1 ) ti C nn t gic ACBM l t gic iu ha, suy ra


AM BM
= . (1)
AC BC
BCQ, BMP ng dng nn
BM MP
= . (2)
BC CQ
Gi I l giao im MK vp PQ, p dng nh l Menelaus APQ, ct tuyn M, K, I,
kt hp (1), (2), ta c IP = IQ . T gic BQJP l t gic iu ha, suy ra tip tuyn ti B,
J vi ng trn (O2 ) c nh J c nh.

Bi ton 12 (IMO 2003). Gi s ABCD l t gic ni tip. Gi P,Q,R l chn cc ng


vung gc h t D ln cc ng thng BC, CA, AB. Chng minh rng PQ = QR khi v
ch khi phn gic cc gc ABC, ADC ct nhau trn AC.

Bi ton 13. Cho tam gic ABC (AB < AC) ni tip ng trn (O), tip tuyn ti B, C
ct nhau ti D. Qua A k ng thng vung gc vi AD ct BC ti E,trn DE ly P v Q
sao cho DP = DQ = DB. Chng minh tam gic ABC v tam gic APQ ng dng.

Li gii.

47
Hi tho khoa hc, Hng Yn 25-26/02/2017

Theo gi thit CBD = CAB, DBA = CBD + ABC = CAB + ABC ==


1800 BCA, gi M l giao im BC v OD th AD l ng i trung, suy ra BAD =
DB sin BAD
CAM. p dng nh l hm s sin cho BAD, CAM, ta c = =
DA sin DBA
sin CAM MC
= . DP = DQ nn DMS = DAE = 900 . Suy ra D, M, A, D nm trn
sin BCA MA
BC MC AC
ng trn AMC = ADQ nn MAC v DAQ ng dng, nn = = ,
PQ DQ AQ
ACB = AQP nn ABC v APQ ng dng.
Bi ton 14 (Japan). Cho t gic ABCD ni tip ng trn (O), ng thi tha mn
AB CD
= . Gi P l giao im ca AD v BC, Q l giao im AB v CD, gi E, F, H, G
AD CB
ln lt l trung im AB, BC, CD, DA. ng phn gic ct EG ti S, ng phn gic
gc ct FH ti T. Chng minh rng ST song song vi BD.

Ti liu
[1] Nguyn Vn Mu (Ch bin, 1997), Hnh hc v mt s vn lin quan, NXB Gio
dc.
[2] Nguyn Minh H, Nguyn Xun Bnh (2005), Bi tp nng cao v mt s chuyn
hnh hc 10, NXB Gio dc.
[3] Yufei Zhao, IMO training 2008, Circles.
[4] Kin Y.Ly, Pole and polar,Mathematical Excalibur.
[5] Ti liu t internet,...

48
Hi tho khoa hc, Hng Yn 25-26/02/2017

XUNG QUANH MT BI TON THI IMO


V Tin Vit
Hc vin An ninh nhn dn

Vi tam gic ABC ta k hiu


- Cc gc: A, B, C
- Cc cnh: a = BC, b = CA, c = AB
- Cc ng cao: h a , hb , hc
- Cc trung tuyn: m a , mb , mc
- Cc phn gic: la , lb , lc
- Bn knh ng trn ni tip, ngoi tip: r, R
- Na chu vi: p = 12 ( a + b + c)
- Din tch: S
Nm 1961 k thi IMO ti Budapest - Hungary c bi ton s 2 sau1 :
Cho a, b, c l di cc cnh v S l din tch ca mt tam gic.

Chng minh bt ng thc a2 + b2 + c2 > 4 3S ().
Khi no th xy ra du (=)?
Chng minh. Cch 1. Ta c
( p a) + ( p b) + ( p c) p
q
3
( p a)( p b)( p c) 6 =
3 3
p3
( p a)( p b)( p c) 6
27
Suy ra

p4
S2 = p( p a)( p b)( p c) 6
27
p 2 ( a + b + c) 2 ( a + b + c2 ) + 2( ab + bc + ca)
2 2
S6 = =
3 3 12 3 12 3
a2 + b2 + c2
6
4 3
Dn n bt ng thc cn phi chng minh.
1 Bi ton ny do R. Weitzenbock
a ra nm 1919.

49
Hi tho khoa hc, Hng Yn 25-26/02/2017

D thy du (=) xy ra khi v ch khi tam gic u.


Theo chng minh trn ta cn thy

( a2 + b2 + c2 ) + 2( ab + bc + ca)
S6
12 3

12 3S 6 ( a2 + b2 + c2 ) + 2( ab + bc + ca)

12 3S + 2( a2 + b2 + c2 ) 2( ab + bc + ca) 6 3( a2 + b2 + c2 )

12 3S + ( a b)2 + (b c)2 + (c a)2 6 3( a2 + b2 + c2 )
2 2 2
1h 2 2 2
i
a + b + c > 4 3S + ( a b) + (b c) + (c a) (1)
3

Cch 2. Ta chng minh bt ng thc "mnh" hn

2 2 1h
2
2 2 2
i
a + b + c > 4 3S + ( a b) + (b c) + (c a) (2)
2
Tht vy, ta thy

2( a2 + b2 + c2 ) = 2( ab + bc + ca) + ( a b)2 + (b c)2 + (c a)2


 1 1 1 
= 4S + + + ( a b )2 + ( b c )2 + ( c a )2
sin A sin B sin C
1
D thy hm f ( x ) = sin x l hm li trn (0, ), nn ta c

f ( A) + f ( B) + f (C ) A+B+C 2
> f( ) = f( ) =
3 3 3 3

T ta c bt ng thc cn phi chng minh. D thy du (=) xy ra khi v ch khi


tam gic u.
Cch 3. Ta d dng thy
 1 1 1 
2 2 2
a + b + c > ab + bc + ca = 2S + +
sin A sin B sin C

n y theo cch 2 ta c bt ng thc cn phi chng minh. D thy du (=) xy ra


khi v ch khi tam gic u.

Theo chng minh ny ta cn c bt ng thc ab + bc + ca > 4 3S (3)

Nm 1938 P. Finsler v H. Hadwiger nu ra bt ng thc "mnh" hn cc bt ng


thc Weitzenbock
v (1), (2) nh sau: Vi mi tam gic ta c

a2 + b2 + c2 > 4 3S + ( a b)2 + (b c)2 + (c a)2 (4)

50
Hi tho khoa hc, Hng Yn 25-26/02/2017

Chng minh. Bt ng thc cn chng minh tng ng vi



[ a2 (b c)2 ] + [b2 (c a)2 ] + [c2 ( a b)2 ] > 4 3S
( a b + c)( a + b c) + (b c + a)(b + c a)+

+(c a + b)(c + a b) > 4 3S


( p a)( p b) + ( p b)( p c) + ( p c)( p a) > 3S
( p a)( p b) + ( p b)( p c) + ( p c)( p a)
q
> 3p( p a)( p b)( p c)

t x = p a > 0, y = p b > 0, z = p c > 0 th x + y + z = p v bt ng thc trn tr


thnh
q
xy + yz + zx > 3( x + y + z) xyz
x2 y2 + y2 z2 + z2 x2 + 2( xy2 z + yz2 x + zx2 y) > 3( x + y + z) xyz

x2 y2 + y2 z2 + z2 x2 ( xy2 z + yz2 x + zx2 y) > 0


1h i
( xy yz)2 + (yz zx )2 + (zx xy)2 > 0 , ng!
2
D thy du (=) xy ra khi v ch khi tam gic u.
Trc Weitzenbock,
nm 1897 I. Ionescu a ra bt ng thc sau:

Chng minh rng khng tn ti tam gic th mn a2 + b2 + c2 < 4 3S
V th ngi ta cn gi () l bt ng thc Ionescu-Weitzenbock.

Ta chng minh bt ng thc "tng qut" hn sau:


Cho cc s thc x, y, z tha mn x + y > 0, y + z > 0, z + x > 0.
Khi vi mi tam gic ta c

xa2 + yb2 + zc2 > 4S xy + yz + zx ()


p

Chng minh. Bt ng thc () c th vit thnh

xa2 + yb2 + z( a2 + b2 2ab cos C ) > 2ab sin C xy + yz + zx


p
 
2 2
p
( x + z) a + (y + z)b > 2ab z cos C + sin C xy + yz + zx
a b  p 
( x + z) + (y + z) > 2 z cos C + sin C xy + yz + zx
b a

51
Hi tho khoa hc, Hng Yn 25-26/02/2017

S dng bt ng thc Cauchy-Schwarz ta c

a b
q
( x + z) + (y + z) > 2 ( x + z)(y + z)
  q b a
2 z cos C + sin C xy + yz + zx 6 2 (z2 + xy + yz + zx )(cos2 C + sin2 C )
p
q q
2
= 2 (z + xy + yz + zx ) = 2 ( x + z)(y + z)

Vy ta c bt ng thc cn phi chng minh.

Ngc chiu vi bt ng thc Finsler-Hadwiger, ta chng minh bt ng thc sau:


Vi mi tam gic ta c

2 2 2
h
2 2 2
i
a + b + c 6 4 3S + 2 ( a b) + (b c) + (c a) (5)

Chng minh. chng minh, ta cn n b

B 1. Cho f : I R l hm li. Khi vi mi x, y, z I ta c

f ( x ) + f (y) + f (z) x+y+z 2h x + y y+z z+x i


+ f( )> f( ) + f( ) + f( )
3 3 3 2 2 2

Bt ng thc (5) c th vit thnh


h i
a2 + b2 + c2 6 4 3S + 2 2( a2 + b2 + c2 ) 2( ab + bc + ca)

4( ab + bc + ca) 2( a2 + b2 + c2 ) 6 4 3S + ( a2 + b2 + c2 )

Mt khc

a2 = b2 + c2 2bc cos A = (b c)2 + 2bc(1 cos A)


1 cos A A
= (b c)2 + 4S = (b c)2 + 4S tan
sin A 2
nn ta c
 A B C
a2 + b2 + c2 = ( a b)2 + (b c)2 + (c a)2 + 4S tan + tan + tan
2 2 2
 A B C 
2( ab + bc + ca) ( a2 + b2 + c2 ) = 4S tan + tan + tan
2 2 2
Do bt ng thc cn chng minh tr thnh
 A B C
8S tan + tan + tan 6 4 3S + ( a2 + b2 + c2 )
2 2 2
 A B C a2 + b2 + c2
2 tan + tan + tan 6 3+
2 2 2 4S

52
Hi tho khoa hc, Hng Yn 25-26/02/2017

a2 + b2 + c2
Ta d dng chng t c h thc cot A + cot B + cot C = , nn bt ng
4S
thc cn chng minh tr thnh
 A B C
2 tan + tan + tan 6 3 + cot A + cot B + cot C
2 2 2
Xt hm f : (0, 2 ) R vi f ( x ) = cot x, ta thy y l hm li, nn p dng b 1, ta
c
1 A+B+C A+B
(cot a + cot B + cot C ) + cot( ) > cot( )
3 3 cyc 2
 A B C
cot a + cot B + cot C + 3 > 2 tan + tan + tan
2 2 2

Ta s chng minh bt ng thc "mnh" hn bt ng thc (5) nh sau:


Vi mi tam gic ta c

2 2 2
6 6h i
a + b + c 6 4 3S + ( a b )2 + ( b c )2 + ( c a )2 (6)
2
Chng minh. chng minh, ta cn n b

B 2. Vi mi tam gic ta c a2 + b2 + c2 > 4(r + R )2


Bt ng thc (6) c th vit thnh
2 2 2
h 2 2 2
i
2( a + b + c ) 6 4 3S + (6 6) ( a b) + (b c) + (c a)

Ta d dng c cc h thc a2 + b2 + c2 = 2( p2 4rR r2 ), ab + bc + ca = p2 + 4rR + r2 nn


bt ng thc trn tr thnh

p2 (4 6) + 4 3S > rR(64 12 6) + r2 (16 3 6)

S dng bt ng thc Gerretsen p2 > 16rR 5r2 ta s chng minh rng



(16rR 5r2 )(4 6) + 4 3S > rR(64 12 6) + r2 (16 3 6)

Bt ng thc ny c vit thnh



4 3S > 4 6rR + (36 8 6)r2

3p2 > 6R2 + 2(9 6 12)rR + (9 2 6)r2

3p2 > 6R2 + 2(9 6 12)rR + (105 35 6)r2

Do b 2, ta cn chng minh bt ng thc



8R + 3r > 2(3 6 4) R + (35 12 6)r

(16 6 6)( R 2r ) > 0

53
Hi tho khoa hc, Hng Yn 25-26/02/2017

Bt ng thc sau cng l ng do bt ng thc Euler R > 2r.


Bi ton m: Tm s k "ti u" bt ng thc sau ng vi mi tam gic
h i
a2 + b2 + c2 6 4 3S + k ( a b)2 + (b c)2 + (c a)2 .

Ti liu
[1] Arthur Engel, Problem solving strategies. Springer Verlag, 1998.

[2] Cezar Lupu, Constantin Mateescu, Vlad Matei, Mihai Opincariu, Refinements of the Finsler-
Hadwiger reverse inequality. Mathematics Subject Classification, 2000.

[3] Emil Stoica, Nicusor Minculete, Catalin Barbu, New aspects of Ionescu-Weitzenbocks inequal-
ity, Gometry Balkan Press, 2016.

54
Hi tho khoa hc, Hng Yn 25-26/02/2017

NH L THNG D TRUNG HOA V MT S


NG DNG

Nguyn Duy Lin


THPT Chuyn Vnh Phc

1 M u
nh l thng d Trung Hoa l tn ngi phng Ty t thm, ngi Trung Quc gi
n l bi ton Hn Tn im binh. Hn Tn l mt danh tng thi Hn S, tng c
phong tc vng thi Hn Cao T Lu Bang ang dng nghip. S k T M Thin vit
rng Hn Tn l tng tri g khng ni, nhng rt c ti v qun s, tc k rng khi Hn
Tn im qun s ng cho qun lnh xp hng 3, hng 5, hng 7 ri bo co s d mi hng,
t ng tnh chnh xc qun s n tng ngi. Cch im qun s c ng th hin
qua bi th sau:
Tam nhn ng hnh tht thp hy.
Ng th mai hoa trp nht chi
Tht t on vin chnh bn nguyt
Tr bch linh ng tin c chi.
Dch.
Ba ngi cng i t by chc
Nm c mai hoa hm mt cnh
By g xum vy va na thng
Tr trm linh nm bit s thnh

(Ngi dch: Trnh i V i nh Minh).


Bn cht ca bi ton Hn Tn im binh y l vic gii h phng trnh ng d bc
nht

x a1 ( mod m1 )

x a ( mod m )
2 2


....
x ak ( mod mk )

Trong m1 , m2 , . . . , mk l cc s nguyn dng i mt nguyn t cng nhau, vi bi


ton ca Hn Tn th k = 3; m1 = 3; m2 = 5; m3 = 7..

55
Hi tho khoa hc, Hng Yn 25-26/02/2017

nh l 1 (nh l Thng d Trung Hoa). Cho ks nguyn dng i mt nguyn t cng


nhau m1 , m2 , . . . , mk v a1 , a2 , . . . , ak l ks nguyn ty khi h phng trnh ng d
tuyn tnh.


x a1 ( mod m1 )

x a2 ( mod m2 )


....
x ak ( mod mk )

c nghim duy nht m un m1 m2 . . . mk


Chng minh nh l.
1. Chng minh s duy nht:
Gi s h c hai nghim x, y dn n x y ( mod mi ) , i = 1; k. V m1 , m2 , . . . , mk i
mt nguyn t cng nhau nn x y ( mod m1 m2 . . . mk ).Tc l y v x cng thuc mt lp
thng d m1 m2 . . . mk .
2. Chng minh s tn ti:
Ta mun vit cc nghim nh l mt t hp tuyn tnh ca cc s a1 , a2 , . . . , ak .Chng hn
x = A1 a1 + A2 a2 + + A k a k
Vi cc Ai phi tm tha mn A j 0 ( mod mi ) , j 6= i v Ai 1 ( mod mi ) .
t N1 = m2 m3 . . . mk ; N2 = m1 m3 . . . mk ; . . . ; Ni = m1 m2 . . . mi1 mi+1 . . . mk ; . . .
Khi ( Ni , mi ) = 1 v (mi , m1 ) = (mi , m2 ) = = (mi , mi1 ) = (mi , mi+1 ) = =
(mi , mk ) = 1 v m j | Ni , j 6= i.V ( Ni , mi ) = 1 nn tn ti Ni1 sao cho Ni Ni1 1 ( mod mi ).
n y ta t Ai = Ni Ni1 th Ai 1 ( mod mi ) ; Ai 0 mod m j , j 6=


i v`i Ni 0 mod m j Ai 0 mod m j .


 

Khi x = A1 a1 + A2 a2 + + Ak ak = N1 N11 a1 + N2 N21 a2 + + Nk Nk1 ak s tha


mn x Ni Ni1 ai ai ( mod mi )(v tt c cc tha s cn li u chia ht cho mi )

Nhn xt 1. nh l thng d Trung Hoa khng nh v s tn ti duy nht ca mt lp


thng d cc s nguyn tha mn ng thi nhiu ng d tuyn tnh. Do c th s dng
nh l gii quyt nhng bi ton v s tn ti v m cc s nguyn tha mn mt h cc
iu kin v quan h ng d, quan h chia ht. . . , hay m s nghim ca phng trnh
ng d, chng minh cho bi ton s hc chia ht. Vic s dng hp l cc b m1 , m2 , . . . , mk
v b a1 , a2 , . . . , ak trong nh l, cho ta nhiu kt qu kh th v v t ta c th lp c
nhiu bi ton hay v kh. Sau y ti a ra mt s ng dng ca nh l thng d Trung
Hoa gii cc bi ton s hc m chng ta thng gp.

2 p dng vo gii h ng d tuyn tnh


Vn dng t tng ca nh l thng d Trung Hoa, chng ta c th xy dngmt
phng php hiu qu nht trong vic gii h phng trnh ng d tuyn tnh.

56
Hi tho khoa hc, Hng Yn 25-26/02/2017

Cch gii.
Bc 1: t m = m1 m2 . . . mn = Ni mi vi i = 1, 2, 3, . . . , n
Bc 2: Tm cc nghim Ni1 ca phng trnh Ni x 1 ( mod m)
n
Bc 3: Tm c mt nghim ca h l: x0 = Ni Ni1 ai
i =1
Bc4: Kt lun nghim: x x0 ( mod m)

V d 1. u tin ta n vi bi th dn gian Vit Nam:

Trung Thu.
Trung thu gi mt trng trong
Ph phng ng c, n lng sao sa
R nhau i m n hoa
Qun quanh, quanh qun bit l ai hay
Kt nm chn s n ny
By n kt li cn hai ngn tha
Chn n th bn ngn d.
n hoa bao ngn m ngn ng lng.
(Cho bit s n trong khong 600 n 700)

Gii. S dng nh l thng d Trung Hoa ta gii nh sau. Gi s n l


x, ( x Z, 600 x 700) theo bi th ta c h phng trnh ng d nh sau:

x 0 ( mod 5)

x 2 ( mod 7)

x 4 ( mod 9)

N1 = 7 9 = 63 3 ( mod 5) N11 = 2 N2 = 5 9 = 45 3 ( mod 7) N21 = 5,


N3 = 5 7 = 35 8 ( mod 9) N31 = 8.
T ta c x = 2.63.0 + 5.45.2 + 8.35.4 = 1570 310 ( mod 315) x = 310 + 315k, k
Z.
Do x Z, 600 x 700 t suy ra k = 1 v x = 625. Vy s n l 625.
Hoc gii theo cc c thi xa nh sau: Gi x l s n (x l s nguyn dng trong
khong 600 n 700 ),x chia ht cho 5, x chia cho 7 d 2, x chia cho 9 d 4. Ch rng s d
khi chia cho 7 v cho 9 u t hn s chia 5 n v, suy ra x + 5 s chia ht cho c 5;7;9. Bi
s chung nh nht ca 5;7;9 nm trong khong 600 n 700 l 315 2 = 630.
Vy s n s l 630 5 = 625. Li gii rt trong sng v p tic rng ti cha chuyn
th v th c thi.

57
Hi tho khoa hc, Hng Yn 25-26/02/2017

V d 2. Gii h phng trnh ng d:



x 2 ( mod 3)

x 3 ( mod 5)

x 5 ( mod 7)

Gii. Ta c N1 = 5 7 = 35 2 ( mod 3) N11 = 2 N2 = 3 7 = 21 1 ( mod 5)


N21 = 1 N3 = 3 5 = 15 1 ( mod 7) N31 = 1 T ta c x = 2.35.2 + 1.21.3 + 1.15.5 =
278 68 ( mod 105) l nghim h phng trnh.

V d 3. Gii h phng trnh ng d:




x 1 ( mod 3)

x 4 ( mod 5)


x 1 ( mod 7)
x 1 ( mod 8)

Gii. Ta c N1 = 5 7 8 = 280 1 ( mod 3) N11 = 1 N2 = 3 7 8 = 168 3 ( mod 5)


N21 = 2 N3 = 3 5 8 = 120 1 ( mod 7) N31 = 1 N4 = 3 5 7 = 105 1 ( mod 8)
N41 = 1 T c x = 1.280.1 + 2.168.4 + 1.120.1 + 1.105.1 = 1849 169 ( mod 840) l
nghim h phng trnh

V d 4. Gii phng trnh ng d x2 1 ( mod 144) .


Gii. V 144 = 16 9, v`a (16, 9) = 1. Do theo nhl thng d Trung Hoa th nghim ca
bi ton chnh l nghim ca h phng trnh
(
x2 1 ( mod 16)
x2 1 ( mod 9)

Phng trnh x2 1 ( mod 16)c 4 nghim x 1, 7 ( mod 16)


Phng trnh x2 1 ( mod 9) c 2 nghim x 1 ( mod 9) do ta c tt c 8 h sau
( (
x 1 ( mod 16) x 1 ( mod 16)
(1) , (2)
x 1 ( mod 9) x 1 ( mod 9)
( (
x 1 ( mod 16) x 1 ( mod 16)
(3) , (4)
x 1 ( mod 9) x 1 ( mod 9)
( (
x 7 ( mod 16) x 7 ( mod 16)
(5) (6)
x 1 ( mod 9) x 1 ( mod 9)
( (
x 7 ( mod 16) x 7 ( mod 16)
(7) , (8)
x 1 ( mod 9) x 1 ( mod 9)
C 8 h u ng vi k = 2 v N1 = 9 9 ( mod 16) N11 = 9 nn N1 N11 = 81.

58
Hi tho khoa hc, Hng Yn 25-26/02/2017

N2 = 16 7 ( mod 9) N21 = 4 nn N2 N21 = 28.


Do phng trnh ban u c tt c 8 nghim sau
(1) : x = 1.81 + 1.64 = 145 1 ( mod 144)
(2) : x = 1.81 + (1) .64 = 17 17 ( mod 144)
(3) : x = (1) .81 + 1.64 = 17 17 ( mod 144)
(4) : x = (1) .81 + (1) .64 = 145 1 ( mod 144)
(5) : x = 7.81 + 1.64 = 631 55 ( mod 144)
(6) : x = 7.81 + (1) .64 = 503 71 ( mod 144)
(7) : x = (7) .81 + 1.64 = 503 71 ( mod 144)
(8) : x = (7) .81 + (1) .64 = 631 55 ( mod 144)
Nhn xt 2. Nh vy da vo nh l thng d Trung Hoa ta c th m c s nghim
ca mt phng trnh ng d. Chng ta hyc th ha tng ny thng qua cc v d 5,
v d 6 sau y
V d 5. Cho m l mt s nguyn dng, tm s nghim ca phng trnh: x2 x ( mod m).
Gii. Gi s m = p11 p22 . . . pk k ( pi , i N). Ta c x2 x ( mod m) khi v ch khi x2

 
x mod pi i (i = 1, 2, . . . , k) x ( x 1) 0 mod pi i (i = 1, 2, . . . , k )

V ( x, x  1) = 1 pt : x ( x 1) 0 mod pi i c hai nghim modulo pi i l x

0 mod pi i v x 1 mod pi i .Theo nh l thng d Trung Hoa, vi mi b a1 , a2 , . . . , ak .
H phng trnh (

x ai mod pi i
i = 1, 2, . . . , k

lun c nghim duy nht modulo m. Do mi phng trnh. x ( x 1) 0 mod pi i u c
hai nghim modulo pi i nn phng trnh cho c 2k nghim.

V d 6 (VMO 2008). Cho m = 20072008 .Hi c bao nhiu s nguyn dng n mtho mn
.
iu kin: n (2n + 1) (5n + 2) ..m.
Gii. Ta c m = 92008 .2232008 = 34016 .2232008 = n1 .n2 .
.
Do (10, m) = 1 suy ra n (2n + 1) (5n + 2) ..m m|10.5.2n. (2n + 1) (5n + 2) =
10n (10n + 5) (10n + 4) m| x ( x + 5) ( x + 4)trong x = 10n.
Ta c: m| x ( x + 5) ( x + 4) h phng trnh ng d sau

x 0 ( mod 10)

x ( x + 5) ( x + 4) 0 ( mod n1 )

x ( x + 5) ( x + 4) 0 ( mod n2 )

V 3 khng l c chung ca x, x + 4, x + 5 nn x ( x + 5) ( x + 4) 0 ( mod n1 ) khi v ch


khi x r1 ( mod n1 ) r1 {0, 4, 5}.
Tng t x ( x + 5) ( x + 4) 0 ( mod n2 ) khi v ch khi x r2 ( mod n2 ) r1
{0, 4, 5}.

59
Hi tho khoa hc, Hng Yn 25-26/02/2017

Vy m|n (2n + 5) (5n + 4) x 0 ( mod 10) ; x r1 ( mod n1 ) ; x r2 ( mod n2 ).(1)


Vy cc s n m tho mn iu kin bng s cc s x 10n1 .n2 tho mn (1).Vi mi
cch chn r1 {0, 4, 5} & r2 {0, 4, 5}theo nh l Trung Hoa ta c duy nht mt
s x 10n1 .n2 tho mn (1).Vy c 9 s tho mn iu kin bi ra.

Bi ton m rng 1. Cho m = p11 p22 . . . pk k ( pi , i N)v f ( x )l mt a thc vi h s


nguyn. Khi phng trnh ng d f ( x ) 0 ( mod m)c nghim khi v ch khi tt c


i 
cc phng trnh ng d f ( x ) 0 mod pi , i = 1, kc nghim. Nu gi s nghim ca

phng trnh f ( x ) 0 mod pi i , i = 1, k l ni th phng trnh f ( x ) 0 ( mod m)c ng
n1 n2 . . . nk nghim (mod m).

3 p dng gii cc ton s hc v chng minh s tn ti


V d 7. Cho p, q N \ {1} , ( p, q) = 1. Chng minh rng tn ti k Z sao cho ta c s
( pq 1)n k + 1 l hp s vi mi n N .
Gii. Do ( p, q) = 1 theo nh l thng d Trung Hoa k N tho mn h phng trnh
ng d
(
k 1 ( mod p)
k 1 ( mod q)
.
Nu n..2 ( pq 1)n 1 ( mod q) ( pq 1)n k 1 ( mod q) ( pq 1)n k + 1
0 ( mod q)
Nu n2 ( pq 1)n 1 ( mod p) ( pq 1)n k 1 ( mod p) ( pq 1)n k + 1
0 ( mod p) Vy ( pq 1)n k + 1l hp s vi mi n N .

Nhn xt 3. Chng minh trn tht gn gng nh vo vic s dng nh l thng d Trung
Hoa. Mu cht ca bi ton l chng ta thy c ( pq 1)n k + 1 l hp s ta cn ch ra
rng khi no ( pq 1)n k + 1 chia ht cho p hoc q(qua vic xt tnh chn l ca n) t ta
xy dng c mt h phng trnh ng d:
(
k 1 ( mod p)
k 1 ( mod q)

V d 8 (IMO 1989). Chng minh rng vi mi n N tn ti ns t nhin lin tip sao cho
bt k s no trong cc s y cng u khng phi l lu tha (vi s m nguyn dng)
ca s nguyn t.
Gii. Cch 1. Mi n N xt ns nguyn t phn bit p1 , p2 , . . . , pn .
Xt h phng trnh

60
Hi tho khoa hc, Hng Yn 25-26/02/2017

x p1 1 mod p21



x p2 1 mod p22





...............................
x pn 1 mod p2n


Theo nh l thng d Trung Hoa th h phng trnh trn c nghim a Z : a


pi 1 mod p2i i = 1, n. T suy ra cc s a + 1, a + 2, . . . , a + n u khng phi l lu
tha vi s m nguyn dng ca mt s nguyn t.
Cch 2. Mi n N xt 2ns nguyn t phn bit p1 , p2 , . . . , pn , q1 , q2 , . . . , qn .Xt h
phng trnh


x 1 ( mod p1 q1 )

x 2 ( mod p q )
2 2


...............................
x n ( mod pn qn )

Theo nh l thng d Trung Hoa th h phng trnh trn c nghim


a Z : a i ( mod pi qi ) i = 1, n. T suy ra cc s a + 1, a + 2, . . . , a + n, u
khng phi l lu tha vi s m nguyn dng ca mt s nguyn t.

Nhn xt 4. qua s chn kho lo b m1 , m2 , . . . , mk cho ta mt dy ns hng tha mn yu


cu
T tng ging nh trn cho cc v d 4,5,6,10 di y.

V d 9 (Nordic 1998). Tm s nguyn dng n sao cho tn ti dy { x1 , x2 , . . . , xn }=


.
{1, 2, . . . , n} tho mn: x1 + x2 + + xk ..k vi mi k = 1, 2, . . . , n 2/ Tn ti hay khng mt
.
dy v hn { x1 , x2 , . . . }= {1, 2, . . . } sao cho xi 6= x j i 6= j v tho mn: x1 + x2 + + xk ..k
vi mi k = 1, 2, . . . , n?
Gii.
1/ n = 1 tho mn,n = 3 tho mn vi dy tng ng l 1, 3, 2.
n n
n ( n +1) .
Gi s n N tho mn bi khi ta c: xi = i = 2 ..n nl s l .
i =1 i =1
n 1 n 1 .
Gi s n 5, t m = n +1
2 . Theo gt xi = i = mn xn ..n 1 nn suy ra
i =1 i =1
xn mn m ( mod n 1) , 1 xn n xn1 = n 1. Tng t ta c
n 2 n 2 .
xi = i = m (n 1) xn1 ..n 2 suy ra
i =1 i =1
xn1 m (n 1) m ( mod n 2) , 1 xn1 n xn1 = m = xn , v l.
Vy ch c n = 1, n = 3 tho mn iu kin bi.

2/ Ta s xy dng mt dy ( xn )+
n=1 tho mn iu kin bi.
Ly x1 = 1, x2 = 3, x3 = 2.Gi s x1 , x2 , x3 , . . . , x N l mt dy tho mn iu kin

61
Hi tho khoa hc, Hng Yn 25-26/02/2017

.
x1 + x2 + + xk ..k vi mi k = 1, 2, . . . , N.t x1 + x2 + x3 + + x N = s
Gi n l s nguyn dng b nht khng nm trong dy x1 , x2 , x3 , . . . , x N .
Do ( N + 1, N + 2) = 1 nn theo nh l thng d Trung Hoa tn ti mt s nguyn
m > x1 , x2 , x3 , . . . , x N tho mn
(
m s ( mod N + 1)
m s n ( mod N + 2)

t x N +1 = m, x N +2 = n, ta c dy x1 , x2 , x3 , . . . , x N , x N +1 , x N +2 tho mn cc iu kin
ca bi ton v
.
+ x1 + x2 + x3 + + x N + x N +1 = s + m..N + 1;x1 + x2 + x3 + + x N +1 + x N +2 =
.
s + m + n..N + 2
.
v x1 + x2 + + xk ..k vi mi k = 1, 2, . . . , N.
.
Do x1 + x2 + + xk ..k vi mi k = 1, 2, . . . , N + 2 hin nhin dy ( xn )+ n=1 xy dng
nh trn tho mn iu kin bi.

Nhn xt 5. Trong bi ton ny ta cn ch n dy { xn }l mt hon v ca tp N,nu


khng c gi thit ny bi ton tr thnh tm thng, trong phn 2 ta cn quy np nh sau,
mi b
.
x , x , . . . , xn tha mn ta lun tm c x
1 2 sao cho x + x + + x .. n + 1. Do vy
n +1 1 2 n +1
ta cn phi xy dng dy { xn } sao cho dy { xn }quyt ht tp N, y l yu cu chnh ca
bi ton

V d 10. Chng minh rng nu p1 , p2 , . . . , pn l cc s nguyn t phn bit th phng trnh


p p p n 1 pn
x1 1 + x2 2 + + x n 1 = xn c v s nghim nguyn dng ( x1 , x2 , . . . , xn ).
Gii. Ta c ng thc (n 1)k + (n 1)k + + (n 1)k = (n 1)k+1 .
| {z }
n 1
k k k k +1
Khi ta chn x1 = (n 1) , x2 = (n 1) p2 , . . . , xn1 = (n 1) pn1 , xn = (n 1) pn .
p1

p p p n 1 pn
Th ta thu c ngay phng trnh x1 1 + x2 2 + + xn 1 = xn . Vy nu ta ch ra c
s nguyn dng k sao cho x1 , x2 , . . . , xn u nguyn th ta c iu phi chng minh.
M iu ny tng ng vi h sau c nghim.


k 0 ( mod p1 )

k 0 ( mod p2 )



()

k 0 ( mod pn1 )





k 1 mod p
( n)

iu ny lun ng theo nh l thng d Trung Hoa, v p1 , p2 , . . . , pn l cc s nguyn


t phn bit.

62
Hi tho khoa hc, Hng Yn 25-26/02/2017

V d 11. Chng minh rng vi mi s nguyn dng n lun tn ti n s nguyn a1 , a2 , . . . , an


Sao cho ai + a j l ly tha ca mt s t nhin vi s m ln hn 1 vi mi i, j {1, 2, . . . , n}
Gii. Ta chn cc s sau a1 = 1x1 +1 .2x2 .3x3 . . . .(2n) x2n ,
a1 = 1x1 .2x2 +1 .3x3 . . . .(2n) x2n ,. . . ,
an = 1x1 .2x2 .3x3 . . . n xn +1 . . . (2n) x2n , xi N
ai + a j = 1x1 .2x2 .3x3 . . . i xi +1 . . . (2n) x2n + 1x1 .2x2 .3x3 . . . j x j +1 . . . (2n) x2n
= 1x1 .2x2 .3x3 . . . (2n) x2n (i + j)
ai + a j = 1x1 .2x2 .3x3 . . . (i + j) xi+ j+1 . . . (2n) x2n
Xt cc s nguyn t phn bit p1 , p2 , . . . , p2n .
Xt cc h phng trnh ng d tuyn tnh.
(
x1 1 ( mod p1 )
x1 0 ( mod pk ) , k {2, 3, . . . , 2n}
(
x2 1 ( mod p2 )
x2 0 ( mod pk ) , k {1, 3, 4, . . . , 2n}
( 
xi+ j 1 mod pi+ j
xi+ j 0 ( mod pk ) , k {1, 2, 3, . . . i + j 1, i + j + 1, . . . , 2n}
(
x2n 1 ( mod p2n )
x2n 0 ( mod pk ) , k = 1, 2n 1
Theo nh l thng d Trung Hoa th cc h ny chc chn c nghim. T suy ra
x1 x2 x i + j +1 x2n
pi + j ; pi + j ; ; pi + j ; ; pi + j cc s ny u l s nguyn.

Khi ai + a j = 1x1 .2x2 .3x3 . . . (i + j) xi+ j +1 . . . (2n) x2n


" x +1
# pi + j
x1 x2 i+ j x2n
pi + j pi + j pi + j pi + j
= 1 2 (i + j ) (2n) l ly tha ca mt s nguyn dng y

l iu phi chng minh


V d 12 (Balkan 2000). Cho A l mt tp hp khc rng cc s nguyn dng. Chng minh
rng tn ti s nguyn dng m sao cho mi phn t ca tp mA u l ly tha ca mt s
t nhin vi s m ln hn 1.
Gii. Gi s A = { a1 , a2 , . . . , ak }. Gi p1 , p2 , . . . , p N l tt c cc c s nguyn t ca s
k N i,j
ai . Vi mi i = 1, 2, . . . , k tn ti cc s nguyn khng m i,j sao cho ai = p j . Gi
i =1 j =1

q1 , q2 , . . . , qk l cc s nguyn t phn bit. Theo nh l thng d Trung Hoa, vi j = 1, N


Tn ti j i,j ( mod qi )vi mi i = 1, 2, . . . , k.
" i,j + j # qi
N j N i,j + j N qi
t m = p j . Khi vi i = 1, 2, . . . , k th mai = p j = pj l s ly
j =1 j =1 j =1
tha

63
Hi tho khoa hc, Hng Yn 25-26/02/2017

Ta c iu phi chng minh.

V d 13. Chng minh rng tn ti v hn s k nguyn dng chn, sao cho vi mi s


nguyn t p th s p2 + k l hp s.
Gii.
+ Nu p = 2 p2 + k l hp s vi mi s k chn.
+ Nu p > 3 p2 1 ( mod 3) suy ra mi k chn v k 2 ( mod 3) th p2 + k l hp s
(bi ca 3 )
+ Nu p = 3 p2 + k = 9 + k 0 ( mod 5) Nu k 1 ( mod 5).
Vy k tha mn iu kin bi ton k l nghim nguyn dng ca h phng trnh
ng d (
k 0 ( mod 2)
k 1 ( mod 5)

theo nh l thng d Trung Hoa th h phng trnh trn c nghim: k 26 ( mod 30)
k = 30h + 26, (h N),p2 + k = p2 + 30h + 26 40 cho nn p2 + k l hp s. Vy c v s
k nguyn dng chn, sao cho vi mi s nguyn t p th s p2 + k l hp s.

Nhn xt 6. Chng minh trn tht n tng nh vo vic s dng nh l thng d Trung
Hoa. Mu cht ca bi ton l chng ta thy c p2 + k l hp s.ta cn ch ra rng khi
no p2 + k chia ht cho 2,3 hoc 5 (qua vic xt cc dng ca p) t ta xy dng c mt
h phng trnh ng d:
k 0 ( mod 2)

k 2 ( mod 3)

k 1 ( mod 5)

T tm c tt c gi tr ca k.

V d 14. (mathlink.ro) Chng minh rng tn ti a thc P ( x ) Z [ x ], khng c nghim


.
nguyn sao cho vi mi s nguyn dng n, tn ti s nguyn dng x sao cho P ( x ) ..n.
Gii. Xt a thc P ( x ) = (2x + 1) (3x + 1).Vi mi s nguyn dng n, ta biu din n di
dng n = 2k (2m + 1). V 2k , 3 = 1 nn tn ti a sao cho 3a 1 mod 2k . T
 

3x 1 mod 2k th ta cn chn x a mod 2k .


 

V (2, 2m + 1) = 1 nn tn ti b sao cho 2b 1 ( mod 2m + 1). T 2x


1 ( mod 2m + 1) th ta cn chn x b ( mod 2m + 1)
Nhng do 2k , 2m + 1 = 1. Nn theo nh l thng d Trung Hoa, tn ti s nguyn x


l nghim ca h phng trnh ng d sau:


(
x a mod 2k


x b ( mod 2m + 1)

.
theo lp lun trn P ( x ) = (2x + 1) (3x + 1)..n.

64
Hi tho khoa hc, Hng Yn 25-26/02/2017

V d 15. Cho tp A = {2, 7, 11, 13} v a thc P ( x ) Z [ x ] c tnh cht vi mi n Z tn


ti p A sao cho p | P (n) . Chng minh rng tn ti p A sao cho p | P (n) vi mi n Z.
Gii.

B 1. x, y Z sao cho x y ( mod p) P ( x ) P (y) ( mod p) (t chng minh)


p dng: Gi s khng tn ti p A sao cho p | P (n) vi mi n Z suy ra a, b, c, d Z
phn bit sao cho
. . .
P ( a) a0 ( mod 2) a0 6 .. 2 P (b) b0 ( mod 7) b0 6 .. 7 P (c) c0 ( mod 11) c0 6 .. 11
.
P (d) d0 ( mod 13) d0 6 .. 13
Xt h phng trnh ng d:


x a ( mod 2)

x b ( mod 7)
() .


x c ( mod 11)
x d ( mod 13)

Theo nh l thng d Trung Hoa h phng trnh () c nghim x0 . Kt hp vi b


ta c

P ( a) a0 ( mod 2)


P ( xo )
P (b) b0 ( mod 7)

P ( xo )
()


P ( xo ) P (c) c0 ( mod 11)
P ( xo ) P (d) d0 ( mod 13)

mu thun vi iu gi s trn. Vy iu gi s l sai t ta c iu phi chng minh.

Bi ton m rng 2. Cho P ( x ) l a thc vi h s nguyn. Gi s rng c mt mt tp


hu hn cc s nguyn t A = { p1 , p2 , . . . , pn }, sao cho vi mi s nguyn alun tn ti s
.
pi A, i = 1, n sao cho P ( a) ..pi . Chng minh rng tn ti s nguyn t p sao cho P ( x )


chia ht cho p vi mi s nguyn x

Nhn xt 7. Qua vic gii hai v d 8 v 9 vic kt hp gia nh l thng d Trung Hoa vi
cc tnh cht ca a thc nguyn cho ta mt kt qu th v

V d 16. Cho n, h N . Chng minh rng tn ti n s t nhin lin tip tha mn mi s


u c t nht h c s nguyn t phn bit.
Gii. Do tp hp cc s nguyn t l v hn nn ta c th chn nh s nguyn t phn bit
p1 < p2 < < ph < < pnh
Theo nh l thng d Trung Hoa th tn ti k N l nghim ca h phng trnh

65
Hi tho khoa hc, Hng Yn 25-26/02/2017



k 1 ( mod p1 p2 . . . ph )




k 2 ( mod ph+1 ph+2 . . . p2h )




 

k i mod p(i1)h+1 p(i1)h+2 . . . pih







 
k n mod p(n1)h+1 p(n1)h+ 2 . . . pn h

,i = 1, n
T ta c n s t nhin lin tip l: k + 1; k + 2; ; k + n tha mn mi s u c t
nht h c s nguyn t phn bit.

V d(17. Chng minh rng vi mi m, n nguyn dng th tn ti x nguyn dng tho


2x 1999 ( mod 3m )
mn:
2x 2009 ( mod 5n )
Gii.
m n
( 2. 2 l cn nguyn( thu ca mod 5 , mod 3 . T tn ti x1 , x2
B :
2x1 1999 ( mod 3m ) 2x1 1 ( mod 3)
do v x1 , x2 chn
2x2 2009 ( mod 5n ) 2x2 4 ( mod 5)
Theo nh l thng d Trung Hoa th h phng trnh ng d sau c nghim
t x21 m 1
( 
 mod 3 
x2 n 1
t 2 mod 4.5
 
mod (3m ) = 2.3m1
(
2t x1 2x 1999 ( mod 3m )
Chn x = 2t th  
2t x2 mod (5n ) = 4.5n1 2x 2009 ( mod 5n )
(pcm)

Nhn xt 8. Bi ton cn vn dng kt hp kin thc gia cn nguyn thy v nh l thng


d Trung Hoa cho ta mt li gii tht cht ch v ngn gn

V d 18 (diendantoanhoc.net 2014). Chopl s nguyn t. Chng minh rng tn ti mt


bi s ca p sao cho 10 ch s tn cng ca n i mt khc nhau.

Gii.
Nu p = 2 th hin nhin lun tn ti mt s tha mn bi v d: 1234567899876543210
Nup = 5 th cng lun tn ti mt s tha mn bi v d: 1234567899876432105
Nu p 6 {2, 5}. Xt h phng trnh ng d tuyn tnh.
(  
x a0 a1 a2 . . . a9 mod 1010
x 0 ( mod p)

66
Hi tho khoa hc, Hng Yn 25-26/02/2017

Trong ai {0, 1, 2, 3, 4, 5, 6, 7, 8, 9} , ai 6= a j , 0 i 6= j 9
 
10
V p , p 6 {2, 5} suy ra gcd p, 10 = 1. Do theo nh l thng d Trung Hoa
th h ny chc chn c nghim, nghim ca h chnh l s tha mn (iu phi chng minh
)

Nhn xt 9. T cc trng hp c s cho cc s nguyn t 2 v 5, xy dng nn h phng


trnh ng d tuyn tnh ti u cho s nguyn t bt k khc 2 v 5.

V d 19 (HSG Tri h Hng Vng 2014). Chng minh rng tn ti 16 s nguyn dng
lin
2 tip sao cho khng c s no trong 16 s c th biu din c di dng
7x + 9xy 5y2 , ( x, y Z).

2 2
2 2
Gii. t 7x + 9xy 5y = A 28A = (14x + 9y) 13.17y . Ta xt s d khi chia A

cho 9,13 v 17 thu c.


A chia cho 9 khng c s d l 3,6.
A chia cho 13 khng c s d l 1,3,4,9,10,12
A chia cho 17 khng c s d l 1,2,4,8,9,13,15,16
Theo nh l thng d Trung Hoa tn ti s nguyn dng n tha mn:


n 4 (mod 9)
n 2 (mod 13)

n 0 (mod 17)

R rng
n + 7; n + 10 khng c dng 7x2 + 9xy 5y2 , ( x, y Z)

n + 3; n + 5; n + 6; n + 11; n + 12; n + 14 khng c dng 7x2 + 9xy 5y2 , ( x, y Z)


n + 1; n + 2; n + 4; n + 8; n + 9; n + 13; n + 15; n + 16 khng c dng 7x2 + 9xy 5y2


Suy ra tn ti 16 s nguyn dng lin tip n + 1; n + 2; . . . ; n + 15; n + 16tha mn ycbt

Nhn xt 10. T cc trng hp c s cho cc s nguyn 9,13 v 17, xy dng nn h phng


trnh ng d tuyn tnh ti u ch ra c 16 s nguyn dng lin tip khng c dng
ca biu thc cho, mt vic lm cn c s nhy bn v tinh t

4 p dng gii cc bi ton v tnh chia ht


V d 20. Chng minh rng phng trnh x2 34y2 1 ( mod m) c nghim vi mi
m N .
Gii. Trng hp 1. (m, 3) = 1 th x2 34y2 1 ( mod m) hay ( x 5y) ( x + 5y)
(3y + 1) (3y 1) ( mod m) .
Tp hp cc s {3y + 1, 3y 1} chy qua cc s khng chia ht cho 3, suy ra
.
y0 Z : (3y0 + 1) (3y0 1) ..m chn x0 = 5y0 suy ra ( x0 , y0 ) cn tm.

67
Hi tho khoa hc, Hng Yn 25-26/02/2017

Trng hp 2. (m, 5) = 1 th x2 34y2 1 ( mod m) hay ( x 3y) ( x + 3y)


(5y + 1) (5y 1) ( mod m) .
Tp hp cc s {5y + 1, 5y 1} chy qua cc s khng chia ht cho 5, suy ra
.
y0 Z : (5y0 + 1) (5y0 1) ..m chn x0 = 3y0 suy ra ( x0 , y0 ) cn tm.
Trng hp 3. (m, 3) = (m, 5) 6= 1, t m = m1 .m2 vi
m1 = 3 ( N ) , m2 N : (m1 ; m2 ) = 1, (m1 , 5) = 1.
2
(3, m2 ) = 1 ( x1 ; y1 ) (Z+ ) : x12 34y21 1 ( mod m2 ) .
2
(5, m1 ) = 1 ( x2 ; y2 ) (Z+ ) : x22 34y22 1 ( mod m1 ) .
T theo nh l thng d Trung Hoa tn ti ( x, y) N sao cho
( (
x x1 ( mod m1 ) x x2 ( mod m2 )
v
y y1 ( mod m1 ) y y2 ( mod m2 )
Vy ta c iu phi chng minh.

Nhn xt 11. Cch gii ca bi ton chnh l dng phng php gen trong phng trnh
ng d, kt hp vi nh l thng d Trung Hoa

V d 21 (Shortlisted IMO 1998). Xc nh tt c n N sao cho vi n ny tn ti m Z sao


cho: 2n 1|m2 + 9.
Gii. Ta chng minh 2n 1|m2 + 9 n = 2s (s N )
iu kin cn. t n = 2s t (s N, t N , (t, 2) = 1).Nu t 3 2t 1|2n 1
2t 1|m2 + 9.
Ta c 2t 1 1 ( mod 4) p : p 1 ( mod 4) , p|2t 1 ( p 6= 3) suy ra p|m2 + 9
 p 1 p 1
p|m2 + 32 theo nh l Fecma 1 m p1 m2 2 (9) 2 1 ( mod p)v
l iu ny khng xy ra nu tn ti t p = 3 mu thun, Nn 2t 11 ( mod p) vy
n = 2s ( s N )
s  2   s 1 
iu kin . 2n 1 = 22 1 = (2 1) (2 + 1) 22 + 1 22 + 1 . . . 22 + 1 t
t


n 2 2 2 2 2
suy ra suy ra 2 1|m + 9 2 + 1|m + 9t = 1, s 1. M 2 + 1; 2 + 1 = 1 ( 6= )
t
 t

Theo nh l thng d Trung Hoa h phng trnh x 22 mod 22 + 1 t = 0, s 2
t
 t +1
  t +1

c nghim nn tn ti c Z : c 2 2 mod 2 2 2
+ 1 c + 1 0 mod 2 2 + 1 t =
n 2 2

0, s 2 t y suy ra 2 1|9 c + 1 = m + 9 trong m = 3c

Nhn xt 12. Ci kh ca bi ton l d on dng ca n(thng qua mt s v d c s),


vi iu kin ta cn xy dng c h phng trnh ng d c m1 , m2 , . . . , ms i mt
nguyn t cng nhau.

V d 22 (Shortlisted IMO 2001). Cho s nguyn dng a ch c c nguyn t dng 4k +


.
1 (k N ). Chng minh rng tn ti b N sao cho b2 + 1..a4 + a2 .
Gii.

68
Hi tho khoa hc, Hng Yn 25-26/02/2017

.
B 3. Cho p , p = 4k + 1 x N : x2 + 1..pn (n N cho trc) Quy np
n = 1chn x = (2k )! tho mn b
.
Gi s b ng vi n = h N xh : x2h + 1..ph x2h + 1 = uph (u N )
2 .
t xh+1 = xh + tph x2h+1 + 1 = xh + tph + 1 = ph (u + 2xh t) + p2h t2 ..ph+1 Ta cn
.
chn t N : u + 2x t..p h
x s
p dng a2 = ph h ( ph 1 ( mod 4)) suy ra a2 + 1 =

ph h ( ph 1 ( mod 4))
h =1 h =1
s . .
a2 a2 +1 =
h = 1, 2, . . . , s bh Z : bh2 + 1..ph (theo b ), b02 + 1..2 (b0 Z)

pk k
k =1
Xt h phng trnh ng d :
(

x b0 mod ph h
x bs ( mod ps s )

. s
Theo nh l thng d Trung Hoa th h c nghim x = b Z : b2 + 1.. ph h
h =1

V d 23. Tm tt c cc s nguyn dng a sao cho : 2n n2 | an n a , vi mi s n nguyn


dng n 5.
Gii. Chn s nguyn t p sao cho p > 2, a , a2 2a .


Theo nh l thng d Trung Hoa th tn ti n N , n 5 l nghim ca h


(
n 2 ( mod p)
n 2 ( mod p 1)

T ta c 2n n2 22 22 0 ( mod p) p | an n a . M an n a a2 2a ( mod p)
suy ra p a2 2a . Vymi p nguyn t tha mn th p a2 2a a2 2a = 0 a = 2, a =

4 do 2a > a2 , a 5 Th li a = 2, a = 4 tha mn

Nhn
 2 13. a Ci
xt hay ca bi ton qua vic chn s nguyn t p sao cho p >
2, a , a 2

V d 24. 1. Cho s nguyn dng n khng c c chnh phng khc 1. Chng minh rng
. .
nu an 1..n th an 1..n2 .
. .
2. Tm n Z+ , n > 1 c tnh cht mi a Z+ nu an 1..n th an 1..n2 .
Gii.
. .
1) n = p , a N co a p 1..p a p a + a 1..p a 1 ( mod p)
T a p 1 = ( a 1) a p1 + a p2 + + a + 1 0 mod p2 . n = p1 .p2 . . . .pk , an
 
. . .
1 = a p1 p2 ...pk 1..p1 p2 . . . pk ( a p1 p2 ...pk ) pi 1..pi ( a p1 p2 ...pk ) pi 1..p2i a p1 p2 ...pk
. .
1..( p1 p2 . . . pk )2 an 1..n2

69
Hi tho khoa hc, Hng Yn 25-26/02/2017

2)n = 2 p1 p2 . . . pk , 0 2, pi , ( pi , 2) = 1, i = 1, k
iu kin . Ta ch cn xt n = 4p1 p2 . . . pk , pi , ( pi , 2) = 1, i = 1, k l
. .
A = a p1 p2... pk , an 1 = A4 1..4, do A l nn A4 1 = ( A 1) ( A + 1) A2 + 1 ..16 suy
.
ra an 1..42
. .
B = a4 , an 1 = B p1 .p2 ...pk 1..p1 p2 . . . pk an 1 = B p1 .p2 ...pk 1..( p1 p2 . . . pk )2 theo(1)
.
M 42 , p21 . . . p2k = 1 an 1..(4p1 p2 . . . pk )2 = n2


iu kin cn. Gi s n = p q, p , ( p, 2) = 1, (q, 2) = 1, q, N



Ly a = pq + 1 an 1 = a p q 1 = p+1 h, (h, p) = 1 (chng minh bng quy np)
Do

an 1...a 1...p .. .. 2
n n +1 .. 2 2
do a 1 .n a 1 .n p h.p q + 1 2 1.
an 1...p

Vy s m ca l 0 1
Xt p = 2, gi s n = 2 t, t N , (t, 2) = 1th 52t 1 = 25t 1 = 8m, m N, m l
Theo nh l thng d Trung Hoa tn ti s nguyn dng a l nghim h pt ng d
(
a 5 ( mod 16)
a 1 ( mod t)

 i
  1

at an A2 A2 A2 A2 A2
 
t A = 1 = 1 = 1 +1 ... +1 .... +1 =
2+2 V, (V, 2) = 1
i
Do A2 1 = a2t 1 25t 1 8 ( mod 16) , A2 + 1 2 ( mod 4) i = 1, s 1
. . .
Do an 1..n th an 1..n2 . 2+2 V ..22 t2 + 2 2 2 {0, 1, 2}
Vy n = 2 p1 p2 . . . pk , 0 2, pi , ( pi , 2) = 1, i = 1, k
Nhn xt 14. Ci kh ca bi ton phn 2 vi iu kin cn, gii quyt c vn ny
ta cn nm vng s m ng ca mt s vi mt s nguyn t,v chn c b a1 , a2 , .., ak
hp l ca h phng trnh ng d.
V d 25 (Selection tests for the BMO and IMO Romanian teams 2006). Cho a, b l cc s
nguyn dng, sao cho vi mi s nguyn dng n ta c an + n| bn + n. Chng minh rng
a = b.
Gii. Gi s ab. Ta c khi n = 1 th a + 1| b + 1 b > a. Gi p l s nguyn t: p > b, Theo
nh l thng d Trung Hoa tn ti n l s nguyn dng lnghi h pt ng d
(
n 1 ( mod ( p 1))
n a ( mod p)

suy ra n = ( a + 1) ( p 1) + 1.

70
Hi tho khoa hc, Hng Yn 25-26/02/2017

Theo nh l Fermats, an = a(a+1)( p1)+1 = a |a p1 .{z


. . a p}1 a ( mod p) an + n
a+1 time
a + n 0 ( mod p), hay p| an + n p| bn + n , (1) . M theo nh l Fermats, bn + n
b a ( mod p) (2).
T (1) v (2) p| b a v l. Vy iu gi s ab l sai, do a = b.
V d 26. Chng minh rng vi mi s N nguyn dng l tch ca 2015 s nguyn t l
phn bit u l c ca v s s c dng a a+1 + ( a + 1) a , vi a l s nguyn dng.
Gii. Nhn xt. vi s p l s nguyn t l, p N p a a+1 + ( a + 1) a

Chn ( (
a 2 ( mod p) a 2 ( mod p)
() ,
a + 1 0 ( mod p 1) a 1 ( mod p 1)

Theo nh l thng d Trung Hoa h () c nghim a = p 2 + kp ( p 1) , k N .



Khi N = p1 .p2 . . . p2015 , p j l s nguyn t l. Theo nhn xt trn vi mi p j j = 1, 2015
a +1 a
j
lun tn ti a j m p j a j + a j + 1 j . Xt h phng trnh ng d
( 
a a j mod p j
j = 1, 2015

theo nh l thng d Trung Hoa h lun tn ti v hn s anh vy ta c iu chng minh


Nhn xt 15. Qua hai v d 6,7 cho ta thy mt li gii p nu ta bit cch s dng thnh
tho nh l h thng d Trung Hoa
V d 27 (The IMO Team Selection tests -2013). Tm tt c a, b, c Z, c 0 sao cho
( an + 2n )| (bn + c) vi mi s nguyn dng n, ng thi 2ab khng l s chnh phng.
Gii. Ta c ( an + 2n )| (bn + c) bn c ( mod an + 2n ) b3n + c3 0 ( mod an + 2n ) , ()
a3n + 23n b3n + c b3n + c 0 mod a3n + 23n , m an + 2n | a3n + 23n
  

suy ra b3n + c 0 ( mod an + 2n ) ()


T () , () c3 c 0 ( mod an + 2n ) an + 2n | c (c + 1) (c 1) , ( )
Khi cho n + |c (c + 1) (c 1)| < | an + 2n |, kt hp( ) c (c + 1) (c 1) =
0 c = 0c = 1
+ Nu c = 0
- Kh nng a = 2 theo bi 2n+1 bn 4| b tha mn iu kin

- Kh nng | a| 6= 2 tp cc c s nguyn t ca { an + 2n , n N } l v hn tht vy


Nu a| 2 a = 2a1 , ( a1 Z , | a1 | > 1) an + 2n = 2n a1n + 1 , m vi mi k, l


N, k 6= l th
 k l

a21 + 1, a21 + 1 2 suy ra { an + 2n , n N} c v hn c s nguyn t.
 k k l l

Nu a l suy ra k, l N, k 6= l a2 + 22 , a2 + 22 = 1 { an + 2n , n N} c v
hn c s nguyn t,do an + 2n | bn bc v hn c s nguyn t v l.

71
Hi tho khoa hc, Hng Yn 25-26/02/2017

+Nu c = 1
- Kh nng 2| a theo bi a2 + 22 b2 + 1 b2 + 1 0 ( mod 4) v l

- Kh nng a l, do 2ab khng l s chnh phng cho nn


(
2a = l 2 p1 p2 . . . ps
b = m 2 p k p k +1 . . . p t + s b = m 2

Vi l, m N , p1 , p2 , . . . , pt+s , t, k N , s N.cc s nguyn t pi i = 1; t + s




phn bit. k = 1, s = 0 v l do 2ab l s chnh phng


Theo nh l thng d Trung Hoa th tn ti s nguyn t p sao cho p 1 ( mod 4)
   p 
p
b = m2 pi = 1, i = 2; t ; p1 = 1.
    pi   
pi p
k > t p = 1, i = 2; t ; p = 1, i = k; t + s & p1 = 1
   p 
p
1 < k t pi = 1, i = 2; t + s , p1 = 1
   p     
p p p
k = 1, s > 0 pi = 1, i = 2; t + s 1 ; pi = 1, i = k; t + s & p1 = tp+s =
1    
2a b
Qua cc trng hp trn ta thy rng p = 1; p = 1 theo tiu chun Eulers
p 1 p 1 p 1 p 1
(2a) = 1 ( mod p) a 2 + 2 2 0 ( mod p) ; b 2 = 1 ( mod p)
2

p 1 p 1 p 1 p 1

Nhng a 2 + 2 b 2 + 1 b 2 1 ( mod p) mu thun ( do p 6= 2)
2

Vy cc b ( a, b, c) = (2, 4k, 0) , k N , k khng l s chnh phng tha mn bi.

Nhn xt 16. Mt v d kh kh ca bi vit, bi ton l s hp gia ng d thc,thng d


bnh phng v mt cht ca nh l thng d Trung Hoa, ci kho ca bi ton qua vic
chn cc s 2a v b sao cho 2ab khng l s chnh phng
2016
V d 28. Tm tt c cc s nguyn dng n s n.22 n 76 l s chnh phng.
22016
Gii. Gi s tn  ti s nguyn
  dng  
n s n.2 n 76 l s chnh
  1 phng.
  0 Ta c

2016 2015 2015 2015 2014
A = 22 1 = 22 + 1 22 1 = 22 + 1 22 + 1 . . . 22 + 1 22 + 1
2016 2015  l 
A=2 2 1 = 3 2 +1 2

 l l =1 m  l
Gi d = 22 + 1, 22 + 1 , 1 l 6= m 2015 d| 2 d = 1 d = 2, do 22 + 1 l suy
ra d = 1
l
Do cc s 22 + 1 , l = 1; 2015 nguyn 

t cng nhau 
nn theo nh l thng d Trung
l 1 l
Hoa, s tn ti s nguyn C sao cho C 22 mod 22 + 1 , l = 1; 2015
l
 l
 l
suy ra C2 + 1 22 + 1 mod 22 + 1 C2 + 1 22 + 1, l = 1; 2015 suy ra
 .. 2015  2l  .. 2
 2016 
3 C + 1 .3 2 + 1 = A 81C + 81.A n N : (9c) + 81 = n 2
2 2 2 1
l =1

72
Hi tho khoa hc, Hng Yn 25-26/02/2017
2016
suy ra (9c)2 + 5 = n22 n 76 v l do mt s chnh phng khi chia cho 9 cho cc s
d l 0, 1, 4, 7 Nhng (9c)2 + 5 5 ( mod 9). Vy iu gi s l sai tng ng vi khng
2016
c s nguyn dng n s n.22 n 76 l s chnh phng.
V d 29. Mt s nguyn n c gi l s tt nu |n| khng l s chnh phng.Xc nh
tt c cc s nguyn m sao cho mc th biu din bng v hn cch l tng ca 3 s tt khc
nhau v tch ca chng l mt s chnh phng l
Gii.
iu kin cn
Gi s m = u + v + w, vi u, v, wl cc s tt v u.v.w l s chnh phng l. Khi ta c


u 1; 3 ( mod 4)

v 1; 3 ( mod 4)
m = u + v + w 3 ( mod 4) .


w 1; 3 ( mod 4)
u.v.w 1 ( mod 4)

iu kin
Ta chng minh vi mi s nguyn m m m 3 ( mod 4) u tha mn yu cu ca bi
ton Trc tin ta chng minh: Vi mi s nguyn dng m = 4k + 3, (k Z) u phn tch
c V dng m = 4k + 3 = xy + yz + zx (1), trong x, y, z l cc s nguyn l.
Tht vy nu chn x = 1 + 2t v`a y = 1 2t (t Z \ {0}) th x, yl hai s l v khi (1),
tr thnh 4k + 3 = 1 4t2 + (1 + 2t) z + (1 2t) z, (2) suy ra z = 2t2 + 2k + 1 z cng l
s nguyn l
Vi mi s nguyn m c dng (1) v vi chnx, y, z nh trn ta c cc kt qu sau
+ t Z\ {0} th c v hn b s xy, yz, zxphn bit tha mn (1)
+ Tch cc s xy, yz, zx l mt s chnh phng l
+Vy ta ch cn chng minh c v hn t Z\ {0} sao cho | xy| , |yz| , |zx | l cc s tt
- Trc ht ta thy | xy| = 4t2 1 l s tt vi t Z\ {0}.
- Chn hai s nguyn t phn bit p, q > m. Ta xt h phng trnh ng d n t
(
1 + 2t p mod p2

 (3)
1 2t q mod q2

Theo nh l thng d Trung Hoa h phng trnh (3)c v s nghim.


- Vi mi t ta c z khng chia ht cho p, ngc li z chia ht cho p th t (2)v (3) ta c p
l c ca 4k + 3 v l v p > m.
- T ta c |zx | chia ht cho p nhng khng chia ht p2 . Tng t |yz| chia ht cho q
nhng khng chia ht q2 . Vy |zx |,|yz|l cc s tt (pcm)
Vy p s ca bi ton l m 3 ( mod 4).
Nhn xt 17. Mt v d kh ca bi vit, bi ton l s kt hp gia nh l thng d Trung
Hoa, vic chn a1 , a2 & m1 , m2 v lp lun logic, s chnh phng. . .

73
Hi tho khoa hc, Hng Yn 25-26/02/2017

V d 30 (Shortlisted IMO 2002). Trong li im nguyn ca mt phng ta Oxy, mt


im c ta l cc s nguyn A ( x; y) Z2 c gi l nhn thy c t im O,nu trn
on OA khng c im no thuc Z2 , tr Ov A. Chng minh rng vi mi s t nhin n
ty , lun tn ti hnh vung n n c cc nh nguyn v mi im nguyn bn trong v
trn bin ca hnh vung u khng nhn thy c t im O.
y
Gii. Ta c nu ( x, y) = d th im M dx ; d l im nguyn thuc on OA vi A ( x; y).
Do A ( x; y) l im nhn thy c t im O khi v ch khi ( x, y) = 1.Gi pi,j l cc s
nguyn t i mt khc nhau, vi 0 i, j n ( c (n + 1)2 s nguyn t nh vy ). Xt hai
h phng trnh ng d tuyn tnh sau y:


x 0 ( mod p0,0 p0,1 p0,2 . . . p0,n )

x 1 ( mod p p p . . . p )
1,0 1,1 1,2 1,n



x n ( mod pn,0 pn,1 pn,2 . . . pn,n )

v

y 0 ( mod p0,0 p0,1 p0,2 . . . p0,n )

y 1 ( mod p1,0 p1,1 p1,2 . . . p1,n )



y n ( mod pn,0 pn,1 pn,2 . . . pn,n )

Theo nh l thng d Trung Hoa th hai h phng trnh trn u c nghimtn ti cc


s T nhin x, y nh vy. M x + i v y + j u chia ht cho pi,j . Do mi im trong hnh
vung n n vi (n + 1)2 im nguyn Ai,j ( x + i ; y + j) trn u khng nh thy c t
im O.
V d 31. (VMO 2013). Tm s cc b sp th t ( a, b, c, a0 , b0 , c0 ) tha mn :


ab + a0 b0 1 ( mod 15)
bc + b0 c0 1 ( mod 15)
ca + c0 a0 1

( mod 15)

Vi a, b, c, a0 , b0 , c0 {0, 1, 2, . . . , 14}.
Gii. Vi mi s nguyn dng k, gi Nk l s b sp th t ( a, b, c, a0 , b0 , c0 ) tha mn iu
kin: ab + a0 b0 bc + b0 c0 ca + c0 a0 1 ( mod k ) v a, b, c, a0 , b0 , c0 {0, 1, 2, . . . , 14}.
Theo nh l thng d Trung Hoa th :Nmn = Nm Nn nu m, n N v (m, n) = 1
Do tnh gi tr ca N15 , ta cn tnh gi tr ca N3 v N5 .
Trc tin ta tnh Np vi mi p l s nguyn t. C nh cc gi tr ( a, b, a0 , b0 ) ca phng
trnh ab + a0 b0 1 ( mod p), ta cn tnh s nghim ca h sau:bc + b0 c0 ca + c0 a0
1 ( mod p) (1)
Ta xt cc trng hp sau.
Nu ( a, a0 ) 6 t (b, b0 ) ( mod p)vi mi t {0, 1, 2, . . . , p 1}. Khi h (1)c mt
0 b0
nghim duy nht : c a0ab 0 ab
b0 a ( mod p ) , c ab0 a0 b ( mod p ).

74
Hi tho khoa hc, Hng Yn 25-26/02/2017

Nu ( a, a0 ) t (b, b0 ) ( mod p)vi mi t 6= 1. Khi h (1) khng c mt nghim


Nu ( a, a0 ) (b, b0 ) ( mod p). Khi h (1) tr thnh mt phng trnh duy nht l:
bc + b0 c0 1 ( mod p)
Do ba + b0 a0 1 ( mod p), ta c th gi s rng b 6= 0.Do vi mi cch chn c0 , ta c
0 0
duy nht mt cch chn c 1bb c ( mod p). iu ny cho thy h (1) c ng p nghim.
t Tp l s b sp th t ( a, b, a0 , b0 ), tha mn ab + a0 b0 1 ( mod p) v a, b, a0 , b0 thuc
tp {0, 1, 2, . . . , p 1}. Vi mi b ( a, a0 ) 6= (0, 0) ,c ng p cp (b, b0 ) tha mn phng
trnh.
Suy ra, Tp = p p2 1 .


t C p l s b sp th t ( a, b), tha mn a2 + b2 t ( mod p) v a, b


{0, 1, 2, . . . , p 1}.
p 1
T lp lun trn ta c. Np = Tp C p (t) + pC p (1) = p p2 1 p2 + C p (0) +

i =1
pC p (1) (2)
Ta d dng tnh c C3 (0) = 1; C3 (1) = 4; C5 (0) = 9, C5 (1) = 4 N3 = 28, N5 = 124
v N15 = 28 124 = 3472. Vy s cc b ( a, b, c, a0 , b0 , c0 ) tha mn iu kin bi l 3472..

Nhn xt 18. Mt v d p dng nh l thng d Trung Hoa kh c bn, ta cn nh v nh


x Phc hi : Z p Zq = Z pq vn l ng dng quan trng nht ca nh l thng d
TrungHoa ny: Mt s thuc Z pq c xc nh mt cch duy nht qua cp s d ca n khi
chia cho p v q.T ta c th chuyn cc bi ton trn Z pq v cc bi ton trn Z p v trn
Zq .

V d 32. Cho n l s nguyn dng l v n > 3. Gi k, t l cc s nguyn dng nh nht


cc s kn + 1 v tn u l s chnh phng. Chng minh rng n l s nguyn t khi v
ch khi min {k, t} > n4
Gii.
+() Gi s n l s nguyn t. Khi n|tn v tn l s chnh phng nn n2 |tn n|t,
iu ny dn n t n > n4 .
. .
Mt khc, t u2 = kn + 1 u2 1 ( mod n) , n (u + 1) ..n (u 1) ..n
u 1 n(do u > 1) kn + 1 (n 1)2 , k n 2 k n4
T hai iu trn ta c min {k, t} > n4 .
+()
Trng hp 1. n ch c mt c s nguyn t. t n = p , ( p , p 3) Nu chn,
ta ly t = 1 < n4 tn = p l s chnh phng mu thun vi gt Nu l 3, ta ly
p
t = p < 4 = n4 tn = p+1 l s chnh phng mu thun vi gt
Do = 1 nn n = p .
Trng hp 2. n c t nht hai c s nguyn t phn bit
Khi n c th biu din di dng n = p m, ( p , p 3, m N , (m, 2) = 1, (m, p) = 1).

75
Hi tho khoa hc, Hng Yn 25-26/02/2017

Theo nh l thng d Trung Hoa tn ti s Z sao cho :


(
s 1 ( mod p )
s 1 ( mod m)

Suy ra n|s2 . Hn na c th chn s Z sao cho |s| n2 . V s 6 1 ( mod m) & s 6


2
1 ( mod p ) Dn n s 6= 1 hay s2 6= 1. By gi ta ly k = s n1 k N , mt khc
2 2 n2
kn + 1 = s2 l s chnh phng v k = s n1 < sn n
4
= n4 , mu thun vi min {k, t} > n4 .
Do trng hp ny khng xy ra. Vy n = p

5 Bi tp tng t


x 1 ( mod 2)

x 2 ( mod 3)
Bi 1. Gii h phng trnh ng d ( vi 0 < x < 120)


x 3 ( mod 4)
x 4 ( mod 5)

Bi 2. Cho cc s nguyn dng n, h, d. Chng minh rng lun tn ti mt cp s cng n s


hng c cng sai d, sao cho mi s hng ca cp s cng u c t nht h c s nguyn t
phn bit.

Bi 3 (Korea MO 1999). Tm tt c cc s t nhin n sao cho 2n 1 chia ht cho 3 v tn ti


m Z sao cho 4m2 + 1 chi ht cho 2n31 .

Bi 4. Cho f ( x ) l a thc vi h s nguyn. Gi s rng c mt tp hu hn cc s nguyn


t A = { p1 , p2 , . . . , pn } sao cho vi mi s nguyn a lun tn ti s pi A sao cho f ( a) chia
ht cho pi . Chng minh rng tn ti mt s nguyn t p sao cho f ( x ) chia ht cho p vi mi
s nguyn x.

Bi 5. Cho cc s nguyn dng a, b. Chng minh rng lun tn ti n s lin tip ca dy s


a + b, a + 2b, a + 3b, . . . , a + nb, . . . l hp s.

Bi 6. Chng minh rng vi mi s nguyn dng n,lun tn ti mt tp hpSgm n phn


t, saocho bt k mt tp con no caScng c tng cc phn t l ly tha ca mt s t
nhin

Bi 7. Chng minh rng vi mi s nguyn dng n, lun tn ti n s lin tip ca dy s


sao cho bt k s no trong dy cng u c c dng 2k 1, vi k l s t nhin.

Bi 8. Chng minh rng khng tn ti a thc f ( x ) vi h s nguyn c bc nguyn dng,


sao cho f (k ) l s nguyn t vi mi s nguyn dng k.

Bi 9 (Czech-Slovak 1997). Chng minh rng tn ti mt dy s tng { an }+
n=1 cc s t nhin

sao cho vi mi k N,dy {k + an }+
n=1 ch cha hu hn cc s nguyn t.

76
Hi tho khoa hc, Hng Yn 25-26/02/2017

Bi 10. Chng minh rng tn ti v hn s nguyn dng a tha mn cc iu kin sau i)


Tn ti x, y Z, ( x, y) = 1 sao cho a2 = x3 + y3 . ii) Tn ti b Z sao cho b2 + 3 chia ht
cho a2 a2 + 3 .

Bi 11. Cho f 1 ( x ) , f 2 ( x ) , . . . , f n ( x ) l n a thc vi h s nguyn khc 0. Chng minh rng


tn ti a thc P ( x ) vi h s nguyn sao cho vi mi i = 1; n ta lun c P ( x ) + f i ( x ) l a
thc bt kh quy trn Z.

Bi 12 (Bulgaria TST 2003). Ta gi mt tp hp cc s nguyn dng C l tt nu vi mi


s nguyn dng k th tn ti a, b khc nhau trong C sao cho ( a + k, b + k ) > 1. Gi s ta c
mt tp tt m tng cc phn t trong bng 2003.Chng minh rng ta c th loi i mt
phn t c trong C sao cho tp cn li vn l tp tt.

Bi 13. Chng minh rng vi mi s nguyn dng n (n 2), lun tn ti hai s nguyn
dng a, b sao cho ( a + i, b + j) > 1, i, j {1, 2, . . . , n 1}.

Bi 14. Ta gi mt hnh vung l hnh vung tt, nu n c 4 nh l cc im nguyn, ng


thi on thng ni tm O vi tt c cc im nguyn trn bin v trong hnh vung cha
t nht mt im nguyn khc hai u mt. Chng minh rng vi mi s nguyn dng n
u tn ti mt hnh vung tt dng n n.
n o
Bi 15. Tm s nguyn dng n sao cho vi mi h thng d thu gn n l a1 , a2 , . . . , a (n)
ta c a1 a2 . . . a (n) 1 ( mod n).

Bi 16 (USA-TST 2009). Chng minh rng tn ti mt dy s tng { an }+ n=1 cc s t nhin
sao cho vi mi n th a1 a2 . . . an 1 l tch ca hai s nguyn lin tip.

Bi 17 (Moldova TST 2009). a) Chng minh rng tp cc s nguyn c th phn hoch thnh
Cc cp s cng vi cng sai khc nhau. b) Chng minh rng tp cc s nguyn khng th
vit c di dng hp ca cc cp s cng vi cng sai i mt nguyn t cng nhau.

Bi 18. Cho s nguyn dng n = p11 p22 . . . pk k , trong p1 , p2 , . . . , pk l cc s nguyn t
i mt khc nhau. Tm s nghim ca phng trnh ng d x2 + x 0 ( mod n)

Bi 19. Cho tp An = { a N| 1 a n, ( a, n) = 1}. Tm | An |.

Bi 20. Cho p l s nguyn t, gi f ( p)l s tt c b sp th t ( a, b, c, a0 , b0 , c0 ) tha mn



0 0
ab + a b x ( mod p)

bc + b0 c0 y ( mod p)
ca + c0 a0 z ( mod p)

Vi a, b, c, a0 , b0 , c0 {0, 1, 2, . . . , p 1} v 0 x, y, z p 1. Tm f ( p).

Bi 21. Tm tt c cc s nguyn dng n tha mn tnh cht sau y: Nu ( x, n) = 1 th


x2 1 ( mod n).
2015
Bi 22. Tn ti hay khng s nguyn dng n n.22 n 81 l s chnh phng.

77
Hi tho khoa hc, Hng Yn 25-26/02/2017

Bi 23. Cho s nguyn dng n. Chng minh rng tn ti s nguyn dng m tha mn h
ng d. (
2m 2015 ( mod 3n )
2m 32015 ( mod 2n )
Bi 24. Ta gi mt s l ly tha ng nu n c dng am , ( a, m Z, m > 1). Vi s nguyn
dng no n th tn ti cc s nguyn b1 , b2 , . . . , bn khng ng thi bng nhau sao cho vi
mi s nguyn dng k s (b1 + k ) (b2 + k) . . . (bn + k )l s ly tha ng
Bi 25. Chng minh rng h phng trnh ng d
(
x a1 (mod m1 )
x a2 (mod m2 )
C nghim khi v ch khi gcd (m1 , m2 ) | ( a1 a2 )
Bi 26. Chng minh rng vi mi s nguyn dng n, lun tn ti n s lin tip ca dy s
sao cho bt k s no trong dy cng u chia ht cho bnh phng ca mt s nguyn t
Bi 27 (Vit Nam TST 2015). Mt s nguyn dng k c tnh cht T (m) nu nh vi mi s
nguyn dng a, tn ti s nguyn dng n sao cho 1k + 2k + 3k + + nk a ( mod m) a)
Tm tt c cc
 s nguyn dng kc tnh cht T (20) b) Tm s nguyn dng knh nht c
tnh cht T 2015

Bi 28 (Saudi Arabia TST 2015). Cho nv k l cc s nguyn dng. Chng minh rng nu n
v 30 nguyn t cng nhau th tn ti cc s nguyn a v b, mi s u nguyn t cng nhau
vi n, sao cho a2 b2 + k chia ht cho n.
Bi 29 (VMO 1997). Chng minh rng vi mi s nguyn dng n, lun tn ti s nguyn
dng k sao cho 19k + 97 chia ht cho 2n .

Ti liu
[1] H Huy Khoi (1997), S hc, NXB Gio dc.
[2] Nguyn Vn Nho, OLYMPIC ton hc Chu Thi Bnh Dng, NXB Gio dc.
[3] B GD v T (2011), Ti liu tp hun gio vin.
[4] Tp ch Ton hc v tui tr, Tuyn tp chuyn .
[5] Dan Brnzei, Ioan Serdean, Vasile Serdean, Junor Balkan Mathematical Olympiads, Math-
ematical Excalibur.
[6] Titu Andreescu, Diophantine Equations. . .
[7] Titu Andreescu, Mathematical Olympiad Treasures, 2004 Birkhauser Boston,USA.
[8] Kenneth H.Rosen, Elementary Number Theory and Its Application

78
Hi tho khoa hc, Hng Yn 25-26/02/2017

L THUYT TH V NG DNG
Hong Phng Anh
THPT Chuyn Hng Yn

Tm tt ni dung

L thuyt th l mt ngnh ton hc hin i, cn rt tr so vi lch s pht trin


hng nghn nm ca nhng chuyn ngnh khc, nhng li c ng dng quan trng trong
nhiu ngnh khoa hc k thut hin i nh: tin hc, cng ngh thng tin, vt l, ha
hc, sinh hc,... Trong khun kh hn hp, khng th trnh by ht c kin thc ca
c mt chuyn ngnh, nn bi vit ny ch cp n nhng khi nim v kt qu c
bn nht, nhm gip ngi c c mt ci nhn tng quan, ng thi trnh by mt s
k thut v ng dng tiu biu ca L thuyt th trong ton hc ph thng.

1 C s l thuyt
1.1 Khi nim
nh ngha 1. Mt th G = (V, E) c hiu l mt b hai tp hp: tp hp nh V v tp hp
cnh E ni cc nh ny vi nhau.

Nu mi cnh ca th l ng ni duy nht gia mt cp hai nh phn bit, ta gi


l th n.
Mt th c gi l th v hng nu tt c cc cnh ca n u l cnh v hng. Mt
th c gi l th c hng nu tt c cc cnh ca n u l cnh c hng.
Trong khun kh bi vit ny, chng ta ch nghin cu cc th n v hng hu hn
nh.

Mt s thut ng khc

1. Hai nh v, w c gi l k nhau nu c mt cnh ni v v w.

2. Bc ca mt nh l s cnh xut pht t nh . Mt nh c gi l nh c lp nu


n c bc bng 0. nh c bc bng 1 gi l nh treo.

3. Mt dy cnh k tip ei = (vi , vi + 1) vi i = 1, 2, ..., m 1 c gi l mt ng i nu


cc nh v1 , v2 , ...vm i mt khc nhau. v1 c gi l nh u v vm c gi l nh
cui ca ng i . S cc cnh ca ng i c gi l di ca ng i.

79
Hi tho khoa hc, Hng Yn 25-26/02/2017

4. Mt dy cnh dng ei = (vi , vi + 1) vi i = 1, 2, ..., m c gi l mt chu trnh nu cc


nh v1 , v2 , ..., vm i mt khc nhau v vm+1 v1 . di ca chu trnh l s cnh ca
chu trnh .

5. Mt th c gi l lin thng nu gia hai nh bt k ca n lun tn ti mt


ng i ni hai nh vi nhau.

6. Cho G = (V, E) l mt th. th b G ca G l mt th vi cng tp nh nh G


v E( G ) = {e
/ E( G )}, ngha l G cha ng cc cnh khng thuc G.

7. th y n nh, k hiu l Kn , l mt th n v hng c n nh v gia hai


nh bt k ca n c ng mt cnh ni.

8. th lng phn l th G = (V, E) m tp nh V c th phn hoch thnh 2 tp


hp V1 V2 sao cho tp cnh E ch gm cc cnh ni 2 nh khng cng mt tp hp.

9. Mt cy l mt th lin thng v hng vi t nht mt nh v khng c chu trnh.

10. Bi l mt th v hng khng c chu trnh.

11. Nhng th c th biu din c trn mt phng sao cho khng c 2 cnh c
biu din no ct nhau c gi l th phng.

12. Cho mt th G = (V, E). Khi mt th G 0 = (V 0 , E0 ) c gi l th con ca


G nu V 0 V, E0 E. th con G 0 = (V 0 , E0 ) c gi l th thnh phn nu nh
mi cnh ca G ni 2 nh ca G 0 cng l cnh ca G 0 .

13. ng i Hamilton l ng i cha mi nh ca th. Chu trnh Hamilton l chu


trnh cha mi nh ca th.

14. ng i Euler l ng i i qua mi cnh ng mt ln. Chu trnh Euler l chu trnh
i qua mi cnh ng mt ln. th Euler l th c chu trnh Euler.

15. Sc s ca th l s nh nht cc mu c th dng t cc nh ca th sao cho


khng c hai nh k nhau no c t cng mt mu.

1.2 Tnh cht


Trc ht, ti xin trnh by mt s nh l c bn m cc bn hc sinh c php s dng
nh kt qu sch gio khoa trong k thi Hc sinh gii Quc gia VMC. y u l cc kt
qu c c qua cc bc chng minh n gin.

nh l 1. Tng s bc ca tt c cc nh trong th gp hai ln s cnh ca th y. T suy


ra, s nh bc l ca mt th lun l s chn.

nh l 2. Trong th n v hng n nh (n Z, n 2), tn ti t nht 2 nh c cng bc.

nh l 3. Nu th G n v hng n nh (n Z, n 2) c ng hai nh cng bc th G phi


c ng mt nh bc 0 hoc c ng mt nh bc n 1.

80
Hi tho khoa hc, Hng Yn 25-26/02/2017

nh l 4. Mi th n v hng hu hn khng lin thng u b phn chia mt cch duy nht


thnh cc thnh phn lin thng.

nh l 5. Nu mi nh ca th G n v hng n nh (n Z, n 0) u c bc khng nh
n
hn th G l th lin thng.
2
nh l 6. th n v hng hu hn l th Euler khi v ch khi hai iu kin sau c ng
thi tha mn
i ) G l th lin thng
ii ) Mi nh ca G u c bc chn.

nh l 7. Nu tt c cc cnh ca mt th n v hng y 6 nh c t bi hai mu th


phi tn ti t nht mt chu trnh n di 3 c tt c cc cnh cng mu.

Ngoi ra, cn c mt s kt qu c bn khc khng ch c nhiu ng dng trong vic gii


cc bi ton, m bn thn k thut chng minh nhng nh l ny cng cho chng ta nhng
cng c hu hiu gii quyt cc bi ton v L thuyt th.

nh l 8. Mt cy bt k vi t nht 2 nh c t nht 2 nh treo.

Chng minh. Trong cy hu hn nh ch c hu hn ng i. Ta xt W = (v1 , v2 , ..., vk ) l


ng i di nht. Khi ta c k > 1.
Hai nh u v cui ca W l v1 v vk l hai nh treo. Tht vy, gi s ngc li, v1 khng
l nh treo th v1 cn c ni vi mt nh u 6= v2 . V W l di nht nn u vi (2 < i k)
no . Nhng nh vy ta s thu c mt chu trnh C = (v1 , v2 , ..., vi , v1 ), tri vi nh
ngha cy. Vy nh l c chng minh.
H qu: th n v hng c bc ca mi nh khng nh hn 2 th c t nht mt chu
trnh.

nh l 9. Mt cy c n nh th c ng n 1 cnh.

Chng minh. Ta chng minh quy np theo s nh n ca cy.


Vi n = 1, r rng cy vi 1 nh khng c cnh no.
Gi s mt cy ty vi n nh c ng n 1 cnh.
Xt G l mt cy c n + 1 nh ty . Theo nh l 8, G c t nht mt nh treo P no . xt
th G { P}.
V P l nh treo, nn th G { P} l mt th lin thng. Tht vy, gi s ngc li,
trong G { P} c hai im A v B khng lin thng vi nhau. M G l th lin thng nn
phi c mt ng i ni A vi B trong G. ng i ny phi i qua P v nhn P l nh
trong ca n. Khi P c bc t nht l 2, mu thun vi gi thit P l nh treo.
Do G khng c chu trnh nn G { P} cng khng c chu trnh. Suy ra G { P} l mt cy
c n nh. Theo gi thit quy np, th G { P} c ng n 1 cnh. Do th G c
ng n cnh v bc ca P trong th G bng 1.

nh l 10. Mt th l lng phn khi v ch khi n khng c chu trnh di l.

81
Hi tho khoa hc, Hng Yn 25-26/02/2017

Chng minh. Gi s G = ( X, Y; E) l mt th lng phn. Khi , dc theo chu trnh bt


k ca G, cc nh thuc tp X v Y ln lt k tip nhau. Do , khi tr v nh xut pht
u tin, ta phi i qua mt s chn cc nh, suy ra s cnh bng s nh ca chu trnh l
mt s chn.
Ngc li, gi s G l mt th m tt c cc chu trnh ca G u c di chn. Ta s
chng minh rng tt c cc thnh phn lin thng ca G u l cc th lng phn, v do
G cng l th lng phn.
Tht vy, gi s G1 l mt thnh phn lin thng ca G v P0 l mt nh ca th G1 , u
tin ta xp P0 vo mt tp Y. Vi mi nh P ca th G1 , ta chn mt ng i W ni nh
P0 vi nh P. Nu ng i W c di chn th ta xp nh P vo tp X, nu W c di
l th ta xp P vo tp Y.
S phn loi cc nh ca th G1 khng ph thuc vo cch chn ng i W. Tht vy,
nu c ng i W c di chn v ng i W 0 c di l cng ni P0 vi P th th
G1 s c chu trnh di l, mu thun vi gi thit ban u.
Vi cch thit lp tp hp X v Y ny, cc nh ca th G1 hoc thuc tp hp X hoc
thuc tp hp Y. By gi ta chng minh G1 ch c cc cnh ni cc nh khng cng tp hp
vi nhau.
Tht vy, gi s c hai nh P v Q k nhau trong th G1 v cng thuc mt tp hp X
hoc Y. Khi , ta c th i t P0 n P, qua Q ri tr v P0 vi mt ng i l cnh. iu
ny khng th xy ra trong G v G ch c chu trnh chn cnh.
Nh vy, G l th lng phn vi hai tp nh X v Y.
nh l 11. Vi mi th G c bc ln nht ( G ), ta c sc s ca G l ( G ) ( G ) + 1.

Chng minh. Ta c mt cch t mu cc nh ca G th sao cho khng c hai nh k


nhau no c t cng mt mu nh sau: Khi ta mun t mu mt nh x th ta t n bi
mt mu khc vi tt c cc mu ca nh k vi n. Cch t ny lun thc hin c, bi v
ta c ( G ) + 1 mu la chn, trong khi x ch c nhiu nht ( G ) nh k vi n.
nh l 12. Ta c th chia cc nh ca mt th G thnh hai nhm sao cho vi mi nh, t nht
mt na s nh k vi n nm nhm kia.

Chng minh. Xt cch chia V thnh V1 V2 c s cnh ni cc nh 2 nhm (mt nh


thuc nhm ny, mt nh thuc nhm kia) l nhiu nht.
Gi s c nh v0 cng nhm vi hn mt na s nh k vi n trong nhm V1 . Khi ,
nu ta chuyn v0 sang nhm V2 v gi nguyn tt c cc im cn li, ta s c cch chia
mi c nhiu cnh ni cc nh 2 nhm hn (mu thun vi gi thit). Vy nh l c
chng minh.

2 ng dng
Bi ton 1. Trong mt hi ngh, mi ngi c t hn hoc bng 3 ngi bn. Hi c th chia
h vo 2 phng sao cho trong mi phng, mi ngi ch c khng qu 1 ngi bn.
Li gii. Ta xt th G vi mi nh i din cho mt ngi trong hi ngh, hai nh c
ni vi nhau bi mt cnh khi v ch khi hai ngi tng ng l bn ca nhau. Khi , ta

82
Hi tho khoa hc, Hng Yn 25-26/02/2017

c mt th vi bc ca mi nh u khng qu 3.
p dng nh l 12, ta c th chia mi ngi thnh hai phng sao cho t nht mt na s
ngi bn ca mi ngi u nm phng kia, hay mi ngi cng phng vi khng qu
mt ngi bn ca mnh.

Bi ton 2 (TOT 1986). Mt gii u c 20 i bng. Ngy u tin mi i u mt trn.


Ngy th hai mi i u mt trn na. Chng minh rng sau 2 ngy, ta c th chn ra 10
i sao cho hai i no cng cha u vi nhau.
Li gii. Coi mi i bng l 1 nh v mi cnh l mt trn u gia 2 i din ra trong
2 ngy. Khi ta thu c mt th G vi mi nh u c bc khng vt qu 2.
Ta chng minh th ny khng c chu trnh l cnh. Tht vy, gi C l mt chu trnh ca
G c di l. V cc i ca C u u ng 2 trn v ch u vi cc i trong C, nn mi
l l
ngy c ng trn gia cc i ca C vi nhau. Suy ra l s t nhin, vy l l s chn.
2 2
Do , G l th lng phn, ngha l ta c th phn hoch G thnh 2 tp hp V1 v V2
sao cho cc cnh u chy t V1 sang V2 , hay cc i trong cng 1 tp i mt cha u vi
nhau.
C 20 i bng nn tn ti 1 tp c t nht 10 i. Vy bi ton c chng minh.

Bi ton 3 (BAMO 2005). t nc Euler c 1000 thnh ph v mt s con ng t gia


chng. Ngi ta c th di chuyn ty t thnh ph ny sang thnh ph khc bng cc con
ng trn. Chng minh rng chnh ph Euler c th lt gch mt s con ng trong s
nhng con ng trn sao cho mi thnh ph c mt s l con ng gch i ra khi n.
Li gii. Coi mi thnh ph l mt nh v mi con ng ni hai thnh ph l mt cnh
gia hai nh tng ng, ta c mt th G lin thng.
Nu G c chu trnh, ta c th xa bt mt cnh thuc chu trnh ny lm mt i chu trnh
m vn m bo tnh lin thng ca G. C lm nh vy n khi khng cn chu trnh no
na, nn ta c th gi s G l mt cy.
G c 1000 nh, c s chn nh bc l (theo nh l 1), nn G cng c s chn nh bc chn.
Gi s G c 2n nh bc chn (n 500). Ta chia 2n nh ny thnh n cp v xt n ng
i ni hai nh trong mt cp vi nhau. Gi W = P1 P2 ... Pn l cch chia cp v ni
ng i c tng di ngn nht, trong Pi l ng i ni cp nh chn th i.
Khi , khng c cnh no xut hin 2 ln ( Pi Pj = , i 6= j). Tht vy, nu tn ti mt
cnh chung AB Pi Pj , ngha l
Pi = ( X1 , X2 , ..., Xm , A, B, Y1 , Y2 , ..., Yn ) c di l ( Pi ) = m + n + 3
Pj = ( Z1 , Z2 , ..., Zs , A, B, T1 , T2 , ..., Tt ) c di l ( Pj ) = s + t + 3
Suy ra l ( Pi ) + l ( Pj ) = m + n + s + t + 6
Ta xy dng ng i mi nh sau
Pi0 = ( X1 , X2 , ..., Xm , A, Zs , ..., Z2 , Z1 ) c di l ( Pi0 ) = m + s + 2
Pj0 = (Yn , ..., Y2 , Y1 , B, T1 , T2 , ..., Tt ) c di l ( Pj0 ) = n + t + 2
Suy ra l ( Pi0 ) + l ( Pj0 ) = m + n + s + t + 4.
Khi , l ( Pi0 ) + l ( Pj0 ) < l ( Pi ) + l ( Pj ), ta thu c cch chia cp v ni ng i mi c tng
di nh hn ca W (mu thun).
Gi ta lt gch tt c cc cnh ca th, tr nhng cnh xut hin trong W th ta s thu
c cch lt gch tha mn bi. Tht vy, khi lt gch nh vy th cc nh ca G c

83
Hi tho khoa hc, Hng Yn 25-26/02/2017

chia thnh cc trng hp sau


+ nh bc l khng nm trn W, tt c cc cnh xut pht t nh ny u c lt gch, ta
c mt s l cnh k n c lt gch.
+ nh bc l nm trn W, c mt s ng i Pi i qua nh ny v nhn im ny lm mt
im nm gia, ng vi mi ng i Pi i qua nh ny th c 2 cnh k n khng c lt
gch, nn s cnh k cn li c lt gch vn l s l.
+ nh bc chn l im u ca Pi no v khng thuc bt k Pj no ( j 6= i ), c mt
cnh xut pht t nh ny khng c lt gch, cn li mt s l cnh k c lt gch.
+ nh bc chn l im u ca Pi v nm trn mt vi Pj no , ng vi mi Pj i qua
nh ny th c 2 cnh k n khng c lt gch, cng vi mt cnh k nm trn Pi cng
khng c lt gch, nn cn li mt s l cnh k c lt gch.
Vy bi ton c chng minh.

Bi ton 4. Trong 1 nhm c mt s ngi quen nhau v mt s ngi khng quen nhau.
Mi bui ti, mt ngi mi tt c ngi quen ca mnh n mt ba tic gii thiu h
vi nhau. Gi s rng sau khi mi ngi t chc t nht mt ba tic m vn cn 2 ngi
cha c gii thiu vi nhau. Chng minh rng 2 ngi s khng c gii thiu ba
tic tip theo.
Li gii. Coi mi ngi l 1 nh v c 2 ngi quen bit nhau th c ni vi nhau bi
1 cnh. Ta chng minh rng, sau khi kt thc qu trnh gii thiu nh trn, tt c cc thnh
phn lin thng ca th u l 1 th con y .
chng minh c khng nh trn, ta ch cn chng minh rng nu th ban u l
lin thng, th th cui cng l y . Tht vy, ta chng minh bng quy np.
+ Vi th c 2 nh, hin nhin ng.
+ Gi v l ngi cui cng s t chc tic, v G 0 l th biu din cc mi quan h quen
bit ngay trc ba tic ca v . Khi G 0 {v} l mt hp ca cc thnh phn lin thng,
mi thnh phn lin thng l mt th y , v v quen vi tt c mi ngi trong 1
th con y (do gi thit quy np). Vy khi v t chc ba tic ca mnh th mi im
c ni li vi nhau thnh 1 th y .
Tr li bi ton, nu c 2 ngi vn cha quen nhau sau qu trnh , ngha l h 2 thnh
phn lin thng khc nhau, do d c t chc thm nhiu ba tic na th 2 ngi cng
s khng c gii thiu vi nhau.

Bi ton 5. Trong mt hi c nhiu hn 6 thnh vin, mi ngi trao i th t vi ng 3


thnh vin khc. Chng minh rng hi ny c th chia thnh hai nhm (khc rng) sao cho
mi ngi trao i th t vi t nht 2 thnh vin trong cng nhm.
Li gii. Coi mi ngi l mt nh, v c 2 ngi trao i th t vi nhau th 2 nh tng
ng ca h s c ni bi mt cnh. Ta thu c th G c bc ca mi nh u l 3,
nn th c t nht mt chu trnh.
Xt chu trnh E l chu trnh c di nh nht. Ta chia nhng ngi thuc chu trnh ny
vo nhm I v nhng ngi cn li vo nhm I I.
T y ta gi bc ni b ca mt nh l s cnh xut pht t nh ti mt im khc
nm trong cng nhm. Hin nhin nhng ngi nhm I trao i th t vi t nht 2 ngi
cng nhm, nn cc nh thuc nhm I u c bc ni b ln hn hoc bng 2.
Gi s c mt ngi X nhm I I nhng li trao i th t vi t hn 2 thnh vin cng

84
Hi tho khoa hc, Hng Yn 25-26/02/2017

nhm, hay ngha l X trao i th t vi t nht 2 ngi A, B trong nhm I.


+ Nu trn mt ng i t A n B trong nhm I c t nht 2 im khc th ta chuyn 2
im sang nhm 2 v chuyn X sang nhm I, ta c nhm I cha mt chu trnh mi c
di nh hn chu trnh E ban u (mu thun).
+ Nu A, B k nhau th A, B, X to thnh mt chu trnh c di l 3, nn chu trnh E cng
c di l 3, gi l 3 im A, B, C.
Ta chuyn X sang nhm I. Khi , trong I c A, B c bc ni b l 3, C v X c bc ni b
l 2. Nu trong I I khng c nh no ni vi 2 nh trong I th ta chia nhm xong. Nu
trong I I c mt nh Y ni vi 2 nh trong I th Y ch c th ni vi C v X, ta chuyn tip
Y sang I. Sau bc ny, trong nhm I ch cn mt im Y c bc ni b l 2, cc im cn li
u c bc 3 trong I nn khng c im no ca nhm I I ni c vi 2 im ca nhm I
na.
+ Nu c 2 ng i ni A vi B trong nhm I u c di l 2, ngha l chu trnh E c
di 4, gi E = ( A, C, B, D, A).
Ta chuyn X sang nhm I th nhm I c 2 nh A, B bc ni b l 3, 3 nh C, D, X c bc ni
b l 2. Gi s cn mt im Y ca I I ni vi hai im trong I, ta chuyn tip Y sang nhm
I, th nhm I c 4 nh bc ni b 3, 2 nh bc ni b 2. Khi , nu khng cn im no
ca I I ni vi 2 nh trong I th nhm I c 6 nh, nhm I I c t nht 1 nh. Nu cn 1 im
Z ni vi 2 nh trong I, chuyn nt Z sang I. V bc ca Z l 3 nn cn 1 nh I I c ni
vi Z, hay I I 6= .
Cui cng, cc nh ca nhm I I u ni vi cc nh ca nhm I ti nhiu nht 1 nh, nn
cc nh u c bc ni b khng nh hn 2, hay mi ngi u trao i th t vi t
nht 2 ngi cng nhm. (pcm)

Bi ton 6. Cho mt nhm gm 12 ngi. Bit rng trong 9 ngi bt k lun tn ti 5 ngi
i mt quen nhau. Chng minh rng trong nhm c t nht 6 ngi i mt quen nhau.
Li gii. Ta t mi ngi tng ng vi mt im, 2 ngi khc nhau c biu din bi
hai im khc nhau. C 2 ngi khng quen nhau ta ni 2 im tng ng vi 2 ngi
bi 1 ng nt lin lm 1 cnh. Khi ta c mt th G l th n, v hng, 12
nh vi tnh cht: trong bt k th con 9 nh no ca G cng tn ti 5 nh i mt
khng k nhau. Ta cn chng minh tn ti 6 nh i mt khng k nhau.
Nu mi chu trnh ca G u c di chn th G l th lng phn, tc l cc nh ca G
c th c phn chia thnh 2 nhm sao cho cc nh trong cng mt nhm i mt khng
k nhau. M G c 12 nh nn c mt nhm c t nht 6 phn t, ta c iu cn chng
minh.
Nu trong G tn ti chu trnh c di l. Gi (C) l chu trnh c di l nh nht trong
G.
+ Trng hp 1. di ca (C) l 3, gi l 3 nh A, B, C.
Nu trong 9 nh cn li khng tn ti 2 nh no k nhau, ta c iu phi chng minh.
Xt trng hp c 2 nh D, E k nhau. Trong 7 nh cn li, ly thm 4 nh ty , cng vi
A, B, C, D, E ta c mt nhm 9 nh, trong phi c 5 nh i mt khng k nhau. Ly
1 nh trong { A, B, C }, 1 nh trong { D, E}, khi phi tn ti t nht 3 nh i mt khng
k nhau trong nhm 4 nh. Do trong nhm 7 nh cha nu tn, nu tn ti 2 cnh bt
k th 2 cnh phi chung 1 nh, suy ra tt c cc cnh phi chung 1 nh, nn 6 nh cn
li (tr nh chung) i mt khng k nhau.

85
Hi tho khoa hc, Hng Yn 25-26/02/2017

+ Trng hp 2. di ca (C) l 5.
Ly thm 4 nh bt k trong s 7 nh cn li, ta c nhm 9 nh, trong tn ti 5 nh
i mt khng k nhau. M trong (C) c 2 nh khng k nhau, nn tn ti t nht 3 nh
i mt khng k nhau trong s 4 nh ty t nhm 7 nh cn li. Chng minh tng t
trng hp di (C) l 3, ta cng thu c 6 nh i mt khng k nhau.
+ Trng hp 3. di ca (C) l 7.
Ly thm 2 nh bt k c nhm 9 nh. M trong (C) c 3 nh khng k nhau, nn 2
nh ly thm khng k nhau. Suy ra 5 nh cn li i mt khng k nhau.
Ta chng minh trong 7 nh ca (C) tn ti mt nh khng k vi c 5 nh kia. Gi s nh
no trong 7 nh ca (C) cng k vi 1 nh trong 5 nh cn li. Khi , theo nguyn l
Dirichlet, tn ti mt nh X trong nhm 5 nh k vi 2 trong 7 nh ca (C), gi s l A v
B. Suy ra A v B phi cng k chung mt nh nm trn chu trnh, gi s l nh C. (V nu
khng th ta s c mt chu trnh mi cha A, X, B c di l v nh hn 7).
Xt chu trnh 7 nh mi (thay C bng X), chng minh tng t ta c 5 nh cn li cng i
mt khng k nhau (mu thun vi gi thit phn chng l C k vi mt nh no trong
5 nh kia m khng phi l X).
+ Trng hp 4. di ca (C) l 9.
Trng hp ny khng th xy ra, v trong 9 nh khng tn ti 5 nh i mt khng k
nhau.
+ Trng hp 5. di ca (C) l 11. Gi im nm ngoi (C) l M.
Trn (C) c ti a 2 nh cng k mt nh v cng k nhau vi M, v nu c hn th tn ti
chu trnh l cnh c di nh hn 11.
Xt 5 nh i mt khng k nhau ca (C) v cng khng k vi M, ta c 6 nh i mt
khng k nhau.
Vy bi ton c chng minh.

Bi ton 7 (IMO 2007). Trong 1 cuc thi Ton, mt s th sinh l bn b ca nhau. Mt nhm
th sinh c gi l "mt hi" nu 2 ngi bt k trong nhm u l bn ca nhau. S
thnh vin ca 1 hi c gi l kch thc ca hi . Bit rng trong cuc thi ny, kch
thc ln nht ca cc hi (kch thc ca hi c s thnh vin nhiu nht) l s chn. Chng
minh rng c th sp xp cc th sinh vo 2 phng sao cho kch thc ln nht ca cc hi
trong phng ny bng kch thc ln nht ca cc hi trong phng kia.
Li gii. Gi C l hi c s thnh vin ln nht v c 2n ngi.
u tin, ta chia tt c nhng ngi C vo phng I v tt c nhng ngi cn li vo phng
II. Mi ln ta chuyn 1 ngi t phng I sang phng II.
Ch rng vi mi ln chuyn nh vy, kch thc ln nht ca cc hi trong phng I gim
i 1, cn kch thc ln nht ca cc hi trong II tng ln 1 hoc gi nguyn. Do , ta c th
chuyn n khi kch thc ln nht ca cc hi trong 2 phng bng nhau, hoc kch thc
ln nht phng I hn phng II l 1 n v m nu chuyn bt k ngi no sang cng u
lm kch thc ln nht ca cc hi phng II tng ln.
Nu trng hp u tin xy ra th bi ton c chng minh.
Ta xt trng hp th hai:
Gi s khi trong phng I cn li 1 hi k + 1 ngi (k n), v hi ln nht trong phng II
c k ngi.
Nu bt k ngi no phng I chuyn sang phng II u to thnh 1 hi k + 1 ngi, ngha

86
Hi tho khoa hc, Hng Yn 25-26/02/2017

l mi ngi phng I u quen bit y (quen bit vi tt c mi ngi trong hi) vi


1 hi k ngi no phng II.
Xt 1 ngi I1 thuc phng I quen y vi 1 hi D c k ngi ca phng II.
Gi s k ngi ny u quen vi mi ngi trong phng I. Khi ta c 1 hi 2k + 1 ngi
trong cuc thi (mu thun vi tnh ln nht ca C).
Nn tn ti 1 ngi J trong hi D ny khng quen vi 1 ngi I2 no trong phng I.
Khi chuyn I1 sang phng II v chuyn J sang phng I, ta c mi phng u c hi ln
nht c k thnh vin.

Bi ton 8. Trn mt hn o n c 10 thnh ph. C 2 hng hng khng iu khin tt c


cc chuyn bay gia cc thnh ph ny. Gia hai thnh ph bt k u c ng 1 chuyn
bay (c 2 chiu). Chng minh rng 1 hng hng khng c th cung cp 2 ng bay khp
kn sao cho mi ng bay qua 1 s l thnh ph v 2 ng bay khng i qua chung 1
thnh ph no.
Li gii. Coi mi thnh ph l 1 nh, mi chuyn bay gia 2 thnh ph l 1 cnh gia 2
nh tng ng, ta c 1 th y K10 .
Ta t mu cc cnh biu din ng bay t hng hng khng th nht mu xanh, cc cnh
biu din ng bay t hng hng khng cn li mu .
Ta cn chng minh th y K10 2 mu c cha 2 chu trnh l cng mu v khng giao
nhau.
Ta s dng nh l 7 v b sau

B 1. Nu cc cnh ca 1 th y K5 c t bi 2 mu v th khng cha 1 tam


gic n mu (tt c cc cnh cng mu) no, th n cha 2 chu trnh n mu c di 5.
Chng minh. Gi v1 , v2 ,..., v5 l cc nh ca K5 .
Nu t 1 nh ca th c 3 cnh cng mu th ta s c 1 tam gic n mu.
C th, gi s v1 v2 , v1 v3 , v1 v4 cng mu . Khi nu 1 trong 3 cnh v2 v3 , v3 v4 , v2 v4 c
mu th ta c mt tam gic mu , nu khng ta c tam gic v2 v3 v4 mu xanh.
V khng c tam gic n mu no trong K5 nn t mi nh c ng 2 cnh mu v 2
cnh mu xanh.
Tm thi khng xt n cc cnh mu xanh, ta s c mt th con 5 nh vi mi nh c
bc l 2. Nn th ny phi l mt chu trnh hoc c th chia thnh mt s chu trnh khng
giao nhau. Nhng v mi chu trnh c t nht 3 nh, m th ny ch c 5 nh nn khng
th chia thnh cc chu trnh khng giao nhau c. Do ta c mt chu trnh 5 cnh mu
.
Chng minh tng t, ta cng c mt chu trnh 5 cnh mu xanh.
Quay tr li bi ton, gi v1 , v2 , ..., v10 l cc nh ca th y K10 2 mu. T nh l 7,
c mt tam gic n mu trong K10 , gi s l v1 v2 v3 . Xt th con K10 {v1 , v2 , v3 } vi 7
nh, ta li c mt tam gic n mu, gi s l v4 v5 v6 . Nu v1 v2 v3 v v4 v5 v6 cng mu th bi
ton c chng minh. Nu khng, gi s v1 v2 v3 mu xanh v v4 v5 v6 mu . Xt 9 cnh
vi v j vi 1 i 3, 4 j 6, theo nguyn l Dirichlet, c 5 cnh cng mu. Khng mt tnh
tng qut, ta gi s 5 cnh cng mu xanh. Do c 1 im v j0 (4 j0 6) sao cho 2
trong 3 cnh v1 v j0 , v2 v j0 , v3 v j0 cng mu xanh. Khi ta c 1 tam gic mu v 1 tam gic
mu xanh vi ng 1 im chung v j0 .
n gin, ta t tn li cc im sao cho v1 v2 v3 mu xanh v v3 v4 v5 mu . Xt th

87
Hi tho khoa hc, Hng Yn 25-26/02/2017

con y K10 {v1 , v2 , v3 , v4 , v5 } = K5 c 5 nh. Nu trong K5 c mt tam gic n mu,


tam gic ny s cng mu vi mt trong hai tam gic v1 v2 v3 v v3 v4 v5 . Nu trong K5 khng
c tam gic no n mu, t B suy ra K5 c chu trnh 5 cnh n mu. Vy bi ton c
chng minh.

Bi ton 9 (USAMO 2007). Ta gi mi hnh lin thng gm cc trn li vung l mt con


vt. Mt con khng long l mt con vt gm nhiu hn hoc bng 2007 . Mt con khng
long khng tch c thnh t 2 con khng long khc c gi l mt con khng long
nguyn s. Tm s nhiu nht c th c ca mt con khng long nguyn s.
Li gii. Xt th G = (V, E) vi mi nh l mt ca con khng long, mi cnh ca
th l mt ng ni gia 2 im c 2 tng ng k nhau. Khi , G l th lin thng
v mi nh c bc khng vt qu 4.
Con khng long l nguyn s khi v ch khi th G biu din n khng chia c thnh 2
th con lin thng v c nhiu hn hoc bng 2007 nh.
Ta s s dng b sau

B 2. Xt th G lin thng n nh c l bc cao nht ca cc nh. Khi , G c th


n1
chia thnh 2 th con lin thng m mi phn c nhiu hn hoc bng nh.

Chng minh. T th G lin thng ta lun c th xa bt mt s cnh thu c mt
cy. Do ta ch cn chng minh b cho cy T nhn c t G.
Nu chn 1 nh x lm gc ca cy th t mi cnh xut pht t x ta c mt nhnh ca cy.
Gi f ( x ) l s nh ca nhnh c nhiu nh nht ca cy gc x. Xt cch chn x f ( x ) t
gi tr nh nht.
n1
V t x c nhiu nht nhnh cy, nn f ( x ) .

n
Ta chng minh f ( x ) .
2
n
Tht vy, gi s f ( x ) > . Gi A l th thnh phn cha tt c cc nh ca nhnh c
2
nhiu nh nht ca x, nn A c f ( x ) nh. Gi a l mt nh thuc A m k vi x.

88
Hi tho khoa hc, Hng Yn 25-26/02/2017

Xt cy gc a. Khi , cc nhnh ca a nm trong A hin nhin c s nh nh hn f ( x ).


n
Nhnh cn li cha x c s nh khng vt qu n f ( x ) < . Suy ra f ( a) < f ( x ) (mu
2
thun vi cch chn x).
n
Do f ( x ) . Gi B l th thnh phn cha tt c cc nh ngoi A, th s nh ca B
2
n n1
ln hn , v cng ln hn nh.
2
n1
Vy lun chia c G thnh 2 th con lin thng, mi phn nhiu hn hoc bng

nh.
Tr li bi ton, nu G c s nh l n > 2006.4 + 1 = 8025 nh. Theo b , G c th chia
thnh 2 th con lin thng m mi phn c s nh ln lt l n1 , n2 vi

n1 2016.4
n1 , n2 > = 2006 n1 , n2 2007
4 4
Hay mi phn l mt con khng long.
Do G l mt con khng long nguyn s th n 8025.
Trng hp du "=" xy ra, ta xt mt cy 8025 nh gc x vi 4 nhnh, mi nhnh 2006 nh.

th ny khng th chia thnh 2 con khng long nguyn s.


Vy s nhiu nht c th c ca 1 con khng long nguyn s l 8025 .

Ti liu
[1] on Qunh, H V Anh, Phm Khc Ban, Vn Nh Cng, V nh Ha, Ti liu
chuyn ton - Hnh hc 12, NXB Gio dc, 2012.

[2] V nh Ha, Mt s kin thc c s v Graph hu hn, NXB Gio dc, 2004.

[3] Po-Shen Loh, Graph Theory, 2008.

89
Hi tho khoa hc, Hng Yn 25-26/02/2017

PHNG PHP O HM TRONG CHNG MINH


BT NG THC

Phm Vn Dng - Hong Th Nhung


THPT Chuyn Hng Yn

Cng vi cc phng php quy np ton hc, phng php phn chng, phng php
s dng bt ng thc kinh in th phng php s dng o hm cng l mt phn kin
thc quan trng khng th thiu trong nhiu bi ton i s cng nh trong bt ng thc.
N thc s l mt cng c hiu qu v c ng dng rng ri trong gii ton, cng l mt
phng php chun mc nht khi ta gp phi cc bt ng thc thng thng.

1 Kin thc c bn
nh l 1. Cho hm s y = f ( x ) xc nh v lin tc trn [ a, b].
*) Nu f ( x ) 0, x [ a, b] th f ( x ) ng bin trn [ a, b] v khi ta c
min f ( x ) = f ( a); max f ( x ) = f (b)
x [ a,b] x [ a,b]

*) Nu f ( x ) 0, x [ a; b] th f ( x ) nghch bin trn [ a; b] v khi ta c


min f ( x ) = f (b); max f ( x ) = f ( a)
x [ a,b] x [ a,b]

nh l 2 (nh l Fermart). Gi s hm s y = f ( x ) xc nh trn mt ln cn b ca


x0 [ a; b] v c o hm ti im x0 . Khi nu hm s y = f ( x ) t cc tr ti x0 th f 0 ( x0 ) = 0.
nh l 3 (iu kin hm s c cc tr). Cho hm s y = f ( x ) xc nh trn [ a; b] v x0 .
Trong mt ln cn b ca x0 , nu f 0 ( x0 ) thay i du khi x qua x0 (c th khng tn ti f 0 ( x0 ))
th f ( x ) t cc tr ti x0 .
*) Nu f 0 ( x ) < 0, x [ x0 ; x0 ] v f 0 ( x ) > 0, x [ x0 ; x0 + ] th x0 l im cc tiu.
*) Nu f 0 ( x ) > 0, x [ x0 ; x0 ] v f 0 ( x ) < 0, x [ x0 ; x0 + ] th x0 l im cc i.

nh l 4. Gi s y = f ( x ) xc nh trn [ a; b] v x0 [ a; b]. Trong mt ln cn b ca x0 ,


hm s y = f ( x ) c o hm cp hai lin tc, ng thi f 0 ( x0 ) = 0 v f 00 ( x ) 6= 0 th x0 l mt im
cc tr ca hm s.
*) Nu f 0 ( x0 ) = 0 v f 00 ( x ) > 0 th x0 l mt im cc tiu ca hm s.
*) Nu f 0 ( x0 = 0 v f ( x ) < 0 th x0 l mt im cc i ca hm s.

90
Hi tho khoa hc, Hng Yn 25-26/02/2017

nh l 5. Nu f l hm li trn on [ a, b] th ta c f ( x ) max { f ( a), f (b)} vi mi x [ a, b],


hay ni mt cch khc, gi tr ln nht ca hm f trn on [ a, b] s t c ti hai u mt ca
on [ a, b]. Tng t, nu f l hm lm trn on [ a, b] th gi tr nh nht ca n trn on [ a, b]
s t c ti hai u mt ca on [ a, b].

2 ng dng o hm trong bt ng thc v bi ton cc tr

2.1 Bt ng thc mt bin s.


Dng 1: Kho st trc tip hm s tm tp gi tr ca hm s.

Bi ton 1 (HBK H Ni, 1997). Cho tam gic ABC c ba gc tha mn A > B > C. Tm
gi tr nh nht ca hm s
r r
x sin A x sin B
f (x) = + 1
x sin C x sin C

Li gii. Ta c A > B > C tc l sin A > sin B > sin C. (1)


x sin A


0
Hm s xc nh khi v ch khi xx sin C
sin B hay x < sin C hoc x > sin A. (2)

0
x sin C
Ta c r r
sin A sin C x sin C sin B sin C x sin C
f 0 (x) = 2
. + 2
. .
2( x sin C ) x sin A 2( x sin C ) x sin B
Do (1) nn f 0 ( x ) > 0, x tha mn (2).
Ta c bng bin thin

r
sin A sin B
Vy min f ( x ) = f (sin A) = - 1.
sin A sin C
Ch : T kt qu trn ta suy ra c phng trnh

x sin A + x sin B = x sin C

91
Hi tho khoa hc, Hng Yn 25-26/02/2017

c ng mt nghim trn [sin A; +). V hm s f ( x ) ng bin c f (sin A) < 0 (do


0 < sin A sin B < sin A sin C ).

Bi ton 2 (B tuyn sinh i hc). Cho s nguyn dng n. Chng minh rng
1
xn . 1 x < , x (0; 1).
2n.e

Li gii. Ta c
1 1
xn . 1 x < x2n (1 x ) < (3)
2n.e 2ne
Xt hm s f ( x ) = x2n (1 x ) vi x (0; 1). Ta c

f 0 ( x ) = x2n1 [2n (2n + 1) x ]

Nn ta c bng bin thin

(2n)2n
Vy max g( x ) = .
(0;1) (2n + 1)2n+1
Ta chng minh

(2n)2n 1 2n 2n+1 1
2n + 1
< ( ) <
(2n + 1) 2ne 2n + 1 e
2n + 1 2n+1
( ) > e (2n + 1)[ln(2n + 1) ln(2n)] > 1
2n
1
ln(2n + 1) ln(2n) > (4)
2n + 1

p dng nh l Lagrange cho hm s f ( x ) = ln x trn [2n; 2n + 1] suy ra tn ti c


f (2n + 1) f (2n)
[2n; 2n + 1] sao cho f 0 ( x ) = . Suy ra
2n + 1 2n
1 1
ln(2n + 1) ln(2n) = > (5)
e 2n + 1

T (3); (4) v (5) suy ra iu phi chng minh.

92
Hi tho khoa hc, Hng Yn 25-26/02/2017

Bi tp tng t
Bi 1 (Ton hoc v tui tr). Cho s nguyn dng n. Chng minh rng
k =1
1
k (2k 1)
< 2ln2.
n

Dng 2: S dng tnh n iu


Trong mt s bi ton c th phi o hm nhiu ln lin tip thm ch phi kho st thm
hm s ph. Ta thng s dng

f ( x ) ng bin trn [ a; b] th f ( x ) > f ( a) vi mi x > a.


f ( x ) nghch bin trn [ a; b] th f ( x ) > f (b) vi mi x < b.
x3
Bi ton 3. Chng minh rng vi mi x > 0 ta c x < sin x < x.
6
x3
Li gii. Xt hm s f ( x ) = x + + sin x trn [0; +). Ta c
6
x2
f 0 (x) = 1 + cos x; f 00 ( x ) = x sin x; f 000 ( x ) = 1 cos x.
2
000 00 00
Ta c f ( x ) = 1 cos x 0, x [0; +) f ( x ) f (0) = 0, nn f 0 ( x ) ng bin
trn [0; +). Suy ra f 0 ( x ) f 0 (0) = 0 f ( x ) ng bin trn [0; +). Do
f ( x ) f (0) = 0, x 0; f ( x ) > f (0) = 0, x > 0.
Tc l
x3 x3
x + sin x > 0 x < sin x, x > 0. (6)
6 6
Lu f 00 ( x ) > f 00 (0) = 0 vi x > 0 ta c sin x < x. (7)
T (6) v (7) ta c iu phi chng minh.

Bi ton 4. Tm gi tr nh nht ca hm s f ( x ) = x2 + x + 1 + x2 x + 1.

Li gii. TX: R. Xt hm s f ( x ) = x2 + x + 1 + x2 x + 1 trn R . Ta c
1 1
0
x+ x
f ( x ) = s 2 + s 2
2 2
1 3 1 3
x+ + x +
2 4 2 4
1 1
x+ x
= s 2
s 2 
2 2
1 3 1 3
x+ + x +
2 4 2 4
   
1 1
= g x+ g x
2 2

93
Hi tho khoa hc, Hng Yn 25-26/02/2017

t
vi mi x, trong g(t) = r , t R. V hm g ng bin trn R nn
3
t2 +
4

f 0 (x) = 0 x = 0

1 1
f 0 ( x ) > 0 g( x + ) > g( x ) x > 0
2 2
Ta c bng bin thin

T bng bin thin suy ra min f ( x ) = 2 x = 0.

Bi tp tng t
x 1 2
Bi 2. Chng minh rng x 1x + x 1x > , x (0; 1).
e
Dng 3: Kt hp vi cc bt ng thc khc nh bt ng thc AM - GM, bt ng thc
Cauchy - Schwarz, bt ng thc Chebyshes,...


Bi ton 5. Chng minh rng nu 0 x < th
2

2sin x + 2tan x 2x+1 .



Li gii. p dng bt ng thc AM - GM ta c 2sin x + 2tan x 2. 2sin x .2tan x . Ta chng
minh
2. 2sin x .2tan x 2x+1 2sin x+tan x 22x sin x + tan x 2x
h i
Xt hm s f ( x ) = sin x + tan x 2x lin tc trn 0; , ta c
2
0 1 2 1 h i
f ( x ) = cos x + 2 > cos x + 2 0, x 0;
cos2 x cos2 x 2
h i 1
(v vi x 0; th cos x > cos2 x v theo bt ng thc AM - GM ta c cos2 x + 2)
2 h i cos2 x
Do f ( x ) ng bin trn 0; .
2

94
Hi tho khoa hc, Hng Yn 25-26/02/2017

i h
Suy ra f ( x ) f (0) = 0, hay sin x + tan x 2x, x 0;
2
y l iu phi chng minh.

Bi ton 6 (Olympic 30 - 4- 1990). Chng minh rng


 3
sin x  
> cos x, x 0; .
x 2

Li gii. Ta bin i
3
sin3 x

sin x
> cos x > cos x sin2 x. tan x x3 > 0.
x x3
h i
2 3
Xt hm s f ( x ) = sin x. tan x x vi x 0; . Ta c
2
0
f ( x ) = 2 sin2 x + tan2 x 3x2 .

t g( x ) = 2 sin x + tan x 3x, x 0; ), th
2 r
0 1 1 1
g ( x ) = 2 cos x + 2
3 = cos x + cos x + 2
3 3 3 cos2 x. 2 3 = 0 vi mi
cos x cos x cos x

x [0; ), nn g( x ) ng bin trn [0; ). Suy ra
2 2

g( x ) g(0) = 0, x [0; ).
2
0 1 2 2
h i
Do f ( x ) (3x ) 3x = 0 nn f ( x ) ng bin trn 0; . Suy ra
3 2
h i
f ( x ) > f (0) = 0, x 0; .
2
Nhn xt 1. Khi trong bt ng thc c cha cc loi hm s khc nhau ta thng c lp
mi loi hm s d xt du ca o hm, hoc ta c th o hm lin tip kh bt mt
loi hm s nh trong bi ton 3.
x3
Cch 2: Theo bi ton 3 ta c x < sin x, suy ra
6
3
sin x 3 x2 x2 x4 x6 x2 x4
  
> 1 = 1 + > 1 + . (8)
x 6 2 12 216 2 24
Ta chng minh c
x2 x4
cos x < 1 + . (9)
2 24
T (8) v (9) ta c iu phi chng minh.

95
Hi tho khoa hc, Hng Yn 25-26/02/2017

Bi ton 7 (VMO - 2003, Bng B). Cho hm s f xc nh trn tp s thc, ly gi tr trn R


v tha mn iu kin

f (cot x ) = sin 2x + cos 2x, x (0; ).

Hy tm gi tr ln nht v gi tr nh nht ca hm s g( x ) = f (sin2 x ). f (cos2 x ) trn R.


Li gii. Ta c
f (cot x ) = sin 2x + cos 2x, x (0; )
2 cot x cot2 x 1 cot2 x + 2 cot x 1
f (cot x ) = + = , x (0; ).
cot2 x + 1 cot2 x + 1 cot2 x + 1
T vi ch rng vi mi t R u tn ti x (0; ) sao cho cot x = t ta c

t2 + 2t 1
f (t) = , t R.
t2 + 1
Dn ti
sin4 2x + 32 sin2 2x 32
g( x ) = f (sin2 x ). f (cos2 x ) = , x R. (10)
sin4 2x 8 sin2 2x + 32
 
1 1
t u = sin 2x. D thy khi x chy qua R th u chy qua 0;
2
. V vy t (10) ta c
4 4
min g( x ) = min h(u)
x R 1
x [0; ]
4
v
max g( x ) = max h(u).
x R 1
x [0; ]
4
u2 + 8u 2
trong , h(u) = . Ta c
u2 2u + 2

0 2(5u2 + 4u + 6)
h (u) = .
(u2 2u + 2)2

1 1
D dng chng minh c h0 (u) > 0, u [0; ]. Suy ra hm h(u) ng bin trn [0; ].
4 4
1 1 1
V vy trn [0; ] ta c min h(u) = h(0) = 1 v max h(u) = h( ) = .
4 4 25
1
Vy min g( x ) = 1, t c chng hn khi x = 0 v max g(u) = , t c chng hn
25

khi x = .
4

Bi tp tng t
3x +2
Bi 3. Chng minh rng 4sin x + 2tan x 2 2 , x [0; ).
2

96
Hi tho khoa hc, Hng Yn 25-26/02/2017

2.2 Bt ng thc c nhiu bin s


chng minh bt ng thc c nhiu bin s bng phng php o hm th iu
quan trng nht l chng ta a c v hm s mt bin v kho st hm s theo bin .
Dng 1: Kho st hm c trng

Bi ton 8. Cho A, B, C l ba gc ca mt tam gic nhn. Chng minh rng



tan A + tan B + tan C + 6(sin A + sin B + sin C ) 12 3.

Li gii. Xt hm s f ( x ) = tan x + 6 sin x 7x vi (0; ).
2
Ta c
1 (cos x 1)(3 cos x + 1)(2 cos x 1)
f 0 (x) = 2
+ 6 cos x 7 = .
cos x cos2 x

V x (0; ) nn
2

f 0 ( x ) = 0 2cosx 1 = 0 x = .
3

Lp bng bin thin ca f ( x ) trn (0; ) ta c
2
7
min f ( x ) = f ( ) = 4 3
3 3
(0; )
2
p dng vo bi ton ta c
7
f ( A ) + f ( B ) + f ( C ) 3(4 3 )
3

tan A + tan B + tan C + 6(sin A + sin B + sin C ) 12 3.
Nhn xt 2. Trong bt ng thc trn A, B, C bnh ng nn ta d dng kim tra c du

bng xy ra khi v ch khi A = B = C = . V vy ta cn chn mt hm s c dng
3
f ( x ) = tan x + 6 sin x + kx

sao cho f 0 ( ) = 0. Do k = 7 v ta tm c hm c trng cn xt.
3

Bi ton 9 (i hc Quc gia H Ni, 2000). Cho a + b + c = 0. Chng minh rng

8 a + 8b + 8c 2 a + 2b + 2c .

Li gii. Xt hm s f ( x ) = (2x )3 2x 2x ln 2 trn R.


Ta c f 0 ( x ) = 3.(2x )2 . ln 2 2x . ln 2 2 ln 2 = (2x 1)(3.2x + 2) ln 2

97
Hi tho khoa hc, Hng Yn 25-26/02/2017

v f 0 ( x ) = 0 2x = 1 x = 0.
Ta c bng bin thin
Suy ra
f ( x ) 0; x R f ( a) + f (b) + f (c) 0.
Do , 8a + 8b + 8c (2a + 2b + 2c ) 2( a + b + c) ln 2 0.
Suy ra 8a + 8b + 8c 2a + 2b + 2c .
Nhn xt 3. Trong mt s bi ton bt ng thc hai bin ta bin i c lp mi bin v mt
v, khi xut hin hm c trng cn kho st.

Bi ton 10 (i hc khi D, 2006). Chng minh rng


 b  a
a 1 b 1
2 + a 2 + b , a b > 0.
2 2

Li gii. Ta c !a
 b  a  a b b
1 1 1 + 4 1 + 4
2a + a 2b + b
2 2 2a 2b
a
(1 + 4a )b 1 + 4b ln(1 + 4a )b ln(1 + 4b ) a


ln(1 + 4a ) ln(1 + 4b )

a b
ln(1 + 4x )
Xt hm s f ( x ) = vi x > 0. Ta c
x
4x ln 4x (1 + 4x ) ln(1 + 4x )
f 0 (x) = < 0,
x 2 (1 + 4 x )

nn f l hm nghch bin trn (0; +). Do f ( a) f (b) (pcm).


Nhn xt 4. Trong bi ton trn ta s dng kt qu:
Cho hm s f ( x ) ng bin trn ( a; b). Khi vi , ( a; b) ta c

< f ( ) < f ( ).

Bi ton 11 (i hc khi A, 2004). Cho tam gic ABC khng t, tha mn iu kin

cos 2A + 2 2 cos B + 2 2 cos C = 3. (11)

Tnh cc gc ca tam gic ABC.

98
Hi tho khoa hc, Hng Yn 25-26/02/2017

A 1
Li gii. T gi thit 0 < A suy ra 0 < sin . Ta c
2 2 2

(11) 1 2 sin2 A + 2 2(cos B + cos C ) = 3
A BC
sin2 A + 2 2 sin . cos = 1. (12)
2 2
A BC A
Li c 2 2 sin . cos 2 2 sin (13)
2 2 2
2
A
T (12) v (13) ta c sin A + 2 2 sin 1 0.
2
A 2 2

t t = sin th 4t (1 t ) + 2 2t 1 0 (14).
2
1
Xt hm f (t) = 4t4 4t2 + 2 2t 1. Ta phi c f (t) 0 vi 0 < t .
5
0 00 1
Tnh f (t) = 16t3 8t + 2 2; f (t) = 48t2 8 suy ra f 00 (t) = 0 t = .
6
00 1 00 1 1
Do , f (t) < 0 0 < t < v f (t) > 0 < t < .
6 6 2
0 0 1 1
Suy ra f (t) f ( ) > 0 nn f (t) ng bin trn (0; ]. Do
6 2
1
f (t) f ( ) = 0
2

ng thc xy ra khi v ch khi

1 A 1
t = sin = A = .
2 2 2 2

BC
Thay vo ta c cos = 1 suy ra B = C = .
2 4
Bi ton 12 (VMO- 2004). Cho cc s thc dng x, y, z tha mn iu kin

( x + y + z)3 = 32xyz.

x 4 + y4 + z4
Tm gi tr nh nht v gi tr ln nht ca biu thc P = .
( x + y + z )4
Li gii. Nhn xt rng vi l mt s thc dng ty , ta lun c

P( x, y, z) = P(x, y, z)

v nu x, y, z tha mn iu kin ca bi th x, y, z cng tha mn iu kin . V th


khng mt tng qut, ta c th gi s x + y + z = 4. Khi , kt hp vi iu kin bi ta
c xyz = 2.

99
Hi tho khoa hc, Hng Yn 25-26/02/2017

1
Bi ton tr thnh: Tm gi tr nh nht v gi tr ln nht ca bi ton P = ( x 4 + y4 + z4 ).
256
Khi cc bin s dng x, y, z thay i sao cho

x+y+z = 4
xyz = 2
t Q = x4 + y4 + z4 , Q l biu thc i xng i vi 3 bin x, y, z nn biu din c qua
cc biu thc i xng c bn v t = xy + yz + zx; ta c
Q = ( x 2 + y2 + z2 )2 2( x 2 y2 + y2 z2 + z2 x 2 )
= (42 2t)2 + 2[t2 2xyz( x + y + z)]
= 2t2 64t + 44 + 32
= 2(t2 32t + 144) (15)
Khi i bin ta cn tm tp xc nh cho bin mi (chnh l tp gi tr ca t(x, y, z)).
2
T cc iu kin i vi x, y, z ta c: y + z = 4 x v yz = . (16)
x
2
Do , t = x (4 x ) = (17).
x
p dng bt ng thc AM - GM cho hai s dng y, z t (16) ta c:
8
(4 x )2 x3 8x2 + 16x 8 0
x

( x 2)( x2 6x + 4) 0 3 5 x 2; ( x (0; 4))

Xt hm s t, c xc nh bi (17), trn on [3 5; 2], ta c

0 2( x 2)( x2 x 1) 5 51
t (x) = 5t .
x2 2

5 51
V hm s f (t) = t2 32t + 144 nghch bin trn (0; 16) v [5; ] (0; 16) nn
2
5 5 1 383 165 5
min f (t) = f = v max f (t) = f (5) = 9.
2 2
Kt hp (15) ta c min Q = 338 165 5 v max Q = 18.
9
V vy, min P = , t c khi x = 2, y = z = 1.
128
Ch
Xt bng bin thin ca t( x ) nh gi t = xy + yz + zx nh sau.
T nh gica x, do tnh ixng vi x, y, z ca cc biu thc x + y + z = 4 v xyz = 2,
suy ra: 3 5 y 2 v 3 5 z 2.
Do , ( x 2
)(y 2)(z 2) 0 (18)
v ( x (3 5))(y (3 5))(z (3 5)) 0 (19)
Ta
 c h (18) v (19) tng ng vi h bt phng trnh
xyz 2t + 4( x + y + z) 8 0
xyz (3 5)t + (3 5) ( x + y + z) (3 5)2 0
2

51

10 2t 0 5
5t .
8 5 14 (3 5) 0 2

100
Hi tho khoa hc, Hng Yn 25-26/02/2017

Bi tp tng t
Cho a, b, c l ba s dng tha mn a2 + b2 + c2 = 1. Chng minh rng

a b c 3 3
+ + .
1 a2 1 b2 1 c2 2
Dng 2:Kt hp vi cc bt ng thc khc nh bt ng thc AM - GM, bt ng thc
Cauchy - Schwarz, bt ng thc Chebyshes,...
Ta thng c lng T ( x, y, z, ...) bi mt hm s ch ph thuc vo mt bin s, t kho
st hm s ny t c mc ch.
i vi cc bi ton phc tp, ta cn phi hp vi phng php chn khong cc bin hoc
cc bt ng thc ph khc nh bt ng thc AM - GM, bt ng thc Cauchy - Schwarz,
bt ng thc Chebyshes,... hoc cc nh gi khc, hoc phi hp vi cc phng php
khc nh phng php ta ,...

Bi ton 13 (Tuyn sinh i hc Vinh, 2001). Chng minh rng nu a, b, c l di ba cnh


ca mt tam gic c chu vi bng 3 th
3a2 + 3b2 + 3c2 + 4abc 13.

Li gii. t T = 3a2 + 3b2 + 3c2 + 4abc. Do vai tr ca a, b, c bnh ng nn khng gim


tng qut ta c th gi s 0 < a b c.
T a + b + c = 3 v a + b > c suy ra
3
1c< .
2
Ta bin i
T = 3( a2 + b2 ) + 3c2 + 4abc = 3[( a + b)2 2ab] + 3c2 + 4abc
= 3(3 c)2 + 3c2 2ab(3 2c).
 2
a+b
Do 2 3c > 0 v ab . (20)
2
Suy ra
1
T 3(3 c)2 + 3c2 ( a + b)2 (3 2c)
2
1
= 3(c2 6c + 9) + 3c2 (3 c)2 (3 2c)
2
3 27
= c3 c2 + = f ( c ).
2 2
0 3
Ta c f (c) = 3c2 3c, nn f (c) ng bin trn [1; ]. V vy
2
T f (c) f (1) = 13.
ng thi T = 13 c = 1. Vi gi thit 0 < a b c v a + b + c = 3 v (20) suy ra
a = b = 1, tc l tam gic ABC u.

101
Hi tho khoa hc, Hng Yn 25-26/02/2017

Bi ton 14 (Tuyn sinh i hc khi A, 2011). Cho x, y, z l cc s thc thuc on [1; 4] v


x y, x z.
Tm gi tr nh nht ca biu thc
x y z
P= + + .
2x + 3y y + z z + x

Li gii. Trc ht ta chng minh vi mi a, b dng, ab 1 th


1 1 2
+ . (21)
1+a 1+b 1 + ab

Tht vy, ta c (21) ( ab 1)( a b)2 0 lun ng do a, b dng v ab 1.
Du bng xy ra khi v ch khi a = b hoc ab = 1.
p dng (21) vi x, y thuc on [1; 4] v x y, x z ta c

x 1 1 1 1
P= + x + x +
3y x
r
2x + 3y 1 + 1+
y z 2+ 1+
x y
z x x
Du bng xy ra khi v ch khi = hoc = 1 (22)
y z y
x t2 2
r
t = t, t [1; 2], khi P 2 + .
y 2t + 3 1 + t
t2 2
Xt hm f (t) = 2 + , t [1; 2] c
2t + 3 1 + t
2[t3 (4 + 3) + 3t(2t 1) + 9]
f 0 (t) = <0
(2t2 + 3)2 (1 + t)2
34
Suy ra f (t) f (2) = .
33
x
Du bng xy ra khi v ch khi t = 2 = 4 x = 4; y = 1 (23)
y
34
Do P .
33
T (22) v (23) suy ra du bng xy ra khi v ch khi x = 9; y = 1, z = 2.

Bi tp tng t
Bi 4 ( thi chn HSG QGTHPT bng B, 1999). Xt phng trnh ax3 x2 + bx 1 = 0 vi
a, b l cc s thc, a 6= 0, a 6= b sao cho cc nghim u l cc s thc dng. Tm gi tr nh
5a2 3ab + 2
nht ca P = .
a2 ( b a )
Dng 3: Kho st hm s theo tng bin
i vi cc bt ng thc nhiu bin, ta c th chn mt bin l bin s bin thin v c
nh cc bin cn li, bi ton lc ny tr thnh bt ng thc mt bin.

102
Hi tho khoa hc, Hng Yn 25-26/02/2017

Bi ton 15. Chng minh rng

2( x3 + y3 + z3 ) ( x2 y + y2 z + z2 x ) 3, x, y, z [0; 1].

Li gii. Bt ng thc cho tng ng vi


f ( x ) = 2x3 yx2 z2 x + 2(y3 + z3 ) y2 z 3.
0 0
Ta c f ( x ) = 6x2 2yz z2 v f ( x ) = 0.
1 p 1 p
Do , x = x1 = (y y2 + 6z2 ) hoc x = x2 = (y + y + 6z2 ).
6 6
V x 0 nn x1 / (0; 1).
Xt hai trng hp
0
+ Nu x2 / (0; 1) f ( x ) 0, x [0; 1].
Suy ra f ( x ) gim trn [0; 1].
Do max f ( x ) = max{ f (0), f (1)}.
x [0;1]
+ Nu x2 (0; 1) th ta c bng bin thin.

T bng bin thin suy ra max f ( x ) = max { f (0), f (1)} .


x [0;1]
Nh vy trong c hai trng hp ta u c max f ( x ) = max { f (0), f (1)} .
x [0;1]
Mt khc

f (0) = 2( y3 + z3 ) y2 z 2( y3 + z3 ) y2 z + (2 y z2 ) = f (1).

Ta s chng minh f (1) 3. Tht vy, t

f (1) = g ( y ) = 2( y3 + z3 ) y2 z + (2 y z2 )

Ta c g0 (y) = 6y2 2zy 1 = 0.


1 1
Do , y = y1 = (z z2 + 6) < 0 hoc y = y2 = (z + z2 + 6)
6 6
+ Nu y2 / (0; 1) thi g0 (y) 0, y [0; 1]. Suy ra g(y) gim trn [0; 1]. Do

max g(y) = max { g(0), g(1)} .


y[0;1]

+ Nu y2 (0; 1) th ta c bng bin thin

T bng bin thin suy ra max g(y) = max { g(0), g(1)} . Nh vy trong c hai trng hp
y[0;1]

103
Hi tho khoa hc, Hng Yn 25-26/02/2017

ta u c max g(y) = max { g(0), g(1)}.


y[0;1]
Ta c
g(0) = 2z3 + 2 z2 2z3 + 2 z2 + (1 z) = g(1) = z(z 1)(2z + 1) + 3 3 vi mi
z [0; 1].

Bi ton 16 ( thi HSG THPT ton quc bng A, 1999). Xt cc s thc dng a, b, c tha
mn abc + a + c = b. Tm gi tr ln nht ca biu thc
2 2 3
P= 2 + 2 .
a2 +1 b +1 c +1
1 a+c
Li gii. Bin i gi thit thnh a + c = b(1 ac) > 0, suy ra a < , b = (24).
c 1 ac
Thay (24) vo biu thc P v bin i c

2 3 2( a + c )2
P= + + 2. (25)
a2 + 1 c2 + 1 (1 + a2 )(1 + b2 )

1 2( x + c )2 1
Xt hm s f ( x ) = P = 2
+ 2 2
vi 0 < x < v coi c l tham s (c > 0).
x + 1 (1 + x )(1 + c ) 2
Ta c
2c( x2 + 2cx 1)
f 0 (x) =
(1 + c2 )(1 + x2 )2
1 1
Trn (0; ) th f 0 ( x ) = 0 c nghim duy nht l x0 = c + c2 + 1 (26) vi 0 < x0 < . Qua
c c
x0 th f 0 ( x ) i du t dng sang m nn f ( x ) t cc i ti x0 nn
c
f ( x ) f ( x0 ) = 1 +
c2 + 1
T theo (25) ta c
3 2c 3
P = 2 f (x) 2 + + 2 = g ( c ).
c2 +1 c2 + 1 c + 1
Xt hm s g(c) vi c > 0. Ta c

2(1 8c2 )
g0 (c) = .
(c2 + 1)(3c + c2 + 1)

104
Hi tho khoa hc, Hng Yn 25-26/02/2017

1
Vi c > 0, th g0 (c) = 0 ti c = v qua c th g0 (c) i du t dng sang m nn g(c) l
8
1 10
gi tr cc i, suy ra P g( ) = .
8 3
10 1 1
Gi tr P = t c khi c = , a = , b = 2 theo (24) v (26).
3 8 2

Bi tp tng t
Bi 5. Xt cc s thc dng x, y, z tha mn h iu kin


0<zyx3

3 2
+ 2 1

xy y

18 4 18
2 + 2 + 2 3


x y y z z x
1 80 18
Hy tm gi tr ln nht ca biu thc P( x, y, z) = xyz + x3 + y3 .
2 27 8

Ti liu
[1] Trn Phng, V p Bt ng thc trong cc k thi Olympic Ton hc, NXB HQGHN 2010.

[2] Trn Phng, Nhng vin kim cng trong bt ng thc Ton hc, NXB HQGHN 2009.

[3] Phm Kim Hng, Sng to bt ng thc, NXB HQGHN 2006.

105
Hi tho khoa hc, Hng Yn 25-26/02/2017

MT S NG DNG CA NH L L AGRANGE
Phm Vn Dng - Hong Th Minh Thy
THPT Chuyn Hng Yn

1 nh l Lagrange
nh l 1 (nh l Lagrange). Nu hm s f ( x ) lin tc trn [ a; b] v c o hm trn khong
( a; b) th tn ti c ( a; b) sao cho

f (b) f ( a)
f 0 (c) = .
ba

ngha hnh hc
nh l ny khng nh vi gi thit ca hm s f (nu trn), th lun lun tn ti t nht
mt im thuc th y = f ( x ) m ti im tip tuyn song song vi ng thng ni
hai im u v cui ca th (nh hnh v minh ha).

Ta thng dng cc h qu sau y ca nh l Lagrange.

H qu 1. Nu hm s f ( x ) lin tc trn [ a; b] v c o hm trn khong ( a; b), ngoi ra


f ( a) = f (b) th tn ti c ( a; b) sao cho f 0 (c) = 0.
c bit, nu hm f tha mn nh l trn, ng thi f ( a) = f (b) = 0 th gia hai nghim
ca phng trnh f ( x ) = 0 c t nht mt nghim ca f 0 ( x ) = 0.

H qu 2. Nu hm s f ( x ) c o hm trn [ a, b] v phng trnh f 0 ( x ) = 0 c duy nht


nghim trn on y, th trn [ a, b] phng trnh f ( x ) khng th c qu hai nghim.
Chng minh. Gi thit phn chng f ( x ) = 0 c qu hai nghim v do ta c th gi s
phng trnh y c qu ba nghim (v nu n c nhiu nghim hn na th lp lun khng
thay i).
Gi x1 , x2 , x3 ( a x1 < x2 < x3 b) l ba nghim y.
Theo h qu 1, tn ti c1 , c2 ( x1 < c1 < c2 , x3 ) sao cho

f 0 (c1 ) = f 0 (c2 ) = 0. (1)

ng thc (1) chng t rng c1 , c2 l hai nghim phn bit ca phng trnh f 0 ( x ) = 0 trn
on [ a, b]. iu v l chng t gi thit phn chng l sai (pcm).

105
Hi tho khoa hc, Hng Yn 25-26/02/2017

H qu 3. Nu hm f ( x ) lin tc trn [ a, b] v f 0 ( x ) = 0 x ( a, b).


Khi f ( x ) const, x [ a, b].

Chng minh. Ly xa0 ty m a < xa0 b. p dng nh l Lagrange trn [ a, xa0 ], ta thy
tn ti c, a < c < xa0 sao m

f ( xa0 ) f ( a) = ( xa0 a) f 0 (c).

Do f 0 (c) = 0 suy ra
f ( xa0 ) = f ( a). (2)
ng thc (2) ng vi mi xa0 m a < xa0 b, v chnh l iu phi chng minh.

2 Mt s ng dng
2.1 Chng minh s tn ti nghim ca phng trnh
V d 1. Cho a0 , a1 , . . . , an l cc s thc v tha mn iu kin sau

a1 a2 an a 22 a 23 a n 2n
a0 + + ++ = a0 + a1 + 2 + 3 + + = 0.
2 3 n+1 3 4 n+1
Chng minh rng phng trnh

a1 + 2a2 x + 3a3 x2 + + nan x n1 = 0

c t nht mt nghim thuc khong (0; 2).

Li gii. Xt hm s

1 1 1
f ( x ) = a0 x + a1 x 2 + a2 x 3 + + a n x n +1 .
2 3 n+1
Ta c
a1 a2 an
f (1) = a0 + + ++ ,
2 3 n+1
a2 22 a 23 a n 2n
f (2) = 2a0 + 2a1 + 2. + 2 3 + + 2.
3 4 n+1
2 3 a n 2n
 
a2 2 a3 2
= 2 a0 + a1 + + ++ ,
3 4 n+1

v th t gi thit suy ra
f (0) = f (1) = f (2) = 0.
p dng nh l Rolle, ta thy tn ti c1 , c2 (0 < c1 < 1 < c2 < 2) sao cho

f 0 (c1 ) = f 0 (c2 ) = 0. (3)

106
Hi tho khoa hc, Hng Yn 25-26/02/2017

T (3) v li p dng nh l Rolle vi hm f 0 ( x ), ta thy tn ti , c1 < < c2 sao cho

f 0 () = 0. (4)

Do (c1 , c2 ) nn (0; 2).


Li thy

f 0 ( x ) = a0 + a1 x + a2 x2 + a3 x3 + + an x n f 0 ( x ) = a1 + 2a2 x + 3a3 x2 + + nan x n1 .

Vy t (4) suy ra l nghim ca phng trnh

a1 + 2a2 x + 3a3 x2 + + nan x n1 = 0.

l iu phi chng minh.

V d 2. Cho P( x ) l a thc bc n c n nghim thc phn bit x1 , x2 , . . . , xn . Chng minh


rng
n P( x )
j
P0 (x j ) = 0.
j =1

Li gii. T gi thit, ta c th vit P( x ) di dng sau

P( x ) = a( x x1 )( x x2 ) . . . ( x xn )

vi a 6= 0.
Suy ra  
0 1 1 1
P ( x ) = P( x ) + ++ .
x x1 x x2 x xn
Do P( x1 ) = P( x2 ) = = P( xn ) = 0, nn theo nh l Rolle phng trnh P0 ( x ) = 0 c
n 1 nghim phn bit y1 , y2 , . . . , yn1 vi
x 1 < y 1 < x 2 < y 2 < x 3 < < y n 1 < x n .
(Theo nh l Rolle th phng trnh P0 ( x ) = 0 c t nht n 1 nghim y1 , y2 , . . . , yn1 , mt
khc do deg P0 ( x ) = n 1, vy phng trnh y c ng n - 1 nghim nh vy).
V th P0 ( x ) c th vit li di dng sau y.

P0 ( x ) = b( x y1 )( x y2 ) . . . ( x yn1 )

vi b 6= 0.
Suy ra  
00 0 1 1 1
P (x) = P (x) + ++ . (5)
x y1 x y2 x y n 1
Theo (5) ta c

1 1 1
P0 (yk ) = P(yk )( + ++ ) = 0k = 1, n 1.
y k x1 y k x2 yk xn

107
Hi tho khoa hc, Hng Yn 25-26/02/2017

Do P(yk ) 6= 0, nn suy ra
1 1 1
+ ++ = 0, k = 1, n 1. (6)
y k x1 y k x2 yk xn
T (5) v (6) suy ra
P00 ( x j ) 1 1
= + + , k = 1, n 1. (7)
P0 ( x j ) x j y1 x j y n 1

Cng tng v n 1 ng thc dng (7), ta c


n P00 ( x j ) n
1 1 1
P 0 ( x j ) ( x j y 1 + x j y n + + x j y n 1
=
j =1 j =1
n 1
1 1 1
= ( y k x1 + y k x2 + + y k x n ). (8)
j =1

By gi t (3) v (8) suy ra


n P( x j )
P0 ( x j )
= 0.
j =1

l iu phi chng minh.


V d 3. Cho s thc dng m v cc s thc a, b, c tha mn iu kin
a b c
+ + = 0.
m+2 m+1 m
Chng minh rng phng trnh ax2 + bx + c = 0 c nghim.

Li gii. Nu a = 0 th hin nhin kt lun ca bi ton ng. Nu a 6= 0, xt hm s

ax m+2 bx m+1 cx m
f (x) = + + .
m+2 m+1 m
Khi f l mt hm s lin tc trn [0, 1], c o hm trn (0,1) v


f 0 ( x ) = ax m+1 + bx m + cx m1
= x m1 ( ax2 + bx + c)



f (0) = 0
a b c


f (1) = + + = 0.


m+2 m+1 m
Theo nh l Lagrange tn ti x0 thuc khong (0;1) sao cho
f (1) f (0) = f 0 ( x0 )(1 0) f 0 ( x0 ) = 0
ax02 + bx0 + c = 0.

Do phng trnh ax2 + bx + c = 0 c nghim trn (0; 1).

108
Hi tho khoa hc, Hng Yn 25-26/02/2017

V d 4 (nh l Cauchy). Nu cc hm s f ( x ), g( x ) l cc hm s lin tc trn [ a; b], c o


hm trn khong ( a; b) v g0 ( x ) khc khng trn khong ( a; b) th tn ti c ( a; b) sao cho

f 0 (c) f (b) f ( a)
0
= .
g (c) g(b) g( a)

Li gii. Theo nh l Lagrange lun tn ti x0 ( a; b) sao cho

g(b) g( a)
g 0 ( x0 ) =
b a)

f (b) f ( a)
suy ra g( a) 6= g(b). Xt hm s F ( x ) = f ( x ) g( x ). Khi , F ( x ) l hm lin tc
g(b) g( a)
trn [ a; b], c o hm trn khong ( a; b) v

f ( a) g(b) f (b) g( a)
F ( a) = F (b) = .
g(b) g( a)

Theo nh l Rolle tn ti c ( a; b) sao cho F 0 (c) = 0. M

f (b) f ( a) 0
F0 (x) = f 0 (x) g ( x ),
g(b) g( a)

f (b) f ( a) 0
suy ra f 0 (c) = g ( c ).
g(b) g( a)
Nhn xt 1. nh l Lagrange l h qu ca nh l Cauchy (trong trng hp g( x ) = x ).

V d 5. Cho a thc P( x ) v Q( x ) = aP( x ) + bP0 ( x ) trong a, b l cc s thc, a 6= 0.


Chng minh rng nu Q( x ) v nghim th P( x ) v nghim.

Li gii. Ta c degP( x ) = degQ( x ). V Q( x ) v nghim nn degP( x )chn.


Gi s P( x ) c nghim, v degP( x ) chn nn P( x ) c t nht hai nghim.
+) Khi P( x ) c nghim kp x = x0 ta c x0 cng l mt nghim ca P0 ( x ) suy ra Q( x ) c
nghim.
+) Khi P( x ) c hai nghim phn bit x1 < x2 .
Nu b = 0 th hin nhin Q( x ) c nghim.
a
Nu b 6= 0 : Xt f ( x ) = e b x P( x ), th f ( x ) c hai nghim phn bit x1 < x2 v

a a a 1 a 1 a
f 0 ( x ) = e b x P( x ) + e b x P0 ( x ) = e b x ( aP( x ) + bP0 ( x )) = e b x P( x ).
b b b
V f ( x ) c hai nghim suy ra f 0 ( x ) c t nht mt nghim hay Q( x ) c nghim.

109
Hi tho khoa hc, Hng Yn 25-26/02/2017

2.2 Chng minh bt ng thc


V d 6. Cho 0 < a < b. Chng minh rng

ba b ba
< ln < .
b a a
1
Li gii. Xt hm s f ( x ) = ln x, x [ a, b]. Ta c f 0 ( x ) = . Theo nh l Lagrange tn ti c
x
thuc khong ( a, b) sao cho

ba
f (b) f ( a) = f 0 (c)(b a) ln b ln a =
c
b ba
ln = .
a c
1 1 1
M 0 < a < c < b nn < < , do
b c a
ba ba ba
< < .
b c a
ba b ba
Suy ra < ln < .
b a a
V d 7. Chng minh rng
  x +1  x
1 1
1+ > 1+ , x > 0.
x+1 x

Li gii. Xt hm s
 
1 t
f (t) = t ln 1 + , g(t) = ln(1 + t) .
t 1+t

Khi f c o hm trn (0; +) v g c o hm trn (1; +), v


 
0 1 1

f (t) = ln 1 +
t 1+t

0 1 1 t
g (t) = 1 + t = .


2
(1 + t ) (1 + t )2
 
1
Theo nh l Lagrange tn ti c1 ( x, x + 1) v c2 0, sao cho
c1
 
0 1
f ( x + 1) f ( x ) = f (c1 )( x + 1 x ) = g


    c1
1 1
g g (0) = g 0 ( c2 ) 0


c1 c1

110
Hi tho khoa hc, Hng Yn 25-26/02/2017

suy ra
 
1
f ( x + 1) f ( x ) = g


  c1
1
g > g (0) = 0


c1

  x +1  x
1 1
nn f ( x + 1) f ( x ) > 0 hay ln 1 + > ln 1 +
x+1 x
  x +1  x
1 1
suy ra 1 + > 1+ .
x+1 x
l iu phi chng minh.

V d 8. Cho 0 < a < b < c < d. Chng minh rng


r r
3 abc + abd + acd + bcd ab + ac + ad + bc + bd + cd
< .
4 6

Li gii. Xt hm s f ( x ) = ( x a)( x b)( x c)( x d). Khi f lin tc v c o hm


trn R v
f ( a) = f (b) = f (c) = f (d) = 0.
Theo nh l Rolle trn tng on [ a, b], [b, c], [c, d] tn ti x1 , x2 , x3 sao cho a < x1 < b <
x2 < c < x3 < d v
f 0 ( x1 ) = f 0 ( x2 ) = f 0 ( x3 ) = 0.
Suy ra x1 , x2 , x3 l nghim ca phng trnh

f 0 ( x ) = ( x b)( x c)( x d) + ( x a)( x c)( x d) + ( x a)( x b)( x d)+


+ ( x a)( x b( x c)
= 4x3 3( a + b + c + d) x2 + 2( ab + ac + ad + bc + bd + cd) x
( abc + abd + acd + bdc) = 0

V d 9. Cho x (0; 1), n N. Chng minh rng:


1
xn 1 x < .
2ne

Li gii. Theo bt ng thc Cauchy:


2n+1
x + x + + x + (2n 2nx )

x . . . x.(2n 2nx )
2n + 1

(c 2n ch s x)

111
Hi tho khoa hc, Hng Yn 25-26/02/2017

Suy ra
 2n+1
2n 2n
2nx (1 x ) . (1)
2n + 1

Mt khc, xt hm s
f ( x ) = ln x, x [2n, 2n + 1],
1
th f 0 ( x ) = . Theo nh l Lagrange c (2n, 2n + 1) sao cho
x
f (2n + 1) f (2n) = f 0 (c)(2n + 1 2n)
2n + 1 1 1 2n 1
ln = > ln >
2n c 2n + 1 2n + 1 2n + 1
1
2n 1 2n
ln < < e 2n + 1
2n + 1 2n + 1 2n + 1
 2n+1
2n
< e 1 . (2 )
2n + 1

T (1) v (2) suy ra


1
2nx2n (1 x ) <
e
suy ra
1
xn 1 x < .
2ne

2.3 Tm gii hn ca dy s
V d 10. Cho dy s thc un xc nh bi

u1 = 2017
(
1
= ln 1 + u2n 2018, n 1 (1)

u n +1
2
Chng minh rng dy s un c gii hn hu hn.

1
Li gii. Xt hm s f ( x ) = ln 1 + u2n 2018 vi x R, f ( x ) l hm s lin tc trn R,

2
0 x
v ta c f ( x ) = suy ra
1 + x2

0 x 1
1 + x2 2 , x R.
f ( x ) =

Gi s rng un c gii hn l a th a l nghim ca phng trnh

1  
x= ln 1 + x2 2018. (13)
2

112
Hi tho khoa hc, Hng Yn 25-26/02/2017

Ta i chng minh (13) c nghim duy nht. Tht vy


1   1  
x= ln 1 + x2 2018 g( x ) = x + 2018 ln 1 + x2 = 0. (14)
2 2
Ta c hm s g( x ) l hm s lin tc v

x2 x + 1
g0 ( x ) = > 0, x R.
x2 + 1
Suy ra g( x ) ng bin trn R. Mt khc
1  
g(0).g(2018) = 2018. ln 1 + 20182 .
2
Suy ra phng trnh (14) c nghim duy nht. Gi nghim l a.
Theo nh l Lagrange, tn ti c R sao cho
  n 1
1 1
|un+1 a| = | f (un ) f ( a)| = f 0 (c) |un a| |un a|

| u1 a | .
2 2

Nh th ta c   n 1
1
0 < | u n +1 a | < | u1 a | ,
2
m   n 1
1
lim |u1 a| = 0,
n+ 2

nn theo nguyn l kp ta suy ra

lim |un+1 a| = 0 lim (un+1 a) = 0 lim un = lim un+1 = a.


n+ n+ n+ n+

Vy dy s un c gii hn hu hn khi lim n +.


V d 11. Cho dy s thc un xc nh bi

u1 = 2017


un
u n +1 = 3 + p 2 , n 1 (1)
un 1

Hy tm lim un .
n+

Li gii.
Bng quy np chng minh c un > 3 vi mi n = 1, 2,. . .
Gi s rng (un ) c gii hn l a th a 3 v a l nghim ca phng trnh a = 3 +
a a2
( a 3)2 = 2 ( a2 3a)2 2( a2 3a) 3 = 0
a2 1 a 1
2 2
a 3a = 1 hoc a 3a = 3

113
Hi tho khoa hc, Hng Yn 25-26/02/2017

3 + 15
a=
2
x
Xt hm s f ( x ) = 3 + trn ( 3, +), th un+1 = f (un ) v f ( a) = a. ta c f 0 ( x ) =
x2 1
1
p suy ra
( x 2 1)3
0 1
f ( x ) < , x ( 3; +).
2 2
Xt hiu sau y v kt hp vi nh l Lagrange ta suy ra:
|un+1 a| = | f (un ) f ( a)| = | f 0 (cn )(un a)| = | f 0 (cn )0 | |un a| (cn (un ; a) cn
( a; un ))
1 1
| u n a | . . . ( ) n | u1 a |
2 2 2 2
Nh th ta c
1
0 < | u n +1 a | < ( ) n | u 1 a |
2 2
m  n
1
lim | u1 a | = 0
n+ 2 2

nn theo nguyn l kp ta suy ra

lim |un+1 a| = 0
n+

hay lim (un+1 a) = 0 nn lim un = lim un+1 = a.


n+ n+ n+

3+ 15
Vy dy s un c gii hn hu hn khi n + v lim un = .
n+ 2
1 n1
V d 12 (VMO 2002). Xt phng trnh = vi n l s nguyn dng.
i =1 1 2 i2 x
a) Chng minh rng vi mi s nguyn dng n, phng trnh nu trn c mt nghim duy
nht ln hn 1; k hiu nghim l xn .
b) Chng minh rng lim xn = 4.

Cch gii.
n 1 1
a) Xt f n ( x ) = , ta c f n ( x ) lin tc v nghch bin trn (1; +).
i =1 i2 x
1 2
1
M lim f n ( x ) = +, lim f n ( x ) = nn f n ( x ) = 0 c mt nghim duy nht ln hn 1.
x 1+ x + 2
b) Vi mi s nguyn dng n, ta c
1
f n (4) = < 0 nn f n (4) < f n ( xn ) xn < 4.
2(2n + 1)
Theo nh l Lagrange, lun tn ti cn ( xn ; 4) tha mn:

f n (4) f n ( xn ) = f 0 (cn )(4 xn ).

114
Hi tho khoa hc, Hng Yn 25-26/02/2017

1 9
M f 0 (cn ) < nn 4 xn < 9( f n (4) f n ( xn )) 4 xn < ,
9 2(2n + 1)
9
suy ra 4 < xn < 4 nn lim xn = 4.
2(2n + 1)

V d 13 (VMO 2007). Cho s thc a > 2 v f n ( x ) = a10 x10+n + x n + x n1 + + x + 1.


a) Chng minh rng vi mi s nguyn dng n, phng trnh f n ( x ) = a lun c ng mt
nghim dng duy nht. K hiu nghim l xn .
a1
b) Chng minh rng dy xn c gii hn bng khi n dn n v cng.
a

Li gii.
t Fn ( x ) = f n ( x ) a, ta c Fn ( x ) lin tc, ng bin trn [0; +) v Fn (0) = 1 a, Fn (1) =
a10 + n + 1 a > 0. Suy ra phng trnh f n ( x ) = a lun c ng mt nghim xn dng duy
nht.
a1
t b = th f n (b) = bn ( a a)[( a 1)9 1] + a nn f n (b) > a, suy ra b > xn , n N .
a
Theo nh l Lagrange, lun tn ti cn ( xn ; b) tha mn:

f n (b) f n ( xn ) = f 0 (cn )(b xn ).

M f 0 (cn ) > 1 nn b xn < f n (b) f n ( xn ) = bn ( a 1)[( a 1)9 1] lim xn = b.


suy ra b bn ( a 1)[( a 1)9 1] < xn < b nn lim xn = b (v b (0; 1)).

Ti liu
[1] V Giang Giai, V Khc Thng, L Quang Tun, ng dng cc tnh cht hm s gii
bi ton, NXB Thanh Ha, 2002.

[2] Phan Huy Khi, Cc bi ton v hm s, NXB H Ni, 2000.

[3] on Qunh, Trn Nam Dng, Nguyn V Lng, ng Hng Thng, Ti liu chuyn
ton i s v gii tch 11, NXB Gio dc, 2012.

115
Hi tho khoa hc, Hng Yn 25-26/02/2017

PHNG PHP LNG GIC GII PHNG TRNH


A THC BC CAO

Mng Thanh Hng


THPT Sn Dng, Tuyn Quang

Tm tt ni dung
Trong bo co ny trnh by mt s h thc i s c xut x t cc php bin i
lng gic, t cho p dng kho st mt s dng phng trnh a thc nhiu n v
a thc mt bin bc cao.

1 Mt s ng nht thc dng i s - lng gic


Nhn xt rng ng thc c bn dn n s phong ph ca h thng cc ng
nht thc lng gic l cng thc

sin2 t + cos2 t = 1, t R. (1.1)

Gn vi h thc (1.1) l ng nht thc Lagrange

(2x )2 + (1 x2 )2 = (1 + x2 )2 , x R. (1.2)

Hai cng thc (ng nht thc) (1.1) v (1.2) l hai cch vit ca mt h thc. Nu ta
t
thay x = tan vo (1.2) th d dng thu c (1.1) v ngc li.
2
Nh vy l mi cng thc lng gic s tng ng vi mt ng nht thc i s
tng ng. Tuy nhin, vi s lng cc cng thc bin i lng gic qu nhiu, bn
thn cc h thc lng gic to thnh mt chuyn c tnh c lp tng i, dn tch
hn c s i s ca n, lm cho chng ta qun i mt lng ln cc h thc i s c
cng xut s t mt h thc lng gic quen bit. c bit, trong chng trnh ton bc
ph thng hin nay, cc hm s lng gic ngc, hm lng gic hyperbolic,... khng
nm trong phn kin thc bt buc th nhng bi ton lin quan n chng s l mt
thch thc ln i vi hc sinh.
Ta nhc li cng thc Euler quen bit

ei = cos + i sin , R.

Khi
ei + ei

cos

= ,
2
i
e e i
sin = .

2i

116
Hi tho khoa hc, Hng Yn 25-26/02/2017

1 1
R rng khi kho st hm s cos t th t ai ngh trong u rng n c dng a+ v
2 a
khi a khng cn l mt s thc. Nhng nu ta ch n biu thc

e + e
, R,
2
th chnh l cos(i) (= cosh ) v v vy, v mt hnh thc, ta s c nhiu bin i thu
c t cc cng thc lin quan n bin x 6 [1, 1] ging nh cng thc i vi hm
cos t (xem [2]).

V d 1. H thc i s ng vi cng thc

cos 3t = 4 cos3 t 3 cos t

chnh l cng thc

1 3 1 h1 1 i3 h 1  1 i
a + 3 =4 a+ 3 a+ ,
2 a 2 a 2 a
hay
1 3 1
4x3 3x = a + 3
2 a
vi
1 1
x= a+ , a 6= 0.
2 a
V d 2. H thc i s ng vi cng thc

cos 5t + cos t = 2 cos 3t cos 2t

chnh l cng thc

1 5 1  1 1 h1 1 ih 1  2 1 i
a + 5 + a+ =2 a3 + 3 a + 2 .
2 a 2 a 2 a 2 a
S dng kt qu khai trin cc hm lng gic cos 3t v cos 2t, ta thu c ng nht
thc i s dng a thc bc 5

1 5 1
a + 5 = m + 2(4m3 3m)(2m2 1),
2 a
trong
1 1
m= a+ .
2 a

By gi ta chuyn sang xt cc h thc i s lin quan n hm s sin t. T cng thc


Euler, ta thu c h thc
eit eit
i sin t = .
2
T y suy ra biu thc i sin(it) nhn gi tr thc. iu ny gi cho ta cch chuyn i
cc ng nht thc i vi hm s sin sang cc ng nht thc i s.

117
Hi tho khoa hc, Hng Yn 25-26/02/2017

V d 3. Xt cng thc khai trin

sin 3t = 3 sin t 4 sin3 t.

T y ta thu c cng thc (hnh thc)

i sin i (3t) = 3(i sin it) + 4(i sin it)3 .

H thc i s ng vi cng thc trn chnh l ng nht thc


1 3 1 h1 1 i h 1  1 i3
a 3 =3 a +4 a ,
2 a 2 a 2 a
hay
1 3 1
4x3 + 3x = a 3
2 a
vi
1 1
x= a , a 6= 0.
2 a
V d 4. Xt cng thc bin i

sin 5t + sin t = 2 sin 3t(1 2 sin2 t). (1.3)

Ta vit li cng thc (1.3) di dng

i sin i (5t) + i sin it = 2i sin i (3t)(1 + 2(i sin it)2 .

H thc i s ng vi cng thc trn chnh l ng nht thc


1 5 1  1 1 h1 1 ih 1 1 2 i
a 5 + a =2 a3 3 1+ a 2 .
2 a 2 a 2 a 2 a
S dng kt qu khai trin cc hm lng gic sin 3t v sin 2t, ta thu c ng nht
thc i s dng nh v d sau y.
V d 5.
1 5 1
a 5 = m + 2(4m3 + 3m)(2m2 + 1),
2 a
trong
1 1
m= a .
2 a
nh ngha 1 (a thc Chebyshev loi 1). Cc a thc Tn ( x ) (n N) c xc nh
nh sau (
T0 ( x ) = 1; T1 ( x ) = x,
Tn+1 ( x ) = 2xTn ( x ) Tn1 ( x ), n > 1
c gi l cc a thc Chebyshev (loi 1).

nh ngha 2 (a thc Chebyshev loi 2). Cc a thc Un ( x ) (n N) xc nh nh sau


(
U0 ( x ) = 0; U1 ( x ) = 1,
Un+1 ( x ) = 2xUn ( x ) Un1 ( x ), n > 1

c gi l cc a thc Chebyshev (loi 2).

118
Hi tho khoa hc, Hng Yn 25-26/02/2017

Tnh cht 1. Tn ( x ) = cos(n arccos x ) vi mi x [1, 1]

Tnh cht 2. Tn ( x ) c ng n nghim phn bit trn [-1, 1 ] l

2k + 1
xk = cos (k = 0, 1, . . . , n 1).
2n
sin(n arccos x )
Tnh cht 3. Un ( x ) = vi mi x (1, 1).
1 x2
1 0 sin nt
Tnh cht 4. Un ( x ) =Tn ( x ) = , cos t = x, a thc bc n 1 c h s bc cao nht
n sin t
bng 2n1 v l hm chn khi n l; l hm l khi n chn.

Tnh cht 5. Tn ( x ) c ng n nghim phn bit trn [-1, 1 ] l

2k + 1
xk = cos (k = 0, 1, . . . , n 1).
2n
Bi ton 1. Chng minh rng vi m, n N; n m v x R th

Tn+m ( x ) + Tnm ( x ) = 2Tn ( x ) Tm ( x ).

Li gii. S dng nh ngha v phng php quy np hoc s dng cc cng thc

cos(n + m) x + cos(n m) x = 2 cos nx cos mx

v
cosh(n + m) x + cosh(n m) x = 2 cosh(nx ) cosh(mx ).

Bi ton 2. Chng minh rng

Tm ( Tn ( x )) = Tmn ( x ), x R, m, n N. (1.4)

Li gii. Ta chng minh (1.4) bng phng php quy np theo m. Vi n c nh tu v


m = 0, ta c T0 ( Tn ( x )) = 1 = T0n ( x ) (theo nh ngha Tn ( x )). Vy (1.4) ng vi m = 0;
n N.
Gi s (1.4) ng ti m. Khi

Tm+l ( Tn ( x )) = 2Tn ( x ) Tm ( Tn ( x ))

= 2Tn ( x ) Tmn ( x ) T(ml )n ( x ) (theo gi thit quy np)


= Tn+mn ( x ) + Tmnn ( x ) Tmnn ( x ) (theo Bi ton?? )
= T(m+1)n ( x ).
Vy
Tm ( Tn ( x )) = Tmn ( x ), x R, m, n N.

119
Hi tho khoa hc, Hng Yn 25-26/02/2017

2 Phng trnh a thc bc cao


Tip theo, ta xt mt s ng dng ca ng thc i s - lng gic vo gii mt s
dng phng trnh a thc bc cao v chuyn phng trnh v t v phng trnh a
thc.

Bi ton 3. Chng minh rng phng trnh

64x6 96x4 + 36x2 3 = 0

c nghim thc x0 tha mn iu kin


q p q p
2+ 2+ 2 2+ 2+ 3
< x0 < .
2 2

1 + cos 2
Li gii. T cng thc cos2 = (vi 0 6 6 ), ta suy ra
2
v r
v u
u 1 + cos u 1 + 1 + cos
u u
t 2 t 2 1
q
cos = = = 2 + 2 + 2 cos .
4 2 2 2

Khi = , ta c
4 r
1
q
cos = 2+ 2+ 2.
16 2

Khi = , ta c
6 r
1
q
cos = 2+ 2+ 3.
24 2
Mt khc, ta cng c

cos 6t = 4 cos3 2t 3 cos 2t


= 4(2 cos2 t 1)3 3(2 cos2 t 1)
= 32 cos6 t 48 cos4 t + 18 cos2 t 1

Suy ra
64 cos6 t 96 cos4 t + 36 cos2 t 3 = 2 cos 6t 1.
T phng trnh 64x6 96x4 + 36x2 3 = 0, ta xt x [1; 1] v t

1
x = cos t, ta c 2 cos 6t 1 = 0 cos 6t = t= .
2 18

Do x0 = cos l mt nghim ca phng trnh. Mt khc
18

< <
24 18 16

120
Hi tho khoa hc, Hng Yn 25-26/02/2017

hay

cos > cos > cos .
24 18 16

Vy phng trnh 64x6 96x4 + 36x2 3 = 0 lun c mt nghim x0 = cos tha mn
18
iu kin q p q p
2+ 2+ 2 2+ 2+ 3
< x0 < .
2 2
Bi ton 4. Cho b s m, n, p R. Gii phng trnh

x 3 + m3 x 3 + n3 3 3xmxnxp
+ 3 3
+ = 0.
( x + m )3 3
x +p 2 2x+mx+nx+p
( x + n) +
( x + p )3

Li gii. Nhn xt rng


x 3 + m3 1 3 ( x m )2
= + .
( x + m )3 4 4 ( x + m )2
V vy phng trnh cho tng ng vi phng trnh sau

1 3 ( x m )2 1 3 ( x n )2 1 3 ( x p )2
+ + + + +
4 4 ( x + m )2 4 4 ( x + n )2 4 4 ( x + p )2

3 3xmxnxp
+ = 0. (1)
2 2x+mx+nx+p
t
xm xn xp
= a, = b, =c
x+m x+n x+p
v rng
1 3 2 1 3 2 1 3 2
+ a + + b + + c
4 4 4 4 4 4
3 3
+ abc = 0
4 2
c th bin i c v dng

( ab + c)2 = (1 a2 )(1 b2 ). (2)

Thay cc gi tr a, b, c theo bin x, m, n, p ta c

4[ x3 + (mn mp np) x ]2
( ab + c)2 = ,
( x + m )2 ( x + n )2 ( x + p )2
4mx 4nx
1 a2 = 2
, 1 b2 = .
( x + m) ( x + n )2
Vy (2) c dng

x2 [ x2 + 2( x + p) mn + mn mp np][ x2 2( x + p) mn + mn mp np] = 0.

121
Hi tho khoa hc, Hng Yn 25-26/02/2017

Gii ra ta c cc nghim ca phng trnh l

x1 = x2 = 0,

x3,4 = ( mp np) mn,

x5,6 = mn ( mp + np.

Bi ton 5. Cho 0 < < . Chng minh rng vi mi a thc Q( x ) R[ x ] bc n th
n+2
a thc
P( x ) = ( x2 2x cos + 1) Q( x )
khng th c tt c cc h s u khng m.
Li gii. Gi s
Q ( x ) = a 0 x n + a 1 x n 1 + + a n 1 x + a n
v
P( x ) = b0 x n+2 + b1 x n+1 + + bn+1 x + bn+2 .
Khi


b0 = a0 ,




b1 = a1 2a0 cos ,

b2 = a2 + a0 2a1 cos ,



bn+1 = an1 2an cos ,






bn + 2 = a n .
Suy ra
bk = ak + ak2 2ak1 cos , an+2 = an+1 = 0, a1 = a2 = 0
v
n +2
bk sin k = 0.
k =0
 
M sin k > 0 v 0, nn tn ti h s b j < 0.
n+2
Bi ton 6. Gii phng trnh
p
1 x2 = 4x3 3x.

Li gii. t x = cos , 0 . Ta c sin = cos 3, hay


 
cos 3 cos = 0.
2
T    
2 sin 2 sin + = 0.
4 4
5 3
n y ta c th d dng tm c = hoc = hoc = . V
8 8 4
v
u 1 + cos
u p
t 4 2+ 2
cos = = ,
8 2 2

122
Hi tho khoa hc, Hng Yn 25-26/02/2017
v
u 1 + cos 5
u p
5 t
4 = 2 2
cos = ,
8 2 2

3 2
cos = ,
4 2
(p p )
2+ 2 2 2 2
nn tp hp , , l tp nghim ca phng trnh.
2 2 2

Bi ton 7. Gii phng trnh


x 35
x+ = .
x2 1 12

1
Li gii. Ch rng x > 1. t x = , 0 < < . Phng trnh c th vit li thnh
sin 2
1 1 35
+ = .
sin cos 12
t2 1
t sin + cos = t th 1 + 2 sin cos = t2 , suy ra sin cos = . Thay vo ta
2
c
2t 35 7 5
= 35t2 24t 35 = 0 t = t = .
t2 1 12 5 7
7
V t = sin + cos > 0 nn ta loi nghim th hai. Xt t = th t y ta tnh c
5
12
sin cos = . Nh vy sin , cos l nghim ca phng trnh
25
7 12
X2 X + = 0.
5 25
3 4
T y ta tnh c sin = hoc sin = . Tng ng ta c nghim ca phng
5 5
5 5
trnh l x = v x = .
3 4
Bi ton 8. Gii phng trnh

8x3 4x 1 =
3
6x + 1.

Li gii. Phng trnh cho tng ng



(2x )3 4x 1 = 3
2x + 4x + 1.

t u = 2x, v = 3 2x + 4x + 1, ta c h phng trnh
( 3 ( 3 (
u 4x 1 = v u v3 = v u (u v)(u2 + uv + v2 + 1) = 0

v3 4x 1 = u u3 4x 1 = v u3 4x 1 = v
(
u=v
8x3 6x = 1. (1)
u3 4x 1 = u

123
Hi tho khoa hc, Hng Yn 25-26/02/2017

Nu | x | > 1 th |8x3 6x | = 2| x |(4x2 3) > 2 nn (1) v nghim. Do vy ta phi c


| x | 1. iu ny cho php ta t x = cos t vi t [0, ]. Khi phng trnh (1) c th
vit li thnh
1 5 7
cos 3t = t1 = t2 = t3 = .
2 9 9 9
5 7
Vy phng trnh cho c ba nghim x = cos , x = cos , x = cos .
9 9 9
Bi ton 9. Tn ti hay khng tn ti cc s a1 , a2 , . . . , an R l cc nghim ca a thc
n
P( x ) = xn + (1)k Cnk akk xnk .
k =1

Li gii. Gi s tn ti cc s nh vy. Khi theo nh l Viette th

Cnk akk = ai1 ai2 aik (k = 1, . . . , n)


i1 <i2 <<ik

(tng ny c Cnk s hng).


Gi s
| ak | = max{| a1 |, | a2 |, . . . , | an |}.
Khi ta c
Cnk | ak |k = | ai1 || ai2 | | aik | Cnk | ak |k .
i1 <i2 <<ik

Vy nn
| a1 | = | a2 | = = | a n |.
Mt khc
| a1 + a2 + + a n | = n | a1 |
nn a1 , a2 , . . . , an cng du v do chng bng nhau v t bng a. Ta c P( x ) =
( x a)n l a thc tho mn iu kin bi ra.
Bi ton 10. Chng minh rng khng tn ti mt tp hu hn cc s thc dng M sao
cho ng vi mi n nguyn dng u tn ti a thc bc n thuc M [ x ] c ng n nghim
u thuc M.
Li gii. Gi s tn ti tp

M = { a1 , a2 , . . . , a n } ( a = a1 < < a n = b )

tho mn yu cu ca bi ton. Khi theo gi thit th tn ti a thc

P( x ) = b0 + b1 x + + bn x n M [ x ]

sao cho P( x ) c n nghim x1 , x2 , . . . , xn thuc M. Theo nh l Vieete th


n  bn 1  2 bn 2  b 2 | bn 2 |
x2j =
bn
2
bn
< n 1 + 2
bn | bn |
.
j =1

Suy ra
n  b 2 b
na2 x2j a
+2
a
n N ,
j =1

124
Hi tho khoa hc, Hng Yn 25-26/02/2017

iu ny l khng th xy ra. Vy khng tn ti mt tp hu hn cc s thc dng M


sao cho ng vi mi n nguyn dng u tn ti a thc bc n thuc M[ x ] c ng n
nghim u thuc M.

Bi ton 11. Chng minh rng phng trnh

64x6 96x4 + 36x2 3 = 0

c nghim thc x = x0 tho mn iu kin


q p q p
2+ 2+ 2 2+ 2+ 3
< x0 < .
2 2
1 + cos 2
Li gii. T cng thc cos2 = (0 6 6 ), ta suy ra
2
v r
v u
u 1 + 1 + cos
u u
u 1 + cos
t 2 t 2 1
q
cos = = = 2 + 2 + 2 cos .
4 2 2 2

Khi = th ta c
4
v
u s r
1t
u
2 1
q
cos = 2+ 2+2 = 2 + 2 + 2.
16 2 2 2


Khi = th ta c
6
v
u s r
1t
u
3 1
q
cos = 2+ 2+2 = 2 + 2 + 3.
24 2 2 2

Ta li c

cos 6t = 4 cos3 2t 3 cos 2t = 4(2 cos2 t 1)3 3(2 cos2 t 1)


= 32 cos6 t 48 cos4 t + 18 cos2 t 1,

nn 64 cos6 t 96 cos4 t + 36 cos2 t 3 = 2 cos 6t 1


T phng trnh 64x6 96x4 + 36x2 3 = 0, ta xt x [1, 1].

t x = cos t, khi ta c 2 cos 6t 1 = 0 t =
18

Do vy x0 = cos l mt nghim ca phng trnh.
18

Ta c < < cos > cos > cos .
24 18 16 24 18 16

Vy phng trnh 64x6 96x4 + 36x2 3 = 0 c mt nghim x0 = cos tho mn
18
q p q p
2+ 2+ 2 2+ 2+ 3
iu kin < x0 < .
2 2

125
Hi tho khoa hc, Hng Yn 25-26/02/2017

Bi ton 12. Tm x (0; 1) tha mn iu kin


1
32x ( x2 1)(2x2 1)2 = 1 .
x
 
Li gii. V x (0; 1) nn ta c th t x = cos vi 0; .
2
Ta c
1
32 cos (cos2 1)(2 cos2 1)2 = 1
cos
1
32 cos sin2 cos2 2 = 1
cos
8 sin2 2 cos2 2 = 1 cos
2 sin2 4 = 1 cos
cos = cos 8.

k2 l2
Do = hoc = (vi k, l Z.)
7  9
V rng 0; nn k = 1, l = 1 v l = 2.
2
Vy phng trnh cho c ba nghim thuc (0; 1) l
2 2 4
x = cos ; x = cos ; x = cos .
7 9 9
Bi ton 13. Gii phng trnh
q q
x3 + (1 x 2 )3 = x 2(1 x 2 ).

Li gii. iu kin hcc biuithc c ngha: 1 6 x 6 1.



t x = sin ( vi ; ), th phng trnh tr thnh
2 2

sin3 + cos3 = 2 sin cos

(sin + cos )3 3 sin cos (sin + cos ) 2 sin cos = 0.
 
t sin + cos = 2 sin + = t vi iu kin |t| 6 2.
4
t2 1
Suy ra sin cos = v phng trnh tr thnh
2
t2 1 t2 1
t3 3 t 2 =0
2 2
t3 + 2t2 3t 2 = 0

(t 2)(t + 2 1)(t + 2 + 1) = 0

Suy ra t = 2 hoc t = 1 2 do |t| 6 2 .
Vi t = 2 th    
2 sin + = 2 sin + = 1
4 4

hay = + k2.
4

126
Hi tho khoa hc, Hng Yn 25-26/02/2017

h i 2
V ; nn = v do x = sin = .
2 2 4 4 2
Vi t = 1 2, suy ra
sin + cos = 1 2
hay p
x+ 1 x2 = 1 2,
tc l ( (
x 6 1 2 x 6 1 2

1 x 2 = (1 2 x )2 x 2 (1 2) x + (1 2) = 0
p
1 2+ 2 21
x= .
2
Vy phng trnh cho c 2 nghim l
p
2 1 2+ 2 21
x= ;x = .
2 2
Bi ton 14. Gii phng trnh
q p hq q i p
1 + 1 x2 (1 + x )3 (1 x )3 = 2 + 1 x 2 .

Li gii. iu kin c ngha: 1 6 x 6 1. t x = cos t (vi t [0; ]), phng trnh tr


thnh

q q  p
1 + sin t (1 + cos t) (1 cos t) = 2 + 1 cos2 t
3 3

hay s
s s
 2  3  3
t t t t p
sin + cos 2 cos2 2 sin2 = 2+ sin2 t (2.1)
2 2 2 2

t h i
V t [0; ] nn 0; . Do
2 2
t t
sin > 0, cos > 0.
2 2
Vy nn phng trnh (2.1) tng ng vi
 t t

3 t 3 t

2 2 sin + cos cos sin = 2 + sin t
2 2 2 2

t t
 
t t

2 2 sin + cos cos sin
2 2 2 2
 
2 t t t 2 t
cos + sin cos + sin = 2 + sin t
2 2 2 2

1

2 2 cos t 1 + sin t = 2 + sin t 2 cos t(2 + sin t) = 2 + sin t
2

2 2
( 2 cos t 1)(2 + sin t) = 0 cos t = x=
2 2

2
Vy phng trnh c nghim l x = .
2

127
Hi tho khoa hc, Hng Yn 25-26/02/2017

Bi ton 15. Gii phng trnh theo ( x, y, z) :


xy yz zx
+ + = 0.
1 + xy 1 + yz 1 + zx

 
Li gii. t x = tan , y = tan , z = tan , vi , , ; . Khi
2 2
xy yz zx
+ +
1 + xy 1 + yz 1 + zx
tan tan tan tan tan tan
= + +
1 + tan tan 1 + tan tan 1 + tan tan
= tan( ) + tan( ) + tan( )

v
xy yz zx
. . = tan( ) tan( ) tan( ).
1 + xy 1 + yz 1 + zx
Ta chng minh ng nht thc

tan( ) + tan( ) + tan( ) = tan( ) tan( ) tan( ).

Tht vy, ta c
tan a + tan b
= tan( a + b)
1 tan a tan b
nn
tan a + tan b = tan( a + b)(1 tan a tan b).
Suy ra

tan( ) + tan( ) + tan( )


= tan( + )[1 tan( ) tan( )] + tan( )
= tan( ) + tan( ) + tan( ) tan( ) tan( )
= tan( ) tan( ) tan( ).

Do
xy yz zx xy yz zx
+ + = . . .
1 + xy 1 + yz 1 + zx 1 + xy 1 + yz 1 + zx
Vy phng trnh c nghim ( x, y, z) = ( a, a, b), ( a, b, a), (b, a, a) vi a, b R v ab 6= 1.

Bi ton 16. Gii h phng trnh



x + y + z = xyz
3x x3 3y y3 3z z3
+ + =0
1 3x2 1 3y2 1 3z2
 
Li gii. t x = tan , y = tan , z = tan vi , , , . Do x + y + z = xyz
2 2
nn tan + tan + tan = tan tan tan , hay

tan + tan = tan (1 tan tan ).

128
Hi tho khoa hc, Hng Yn 25-26/02/2017

Do tan tan 6= 1 nn ng thc (2) tng ng vi


tan + tan
= tan ,
1 tan tan
hay
tan( + ) = tan .
Do
+ + = n, n Z.
Suy ra
3 + 3 + 3 = 3n, n Z,
v v vy
tan 3( + ) = tan 3
hay
tan 3 + tan 3 + tan 3 = tan 3 tan 3 tan 3.
Tip theo, s dng cng thc gc nhn ba

3 tan tan3
tan 3 =
1 3 tan2
ta nhn c
3x x3 3y y3 3z z3 3x x3 3y y3 3z z3
+ + = .
1 3x2 1 3y2 1 3z2 1 3x2 1 3y2 1 3z2

3+a   3a 
Vy h c nghim ( x, y, z) = (0, a, a), 3, a, , 3, a, v cc
3a 1 3a + 1
hon v ca n.

3 Kt lun
Bng phng php tng t, ta cng c th gii c nhiu dng phng trnh a
thc bc cao bng cch p dng cc ng nht thc Chebyshev nhn c phn
trn.

Ti liu
[1] Trng Ngc c (2015), Mt s dy s sinh bi cc hm lng gic, K yu HTKH
Bun Ma Thuc, 14-15/03/2015, trang 58-64.
[2] Nguyn Vn Mu, Phm Th Bch Ngc (2003), Mt s bi ton chn lc v lng gic,
NXB Gio dc.
[3] T-L.T. Radulescu, V.D. Radulescu, T.Andreescu (2009), Problems in Real Analysis: Ad-
vanced Calculus on the real axis, Springer Sciences+Business Media.
[4] Paulo Ney de Sausa, Jorge- Nume Silva (1998), Berkeley Problems in Mathematics,
Springer.

129
Hi tho khoa hc, Hng Yn 25-26/02/2017

PHNG PHP HM SINH XC NH DY S


Lng Th Hng
THPT Chuyn Hng Yn

1 C s l thuyt
Phng php hm sinh l mt phng php hin i, s dng kin thc v chui, chui
hm (Cng thc Taylor), chuyn cc bi ton v dy s thnh nhng bi ton v hm s. y
l phng php mnh gii cc bi ton v dy s m i khi ta hon ton b tay vi cc
phng php khc.
tng ca phng php hm sinh n gin nh sau: Gi s ta cn tm cng thc tng

qut ca dy s { an } no . T cng thc truy hi ta tm c hm sinh f ( x ) = an x n
n =0
ca dy s. V t , h s ai ca xi trong khai trin ca f ( x ) thnh chui ly tha chnh l
s hng th i ca dy { an }. Hay ni cch khc, ta tm f ( x ) ri ly o hm cp n ca n ti
0 l tm c an .
Cc loi hm sinh gm hm sinh thng, hm sinh m, hm sinh Dirichlet. . . . Trong bi ny
ta ch cp n mt loi hm sinh thng dng: Hm sinh thng.

1.1 nh ngha
Cho dy s a0 ; a1 ; a2 ; . . . an ; . . . . Chui ly tha hnh thc

f ( x ) = a0 + a1 x + a2 x 2 + = an x n (1.1)
n =0

c gi l hm sinh thng ca dy { an }.
K hiu tng ng gia mt dy s v hm sinh nh sau:

{ an } f ( x ) = a0 + a1 x + a2 x2 + . . .
1
Chng hn nh {1, 1, 1, . . . } 1 + x + x2 + = .
1x
Khai trin Taylor

f ( n ) (0) n
f (x) = n!
x (1.2)
n =0

131
Hi tho khoa hc, Hng Yn 25-26/02/2017

v cng thc khai trin Newton m rng



x2 xn
(1 + x ) =
Cn xn = 1 + x + ( 1) 2! + + ( 1) . . . ( n + 1) n! + . . . (1.3)
n =0

l nhng c s quan trng chng ta tm cng thc tng minh cho hm sinh ca hng
lot cc dy s.

1.2 Cc php ton



Cho f ( x ) = an x n v g( x ) = bn x n l hm sinh tng ng ca cc dy { an } v {bn }.
n =0 n =0
Khi ta nh ngha cc php ton nh sau:
a) Php cng

f ( x ) g( x ) = an x n bn x n = ( a n bn ) x n (1.4)
n =0 n =0 n =0

V d 1.

1
{1, 1, 1, . . . }
1 1 2
1x

{2, 0, 2, 0, . . . } + = (1.5)
1 1x 1+x 1 x2
{1, 1, 1, 1, . . . }

1+x

b) Php nhn vi mt s k f ( x ) = k ( an x n ) = (kan ) x n
n =0 n =0

1 2
V d 2. {1, 0, 1, 0, . . . } 2
, nhn vi 2 ta c {2, 0, 2, 0, . . . }
1x 1 x2
c) Tch

f ( x ).g( x ) = ( an x n ).( bn x n ) = cn x n (1.6)
n =0 n =0 n =0
n
vi cn = ai bni .
i =0

V d 3. Gi s { an } f ( x ) = an x n = a0 + a1 x + a2 x2 + . . . .
n =0
1 f (x)
Ta bit = 1 + x + x2 + .., do = a0 + ( a0 + a1 ) x + ( a0 + a1 + a2 ) x 2 + . . .
1x 1x
f (x) n
suy ra { ai }
1x i =0

d) Nghch o Ta ni f ( x ) = an x n c nghch o l g( x ) = bn x n nu f(x).g(x)=1.
n =0 n =0
a0 .b0 = 1 1 n
= n suy ra bn = . ai .bni
ai .bni = 0, n 1 a 0 i =1
i =0
e) Dch chuyn sang phi: Nu { a0 ; a1 ; a2 ; . . . } f ( x ) th

132
Hi tho khoa hc, Hng Yn 25-26/02/2017

{0; 0; . . . 0; a0 ; a1 ; a2 ; . . . } x k . f ( x )
| {z }
k s 0

1
V d 4. {1, 1, 1, . . . } th
1x
xk
{0; 0; . . . 0; 1; 1; 1; . . . }
| {z } 1x
k s 0

f) o hm: Nu { a0 ; a1 ; a2 ; . . . } f ( x ) = an x n th
n =0

{ a1 ; 2a2 ; 3a3 ; . . . } f 0 (x) = nan x n 1
n =1
o hm cp n>1 c xc nh bi f (n+1) = ( f (n) )0 .

1
V d 5. {1, 1, 1, . . . } f ( x ) = 1 + x + x2 + x3 + = .
1x
Ly o hm ta c

1
2
= 1 + 2x + 3x + = (n + 1) x n
2
(1 x ) n =0

l hm sinh ca dy {1, 2, 3, . . . }.
Mt h qu tt yu ca o hm l: Nu { an } f ( x ) th {nan } x. f 0 ( x ).

V d 6. Tm hm sinh cho dy {0, 1, 4, 9, . . . }.


1
Ta xut pht t {1, 1, 1, . . . }
1x
1
Ly o hm ta c {1, 2, 3, . . . }
1 x2
x
Dch chuyn sang phi 1 v tr, ta c {0, 1, 2, 3, . . . } .
1 x2
1+x
Ly o hm, ta c {1, 4, 9, . . . }
(1 x )3
x (1 + x )
Dch chuyn sang phi 1 v tr, ta c {0, 1, 4, 9, . . . } .
(1 x )3
x (1 + x )
Vy hm sinh ca dy l f ( x ) = .
(1 x )3

1.3 Mt s kt qu quan trng


f ( x ) a 0 a 1 x a k 1 x k 1
nh l 1. Nu { an } f ( x ) th vi k > 0, { an+k } .
xk
nh l 2. Dy s { an } c gi l dy xc nh kiu tuyn tnh nu dy c dng

a0 = 0 ; . . . ak1 = k1 ; ak+n = 1 ak+n1 + 2 ak+n2 + k an , n 0

133
Hi tho khoa hc, Hng Yn 25-26/02/2017

p( x )
. Khi , hm sinh thng f ( x ) ca dy { an } l mt hm hu t ( f ( x ) = vi p(x) l a
q( x )
thc c bc nh hn q(x)) khi v ch khi { an } l dy xc nh kiu tuyn tnh.

nh l 3. Nu dy s { an } l dy xc nh kiu tuyn tnh v ak+n = 1 ak+n1 +


2 ak+n2 + k an , n 0. Gi r1 , r2 , . . . , rt l cc nghim bi tng ng s1 , s2 , . . . , st
ca phng trnh x k + 1 x k1 + 2 x k2 + + k = 0. Khi , tn ti cc a thc
p1 (n), p2 (n), . . . , pt (n) tha mn 0 deg pi (n) si 1, i = 1, 2, . . . , t v an = p1 (n).r1n +
p2 (n).r2n + + pt (n).rtn .

1.4 Mt s khai trin thng gp

1
1 + x + x2 + x3 + = x n
1x n =0
1
1 + ax + a2 x 2 + a3 x 3 + = an x n
1 ax n =0
1
1 x + x2 x3 + = (1)n x n
1+x n =0
1
1 + x k + x2k + x3k + = x nk
1 xk n =0
1
1 x k + x2k x3k + = (1)n x nk
1 xk n =0
1
1 + 2x + 3x2 + = ( n + 1) x n
(1 x ) 2
n =0 .
1
2
1 + 2ax + 3a2 x2 + = (n + 1) an x n
(1 ax ) n =0
n
(1 + x ) n 1 + Cn1 x + Cn2 x2 + + Cnn x n = Cnk x k
k =0
x2 xn
(1 + x ) 1 + x + ( 1) + + ( 1) . . . ( n + 1) + = Cn x n
2! n! n =0
1 x 2 x k
1 + nx + n(n + 1) + + n(n + 1) . . . (n + k 1) + = Cnk +k1 x k
(1 x ) n 2! k! k =0
x x 2 x 3 xn
ex 1+ + + + =
1! 2! 3! n=0 n!

2 ng dng hm sinh gii cc bi ton dy s


(
a0 = 0
V d 7. Cho dy s xc nh nh sau . Tm an .
an+1 = 2an + 1, n 1
Li gii. Gi s { an } f ( x ), ta suy ra {0; 2a0 + 1; 2a1 + 1; . . . } f ( x ).
Ta xut pht t

134
Hi tho khoa hc, Hng Yn 25-26/02/2017

1 x
{1; 1; 1; . . . } {0; 1; 1; 1; . . . } . (1).
1x 1x
v
{2a0 ; 2a1 ; 2a2 ; . . . } 2 f ( x ) {0; 2a0 ; 2a1 ; 2a2 ; . . . } 2x f ( x ). (2).
x
Cng (1) v (2) suy ra {1; 2a0 + 1; 2a1 + 1; 2a2 + 1; . . . } 2x f ( x ) + .
1x
x x
Do , f ( x ) = 2x f ( x ) + f (x) = .
1x (1 x )(1 2x )
x
Nh vy hm sinh ca dy l f ( x ) = .
(1 x )(1 2x )
By gi ta khai trin f ( x ) thnh chui ly tha
x 2 1
f (x) = = x( )
(1 x )(1 2x ) 1 2x 1 x

f ( x ) = x (2 2n x n x n ) = (2n +1 1 ) x n +1 .
n =0 n =0 n =0
Vy s hng tng qut ca dy l an = 2n 1.
(
a1 = 1
V d 8. Tm an bit .
a n = a n 1 + a n 2 + + a 1 , n 2
Li gii. Gi f ( x ) l hm sinh ca dy cn tm.
f ( x ) = a1 x + a2 x 2 + + a n x n + . . .
= a 1 x + a 1 x 2 + ( a 1 + a 2 ) x 3 + ( a 1 + a 2 + a 3 ) x 4 + ( a 1 + a 2 + + a n 1 ) x n + . . .
= a1 ( x + x 2 + + x n + . . . ) + a2 ( x 3 + x 4 + + x n +
. . . ) + a3 ( x 4 + x 5 + + x n + . . . ) + . . .
= xa1 + xa1 ( x + x2 + . . . ) + x2 a2 ( x + x2 + . . . ) + . . .
= x + ( x + x2 + . . . )( a1 x + a2 x2 + . . . )
1 x
= x+( 1). f ( x ) = x + ( ). f ( x )
1x 1x
x (1 x )
suy ra f ( x ) =
1 2x
Khai trin f ( x ), ta c:

f ( x ) = x (1 x ). 2n x n = 2n x n +1 2n x n +2 = x + x 2 + 2n 2 x n .
( n =0 n =0 n =0 n =2
a1 = 1; a2 = 1
Vy .
a n = 2n 2 , n 2
(
a0 = 2; a1 = 0; a2 = 2
V d 9. Cho dy s . Tm an .
an+3 = 6an+2 11an+1 + 6an , n 0
f ( x ) 2 + 2x2 f (x) 2 f (x) 2
Li gii. p dng nh l 1, ta c: 3
= 6. 2
11. + 6 f (x)
x x x
20x2 12x + 2 20x2 12x + 2 5 4 1
suy ra f ( x ) = 2 3
= = + .
1 6x + 11x 6x (1 x )(1 2x )(1 3x ) 1 x 1 2x 1 3x

135
Hi tho khoa hc, Hng Yn 25-26/02/2017

Khai trin f ( x ) thnh chui ly tha ta c:


f ( x ) = 5 x n 4 2n x n + 3n x n = (5 2n +2 + 3n ) x n .
Vy an = 5 2n+2 + 3n .
(
a0 = a1 = 0
V d 10. Cho dy . Tm an .
an+2 6an+1 + 9an = 2n + n, n 0
1 x
Li gii. Ta bit hm sinh ca dy {2n } l v hm sinh ca dy {n} l .
1 2x (1 x )2
Gi s f ( x ) l hm sinh ca dy { an }. Khi ta c:
f (x) f (x) 1 x
2
6. + 9 f (x) = +
x x 1 2x (1 x )2
x4 x3 + x2 1 1 5
suy ra f ( x ) = 2 2
= 2
+ +
(1 2x )(1 x ) (1 3x ) 4(1 x ) 1 2x 3(1 3x )
5
.
12(1 3x )2
Khai trin thnh chui ly tha ta c:
2n +2 + n + 1 + 5 ( n 3 )3n 1 n
f (x) = x .
4
2n +2 + n + 1 + 5 ( n 3 )3n 1
Vy s hng tng qut ca dy l an = .
4
V d 11. Cho dy s

a1 = 1 F ( n ) (0)
1 a1 a2 a . Chng minh rng an = ,
an = + + + + n 1 , n > 1 n!
n! (n 1)! (n 2)! 1!
1
vi F ( x ) = x .
e 2

Li gii. t a0 = 1 v gi f ( x ) = an x n l hm sinh ca dy cho. Khi ta c:
n =0

x x2 xn
f ( x )(e x 1) = ( a0 + a1 x + a2 x2 + + an x n + . . . )( + ++ )
1! 2! n!
1 a 1 a a
= x + x2 ( + 1 ) + x3 ( + 1 + 2 ) + . . .
2! 1! 3! 2! 1!
2 3 n
= a1 x + a2 x + a3 x + + a n x + . . .
= f (x) 1

1
T suy ra f ( x ) = .
ex2
F ( n ) (0) 1
Theo khai trin Taylor th an = , vi F ( x ) = x .
n! e 2

136
Hi tho khoa hc, Hng Yn 25-26/02/2017
(
a0 = 0; a1 = 1
V d 12. Dy s Fibonacci:
a n = a n 1 + a n 2 , n 2
an 1
Tm s hng tng qut an ca dy v chng minh rng = .
n =0 4 +1
n 11
Li gii. Gi s f ( x ) l hm sinh ca dy cho.
f (x) f (x) x x 1 1
Theo nh l 1, ta c: f ( x ) = + 2
f ( x ) = 2
= (
x
x 1 x x 5 1 x
1 1+ 5 1 5
), vi = ; = .
1 x 2 2
1
Khai trin f ( x ) thnh chui ly tha ta c: f ( x ) = (n n ) x n .
5 n =0
1 1+ 5 1 5
Vy an = (n n ), vi = ;= .
5 2 2
x an
n , nn vi x = 1 ta suy ra c 1
Hn na, ta c f ( x ) = 2
= a n x n + 1
= .
1xx n =0 4 n =0 4 11
Trong trng hp hm sinh ca dy l hm phn thc vi mu l a thc c nghim phc,
ta vn thao tc nh bnh thng!
(
a0 = 0; a1 = 2
V d 13. Cho dy s: . Tm s hng tng qut an ca dy.
an+2 = 4an+1 8an , n 0
Li gii. Gi s f ( x ) l hm sinh ca dy { an }.
f ( x ) 2x f (x)
Ta c 2
= 4. 8 f (x)
x x
i
2x 2x
suy ra f ( x ) = = = 2 +
1 + 4x + 8x 2 [1 (2 + 2i ) x ][1 (2 2i ) x ] 1 (2 + 2i ) x
i
2 .
1 (2 2i ) x
i i
Khai trin f ( x ) thnh chui ly tha ta c: f ( x ) = [ .(2 + 2i )n + .(2 2i )n ] x n .
2 2
i i
Nh vy, s hng tng qut ca dy l an = .(2 + 2i )n + .(2 2i )n .
2 2
3 3
Ta c th chuyn i nh sau: 2 2i = 2 2(cos i sin )
4 4
3n 3n
suy ra (2 2i )n = (2 2)n (cos i sin ).
4 4

137
Hi tho khoa hc, Hng Yn 25-26/02/2017

Do ,
i n 3n 3n 3n 3n
an = (2 2) [(cos i sin ) (cos + i sin )]
2 4 4 4 4
i 3n
= (2 2)n (2i sin )
2 4
3n
= (2 2)n sin
4
0 nu n = 8k; n = 8k + 4




(2 2)n nu n = 8k + 6




(2 2)n nu n = 8k + 2

= 1

(2 2)n nu n = 8k + 1; n = 8k + 3
2


1


(2 2)n nu n = 8k + 5; n = 8k + 7



2
(
a0 = 5; a1 = 13; a2 = 35
V d 14. Cho dy s .
an+3 = 6an+2 11an+1 + 6an , n 0
Chng minh rng an 2n+1 (mod3n+1 ) v an 3n+1 (mod2n+1 ).
Li gii. p dng nh l 3, xt phng trnh x3 6x2 + 11x 6 = 0 c cc nghim n
x1 = 1; x2 = 2; x3 = 3.
Do s hng tng qut ca dy s c dng an = a1n + b2n + c3n = a + b2n + c3n .
T iu kin cho a0 = 5; a1 = 13; a2 = 35, ta tm c an = 2n+1 + 3n+1 . Ta suy ra iu
phi chng minh.
(
a0 = 11; a1 = 6; a2 = 18; a3 = 104; a4 = 346
V d 15. Cho dy s . Tm s
an+5 = 6an+4 13an+3 + 14an+2 12an+1 + 8an , n 0
nguyn dng ln nht m a2017 chia ht cho 2m .
Li gii. Xt a thc p( x ) = x5 6x4 + 13x3 14x2 + 12x 8 = ( x 2)3 ( x i )( x + i ).
p dng nh l 3, ta c an = p1 (n).2n + a.in + b.(i )n vi 0 deg p1 (n) 2. T gi thit
ca bi ton, ta tm c an = (n2 + n + 1)2n + 5.in + 5.(i )n .
suy ra a2017 = (20172 + 2017 + 1)22017 .
Vy s m cn tm l 2017.
(
c0 = 1
V d 16 (S Catalan). Cho dy s . Tm cng
c n = c 0 c n 1 + c 1 c n 2 + + c n 1 c 0 , n 1
thc tnh cn .
Li gii. Gi f ( x ) l hm sinh ca dy {cn }.
T cng thc truy hi trn, ta c
cn+1 = c0 cn + c1 cn1 + + cn c0 , n 0. (3)
f (x) 1
Theo quy tc nhn th {cn+1 } c hm sinh l f ( x ). f ( x ). Do = f 2 ( x ),
x
1 1 4x
suy ra f ( x ) = .
2x

138
Hi tho khoa hc, Hng Yn 25-26/02/2017

1 1 4x
Nhng do f (0) = c0 = 1 nn f ( x ) = .
2x
Khai trin f(x):
1 1 1 1 1
( 1)( 2) . . . ( n + 1)
Ta c (1 4x ) 2 = C n1 (4x )n , vi C n1 = 2 2 2 2 =
n!
2 2
(1)n1 [2(n 1)]!
. .
22n (n 1)!n!
1
2
suy ra (1 4x ) 2 = 1 .C2nn x n +1 .
n =0 n + 1
1 2 Cn
n n + 2n
suy ra f ( x ) = .C2n x 1 = .x n .
2x n=0 n + 1 n =0 n + 1
Cn
Vy s hng tng qut ca dy l an = 2n .
n+1
(
a0 = 12; a1 = 4; a2 = 31
V d 17. Cho dy s ( an ) xc nh bi: .
an+3 = 4an+2 + 3an+1 18an , n 0
Chng minh rng a2016 1 (mod 2017).
Li gii. Gi hm sinh ca dy l f ( x ) = a0 + a1 x + . . . an x n + . . . . Theo nh l 1 ta c:

f ( x ) 2 4x 31x2 f ( x ) 2 4x f (x) 2
= 4 + 3 18 f ( x )
x3 x2 x

9x2 4x + 2 1 1
suy ra f ( x ) = 2 3
= + .
1 4x 3x + 18x 1 + 2x (1 3x )2
Khai trin f ( x ) ta c f ( x ) = ((2)n + (n + 1)3n ) x n . Do an = (2)n + (n + 1)3n , v
ta c a2016 = 22016 + 2017.32016 .
V 2017 l s nguyn t v (2,2017)=1 nn 22016 1 (mod 2017). Vy a2016 1 (mod 2017).
1.3.5 . . . (2n + 1)
V d 18. Cho dy ( an ) xc nh bi an = , n 0. Tnh an .
2017n .n! n =0
Li gii. D thy t cng thc tng qut ca dy s ta suy ra cng thc truy hi:

2n + 3
a0 = 1; an+1 = an , n 0
2017(n + 1)

a n +1 2
, hay 2017(n + 1) an+1 = 2nan + 3an v , n +.
an 2017
Gi f ( x ) l hm sinh ca dy ( an ).
Ta c f ( x ) = an x n hi t v 2017( an+1 x n+1 )0 = 2x ( an x n )0 + 3( an x n ).
suy ra 2017( f ( x ) 1)0 = 2x ( f ( x ))0 + 3 f ( x )
nn (2017 2x ) f 0 ( x ) = 3 f ( x )
f 0 (x) 3
= .
f (x) 2017 2x

139
Hi tho khoa hc, Hng Yn 25-26/02/2017

3
3
Ly nguyn hm hai v ta c ln f ( x ) = ln(2017 2x ) + a f ( x ) = (2017 2x ) 2 .e a .
2
Do f (0) = a0 = 1 nn suy ra e a = 20173/2 .
3 3

Vy f ( x ) = 2017 2 .(2017 2x ) 2 .
Nh vy

3

3  2017  3
an = f (1) = 2017 2 .(2015) 2 = 2.
n =0 2015

V d 19 (VMO-2011). Cho dy s nguyn { an } c xc nh nh sau:


a0 = 1, a1 = 1 v an = 6an1 + 5an2 vi mi n=2,3,4,. . . . Chng minh rng a2012 2010
chia ht cho 2011.
gii. p dng nh l 3, xt phng trnh x2 6x 5 = 0
Li c hai nghim n x =
3 14 nn s hng tng qut c dng an = a(3 + 14) + b(3 14)n .
n
1 2 1 2
T gi thit a0 = 1; a1 = 1 ta suy ra a = ; b = + .
2 14 2 14
1 2 n 1 2 n
Do an = ( )(3 + 14) + ( + )(3 14) .
2 14 2 14
1 2 1 2
Vi s nguyn t p=2011 ta c a p+1 = ( )(3 + 14) p+1 + ( + )(3 14) p+1 .
p +1 2 14 p+1 2 14
M (3 + 14) = A p+1 + B p+1 . 14; (3 14) = A p+1 B p+1 . 14,
( p+1)/2
p+1 ( p+1)/2
p+1
i i
2i 2i 2i 1 2i 1
(trong A p+1 = C p+1 .3 .14 2 v B p+1 = C p+1 .3 .14 2 )
i =0 i =0
suy ra a p+1 = A p+1 4B p+1 .
Do p l s nguyn t nn ta c C kp 0 (mod p) k=1;2;. . . ;p-1, m C kp+1 = C kp + C kp1 nn
p+1 p1
C kp+1 0. T suy ra A p+1 (14 2 + 3 p+1 ) (mod p) v B p+1 3( p + 1)(14 2 +
p1
3 ) 3(14 2 + 3 p1 ) (mod p). Vy a p+1 (3 p + 2.14( p1)/2 ) (mod p).
p 1

Mt khc, ta c 452 14 (mod p) v (p,45)=1 nn theo nh l Phc-ma nh ta c 3 p 3


(mod p) v 14( p1)/2 45 p1 1 (mod p).
Nh vy a2012 3 + 2 = 1 2010 (mod p), hay a2012 2010 chia ht cho 2011.

V d 20 (VMO-2015). Cho f n ( x ) l dy a thc xc nh bi f 0 ( x ) = 2; f 1 ( x ) = 3x; v


f n ( x ) = 3x f n1 ( x ) + (1 x 2x2 ) f n2 ( x ), n 2. Tm tt c cc s nguyn dng n f n ( x )
chia ht cho x3 x2 + x.
Li gii. ( Ta coi x l hng s v tm cng thc tng qut cho dy s ( an ) xc nh bi cng
a0 = 2; a1 = 3x
thc: .
an = 3xan1 + (1 x 2x2 ) an2 , n 2
p dng nh l 3 ta xt phng trnh t2 3xt 1 + x + 2x2 = 0 c hai nghim n t = x + 1
v t = 2x 1, do an = a( x + 1)n + b(2x 1)n . T gi thit a0 = 2; a1 = 3x ta suy ra a=b=1.
Vy f n ( x ) = ( x + 1)n + (2x 1)n .

140
Hi tho khoa hc, Hng Yn 25-26/02/2017

V f n ( x ) chia ht cho a thc g( x ) = x3 x2 + x nn f n (0) = 0 hay 1 (1)n = 0 nn n l.


Li c f n (2) = (5n + 1) chia ht cho (2)2 (2) + 1 = 7 (do n l). M 125 1 (mod 7)
nn ta xt cc trng hp:
TH1: n=3k, k l, th 5n + 1 = 53k + 1 (1)k + 1 = 0 (mod 7).
TH2: n=3k+1, k chn, th 5n + 1 = 5.53k + 1 6 (mod 7).
TH3: n=3k+2, k l, th 5n + 1 = 25.53k + 1 24 3 (mod 7).
Vy iu kin cn ca n l n = 3k vi k l. Khi , f n ( x ) = ( x + 1)3k + (2x 1)3k chia ht
cho ( x + 1)3 + (2x 1)3 = 9( x3 x2 + x ).
Nh vy tt c cc s n cn tm c dng n=3k vi k l, hay n=6m+3 vi m l s nguyn
dng.

V d 21. Xt dy ( an ) c a1 = 1; an = 1.an1 + 2.an2 + (1)n1 (n 1) a1 ; n 2.


a2 = 1

Chng minh rng a3 + 3a2 = 0 . T tm d ca php chia an cho 3.

an+2 + 3an+1 + an = 0, n 2

Li gii. Gi f ( x ) = a1 x + a2 x2 + . . . l hm sinh ca dy ( an ). Xt tch
F ( x ) = f ( x )(1 + 2x2 3x3 + . . . )
= 1.a1 x2 + (1.a2 + 2a1 ) x3 + (1.a3 + 2a2 3a1 ) x4 + . . .

= a2 x2 + a3 x3 + a4 x4 + = f ( x ) x. (4)
1
Li c khai trin = 1 x + x2 x3 + . . .
1+x
1 2 + 4x 3 x
nn = 1 + 2x 3x = 1.x + 2x2 3x3 + 4x4 . . . .
(1 + x )2 (1 + x )2
x
Thay vo (4) ta c f ( x )( ) = f ( x ) x f ( x )( x2 + 3x + 1) = x3 + 2x2 + x
(1 + x )2
hay
( a1 x + a2 x2 + . . . )( x2 + 3x + 1) = x3 + 2x2 + x. T suy ra ngay
a2 = 1

a3 + 3a2 = 0 .

an+2 + 3an+1 + an = 0, n 2

Ta c a3 0 (mod 3). M an+ 2 + 3an+1 + an = 0 nn an+2 + an 0 (mod 3) vi n 2. Do


a2k+1 0( mod 3 )

, vi s nguyn k 1 ta c a4k+2 a2 2( mod 3 ) .

a4k 1( mod 3 )

3 Mt s bi tp tng t
Bi 1 (VMO-1997). Cho dy s nguyn { an } c xc nh nh sau: a0 = 1, a1 = 45 v
an+2 = 45an+1 7an vi mi n N.
(i) Tnh s c dng ca a2n+1 an an+2 theo n.
(ii) Chng minh rng 1997a2n + 7n+1 .4 l s chnh phng vi mi n.

141
Hi tho khoa hc, Hng Yn 25-26/02/2017

Bi 2 (VMO-1998-A). Cho dy s nguyn { an } c xc nh nh sau: a0 = 20, a1 = 100 v


an+2 = 4an+1 + 5an + 20 vi mi n N.
(i) Tm s nguyn dng h nh nht c tnh cht an+h an chia ht cho 1998.
(ii) Tm s hng tng qut ca dy.

Bi 3. Dy s ( an ) xc nh bi a1 = 1 v an = 1.2.an1 + 2.3.an2 + + (n 1).n.a1 , n 2.


n
Chng minh rng an+3 4an+2 an+1 = 2 ak ; n 2.
k =1
n
Bi 4. Vi s nguyn dng n, tnh tng an = (1)k Cn3k . Xt tnh tun hon ca dy ( an )
k =1
v ch ra an chia ht cho 3[n/2]1 .
Bi 5. Xt dy s ( an ) vi a1 = 1; an = 12 an1 + 22 an2 + + (n 1)2 a1 , n 2.
a2 = 4a1 3

Chng minh rng a3 = 4a2 2a1 + 3 . T tm cng thc tng qut an .

an+3 = 4an+2 2an+1 + an , n 2

(
a0 = 1,
Bi 6. Tm s hng tng qut ca dy s ( an ) bit
an = 2an1 + n.3n .
(
u0 = 1
Bi 7. Tm cng thc tng qut cho dy (un ) vi .
un = a.un1 + bn , n 1
(
a0 = a1 = 0
Bi 8. Tm s hng tng qut ca dy ( an ) cho bi .
an+2 6.an+1 + 9an = 2n + n, n 0

a1 = 1

Bi 9. Cho dy s ( an ) bit a2n = an . Tm s hng tng qut ca dy.

a2n+1 = an + an+1

2n + 3
Bi 10. Cho dy ( an ) xc nh bi a1 = 1 v an+1 = an , n 0. Tnh tng T = an .
4( n + 1) n =0
a n 1 a a1
Bi 11. Dy ( an ) xc nh bi a1 = 1 v an = ( + n 2 + + ), n 2. Tm
1! 2! ( n 1) !
tt c cc s nguyn dng n sao cho n!an+1 = 1.

Ti liu
[1] Srini Devadas and Eric Lenhman, Generating Funtions, Lectures Notes, April 2005.
[2] Trn Nam Dng, Hm sinh, Bi ging 2011.
[3] Nguyn Vn Mu (Ch bin), Ton ri rc v cc dng ton lin quan, NXB Gio dc 2008.

142
Hi tho khoa hc, Hng Yn 25-26/02/2017

PHNG TRNH, BT PHNG TRNH, H


PHNG TRNH QUA CC K THI HC SINH GII

Trn Th Lan Hng


THPT Chuyn Hng Yn

Tm tt ni dung

Phng trnh, bt phng trnh, h phng trnh l mt trong nhng dng ton cng
thng xut hin trong cc thi hc sinh gii. Bo co trnh by mt s phng php
c bn gii quyt cc dng ton phng trnh, bt phng trnh, h phng trnh qua
cc k thi hc sinh gii gn y.

1 Mt s phng php thng dng


gii quyt cc bi ton dng phng trnh, bt phng trnh, h phng trnh ta
c th chia thnh 4 phng php chnh nh sau:
1. Phng php bin i tng ng
2. Phng php t n ph
3. Phng php hm s
4. Phng php nh gi.
Sau y ta i vo tng phng php c th:

1.1 Phng php bin i tng ng


p
Bi ton 1 (VMO - 2002 - Bng A). Gii phng trnh 4 3 10 3x = x 2 (1)

74 10
Gii. iu kin: x .
27 3
(1) 4 3 10 3x = x2 4x + 4 9(10 3x ) = x2 (4 x )2
x4 8x3 + 16x2 + 27x 29 = 0 ( x 3)( x + 2)( x2 7x + 15) = 0
x = 3 ( V x2 7x + 15 = 0 v nghim v x = -2 khng tha mn).
Vy phng trnh c nghim duy nht x = 3.

r r
1 1
Bi ton 2 (CMO - 1998). Gii phng trnh x = x + 1 .
x x
Gii. k x>1. !2 !2
r r r r
1 1 1 1
Phng trnh x x = 1 x x = 1
x x x x
p
x 2 1 2 x ( x 2 1) + x = 0 ( x 2 1 x )2 = 0 x 2 1 = x x 2 x

144
Hi tho khoa hc, Hng Yn 25-26/02/2017

1=0
1+ 5 1 5
x= (x= khng tha mn).
2 2
1+ 5
Vy phng trnh c nghim duy nht x = .
2
hai bi ny s dng phng php bin i tng ng vi k thut c bn l bnh
phng hai v khi hai v khng m th bi ton c gii quyt.
Bipton 3 (IMO - 1959).p Vi nhng gi tr thc no ca x th mi ng thc sau l ng?

a) px + 2 2x 1 + px 2 2x 1 = 2.

b) p x + 2 2x 1 + p x 2 2x 1 = 1.

c) x + 2 2x 1 + x 2 2x 1 = 2.
2x 1 0

1
Gii. k: x + 2 2x 1 0 x .
2
x 2 2x 1 0

p p p
Ta c ( p x + 2 2x 1 + x 2 2x 1)2 = 2x + 2 x2 (2x 1)
= 2x + 2 ( x 1)2 = 2x + 2| x 1|.
1
a) Phng trnh tng ng vi x + | x 1| = 1 | x 1| = 1 x x 1.
2
1 1 1 1 1
b) PT x + | x 1| = | x 1| = x x . M iu kin x x = .
2 2 2 2 2
Thay vo phng trnh khng tha mn.
Vy phng trnh v nghim.
c) Ta c x + | x 1| = 2 | x 1| = 2 x x 2. Bnh phng hai v ta c:
3
x2 2x + 1 = 4 4x + x2 x = (tm).
2
Vy
 tp
 nghim ca mi phng trnh trong cu a, b, c ln lt l
1 3
S1 = ; 1 ; S2 = ; S3 = .
2 2
bi ny kt hp vi k thut bnh phng hai v ta thy c khai cn mt biu thc
dng A2 = | A|, a phng trnh v t v phng trnh cha du gi tr tuyt i.

4x2
Bi ton 4 (IMO - 1960). Gii bt phng trnh < 2x + 9.
(1 + 1 + 2x )2
1
Gii. k: x , x 6= 0.
2
2x 2x (1 + 1 + 2x )
Ta c: = = (1 + 1 + 2x )
1 1 + 2x 1 (1 + 2x )
4x2

2
= (1 + 1 + 2x )2 = 2 + 2x + 2 1 + 2x nn bt phng trnh vit
(1 1 + 2x )
7 49 45
di dng: 2 + 2x + 2 1 + 2x < 2x + 9 1 + 2x < 1 + 2x < x < . Kt
2 4 8
1 45
hp iu kin suy ra x < v x 6= 0.
2 8
1 45
 
Vy tp nghim ca bt phng trnh l ; \ {0}.
2 8
bi ny ta s dng k thut nhn chia biu thc lin hp v cng nh vy vi cc bi
ton sau:

145
Hi tho khoa hc, Hng Yn 25-26/02/2017

Bi ton 5 (HSG( khu vc Duyn hi v ng bng Bc B 2015 - 2016 lp 10). Gii h


7x3 + y3 + 3xy( x y) = 12x2 6x + 1 (1)
phng trnh .
2 x2 + 3 9 x2 + y = 1 (2)
Gii. k: 3 y 3.
(1) (y x )3 = (1 2x )3 y x = 1 2x x + y = 1 x = 1 y. Th vo (2) ta
c:
p p p 1
2 (1 y)2 + 3 9 y2 + y = 1 2( y2 2y + 4 + y 2) + 3 9 x2 = 0
2
3y2
2 y2
p + =0
1
p
2
y2 2y + 4 + 2 y 3 + 9 y
2
2 3 1
y ( p + )=0y=0x=1
2
1 3 + 9 x 2
2( y 2x + 4 + 2 y)
2
1
(do |y| 3 2 y > 0).
2
Vy h c nghim duy nht ( x; y) = (1; 0).

Bi ton 6 (HSG( khu vc Duyn hi v ng bng Bc B 2014 - 2015 lp 10). Gii h


y4 2xy2 + 7y2 = x2 + 7x + 8 (1)
phng trnh p 3 2 2
.
2
3 x + y + 1 = x + x 4y + 3 (2)
Gii. iu kin: x 3
(1) y4 2xy2 + 7y2 + x2 7x 8 = 0
(y2 xy2 + 8)(y2 x 1) = 0
y2 = x + 1 hoc y2 = x 8.
Suy ra x 8.
Thay y2 = x + 1 vo (2) ta c: 3 x + x + 2 = x3 + x2 4x 1
2x x2
+ = ( x 2)( x + 1)( x + 2)
3x+1 x+2+2
1 1
(2 x )[ + + ( x + 1)( x + 2)] = 0
3x+1 x+2+2
x = 2 (v
x 1).
y = 3.
Vi y2 = x 8, m x 3 x 8 5 2
n y 5 (v l). o
Vy h phng trnh c nghim ( x; y) (2; 3); (2; 3) .
Li gii bi 6 ngoi k thut nhn chia biu thc lin hp kt hp vi k thut phn
tch thnh nhn t rt n ny theo n kia.


2
6x y + z = 3

Bi ton 7 (VMO - 2016). Gii h phng trnh x2 y2 2z = 1 .

2 2 2
6x 3y y 2z = 0
6x y + z (6x 3y y 2z2 )
2 2 2
Gii. T h ta c: + ( x2 y2 2z + 1) = 0
3
( x 1)2 = (z 1)2 x = z hoc x = 2 z.

146
Hi tho khoa hc, Hng Yn 25-26/02/2017

2 2
6x y + z = 3
6x y + z = 3

Vi x=z, h tr thnh: x2 y2 2x = 1 ( x 1)2 = y2

2
4x 3y2 y = 0
2
4x 3y2 y = 0

2
6x y + z = 3
(1)
2 2
4x 3y y = 0 .

x = y + 1 hoc x = 1 y


5 33
Vi x=y+1 th vo (1) c: 6x ( x 1) + = 3 x2 x2 + 5x 2 = 0 x =
2
5 33 7 33
z= y= .
2 2
7 65
Vi y=1-x th vo (1) c: x =
2
7 65 9 65
z= y= .
2 2
2
y = ( x + 1)

Vi x=2-z th h tr thnh y2 + 4 = ( x + 1)2 y = y2 + 4 (v nghim).

2
6x 3y2 y 2z2 = 0
Vy h phng trnh c nghim
( ! !)
5 33 7 33 5 33 7 65 9 65 7 65
( x; y; z) ; ; ; ; ; .
2 2 2 2 2 2
Bi 7 s dng k thut quen thuc khi gii h phng trnh, l cng i s v th.

Bi(ton 8 (USAOP - 1995). Gii h phng trnh


p
3 2x2 y x4 y2 + x2 (1 2x2 ) = y2
p .
1 + 1 + ( x y)2 = x3 ( x3 x 2y2 )
(p
4 (1 x2 y)2 = 2x4 x2 + y2 (1)
Gii. H phng trnh p 6 4 3 2
.
2
1 + ( x y) = 1 x + x 2x y (2)
p p
2 2
p (1) v (2) ta cp 4 (1 x y) 3 1 +
Cng ( x y )2 = ( x 3 y2 )2 + 1
2 2 2
4 (1 x y) = 1 + ( x y) + ( x y ) + 1 (3). 2 2

Tuy(nhin
p
4 (1 x 2 y )2 2
p
1 + ( x y )2 + ( x 3 y2 )2 + 1 2
p
( 2 y )2 = 2 2

p 4 1 x x y = 1

nn (3) xy ra 1 + ( x y )2 = 1 x=y x = y = 1.

3 2 2
(x y ) + 1 = 1
3
x =y 2

Vy h c nghim duy nht ( x; y) = (1; 1).

147
Hi tho khoa hc, Hng Yn 25-26/02/2017

Bi tp cng phng php


12

(1 ) x=2


y + 3x
Bi 1 (VMO - 2007). Gii h phng trnh 12 .

( 1 + ) y = 6
y + 3x



1
3x (1 + )=2


x+y
Bi 2 (VMO - 1996 - Bng A). Gii h phng trnh 1 .
7y(1 )=4 2


x+y

3 2
x + x (y z) = 2

Bi 3 (VMO - 2004 - Bng A). Gii h phng trnh y3 + y(z x )2 = 30 .

3
z + z( x y)2 = 16

3

Bi 4. Gii bt phng trnh x2 1 + x x3 2.

Bi 5. Gii phng trnh 2x2 + x + 9 + 2x2 x + 1 = x + 4.

Bi 6. Gii bt phng trnh 7 3x 7 + (4x 7) 7 x 32.

Bi 7. Gii phng trnh ( x + 2)( 2x2 + 4x + 6 + 2x 1) = 2x2 + 6x + 7.

1.2 Phng php t n ph



Bi ton 9 (VMO - 1995 - Bng B). Gii phng trnh 2x2 11x + 21 3 3 4x 4 = 0
y3 + 4 y6 + 8y3 + 16
Gii. t 3 4x 4 = y x = x2 = .
4 6
1 11 3
T ta c phng trnh (y6 + 8y3 + 16) (y + 4) 3y + 21 = 0
8 4
y6 14y3 24y + 96 = 0 (1)
(y 2)2 (y4 + 4y3 + 12y2 + 18y + 14) = 0 (2)
Nu y 0 th VT(1)>0 (v
nghim) nn y > 0. Khi y4 + 4y3 + 12y2 + 18y + 14 > 0 nn
t (2) suy ra y = 2, hay 3 4x + 4 = 0 x = 3. Th li tha mn.
Vy phng trnh c nghim duy nht x = 3.

Bi ton 10 (HSG khu vc Duyn hi v ng


bng BcB nm 2013-2014 lp 10). Gii
phng trnh (6x 3) 7 3x + (15 6x ) 3x 2 = 2 9x2 + 27x 14 + 11.
2 7
Gii. k: x .
( 3
3
a = 7 3x
t , a, b 0.
b = 3x 2
(
a2 + b2 = 5
Ta c:
(2b2 + 1) a + (2a2 + 1)b = 2ab + 11
(
a+b = S
t , (S2 4P),
ab = P
ta c h

148
Hi tho khoa hc, Hng Yn 25-26/02/2017
( (
S2 2P = 5 2P = S2 5

2PS + S = 2P + 11 2S(S2 5) + S = S2 5 + 11
(
2P = S2 5
(S 3)(S2 + 2S + 2) = 0
(
S=3
(v S2 + 2S + 2 > 0).
P=2
T
( suy ra (
a=2 a=1
hoc x = 1 hoc x = 2. Th li thy tha mn.
b=1 b=2
Vy phng trnh c hai nghim x=1; x=2.

(
x4 y4 = 240
Bi ton 11 (VMO - 2010). Gii h phng trnh
x3 2y3 = 3( x2 4y2 ) 4( x 8y)
(
x4 + 16 = 16(t4 + 16) (1)
Gii. t y = 2t, h tr thnh:
x3 3x2 + 4x = 16(t3 3t2 + 4t) (2)
Nhn cho hai phng trnh ta c: ( x4 + 16)((t3 3t2 + 4t)) = (t4 + 16)( x3 3x2 +
4x ).
2 2
D
 thy nu  th x.t 6=
 (x,t) l nghim  0 nn ta chia
 hai v phng trnh cho x t c
16 4 16 4
x2 + 2 t3+ = t2 + 2 x3+ .
x t t x
x + 4 = u

t x . Ta c phng trnh
4
t + = v

t
(u2 8)(v 3) = (v2 8)(u 3) u2 v v2 u 3(u2 v2 ) + 8(u x ) = 0
(u v)(uv 3(u + v) + 8) = 0 u = v hoc uv 3(u + v) + 8.
T (1) suy ra x, t cng du. Do p dng BT AM-GM ta c u, v 4
hoc u, v 4 (u 3), (v 3) 1 hoc (u 3), (v 3) 7
uv 3(u + 3) + 8 = (u 3)(v 3) 1 0.
Du bng xy ra khi v ch khi u = v =4.
4
T (2) suy ra u = v x = t hoc x = .
t
Vi x = t, thay vo (1) ta c t4 + 16 = 16(t4 + 16) (v nghim).
4 256
Vi x = thay vo (1) ta c 4 + 16 = 16(t4 + 16) t8 + 15t4 16 = 0 t = 1.
t t
Vy h c nghim ( x; y) {(4; 2); (4; 2)}.

8 (VMO - 2004 - Bng B). Gii h phng trnh


Bi(
x3 3xy2 = 49
x2 8xy + y2 = 8y 17x
x = u + v
(
x+y = u 2
Gii. t =
xy = v y = u v
( 2
u3 + v3 = 98 (1)
Ta c h 2 2
.
3u + 5v = 9u 25v (2)

149
Hi tho khoa hc, Hng Yn 25-26/02/2017

Ly (2)x3+(1) ta c (u 3)3 + (v + 5)3 = 0 u 3 = v 5 u = (v + 2). Th


vo (2) ta c v2 + 2v 15 = 0 v = 3 hoc v=-5.
Vi v = 3 th u = -5, suy ra x = -1; y = -4.
Vi v = -5 th u = 3, suy ra x = -1; y = 4.
Vy h c nghim ( x, y) {(1; 4); (1; 4)}.

9 (Olympic Austria - 2000). Gii h phng trnh


Bi(
( x 1)(y2 + 6) = y( x2 + 1)
.
(y 1)( x2 + 6) = x (y2 + 1)
5 5
Gii. Ta cng hai phng trnh trn cho nhau v rt gn ta c: ( x )2 + (y )2 =
2 2
1
.
2
Tr hai phng trnh trn cho nhau ta c:
xy(y x ) + 6( x y) + ( x + y)( x y) = xy( x y) + (y x )
( x y)(7 + x + y 2xy) = 0 x = y hoc x + y 2xy + 7 = 0.
Vi x = y thay vo (1) ta c x = y =2 hoc x = y =3.
1 1 15
Nu x 6= y th t x + y - 2xy + 7 = 0 ( x )(y ) = .
2 2 4
a = x 5

t 2
5 , ta c
b = y

2
a + b = 1
2 2 (1)
2
15
( a + 2)(b + 2) =

4
1 1
ab + 2( a + b) = 2ab + 4( a + b) = (2)
4 2
2
Cng (1) v (2) ta c ( a + b) + 4( a + b) = 0 a + b = 0 hoc a + b = -4.
Ly (2)-(1) ta c ( a b)2 4( a + b) = 1 (3)
Nu a + b = -4 = a = -b - 4 th vo (3) ta c (2b 4)2 + 42 = 1 (v nghim).
Nu (a + b = 0 th vo (3) ta c ( a b)2 = 1 a b = 1 hoc a b = 1.
a+b = 0
= hoc
ab = 1

a = 1
(
a+b = 0 2
1 hoc
a b = 1 b =

2
a =
1
2 .
1
b =

2
T ta c nghim ca h l ( x; y) {(2; 2); (3; 3); (2; 3); (3; 2)}.

150
Hi tho khoa hc, Hng Yn 25-26/02/2017

Bi tp cng phng php


(p 10 (HSG
Bi
p
Quc Gia - 2001 - Bng B). Gii h phng trnh
7x + y + 2x + y = 5
p .
2x + y + x y = 2

3 4
Bi 11. Gii phng trnh x2 3x + 1 = x + x2 + 1.
3

Bi 12. Gii bt phng trnh 5x2 + 14x + 9 x2 x 20 5 x + 1.

Bi 13. Gii phng trnh x = 2 x. 3 x + 3 x. 5 x + 5 x. 2 x.
q
Bi 14. Gii phng trnh 2 2( x + 1) + 4 2x = 1.

6 2x 6 + 2x 8
Bi 15. Gii phng trnh + = .
5x 5+x 3

Bi 16. Gii phng trnh ( x + 1) x2 2x + 3 = x2 + 1.
4
Bi 17. Gii phng trnh 3
81x 8 = x3 2x2 + x 2.
3

1.3 Phng php hm s


Bi ton 12 (HSG khu vc Duyn hi v ng bng Bc B 2013-2014 lp 11). Gii h
p
2x
2y + 2x + y + 2xy + 1 = 1 (1)
phng trnh 3 3y + 1 = 8y3 2y 1
p
(2) .

x>0

p
Gii. (1) (2x + 1) 2(y + 1) + (2x + 1( )(y + 1) = 0. (
2x + 1 > 0 x>0
k: (2x + 1)(y + 1) 0 m x > 0 nn suy ra = .
y 1 y 1
p p p
(1) ( 2x + 1 y + 1)( 2x + 1 + 2 y + 1) 2x + 1 y + 1 = 0 y = 2x.

Th vo (2) ta c 3 6x + 1 = 8x 6x + 1 + 3 6x + 1 = (2x )3 + 2x (3)
Xt hm s f (t) = t3 + t trn R c f 0 (t) = 3t2 + 1 > 0 vi mi t R, tc l hm s f(t)
1
ng bin trn R. Vy suy ra f ( 3 6x + 1) = f (2x ) 3 6x + 1 = 2x 4x3 3x = .
2
Nhn xt 1. x > 1 khng l nghim ca phng trnh.

Vi 0 . Ta c:
2
1 1 k2 k2
4 cos3 = 3 cos = cos(3) = = + hoc = + (k Z)
2 2 9 3 9 3

Do 0 nn = .
2 9  
Vy h c nghim duy nht (x;y)= cos ; 2 cos .
9 9
Bi ton ny a phng trnh v dng f(u) = f(v), trong hm y = f (t) l hm s
ng bin hoc nghch bin. Theo tnh cht ca hm s th u = v.

151
Hi tho khoa hc, Hng Yn 25-26/02/2017
(
x2 + y3 = 29 (1)
Bi ton 13 (VMO - 2008). Tm s nghim ca h .
log3 x. log2 y = 1 (2)
Gii.( k: x, y > 0 nn t (2) suy ra x, y > 1 hoc x, y < 1. Kt hp vi (1) suy ra x, y > 1.
log3 x = a
t , do x, y > 1 nn a, b > 0.
log2 y = b
(
9a + 8b = 29 (3) 1
Ta c h 9a + 8 a = 29 (5), a>0.
ab = 1 (4)
1
Xt hm s f (t) = 9t + 8 t 29, t > 0.
1 1
Hm s lin tc v c aoh hm mi cp vi mi t > 0 v f 0 (t) = 9t . ln 9 8 t . 2 . ln 8.
t
2 1 1 2 1 2
t
f (t) = 9 . ln 9 + 8 t . 4 . ln 8 + 8 t . 3 . ln 8 > 0 vi mi t >0. Do hm s f(t) ng
t t
bin trn (0; +) nn tn ti duy nht t0 > 0 sao cho f 0 (t0 ) = 0.
Mt khc lim f (t) = + = lim f (t).
t 0+ t+
f(1)<0 nn f (t0 ) < 0, ta c bng bin thin

Do tnh lin tc ca f(t) nn (5) c hai nghim dng. Vy h cho c hai nghim.

Bi ton 14 (VMO - 2006 - Bng A). Gii h phng trnh

2
px 2x + 6. log3 (6 y) = x

y2 2y + 6. log3 (6 z) = y .
2

z 2z + 6. log3 (6 x ) = z
Gii. k: x, y, z < 6.
x

log3 (6 y) = (1)

x 2 2x + 6
y


H tng ng vi log3 (6 z) = p 2 (2) .
y 2y + 6

z
log3 (6 x ) = 2 (3)


z 2z + 6
x
Nhn thy f ( x ) = l hm s ng bin vi x<6
2
x 2x + 6
6x
(v f 0 ( x ) = > 0 vi mi x < 6).
( x2 2x + 6) x2 2x + 6
1
Cn g( x ) = log3 (6 x ) l hm s nghch bin vi x < 6 v g0 ( x ) = < 0 vi
(6 x ) ln 3
mi x < 6.
Gi s ( x; y; z) l mt nghim ca h. Ta chng minh x = y = z.

152
Hi tho khoa hc, Hng Yn 25-26/02/2017

Khng mt tng qut ta gi s x = max { x, y, z} th xy ra cc trng hp sau:


TH1: x y z. Do f(x) l hm s ng bin nn t (1), (2), (3) suy ra
log3 (6 y) log3 (6 z) log3 (6 x ). T g(x) l hm s nghch bin nn x z y,
suy ra y = z. Chng minh tng t ta suy ra x=y=z.
TH2: x z y, chng minh tng t trn suy ra x = y = z.
Thay x = y = z vo (1) ta c f ( x ) = g( x ) vi mi x < 6.
V f(x) ng bin, g(x) nghch bin trn (; 6) nn f ( x ) = g( x ) c nhiu nht mt
nghim. D thy x = 3 tha mn.
Vy h c nghim duy nht ( x; y; z) = (3; 3; 3).

Bi
( ton 15 (VMO - 1999 - Bng A). Gii h phng trnh
(1 + 42xy )5y2x+1 = 22xy+1 + 1 (1)
.
y3 + 4x + ln(y2 + 2x ) + 1 = 0 (2)
Gii. k: y2 + 2x > 0.
1 + 4t 1 + 2t +1
t t=2x-y th (1) tr thnh (1 + 4t )51t = 1 + 2t+1 (3) = .
5t 5
V tri l hm s nghch bin cn v phi l hm s ng bin nn t=1 l nghim duy
nht ca (3).
y+1
T suy ra 2x-y=1 x = , th vo (2) ta c y3 + 2y + 3 + ln(y2 + y + 1) = 0
2
(4).
2y + 1
Xt hm s g(y) = y3 + 2y + 3 + ln(y2 + y + 1) trn R, c g0 (y) = 3y2 + 2 + 2 >
y +y+1
0 vi mi y. Suy ra hm s g(y) ng bin v y = 1 l nghim duy nht ca (4).
Vi y = 1 th x = 0.
Vy h c nghim duy nht ( x; y) = (0; 1).

Bi tp cng phng php


(
x3 3x + ln(2x + 1) = y
Bi 18 (VMO - 1994 - Bng B). Gii h phng trnh .
y3 3y + ln(2y + 1) = x

Bi 19. Gii bt phng trnh (2x + 1)(2 + 4x2 + 4x + 4) + 3x (2 + 9x2 + 3) > 0.
3
Bi 20. Gii phng trnhx3 4x2 5x + 6 = 7x2 + 9x 4.
3
Bi 21. Gii phng trnh 2x3 + 10x2 17x + 8 = 2x2 5x x3 .

Bi 22. Gii phng trnh x + 1 + 3 5x 7 + 4 7x 5 + 5 13x 7 < 8.

Bi 23. Gii phng trnh (sin x 2)(sin2 x sin x + 1) = 3 3 3 sin x 1 + 1.
r
9 2
3 x + 9x 1
Bi 24. Gii phng trnh = 2x + 1.
3

Phng php nh gi

Bi ton 16 (VMO - 1995 - Bng A). Gii phng trnh x3 3x2 8x + 40 8 4 4x + 4 = 0.

153
Hi tho khoa hc, Hng Yn 25-26/02/2017

Gii.
iu kin: x 1.
Khi ta xt hm: f ( x ) = x3 3x2 8x + 40 v hm g( x ) = 8 4 4x + 4 trn on
[1; +) . Phng trnh tng ng: f ( x ) = g( x ). p dng bt ng thc Cauchy cho
4 s khng m ta c:
p 1 4
2 + 24 + 24 + (4x + 4) = x + 13 (1)

g( x ) = 4 24 24 24 (4x + 4)
4
Du bng xy ra khi x = 3.
Mt khc, f ( x ) = x3 3x2 8x + 40 x + 13 ( x 3)( x2 9) 0
( x 3)2 ( x + 3) 0 (2)
Du bng xy ra khi x = 3.
T (1) v (2) suy ra g( x ) x + 13 f ( x ), c hai du bng xy ra u khi x = 3 nn x = 3
l nghim duy nht ca phng trnh.
Vy phng trnh c nghim duy nht x = 3.

Bi ton 17 (VMO - 2009). Gii h phng trnh


1 1 2

+p =p (1)
1 + 2x2 1 + 2y2 1 + 2xy
p x (1 2x ) + py(1 2y) = 2 (2)

9
Gii.
1 1
iu kin: 0 x ; 0 y .
2 2
u tin ta chng minh bt ng thc:
1 1 2
+ . Du bng xy ra khi a = b.
1+a 2 1+ b 2 1 + ab
 2  
1 1 1 1 4 1
Tht vy, ta c: + 2 2
+ 2
+
1+a 2 1+b 2 1 + a 1 + b 1 + ab 1 + a2
1 2
.
1 + b2 1 + ab
Du bng xy ra khi a = b.
1 1 2
p dng vo phng trnh (1) ca h ta c: +p p .
1 + 2x2 1 + 2y2 1 + 2xy
Du bng xy ra khi x = y. Th x = y vo phng trnh (2) ta c:
p 1
x (1 2x ) = 81x (1 2x ) = 1.
9
9 73
T ta c cc nghim tha mn ca phng trnh l: x = y = .
!36
9 73 9 73
Vy nghim ca h phng trnh l: ( x; y) = ; .
36 36

ton 18 (VMO - 2013).sGii h phng trnh


Bi s
r
2 1 2
1 20x
sin x + sin2 x + cos y + cos2 y = x + y



s r s .
2 1 2
1 20y
sin y + sin2 y + cos x + cos2 x = x + y


Gii.
iu kin: sin x, cos x, siny, cos y 6= 0, xy > 0.

154
Hi tho khoa hc, Hng Yn 25-26/02/2017

Nhn v vi v ca shai phng trnh ! trong h ta c: !


r s r
1 1 1 1
sin2 x + + cos2 y + + sin2 y + + cos2 x +
sin2 x cos2 y sin2 y cos2 x
xy
r
= 20 (3)
( x + y )2
p dng bt ng thc s Cauchy - Schwarz ! vs AM - GM ta c: !
r r
1 1 1 1
sin2 x + + cos2 y + + sin2 y + + cos2 x +
sin2 x cos2 y sin2 y cos2 x
2 2
| sin 2x |
 
1 1 3
| sin x. cos x | + = + +
| sin x. cos x | 2 2| sin 2x | 2| sin 2x |
 2  2
3 5
1+ = .
2 2
Hon
s ton tng t ta c:
r  2
2 1 2
1 5
sin y + 2
+ cos x + 2
.
sin y cos x 2
Do theos bt ng thc AM - GM ta c:
     
1 1 1 1
VT (3) 4 4 sin2 x + . cos 2x+ sin2
y + cos2x+
sin2 x cos2 x sin2 y cos2 x
r
5 4 xy
r
4
4 ( ) = 10 20 = VP(3).
2 ( x + y )2

Du bng xy ra khi | sin 2x | = 1; x = y x = y = + k , k Z.
4 2
Th li thy tha mn.  
Vy nghim ca h phng trnh l ( x; y) = +k ; +k ; k Z.
4 2 4 2

Bi tp cng phng php



Bi 25. Gii phng trnh 13 x2 x4 + 9 x2 + x4 = 16.

Bi 26. Gii bt phng trnh 1 2x + 1 + 2x 2 x2 .
r r
6 8
Bi 27. Gii phng trnh + = 6.
3x 2x

28. Gii phng trnh
Bi x2 x + 19 + 7x2 + 8x + 13 + 13x2 + 17x + 7 =
3 3( x + 2).

r  
2
1 1
Bi 29. Gii phng trnh 2 x + 2 2 = 4 x + .
x x
1
Bi 30. Gii phng trnh 3x2 1 + x2 x x x2 + 1 = (7x2 x + 4).
2 2
r r
28 27
Bi 31. Gii phng trnh 2 4 27x2 + 24x + = 1+ x + 6.
3 2
p 3
Bi 32. Gii phng trnh x y 1 + 2y x 1 = xy.
2
16 1 p
Bi 33. Gii phng trnh +p = 10 ( x 2016 + y 2017).
x 2016 y 2017

155
Hi tho khoa hc, Hng Yn 25-26/02/2017

2 Mt s phng php khc


2.1 Lng gic ha
Bi ton 19 (VMO - 1984 - Bng A). Gii phng trnh
p p p
1+ 1x 2 3 3
(1 + x ) (1 x ) = 2 + 1 x . 2

Gii.
iu kin: 1 x 1.
t x = cos t, 0 t
Phng trnhptr thnh:
p 
1 + sin t (1 + cos t)3 (1 cos t)3 = 2 + sin t
s
t 2
  
t t t
cos + sin cos3 sin3 .2 2 = 2 + sin t
2 2 2 2

  
t t t t
cos2 sin2 1 + cos sin .2 2 = 2 + sin t
2 2 2 2
cos t(2 + sin t) 2 = 2 + sin t
(2 + sin t)( 2 cos t 1) = 0
1
cos t = (do 2 + sin t > 0)
2
1
x = ( tha mn )
2
1
Vy phng trnh c nghim duy nht:x = .
2
x
tonx 20 (USAMO - 1978). Gi s x l nghim ca phng trnh(3 +2 2) x =
Bi
( 2 1) + 3. Chng minh rng khi x cng l nghim ca phng trnh ( 2 + 1) =

2 cos .
9
Gii.
Gi s x l nghim ca phng trnh
1
(3 + 2 2) x = ( 2 1) x + 3 ( 2 + 1)2x = + 3 (1)
( 2 + 1) x
1 1
t 2t = ( 2 + 1) x > 0. Khi (1) tr thnh: 4t2 = + 3 4t3 3t = (2)
2t 2
Trc ht ta tm cc nghim t (1; 1) ca phng trnh (1).
Do t (1; 1) nn t t = cos , (0; ). Ta c:
1 1 2
4 cos3 3cos = cos 3 = = + k .
2  2 9 3
5 7
M (0; ) ; ; .
9 9 9
5 7
Suy ra cc nghim ca phng trnh (2) l: t1 = cos ; t2 = cos ; t3 = cos .
9 9 9
5
Ta thy (2) l phng trnh bc 3 c 3 nghim t1 = cos ; t2 = cos ; t3 =
9 9
7
cos trong (1; 1) nn ta khng cn xt cc nghim ngoi khong (1; 1) na. Do
9
5 7
t2 = cos ; t3 = cos khng tha mn iu kin t > 0 nn ch c t1 tha mn
9 9

( 2 + 1) x = 2cos hay x l nghim ca phng trnh ( 2 + 1) x = 2cos (pcm).
9 9

156
Hi tho khoa hc, Hng Yn 25-26/02/2017

Bi ton 21 (USAOP -1990). Gii h


 phng  trnh
 
3 x + 1 1 1
= 4 y+ = 5 z+
x y z
xy + yz + zx = 1

Gii.
Trc tin ta nhn xt rng: Nu ( x; y; z) l mt nghim ca h th x; y; z phi cng du
v ( x; y; z) cng l nghim  cah. Do  ta gi
 s rng: x; y; z 0.
1 1 1
T phng trnh 3 x + = 4 y+ = 5 z+
 2   2 x  2 y z
x +1 y +1 z +1
3 =4 =5 .
2x 2y 2z

t x = tan ; y = tan ; z = tan , , , [0; ). Ta c:
2 2 2
1 x2 + 1 1 y2 + 1 1 z2 + 1 3 4 5
= ; = ; = = = (3)
sin 2x sin 2y sin 2z sin sin sin
Ta c: tan a.tanb + tan b.tanc + tan c.tana = 1th tan( a + b + c) khng xc nh. p ng
++
vo bi ta c: xy + yz + zx = 1 nn tan khng xc nh.
2
++
M , , [0; ) nn = + + = hay , , l 3 gc ca mt
2 2
tam gic. Theo (3) v theo nh l hm sin ta c th gi thit rng tam gic c di 3
1 cos 1 1 cos 1
cnh l 3, 4, 5 n v, t x = tan = = ;y = tan = = ;
2 sin 3 2 sin 2

z = tan = tan = 1.
2 4  
1 1
Vy nghim ca h phng trnh l: ( x; y; z) = ; ;1 .
3 2

Bi tp cng phng php



Bi 34. Gii phng trnh 4x 1 + 4x2 1 = 1.
1 1
Bi 35. Gii phng trnh + = 2 2.
x 1 x2
p p
Bi 36. Gii phng trnh x3 + (1 x2 )3 = x 2(1 x2 .
x2
Bi 37. Gii phng trnh 1+x+ 1x 2 .
4

Bi 38. Gii phng trnh x3 + 4x2 + 2x 3 = 2 x + 3.
x2 + 1 ( x 2 + 1)2
Bi 39. Gii phng trnh x2 + 1 + = .
2x 2x (1 x2

2.2 S dng ta vct


Trong mt phng tap Oxy, ~u( x1 ; y1 ), ~v( x2 ; y2 ) ta
pc: p
1. |~u + ~v| |~u| + |~v| ( x1 + x2 )2 + (y1 + y2 )2 ( x1 )2 + ( x2 )2 + (y1 )2 + (y2 )2 .
Du bng xy ra khi ~u, ~v cng hng.
2. |~u|.|~v| ~u.~v |~u|.|~v|

157
Hi tho khoa hc, Hng Yn 25-26/02/2017

|~u|.|~v| = ~u.~v khi ~u, ~v ngc hng.


|~u|.|~v| = ~u.~v khi ~u, ~v cng hng.
3. ||~u| |~v|| |~u ~v|.
Du bng xy ra khi ~u, ~v cng hng.
4. ||~u| |~v|| |~u + ~v|.
Du bng xy ra khi ~u, ~v ngc hng.


Bi 40. Gii phng trnh | x2 4x + 5 x2 10x + 50| = 5.
Gii.
Trong mt phng ta Oxy, gi p ~u( x 2; 1), ~v( xp 5; 5) ~u ~v = (3; 4) |~u ~v| = 5.
Ta c:
||~
u | |~
v || |~
u
~
v | ( x 2)2 + 1 ( x 5)2 + 25 5
| x2 4x + 5 x2 10x + 50| 5.
Du bng xy ra khi ~u, ~v cng hng tc l ~u = k~v, k > 0
x = 5
(
x 2 = k ( x 5)
4 .
1 = 5k k = 1 > 0
5
Vy phng trnh c nghim duy nht x = .
4
p
Bi ton 22. Gii bt phng trnh (5x2 6x + 2)( x2 + 4x + 5) 2x2 + 4x 3. Gii.
Bt
p phng trnh tng png:
(2x 1) + ( x 1) ( x + 2)2 + 1 (2x + 1)( x + 2) + x 1.
2 2

Trong mt phng ta p Oxy, gi ~u(2x 1; xp 1), ~v( x + 2; 1).


Ta c: |~u|.|~v| ~u.~v (2x 1)2 + ( x 1)2 ( x + 2)2 + 1 (2x 1)( x + 2) + x 1.
Du bng xy ra khi ~u, ~v cng hng tc l ~u = k~v, k > 0.
Khi VT VP. M bt phng trnh c dng VT ( VP nn bt phng
2x 1 = k ( x + 2)
trnh tng ng VT = VP hay ~u = k~v, k > 0
x1 = k

x =
1 + 5
2 .
k = 1 5 > 0

2
1+ 5
Vy phng trnh c nghim duy nht l: x = .
2

Bi tp cng phng php



Bi 41. Gii phng trnh 3x2 + 6x + 7 + 5x2 + 10x + 14 = 4 2x x2 .

Bi 42. Gii phng trnh 2x2 1 + x2 3x + 2 = 2x2 + 2x + 3 + x2 x + 6.

Bi 43. Gii bt phng trnh | x2 x 3| | x2 2| + | x 3 2|.

Bi 44. Gii phngrtrnh


17 1
13x2 6x + 10 + 5x2 13x + + 17x2 48x + 36 = (36x 8x2 21).
2 2

Bi 45. Gii phng trnh x2 4x + 20 + x2 + 4x + 29 = 97.

158
Hi tho khoa hc, Hng Yn 25-26/02/2017

2.3 S dng tnh hon v vng quanh


Bi ton 23 (VMO - 1994). Gii h phng trnh

3 2
x + 3x 3 + ln( x x + 1) = y

y3 + 3y 3 + ln(y2 y + 1) = z . Gii. Xt hm s: f (t) = t3 + 3t 3 + ln(t2

3
z + 3z 3 + ln(z2 z + 1) = x
t + 1) trn R
2t2 1
f 0 (t) = 3t2 + 3 + 2 > 0 t R
t t+1
Suy ra hm s y = f (t) ng bin trn R. Ta vit li h nh sau:

f (x) = y

f (y) = x .

f (z) = x

Khng mt tnh tng qut gi s x = min { x, y, z}. Lc :
Do x y f ( x ) f (y) y z f (y) f (z) z x hay x y z x x =
y = z.
Thay x = y = z th vo phng trnh u tin ta c: x3 + 3x 3 + ln( x2 x + 1) =
0 f ( x ) = 0.
M f(x) l hm ng bin trn R nn phng trnh c nhiu nht mt nghim. D
thy x = 1 l nghim ca phng trnh nn phng trnh c nghim duy nht x = 1.
Vy h phng trnh c nghim duy nht: ( x; y; z) = (1; 1; 1).

Bi ton 24 (Olympic c - 2000). Gii h phng trnh



3
x = 2y 1

y3 = 2z 1 .

3
z = 2x 1
Gii.
Trc ht ta chng minh x = y = z.
Tht vy, khng mt tnh tng qut gi s x = max { x, y, z}, gi s x 6= y.
x3 + 1 y3 + 1
Nu x > y y = > = z y > z, v tng t th z > x ( mu thun).
2 2
Suy rax = y = z th vo phng trnh u tin ca h ta c: x3 = 2x 1 x =
1 5
hoc x = 1.
2 ( !)
1 5 1 5 1 5
Vy h phng trnh c nghim: ( x; y; z) (1; 1; 1); ; ; .
2 2 2

Bi ton 25 (VMO - 2006 - Bng B). Gii h phng trnh



3 2
x + 3x + 2x 5 = y

y3 + 3y2 + 2y 5 = z

3
z + 3z2 + 2z 5 = x
Gii.
Gi s x = max { x; y; z}. Xt 2 trng hp:
( Nu x y z v t h ta c:
+) ( (
x3 + 3x2 + 2x 5 x ( x 1)(( x + 2)2 + 1) 0 x1
x = y =
z3 + 3z2 + 2z 5 z (z 1)((z + 2)2 + 1) 0 1z

159
Hi tho khoa hc, Hng Yn 25-26/02/2017

z(= 1. +) Nu x z y v t h
( ta c: (
3 2
x + 3x + 2x 5 x ( x 1)(( x + 2)2 + 1) 0 x1
3 2
2
x = y =
y + 3y + 2y 5 y (y 1)((y + 2) + 1) 0 1y
z = 1.
Th li, x = y = z = 1 l nghim ca h cho.
Vy h phng trnh c nghim duy nht: ( x; y; z) = (1; 1; 1).

Bi tp cng phng php



3 2
x 9(y 3y + 3) = 0

Bi 46. Gii h phng trnh y3 9(z2 3z + 3) = 0 .

3
z 9( x2 3x + 3) = 0
(
2x 2 = 3y 3x
Bi 47. Gii h phng trnh
2y 2 = 3x 3y

2
x + x 1 = y

Bi 48. Gii h phng trnh y2 + y 1 = z

2
z +z1 = x


x12 = x2 + 1

2
x2 = x3 + 1



Bi 49. Gii h phng trnh . . . . . . . . . . . . . . . ..

xn2 1 = xn + 1





2
x n = x1 + 1

2.4 Bi ton cha tham s


Bi ton 26 (Olympic Bulgari - 2000). Tm tt c cc s thc m phng trnh sau c 3
nghim phn bit:
( x2 2mx 4(m2 + 1)( x2 4x 2m(m2 + 1).
Gii.
Phng trnh tng ng vi: x2 2mx 4(m2 + 1 = 0 ( x m)2 = 5m2 + 4 (1)
hoc x2 4x 2m(m2 + 1 = 0 ( x 2)2 = 2(m3 + m + 1) (2)
Phng trnh ban u c 3 nghim phn bit th xy ra cc trng hp sau: (1) c nghim
kp khc vi 2 nghim phn bit ca (2) hoc (2) c nghim kp khc vi 2 nghim phn
bit ca (1) hoc (1),(2) u c 2 nghim phn bit nhng c 1 nghim chung.
+) Do 5m2 + 4 > 0 nn (1) ch c th c 2 nghim phn bit ch khng th c nghim
kp suy ra trng hp th nht khng xy ra.
+) (2) c nghim kp khi v ch khi 2(m3 + m + 1) = 0 m = 1. khi (2) c nghim
kp x = 2; (1) ( x + 1)2 = 9 x = 2; x = 4. Suy ra phng trnh ban u ch c 2
nghim phn bit l x = 2; x = 4 ( khng tha mn)
+) Vi trng hp cui cng, ta gi r l nghim chung ca (1) v (2) th (x - r) l
tha s chung ca 2 biu thc: x2 2mx 4(m2 + 1; x2 4x 2m(m2 + 1. Tr 2 biu
thc cho nhau ta c (x - r) l tha s ca (2m 4) x (2m3 4m2 + 2m 4) hay
(2m 4)r = (2m 4)(m2 + 1). V vy, m = 2 hoc r = m2 + 1.

160
Hi tho khoa hc, Hng Yn 25-26/02/2017

Nu m = 2 th c (1) v (2) u tr thnh ( x 2)2 = 24 nn phng trnh ban u ch c


2 nghim phn bit, suy ra m = 2 khng tha mn.
Nu r = m2 + 1 (r 2)2 = 2(m3 + m + 2) (m2 1)2 = 2(m3 + m + 2)
(m + 1)(m 3)(m2 + 1) = 0 m = 1 hoc m = 3. Nhng m = -1 ta loi trn nn
suy ra m = 3.
Vi m = 3 (1) ( x 3)2 = 49 x = 4; x = 10; (2) ( x 2)2 = 64 x = 6; x = 10
Suy ra phng trnh ban u c 3 nghim phn bit x = 4; x = 6; x = 10 (tha mn)
Vy vi m = 3 thi phng trnh c 3 nghim phn bit.


x + y + z = a (1)

Bi ton 27 (IMO - 1961). Gii h x2 + y2 + z2 = b2 (2) trong a,b l nhng hng

xy = z2 (3)

s cho trc. Cc s a, b phi tha mn iu kin g cc nghim x, y, z ca h l dng
phn bit?
Gii.
Bnh phng 2 v ca (1) ta c: a2 = x2 + y2 + z2 + 2xy + 2( x + y)z. M x + y = a z
a2 b2
v t (2), (3) ta c: a2 = b2 + 2z2 + 2( a z)z a2 = b2 + 2az z = .
2a
2 2
x + y = a z = a + b

Khi ta c: 2a
2 2 2
xy = z2 = ( a + b )

4a2
2 2 2 2 4 4
x = a + b 10a b 3a 3b

4a 4a .
a2 + b2 10a2 b2 3a4 3b4

y =

4a 4a
a2 + b2
x,y,z > 0 th x + y > 0 > 0 a > 0. Vi iu kin ny th x > 0, y > 0 v
2a
xy > 0.
a2 b2
Mt khc, z = > 0 a2 > b2 a > |b| (4)
2a
x 6= y 10a2 b2 3a4 3b4 > 0 (5)
|b|
t t = theo (4) ta c: 1 > t 0 v c th vit (5) di dng: 3t4 + 10t2 3 > 0
a
 
  
 1 1 
3 t + 3 t + t t 3 > 0 (6)
3 3
1 1
V t > 0 nn t + 3; t + > 0 v v t < 1 nn t 3 < 0 (6) t > 0 t >
3 3
1 1 |b| 1 a
. Nh vy 1 > t > 1 > > a > |b| > , a > 0.
3 3 a 3 3
Vy nghim ca h l:
!
a2 + b2 10a2 b2 3a4 3b4 a2 + b2 10a2 b2 3a4 3b4 a2 b2
( x, y, z) = ; ;
4a 4a 4a 4a 2a
a
v a > |b| > , a > 0 l iu kin cc nghim x, y, z ca h l dng phn bit.
3

Bi ton 28 (VMO - 1995 - 1996 - Bng A). Bin lun s nghim thc ca h

161
Hi tho khoa hc, Hng Yn 25-26/02/2017
(
x 3 y y4 = a2
trong a,b l nhng hng s cho trc.
x2 y + 2xy2 + y3 = b2
Gii. (
y ( x 3 y3 ) = a2 (1)
H tng ng:
y ( x + y )2 = b2 (2)
Xt cc trng hp ( sau:
y=o
1. b = 0. khi : (2)
y = x
Do
( vy h cho tng ng (
y=0 y = x
3 3 2
(I) hoc (II)
y( x y ) = a y ( x 3 y3 ) = a2
(
y = x
(II)
2x4 = a2
+) Nu a 6= 0 th (I) v (II) cng v nghim nn h v nghim.
+) Nu a = 0 th (I) c v s nghim dng ( x mathbbR; y = 0) cn (II) c 1 nghim (0;
0) nn h v nghim.
2. b 6= 0. Khi , t (1), (2) ta thy nu (x; y) l nghim ca " h th phi c x#> 0, y > 0. V
3
|b| |b|
th (2) x = y (3). Th vo (3) vo (1) ta c: y y y3 = a2 (4)
y y

t y = t, t > 0. T (4) ta c phng trnh sau:
" 3 #
| b |
t2 t2 t6 = a2 t9 (|b| t3 )3 + a2 t = 0 (5)
t
Xt f (t) = t9 (|b| t3 )3 + a2 t trn [0; +) c: f 0 (t) = 9t8 + 9(|b| t3 )2 t2 + a2 0t
[0; +). Suy ra ng binp trn [0; +) nn (5) c ti a 1 nghim trong (0; +). M
f (0) = (|b|)3 < 0, f ( 3 |b| = (|b|)3 + |b| a2 > 0 nn (5) c 1 nghim  duy nht.K hiu
|b|
nghim l: t0 (0; +) Suy ra h c nghim duy nht: ( x; y) = t0 ; t20 .
t0
Vy:
Nu a = b = 0 th h c v s nghim.

Nu a ty , b 6= 0 th h c nghim duy nht.

Nu a 6= 0, b = 0 th h v nghim.

2.5 Mt s phng trnh c cch gii c bit


p p
Bi ton 29. Gii phng trnh x2 3 2x + 9 + x2 4 2x + 16 = 5.
Gii.
Nu x 0 th VT 3 + 4 = 7 > 5 = VP nn phng trnh v nghim.
Nu x > 0 th ta xt tam gic vung ABC vi A = 900 , AB = 4, AC = 3. Gi AD l phn
gic ca gc A, ly M thuc AD. t AM = x.

162
Hi tho khoa hc, Hng Yn 25-26/02/2017


Xt tam gic ACM c: CM2 = AM2 + AC2 2.AM.AC.cos450 = x2 + 9 3 2x.
Xt tam gic ABM c: BM2 = AM2 + AB2 2.AM.AB.cos45
0 = x 2 + 16 4 2x.

Suy ra VT = CM + BM BC = AB2 + AC2 = 32 + 42 = 5.


CM AB 3
Du bng xy ra M D hay = = 16CM2 = 9BM2
BM AC 4
2
2
12 2
16x + 16.9 48 2x = 9x + 16.9 36 2x 7x 12 2x = 0 x = .
7
12 2
Vy phng trnh c nghim duy nht: x = .
7
p
Bi ton 30. Gii phng trnh 4 x 2 + 4x + 1 + x2 + y2 2y 3 = 5 y +
4
x4 16.
Gii.


4 x2 0

4x + 1 0
iu kin: x = 2.


x2 + y2 2y 3 0
x4 16 0


y 2

3
Khi phng trnh tr thnh: |y 1| = 2 y y 1 = 2 y y = .
2
y1 = y2

3
Vy phng trnh c nghim x = 2; y = .
2
p p
Bi ton 31. Gii phng trnh 1 + 2x x2 + 1 2x x2 = 2( x 1)4 (2x2
4x + 1).
Gii. (
p 0y1
t y = 2x x2 = 1 ( x 1)2
( x 1)2 = 1 y2
Phng trnh tr thnh: 1 + y + 1 y = 2(1 y2 )2 (1 2y2 ) (1)
p p
p
M 1 + y + 1 y 1 + 1 y2 2 y2 (2)
p p

T (1), (2) 2(1 (y2 )2 (1 2y2 ) 2 y2 .


0z1
t y2 = z
2(1 z)2 (1 2z) 2 z

163
Hi tho khoa hc, Hng Yn 25-26/02/2017

z(4z2 10z + 7) 0 z 0. Do z = 0 nn y = 0 hay 2x x2 = 0 x = 0


hoc x = 2.
Vy phng trnh c nghim: x = 0; x = 2.

Bi ton 32 (Olympic Ukrain 1998 -1999). Gii phng trnh


Tm(tt c cc nghim thc ca h
x1 + x2 + + x1997 = 1997
4
x14 + x24 + + x1997 = x13 + x23 + + x1997
3

Gii.
Ta s chng minh h trn ch c nghim: x1 = x2 = = x1997 .
t Sn = x1n + + x1997
n Theo bt ng thc ly tha trung bnh ta c:
  41   14   13   43
S4 S1 S4 S3 S4
= 1 v = .
1997 1997 1997 1997 1997
S4
Do 1.
1997
V vy bt ng thc xy ra du bng x1 = x2 = = x1997 = 1.
Vy h c nghim duy nht ( x1 ; x2 ; . . . ; x1997 ) = (1; 1; . . . ; 1).

3
3
Bi ton 33. Gii phng trnh 3 7x + 1 x2 x 8 + x2 8x 1 = 2.
Gii.
3 3 2 x 8; t = 3 x 2 8x 1
t y =( 7x + 1; z = x
y+z+t = 2
.
y3 + z3 + t3 = 8
T y + z + t = 2 (y + z + t)3 = 8 (y + z + t)3 (y3 + z3 + t3 ) = 0
3(y + z)(z + t)(t + y) = 0 y = z; z = t; t = y.
+) Nu y = -z ta c: x = -1; x = 9
+) Nu t = -y ta c: x = 1; x = 0
+) Nu z = -t ta c: x =1
Vy tp nghim ca phng trnh l: S = {1; 0; 1; 9}.

ton 34 (IMO
Bi - 1965). Tm tt c cc gi tr ca x [0; 2 ] sao cho: 2 cos x
| 1 + sin 2x 1 sin 2x | 2.
Gii.
t y = | 1 + sin 2x 1 sin 2x | y2 = 2 2| cos 2x |.
Nu cos x 0 m 2 cos x 2 | cos2x | = |2 cos2 x 1| = 1 2 cos2 x. Khi bi
1 h i 3 7
tr thnh: cos x x ; ; .
2 4 2 4 4
Nu cos x < 0    
1 1 3 5 3
+) Gi s | cos x | ta c: cos x | cos x | x ; ; .
2 2 2 4 4 2
1
+) Gi s | cos x | > ta c: | cos 2x | = 2 cos2 x 1 y = 2| sin x | x
  2
3 5
; .
4 4  
7
Vy bt phng trnh c nghim l: x ; .
4 4

164
Hi tho khoa hc, Hng Yn 25-26/02/2017

3 2
x + x (y z) = 2

Bi ton 35 (VMO - 2004 - Bng A). Gii h phng trnh y3 + y(z x )2 = 30 .

3
z + z( x y)2 = 16
Gii.
H tng ng vi:

2 + y2 + z2 ) 2xyz = 2 2 2 2

x ( x x ( x + y + z ) 2xyz = 2

y( x2 + y2 + z2 ) 2xyz = 30 (y z)( x2 + y2 + z2 ) = 14 .

2
2 2
z( x + y + z ) 2xyz = 16
2 2 2
(z x )( x + y + z ) = 14
D thy (0; 0; 0) khng l nghim ca h, do h tng ng vi:

x ( x2 + y2 + z2 ) 2xyz = 2
2x3 2x2 z + xz2 = 2

(y z)( x2 + y2 + z2 ) = 14 5z3 16xz2 + 20x2 z 16x3 = 0()

y = 2z x y = 2z x

z
V x, z 6= 0 nn t t = , t (*) ta c: 5t3 16t2 + 20t 16 = 0 (t 2)(5t2 6t +
x
8) = 0 t = 2 z = 2x Khi h tng ng:

3 2 2
2x 2x z + xz = 2
x = 1

z = 2x y=3

y = 2z x z=2

Vy h c nghim duy nht: ( x; y; z) = (1; 3; 2).

3 Bi tp tng t
Bi 50 (Hc sinh
( gii khu vc Duyn hi v ng bng Bc B - 2012 - Lp 10). Gii h
x3 y3 3y2 = 9
phng trnh
x2 + y2 = x 4y

Bi 51 (Hc sinh gii khu vc Duynhi v ng bng Bc B - 2013 - Lp 10). Gii


phng trnh 3x2 10x + 6 + ( x + 2) 2 x2 = 0

Bi 52 (Hc sinh gii khu


vc Duyn hi v ng bng Bc B - 2011 - Lp 10). Gii
phng trnh 4x2 + 12x x + 1 = 27( x + 1).

Bi 53 (Hc sinh
gii khu vc Duyn
hi v ng
bng Bc B - 2010 - Lp 10). Gii
phng trnh 5x2 + 14x + 9 x2 x 20 = 5 x + 1.
r r
1 1 x1
Bi 54 (VMO - 1977 - Bng A). Gii bt phng trnh x 1 > .
x x x

2 2 2 2
x + y + z + t = 50


x2 y2 + z2 t2 = 24

Bi 55 (VMO - 1981 - Bng A). Gii h phng trnh .


xz = yt
x y + z + t = 0

x2 2x 8
Bi 56 (APMO). Gii phng trnh 2
= ( x 1) x + 2 2.
x 2x + 3

165
Hi tho khoa hc, Hng Yn 25-26/02/2017

Ti liu
[1] V Dng Thy, Nguyn Vn Nho (2001), 40 nm Olympic Ton hc quc t, NXB
Gio dc.
[2] V Dng Thy, Nguyn Vn Nho (2002), Tuyn tp cc bi ton t nhng cuc thi ti
M v Canaa, NXB Gio dc.
[3] Nguyn Vn Nho, OLYMPIC ton hc Chu Thi Bnh Dng, NXB Gio dc.
[4] B GD v T (2011), Ti liu tp hun gio vin.
[5] Tp ch Ton hc v tui tr, Tuyn tp chuyn .

166
Hi tho khoa hc, Hng Yn 25-26/02/2017

MT S DNG TON V BT NG THC I VI


HM PHN THC BC HAI TRN BC NHT

L Th Mai
THPT An Lo, Hi Phng

Tm tt ni dung
Trong bo co ny trnh by kt qu kho st tng t trong [1]-[2]. Trc ht ta xt
bi ton v xc nh cc khong (, ) vi ( < +) sao cho ng vi mi
g( x )
hm s bc hai g( x ) = ax2 + bx + c, a > 0, trn bc nht dng f ( x ) = , g(d) 6= 0,
xd
ta u c bt ng thc
f (u) f (v)
0
0 + u v, u, v (, ), u 6= v ()
f (v) f (v)
hoc
f (u) f (v)
0
0 + u v, u, v (, ), u 6= v ()
f (v) f (v)
v xt cc p dng lin quan.

1 Kt qu c bn
Xt hm s li kh vi ( x ) ( 00 ( x ) 0) trn (, ). Khi , ta c
nh l 1 (Bt ng thc Karamata). Xt hai dy s { xk , yk ( a, b), k = 1, 2, . . . , n}, tha
mn iu kin
x1 x2 x n , y1 y2 y n
v

x1 y1

x1 + x2 y1 + y2



......

x 1 + x 2 + + x n 1 y 1 + y 2 + + y n 1





x1 + x2 + + x n = y1 + y2 + + y n

Khi , ng vi mi hm s li kh vi ( x )( 00 ( x ) 0) trn (, ), ta u c
( x1 ) + ( x2 ) + + ( x n ) ( y1 ) + ( y2 ) + + ( y n ).
ng thc xy ra khi xi = yi , i = 1, 2, . . . n.
Ta cng pht biu tng t i vi hm s lm bng cch i chiu du bt ng
thc. Chng minh nh l Karamata da vo b c bn sau y.

166
Hi tho khoa hc, Hng Yn 25-26/02/2017

B 1 (B c bn, xem [3-5]). Cho hm s y = ( x ) lin tc v c o hm cp 2


trn (, ).
a. Nu 00 ( x ) 0, x (, ) th ( x ) 0 ( x0 )( x x0 ) + ( x0 ), x, x0 (, ).
b. Nu 00 ( x ) 0, x (, ) th ( x ) 0 ( x0 )( x x0 ) + ( x0 ), x, x0 (, ).
ng thc trong hai bt ng thc trn xy ra khi x = x0 .

Nhn xt rng x d 6= 0 vi mi x (, ) th d
/ (, ). iu ny tng ng
vi hoc d hoc d .
T suy ra
Mnh 1. iu kin cn c bt ng thc () vi mi u, v (, ) v u 6= v l hoc
d hoc d .
Tip theo, ta xc nh iu kin c () .
Ta c
g(d)
f ( x ) = ax + ad + b +
xd
v
g(d) 2g(d)
f 0 (x) = a , f 00 ( x ) = . (1.1)
( x d )2 ( x d )3
Trng hp 1. Xt g(d) < 0. Khi f 0 ( x ) > 0 vi mi x 6= d. Ta c f 00 ( x ) > 0 khi
x < d v f 00 ( x ) < 0 khi x > d.
Vy nn khi x < d th f 00 ( x ) > 0 v f 0 ( x ) > 0. Theo B c bn, ta ta c () lun
ng ng vi mi u, v < d, u 6= v.
Khi x > d th f 00 ( x ) < 0 v f 0 ( x ) > 0. v () lun ng vi mi u, v > d, u 6= v.
Trng hp 2. Xt g(d) > 0. Khi f 00 ( x ) < 0 khi x < d v f 00 ( x ) > 0 khi x > d.
Xt phng trnh
r
g(d)
f 0 ( x ) = 0 x1,2 = d , x1 < d < x2 .
a
Khi f 0 ( x ) > 0 khi x > x2 hoc x < x1 v f 0 ( x ) < 0 x1 < x < x2 .
Vy nn
- khi x < x1 th f 0 ( x ) > 0 v f 00 ( x ) < 0. Khi () ng vi mi u, v (, ),
- khi x > x2 th f 0 ( x ) > 0 v f 00 ( x ) > 0. Khi () ng vi mi u, v (, ),
- khi x ( x1 , d) th f 0 ( x ) < 0 v f 00 ( x ) < 0. Khi () ng vi mi u, v ( x1 , d),
- khi x (d, x2 ) th f 0 ( x ) < 0 v f 00 ( x ) > 0. Khi () ng vi mi u, v (d, x2 ).
Vy ta c th pht biu cc kt qu nhn c nh sau.
g( x )
nh l 2. Gi s f ( x ) = vi g( x ) = ax2 + bx + c, a > 0 v g(d) < 0.
xd
Khi , iu kin cn v (i vi (, )) c bt ng thc () l d v iu
kin cn v (i vi (, )) c bt ng thc () l d.
Tng t, ta cng c kt qu sau
g( x )
nh l 3. Gi s f ( x ) = vi g( x ) = ax2 + bx + c, a > 0 v g(d) > 0
xd
Khi , iu kin cn v (i vi (, )) c bt ng thc () l x1 hoc
d < < x2 , tng t, iu kin cn vr (i vi (, )) c bt ng thc () l
g(d)
x2 hoc x1 < d vi x1,2 = d .
a

167
Hi tho khoa hc, Hng Yn 25-26/02/2017

2 H qu
T cc nh l 2-3, ta pht biu cc h qu s dng trong cc bi ton p dng
phn sau.
g( x )
H qu 1. Gi s f ( x ) = vi g( x ) = ax2 + bx + c, a > 0 c g(d) < 0 v d
xd
hoc g(d) > 0 v x2 hoc x1 < < d.
Khi , vi mi b s u1 , u2 , u1 (, ) v v1 , v2 , v3 (, ) sao cho u1 + u2 + u3 =
v1 + v2 + v3 , ta u c

g(uk ) g(vk )
3 3
uk d vk d
g(d)

g(d)
.
k =1 a k =1 a
( v k d )2 ( v k d )2

g( x )
H qu 2. Gi s f ( x ) = vi g( x ) = ax2 + bx + c, a > 0 c g(d) < 0 v d
xd r
g(d)
hoc g(d) > 0 v x1 hoc d < < x2 vi x1,2 = d .
a
Khi , vi mi b s u1 , u2 , u1 (, ) v v1 , v2 , v3 (, ) sao cho u1 + u2 + u3 =
v1 + v2 + v3 , ta u c

g(uk ) g(vk )
3 3
uk d vk d
g(d)

g(d)
.
k =1 a k = 1 a
( v k d )2 ( v k d )2

3 V d p dng
Bi ton 1. Xt cc s x, y, z 1 c x + y + z = 10. Chng minh bt ng thc

9( x 1)2 4( y 1)2 9( z 1)2 157


+ + .
4( x + 1) 3( y + 1) 8( z + 1) 30

Li gii. Xt hm s
( x 1)2 4
f (x) = = x3+ .
x+1 x+1
Ta c
( x 1)( x + 3)
f 0 (x) = 0, x 1
( x + 1)2
v
8
f 00 ( x ) = > 0, x 1.
( x + 1)3
Theo h qu 1, ta c

f (u) f (v)
0
0 + u v, u, v [1, +), u 6= v ()
f (v) f (v)

168
Hi tho khoa hc, Hng Yn 25-26/02/2017
n4 3 8o
Chn 3 s v [1, +) sao cho f 0 (v) ; ; v c tng bng 10. Ta thu c
9 4 9
v1 = 2, v2 = 3, v3 = 5. Tip theo, th vo (), ta c

9( x 1)2 f (2)
0 + x 2, x [1, +)
4( x + 1) f (2)
4( y 1)2 f (3)
0 + y 3, y [1, +)
3( y + 1) f (3)
9( z 1)2 f (5)
0 + z 2, z [1, +)
8( z + 1) f (5)

hay

9( x 1)2 1/2
+ x 2, x [1, +)
4( x + 1) 5/9
4( y 1)2 1
+ y 3, y [1, +)
3( y + 1) 3/4
9( z 1)2 8/3
+ z 2, z [1, +)
8( z + 1) 8/9

Cng cc v tng ng ta thu c

9( x 1)2 4( y 1)2 9( z 1)2 1/2 1 8/3 157


+ + + + = ,
4( x + 1) 3( y + 1) 8( z + 1) 5/9 3/4 8/9 30

pcm.

Bi ton 2. Xt cc s x, y, z (1, 1) c x + y + z = 1. Tm gi tr ln nht ca biu thc

9( x 1)2 4(y 1)2 49(z 1)2


M := + + .
5( x + 1) 5( y + 1) 95(z + 1)

Li gii. Xt hm s
( x 1)2 4
f (x) = = x3+ .
x+1 x+1
Ta c
( x 1)( x + 3)
f 0 (x) = < 0, x (1, 1)
( x + 1)2
v
8
f 00 ( x ) = > 0, x (1, 1).
( x + 1)3
Theo h qu 2, ta c

f (u) f (v)
0
0 + u v, u, v (1, 1), u 6= v ()
f (v) f (v)

169
Hi tho khoa hc, Hng Yn 25-26/02/2017
n 9 4 49 o
Chn 3 s v (1, 1) sao cho f 0 (v) ; ; v c tng bng 1. Ta thu
5 5 95
1 1 1
c v1 = , v2 = , v3 = . Tip theo, th vo (), ta c
2 3 6
9( x 1)2 f (1/2) 1
0 + x , x (1, 1)
5( x + 1) f (1/2) 2
4( y 1)2 f (1/3) 1
0 + y , y (1, 1)
5( y + 1) f (1/3) 3
49(z 1)2 f (1/6) 1
0 + z , z (1, 1)
95(z + 1) f (1/6) 6

hay

9( x 1)2 9 1
+ x , x (1, 1)
5( x + 1) 30 2
4( y 1) 2 4 1
+ y , y (1, 1)
5( y + 1) 15 3
49(z 1)2 35 1
+ z , z (1, 1)
95(z + 1) 114 6

hay

9( x 1)2 9 1
x + , x (1, 1)
5( x + 1) 30 2
4( y 1) 2 4 1
y + , y (1, 1)
5( y + 1) 15 3
49(z 1)2 35 1
z + , z (1, 1)
95(z + 1) 114 6

Cng cc v tng ng ta thu c

9( x 1)2 4(y 1)2 49(z 1)2


M= + +
5( x + 1) 5( y + 1) 95(z + 1)

9 4 35

+ + .
30 5 114
9 4 35 1 1 1
Vy max M = + + t c khi x = , y = , z = .
30 5 114 2 3 6

4 Kt lun
Ta c th nhn cc kt qu tng t i vi lp cc hm s dng a thc bc cao,
dng phn thc bc hai trn bc hai,. . .

170
Hi tho khoa hc, Hng Yn 25-26/02/2017

Ti liu
[1] L Th Mai, 2014, Mt s dng ton v bt ng thc i vi hm s bc nht trn bc nht
trn mt khong, K yu HTKH Cc chuyn Ton hc bi dng HSG vng Ty
Bc", NXB HSP H Ni, 82-86, 2014.
[2] L Th Mai, 2015, Mt s dng bt ng thc phn thc c rng buc bi a thc Vite,
Lun vn Thc s HKH H Thi Nguyn, 2015.
[3] Nguyn Vn Mu, 1998, a thc i s v phn thc hu t, NXB Gio Dc.
[4] Nguyn Vn Mu, 2006, Bt ng thc, nh l v p dng, NXB Gio Dc.
[5] Nguyn Vn Mu, Trnh o Chin, Trn Nam Dng, Nguyn ng Pht, Chuyn
chn lc v a thc v p dng, NXB Gio Dc, 2008.
[6] Cc bi thi Olympic Ton trung hc ph thng Vit Nam (1990-2006), NXB Gio Dc.

171
Hi tho khoa hc, Hng Yn 25-26/02/2017

MT S PHNG PHP GII PHNG TRNH HM


TRN TP S NGUYN

ng Th Mn
THPT Chuyn Hng Yn

Tm tt ni dung

Cc hm s xc nh trn tp s nguyn l rt a dng. Ngoi cc vn chung ca


hm s nh: n iu, b chn, lin tc, tun hon, n nh,. . . chng cn lin quan n
cc khi nim ca s hc nh: tnh chia ht, s nguyn t, s chnh phng, quan h
ng d, hm nhn tnh,. . . Trong mt s trng hp, bn cht ca bi ton v cc hm
s xc nh trn tp s nguyn chnh l cc bi ton s hc. Mt khc, khi xt cc bi ton
v hm f : N R ta c th chuyn v bi ton ca dy s. Dng kt qu ca dy s, ta
thu c tnh cht cn thit gii cc bi ton lin quan n phng trnh hm. Di
y ta xt mt vi phng php gii phng trnh hm trn tp s nguyn.

1 S dng cc kt qu ca dy s
1.1 Da vo cng thc xc nh s hng tng qut ca dy s cho bi cng
thc truy hi
Dng 1. Tm Un bit aUn+1 + bUn + cUn1 = 0, n N; n 2 vi U1 , U2 cho trc.
Phng php gii
Gii phng trnh c trng
a2 + b + c = 0. (1)
a) Nu 1 , 2 l hai nghim thc phn bit ca (1) th Un = A1n + B2n , trong A, B c
xc nh theo U1 , U2 .
b) Nu l nghim kp ca (1) th Un = ( A + nB)n , trong A, B c xc nh theo U1 ,
U2 .
c) Nu = x + iy l nghim phc ca (1) th Un = r n ( A cos n + B sin n), vi
p y
r = || = x2 + y2 ; ( ; ) sao cho tan = v A, B c xc nh theo U1 , U2 .
2 2 x
Dng 2. Tm Un bit aUn+1 + bUn + cUn1 = f n , U1 , U2 cho trc, f n l a thc bc
n cho trc.
Phng php gii
Gii phng tnh c trng

172
Hi tho khoa hc, Hng Yn 25-26/02/2017

a2 + b + c = 0. (2)
0 1
Ta c Un = Un + Un trong
0
Un l nghim tng qut ca phng trnh thun nht dng 1.
aUn+1 + bUn + cUn1 = 0. (3)
Un1 l nghim ring ca phng trnh khng thun nht.
aUn+1 + bUn + cUn1 = f n (4)
0 1
Theo dng (1) ta tm c Un trong A, B cha xc nh. Un xc nh nh sau
a) Nu 6= 1 l nghim ca phng trnh (2) th Un1 l a thc cng bc vi f n .
b) Nu = 1 l nghim n ca phng trnh (2) thi f Un1 = n.gn , trong gn l a thc
cng bc vi f n .
c) Nu = 1 l nghim kp ca phng trnh (2) th Un1 = n2 .gn , trong gn l a thc
cng bc vi f n .
Thay Un1 vo phng trnh (4), ng nht h s tm c Un1 .
Bit U1 , U2 t h thc Un = Un0 + Un1 ta tnh c A, B.

V d 1. Tm tt c cc hm s f xc nh trn N tha mn cc iu kin sau


a) f (1) = 1
b) 2 f (n). f (n + k) 2 f (k n) = 3 f (n). f (k ); k, n N.

Li gii. Cho k = n = 0 t (1) ta c ( f (0))2 + 2 f (0) = 0, suy ra f (0) = 0 hoc f (0) = 2


Nu f (0) = 0 cho n = 0 t (1) ta c f (k ) = 0, k N suy ra f (1) = 0 mu thun vi gi
thit, vy f (0) = 2.
T (1) cho n = 1 ta c 2 f (k + 1) 3 f (k ) 2 f (k 1) = 0; k N.
1
Xt phng trnh c trng 22 3 2 = 0 c nghim 1 = v 2 = 2. Ta tm c f
2
1 n
di dng f (n) = A.2n + B. vi f (0) = 2; f (1) = 1. Ta c
2
A + B = 2
( 
A=0
1
2A B = 1 B = 2
2
1 1
Suy ra f (n) = (1)n+1 . , thay vo tha mn bi. Vy f (n) = (1)n+1 . , n N
2n 1 2n 1
l nghim ca bi ton.

V d 2. Tm tt c cc hm f : N Z tha mn
a) f (1) = 1
b) f (k + n) 2 f (n) f (k ) + f (k n) = 3n.2k , k; n N (1)
Li gii. T (1) cho n = k = 0 ta c f (0) ( f (0))2 = 0 suy ra f (0) = 0 hoc f (0) = 1.
Nu f (0) = 0 t (1) cho n = 0 th f (k ) = 0, k N suy ra f (1) = 0 mu thun vi gi thit,
nn f (0) = 1. T (1) cho n = 1 ta c f (k + 1) 2 f (k) + f (k 1) = 3.2k (2).
Xt phng trnh c trng 2 2 + 1 = 0 c nghim kp = 1. Ta tm c nghim di
dng f (k) = f 0 (k) + f 1 (k ), trong f (k) = A + B.k v f 1 (k ) = a.2k .
Thay f 1 (k) vo phng trnh (2) c a2k+1 2a2k + a2k1 = 3.2k , k N nn a = 6 do

173
Hi tho khoa hc, Hng Yn 25-26/02/2017

f (k ) = A + B.k + 6.2k . Li c f (0) = 1; f (1) = 1 nn A, B xc nh nh sau


 
A+6 = 1 A = 5

A + B + 12 = 1 B = 6

suy ra f (n) = 6.2n 6n 5; n N l nghim ca bi ton.

1.2 Kt hp cc tnh cht ca hm s nh: tun hon, n nh,. . . vi cc


tnh cht ca dy s nh: cp s cng, cp s nhn,. . . v cc bt ng
thc, cc hng ng thc i s v lng gic gii cc bi ton v
phng trnh hm
V d 3. C tn ti hay khng hm s f : Z Z sao cho
f (m + f (n)) = f (m) n; m, n Z. (1)
Li gii. Gi s tn ti hm f tha mn bi. T (1) cho m = 0 ta c
f ( f (n) = f (0) n. (2)
Vi n1 , n2 Z m f (n1 ) = f (n2 ) th f ( f (n1 )) = f ( f (n2 )), t (2) suy ra f (0) n1 =
f (0) n2 , do n1 = n2 nn f l n nh.
Cho n = 0 t (1) ta c f (m + f (0)) = f (m) m + f (0) = m
T ta c f (0) = 0 thay vo (2) c f ( f (n)) = n, n Z (3)
T (1) thay m bng f (m) v p dng (3) ta c f ( f (m) + f (n)) = m n
Xt m, n, p, q l cc s nguyn sao cho m + n = p + q, khi

f ( f (m) + f (n)) = m n = p q = f ( f ( p) + f (q))

Theo chng minh trn, f l n nh, nn suy ra f (m) + f (n) = f ( p) + f (q)


Do vi mi n Z ta c f (n + 1) + f (n 1) = f (n) + f (n) f (n + 1) f (n) =
f ( n ) f ( n 1) f ( n + 1) f ( n ) = f ( n ) f ( n 1) = = f (2) f (1) = f (1) f (0)
nn { f (n)} l mt cp s cng vi s hng u l U1 = f (0) = 0 v cng sai d = f (1) suy ra
f (n) = Un+1 = U1 + nd = nd, n 0.
Ta xt vi hai s n > 0, m 0 sao cho m + nd 0 thay vo (1) c

f (m + f (n)) = f (m + nd) = f (m) n f (m + nd) = md n (m + nd)d = md n

T c d2 = 1 (v l), do vy khng tn ti hm f tha mn yu cu ca bi.

V d 4. Tm tt c cc hm f : N R tha mn cc iu kin sau


a) f (0) = c
3 f (n) + 1
b) f (n + 1) = , n N (1)
3 f (n)
f (n) + 1
3 f (n) + tan 6 f (0) + tan 6
Li gii. T (1) ta c f (n + 1) = = suy ra f ( 1 ) = .
1 1 f (n) 1 f (n) tan 6 1 f (0) tan 6
3

174
Hi tho khoa hc, Hng Yn 25-26/02/2017


Do ta t f (0) = c = tan th f (1) = tan( + ),
6

f (1) + tan tan( + ) + tan
f (2) = 6 = 6 6 = tan( + 2 ).
6
1 f (1) tan 1 tan( + ) tan
6 6 6
Ta chng minh quy np cng thc
n
f (n) = tan( + ), n N. (2)
6
Tht vy, vi n = 0; 1; 2 cng thc (2) ng.
n
Gi s f (n) = tan( + ).
6
f (n) + tan 6 tan( + n
6 ) + tan 6
 
Ta c f (n + 1) = = = tan + ( n + 1 ) , hay (2)
1 f (n) tan 6 1 tan( + 6 ) tan
6
6
ng vi n + 1.

Nghim ca bi ton l f (n) = tan( + n ), n N.
6

2 S dng phng php chng minh quy np


Nguyn l quy np: Cho T N; 1 T v nu n T suy ra n + 1 T th T = N .
Khi gii cc bi ton lin quan n phng trnh hm ta c th s dng cc phng php
quy np khc nhau. Thng thng ta tm cch tnh mt vi gi tr ban u pht hin quy
lut tng qut f (n) v chng minh bng quy np theo n.

V d 5. Tm tt c cc hm f : N N tha mn ng thi cc iu kin sau


a) f (1) = 1,
b) f (m + n) = f (m) + f (n) + mn, m, n N .
Li gii. Cho m = 0 t b) ta c
f (n + 1) = f (n) + n + 1, n N (1)
suy ra f (n) = f (n 1) + n, n N, n > 1. (2)
Ta tnh vi gi tr ban u ca f (n).
1(1 + 1)
n = 1 t a) suy ra f (1) = 1 = ,
2
2(2 + 1)
n = 2 t (2) suy ra f (2) = f (1) + 2 = 3 = ,
2
3(3 + 1)
n = 3 t (2) suy ra f (3) = f (2) + 3 = 6 = .
2
Ta chng minh quy np
n ( n + 1)
f (n) = , n N . (3)
2
Vi n = 1 th (3) ng.
n ( n + 1)
Gi s f (n) = vi n 1.
2
n ( n + 1) (n + 1)(n + 2)
Ta c f (n + 1) = f (n) + n + 1 = +n+1 = , tc l (3) ng vi
2 2

175
Hi tho khoa hc, Hng Yn 25-26/02/2017

n + 1 nn (3) ng vi mi n N . Mt khc do c (1) nn nu f tha mn bi th f c


n ( n + 1)
xc nh duy nht. Vy hm s cn tm l f (n) = , n N .
2
V d 6 (IMO - 1981). Cho hm f (m, n) vi m, n Z tha mn ng thi cc iu kin sau
a) f (0, n) = n + 1,
b) f (m + 1, 0) = f (m, 1),
c) f (m + 1, n + 1) = f (m, f (m + 1, n)); m, n 0.
Hy tnh f (4, 1981).
Li gii. Theo c) c f (4, 1981) = f (3, f (4, 1980)). Ta i xc nh f (3, n), mun vy ta phi
tnh f (1, n) v f (2, n).
* Tnh f (1, n)
f (1, 0) = f (0, 1) = 1 + 1 = 2,
f (1, 1) = f (0, f (1, 0)) = f (0, 2) = 2 + 1 = 3,
f (1, 2) = f (0, f (1, 1)) = f (0, 3) = 3 + 1 = 4.
Ta chng minh quy np f (1, n) = n + 2, n N.
Gi s c f (1, n) = n + 2, ta phi chng minh f (1, n + 1) = n + 3.
Tht vy, c f (1, n + 1) = f (0, f (1, n)) = f (0, n + 2) = n + 3, l iu phi chng minh.
* Tnh f (2, n)
f (2, 0) = f (1, 1) = 3 = 2.0 + 3,
f (2, 1) = f (1, f (2, 0)) = f (1, 3) = 5 = 2.1 + 3,
f (2, 2) = f (1, f (2, 1)) = f (1, 5) = 7 = 2.2 + 3.
Ta chng minh quy np f (2, n) = 2n + 3, n N.
Gi s c f (2, n) = 2n + 3, ta phi chng minh f (2, n + 1) = 2n + 5.
Tht vy, c f (2, n + 1) = f (1, f (2, n)) = f (1, 2n + 3) = 2n + 5, l iu phi chng minh.
* Tnh f (3, n).
Ta c f (3, 0) = f (2, 1) = 5,
f (3, 1) = f (2, f (3, 0)) = f (2, 5) = 13 = 21+3 3,
f (3, 2) = f (2, f (3, 1)) = f (2, 13) = 29 = 22+3 3.
Ta chng minh quy np f (3, n) = 2n+3 3, n N.
Gi s c f (3, n) = 2n+3 3 ta chng minh f (3, n + 1) = 2n+4 3.
Tht vy, c f (3, n + 1) = f (2, f (3, n)) = f (2, 2n+3 3) = 2(2n+3 3) + 3 = 2n+4 3, suy
ra iu phi chng minh.
* Tnh f (4, n).
Ta c f (4, 0) = f (3, 1) = 24 3 = 13,
22
f (4, 1) = f (3, f (4, 0)) = f (3, 13) = 216 3 = 22 3,
2
16 22
f (4, 2) = f (3, f (4, 1)) = f (3, 216 3) = 22 3 = 22 3.
2
Chng minh quy np vi n N th f (4, n) = 2 2 3, trong c (n + 3) s 2.
2 2
Gi s f (4, n) = 22 3 vi (n + 3) s 2. Ta chng minh f (4, n + 1) = 22 3 vi (n + 4)
s 2.

176
Hi tho khoa hc, Hng Yn 25-26/02/2017

Tht vy, c
2
f (4, n + 1) = f (3, f (4, n)) = f (3, 22 3) vi (n + 3) s 2
2 3)+3 2
= 2(2 3 = 22 3 vi (n + 4) s 2

l iu phi chng minh.


2
T ta c f (4, 1981) = 22 3 vi 1984 s 2.
Nhn xt 1. Nguyn l quy np khng cn ng trong tp Z, nhng trong cc bi ton ta
c th chia tp Z thnh phn dng, phn m v quy np ri rc, c th lm nh sau: Gi
s bi ton ng vi n; n + 1; . . . ; 1; 0; 1; 2; . . . ; n ta chng minh bi ton cng ng vi
n 1 v n + 1, chng hn xt v d sau
V d 7. Tm tt c cc hm f : Z Z tha mn cc iu kin sau
a) f (0) = 1,
b) f ( f (n)) = n, n Z,
c) f ( f (n + 2) + 2) = n, n Z.
Li gii. T a) v b) ta c 0 = f ( f (0)) = f (1), vy f (0) = 1 v f (1) = 0. Cho n = 2 t c)
ta c f ( f (0) + 2) = 2 suy ra f (3) = 2 = 1 3.
Cho n = 3 t b) ta c f ( f (3)) = 3 suy ra f (2) = 3 = 1 (2).
Cho n = 1 t c) ta c f ( f (1) + 2) = 1 suy ra f (2) = 1 = 1 2
Cho n = 2 t b) ta c f ( f (2)) = 2 suy ra f (1) = 2 = 1 (1).
Ta chng minh quy np h thc sau ng f (n) = 1 n, n Z. (1)

ng thc (1) ng vi n = 2; 1; 0; 1; 2.
Gi s (1) ng vi n {2k; 2k + 1; . . . ; 1; 0; 1; . . . ; 2k; 2k + 1}.
Ta chng minh (1) ng vi n {2k 2; 2k 1; 2k + 2; 2k + 3}.
Tht vy, theo gi thit quy np ta c f (2k ) = 1 (2k ) = 2k + 1 v f (2k + 1) =
1 (2k + 1) = 2k suy ra
f (2k + 2) = f ( f (2k 1 + 2) + 2) = 2k 1 = 1 (2k + 2),
f (2k + 3) = f ( f (2k 2 + 2) + 2) = 2k 2 = 1 (2k + 3),
f (2k 2) = f ( f (2k + 3)) = 2k + 3 = 1 (2k 2).
f (2k 1) = f ( f (2k + 2)) = 2k + 2 = 1 (2k 1)
Chng t (1) ng vi n {2k 2; 2k 1; 2k + 2; 2k + 3}.
Vy f (n) = 1 n, n Z l nghim ca bi ton.

3 S dng nguyn l th t
nh l 1. Mi tp con khc rng ca N u c phn t nh nht.
nh l 2. Mi tp con khc rng v b chn ca N u c phn t ln nht v nh nht.
V d 8 (IMO - 1977). Gi s f : N N . Chng minh rng nu f (n + 1) > f ( f (n)), n
N th f (n) = n, n N .

177
Hi tho khoa hc, Hng Yn 25-26/02/2017

Li gii. Gi A l tp gi tr ca hm s. Khi A l mt tp con khc rng ca N nn A c


phn t nh nht.
T gi thit c f (2) > f ( f (1)), f (3) > f ( f (2)).
V vy phn t nh nht ca A khng th l mt trong cc s f (2), f (3), f (4), . . . M phn
t nh nht ca A l f (1) v n c xc nh duy nht.
T f (1) 1 suy ra f (n) > 1, n > 1.
V vy ta c th hn ch hm f trn N \{1}, f : N \{1} N \{1}.
Lp lun tng t nh trn, f (2) l phn t nh nht ca min gi tr hm ny, nn ta c
f (1) < f (2) suy ra f (n) > 2, n > 2.
Li tip tc hn ch hm f trn N \{1; 2}, f : N \{1; 2} N \{1; 2}
Lp li qu trnh trn ta c f (1) < f (2) < f (3) < . . . dn n f l hm tng v f (n)
n, n N .
Gi s f (n) > n vi n no , n N th ta c f (n) n + 1 suy ra f ( f (n)) f (n + 1), do f
l hm tng, iu ny mu thun vi gi thit f ( f (n) < f (n + 1).
V vy f (n) = n, n N . l iu phi chng minh.

Nhn xt 2. Mt s phng trnh hm c gii bng cch kt hp nguyn l quy np v


nguyn l th t cng vi mt s tnh cht ca hm s, ta xt v d sau

V d 9 (Ailen - 2001). Tm tt c cc hm f : N N tha mn f (m + f (n)) = f (m) +


n, m; n N.
Li gii. Gi s f (0) = a > 0
T gi thit cho n = 0 ta c f (m + f (0)) = f (m) suy ra f (m + a) = f (m), m N. Chng
t f l hm tun hon, v nh th tp gi tr ca f l

A = { f (0); f (1); f (2); . . . ; f ( a 1)}

Gi M l s ln nht trong A th f (n) M, n N.


Mt khc, t gi thit cho m = 0 ta c f ( f (n)) = f (0) + n = a + n + khi n +, iu
ny mu thun vi f (n) M, n N nn phi c f (0) = a = 0, khi f ( f (n)) = n, n N
Nu f (1) = 0 th 0 = f (0) = f ( f (1)) = 1, v l, vy f (1) = b > 0.
Ta chng minh quy np f (n) = bn, n N (1)
Tht vy, vi n = 0; 1 th (1) ng.
Gi s c f (n) = bn, khi f ( f (n)) = f (bn) suy ra n = f (bn).
f (n + 1) = f (1 + f (bn)) = f (1) + bn = b + bn = b(n + 1), vy (1) ng vi n + 1.
Do f (n) = bn, n N.
Li c f (bn) = b2 n = n nn b = 1 v f (n) = n. Hm s tha mn bi l f (n) = n, n N.

4 S dng tnh cht ca hm nhn tnh mnh


Ta bit i vi hm nhn tnh mnh rt thun li khi tnh gi tr ca n ti mt im

ty l: Cho n = p11 p22 . . . pk k l s phn tch tiu chun ca s t nhin n, nu f l hm
s nhn tnh mnh th f (n) = ( f ( p1 ))1 ( f ( p2 ))2 . . . ( f ( pk ))k . Do xc nh gi tr ca
hm f ta ch cn xc nh gi tr ca n ti cc im nguyn t.

178
Hi tho khoa hc, Hng Yn 25-26/02/2017

V d 10 (THTT). Tm mt hm s f : N N tha mn f (m f (n)) = n2 f (m), m; n N


(1).
Li gii. T (1) cho m = 1 ta c f ( f (n)) = n2 f (1), n N .
Nu n1 , n2 N m f (n1 ) = f (n2 ) th f ( f (n1 )) = f ( f (n2 )) n21 f (1) = n22 f (1).
Do f (n) N nn f (1) 6= 0 suy ra n1 = n2 , vy f l n nh.
T (1) cho m = n = 1 v do f l n nh ta c f ( f (1)) = f (1) f (1) = 1.
Suy ra f ( f (n)) = n2 , n N .
Mt khc vi mi m, n N ta c f ( f (m) f (n)) = n2 f ( f (m)) = n2 m2 = f ( f (mn))
f (m) f (n) = f (mn).
T ta c f l hm nhn tnh mnh. V vy ta quan tm n gi tr ca f ti cc im
nguyn t.
Gi p l mt s nguyn t ty . Nu f ( p) l hp s th tn ti a, b N , a b > 1 sao cho
f ( p) = ab.
Ta c p2 = f ( f ( p)) = f ( ab) = f ( a) f (b) xy ra cc trng hp sau
+ Nu f ( a) = p2 th f (b) = 1 suy ra b = 1, v l.
+ Nu f (b) = p2 th f ( a) = 1 suy ra a = 1, v l.
+ Nu f ( a) = f (b) = p, do f n nh nn a = b suy ra f ( p) = a2 .
Li xt, nu a = m.n vi m, n N th p = f ( a) = f (mn) = f (m) f (n) suy ra f (m) = 1 hoc
f (n) = 1, ngha l m = 1 hoc n = 1, vy a l s nguyn t.
T chng minh, nu p l s nguyn t th f ( p) hoc l s nguyn t hoc l bnh phng
ca mt s nguyn t.
Mt khc ta li c
+ Nu f ( p) = p th f ( f ( p)) = f ( p) suy ra p2 = p nn p = 1, mu thun vi p l s nguyn
t.
+ Nu f ( p) = p2 th f ( f ( p)) = f ( p2 ) suy ra p2 = ( f ( p))2 = p4 nn p = 1, mu thun vi p
l s nguyn t. Do f ( p) 6= p v f ( p) 6= p2 .
V l , ta c th xy dng hm s f nh sau
Gi pi l s nguyn t th i trong dy cc s nguyn t.
( p1 = 2, p2 = 3, p3 = 5, . . . ) th f ( p2k1 ) = p2k ; f ( p2k ) = p22k1 vi mi k = 1, 2, 3, . . .
Hm f xc nh nh trn tha mn iu kin ca bi ton.

V d 11. Xt hm f : N N tha mn f (m2 f (n)) = n.( f (m))2 , m, n N (1)


Xc nh gi tr nh nht c th c ca f (2005).
Li gii. t f (1) = a > 0. T (1) cho m = 1 f ( f (n)) = a2 n, n N (2)
T (1) cho n = 1 f ( am2 ) = ( f (m))2 , m N (3)
T (1), (2), (3) ta c:

( f (m) f (n))2 = ( f (m))2 ( f (n))2 = f ( am2 )( f (n))2


= f (n2 f ( f ( am2 ))) = f (n2 .a2 .a.m2 )
= f ( a.( amn)2 ) = f (( amn)2 f (1)) = ( f ( amn))2
f (m) f (n) = f ( amn)

Vy f ( an) = a f (n); a f (mn) = f ( amn) = f (m) f (n) (4)


T (3) v (4) ta c f ( an2 ) = a f (n2 ) = ( f (n))2 (5)

179
Hi tho khoa hc, Hng Yn 25-26/02/2017

Vi mi n N , ta chng minh cng thc sau bng quy np

ak+1 f (nk+1 ) = ( f (n))k+1 , k N (6)


Vi k = 1 cng thc (6) ng do c (5).
Gi s c ak f (nk+1 ) = ( f (n))k+1 . Khi ta c

ak f (nk+2 ) = ak .a. f (nk+2 ) = ak f ( ank+2 )


= ak . f ( a.n.nk+1 ) = ak . f (nk+1 ). f (n)
= ( f (n))k+1 . f (n) = ( f (n))k+2

Suy ra (6) ng vi k + 1, hay cng thc (6) ng vi mi k N . T ta c ak |( f (n))k+1 .


Ta chng minh a| f (n), tht vy
Gi s s m ca s nguyn t p trong phn tch tiu chun ca a l v s m ca p trong
phn tch tiu chun ca f (n) l . Khi s m ca p trong phn tch tiu chun ca ak l
k, cn s m ca p trong phn tch tiu chun ca ( f (n))k+1 l (k + 1). Nu > th lun
1
tn ti s nguyn dng k0 sao cho > (1 + ). Suy ra k0 > (k0 + 1) hay k0 khng l
k0
c ca ( f (n))k0 +1 mu thun (6). Vy tc l a| f (n).
f (n)
Khi t g(n) = N , ta c
a
f (1) f ( a) f ( f (1)) a2
g (1) = = 1; g( a) = = = =a
a a a a
f (m) f (n) a f (mn) f (mn)
g(m) g(n) = 2
= 2
= = g(mn), m, n N
a a a
f ( f (m)) am2
ag( g(m)) = g( a) g( g(m)) = g( ag(m)) = g( f (m)) = = = am
a a
Suy ra g( g(m)) = m hay g(m2 g(n)) = g(m2 ) g( g(n)) = ( g(m))2 .n = n.( g(m))2
f (n)
V vy g tha mn bi v n c gi tr g(n) = < f (n) vi a > 1 v g(1) = 1.
a
Do mun tm gi tr nh nht ca f (2005) ta s tm cc hm f tha mn bi v c
f (1) = a = 1.
Theo chng minh trn, f l hm n nh v l hm nhn tnh mnh.
Vi p nguyn t m f ( p) = mn; m, n N th ta c f ( f ( p)) = f (mn) = f (m) f (n) p =
f (m) f (n)
Suy ra f (m) = 1 hoc f (n) = 1, ngha l m = 1 hoc n = 1. V vy f ( p) l s nguyn t. Do
f nhn cc gi tr nguyn t phn bit ti cc im nguyn t phn bit.
Vi f tha mn trn th f (2005) = f (5.401) = f (5) f (401) 2.3 = 6. Ta ch ra tn ti mt
hm s tha mn bi c f (1) = 1 v f (2005) = 6 c xc nh nh sau
f (1) = 1; f (2) = 5; f (3) = 401; f (5) = 2; f (401) = 3;
f ( p) = p, p nguyn t p / {2, 3, 5, 401}, do vy gi tr nh nht c th c ca f (2005) l 6.

Nhn xt 3. Theo cch chng minh trn ta c th xc nh gi tr nh nht c th c ca f (n)


vi mi gi tr c th ca n l hm f tha mn bi.

180
Hi tho khoa hc, Hng Yn 25-26/02/2017

5 S dng mt s tnh cht ca s hc


Trong phn ny, ta xt mt s phng trnh hm gii c bng cch p dng cc tnh
cht ca s hc nh: tnh chia ht, nguyn t, quan h ng d, phn nguyn,. . .

V d 12. C bao nhiu hm f : N N tha mn ng thi cc iu kin sau


a) f (1) = 1
b) f (n) f (n + 2) = ( f (n + 1))2 + 1997, n N
Li gii. Gi D l tp hp tt c cc hm s f tha mn iu kin bi ton. Theo gi thit b)
ta c
f (n) f (n + 2) = ( f (n + 1))2 + 1997; f (n + 1) f (n + 3) = ( f (n + 2))2 + 1997
Suy ra f (n) f (n + 2) ( f (n + 1))2 = f (n + 1) f (n + 3) ( f (n + 2))2 = 1997
f ( n ) + f ( n + 2) f ( n + 1) + f ( n + 3)
= , n N
f ( n + 1) f ( n + 2)
V vy ta c
f (1) + f (3) f (2) + f (4) f ( n ) + f ( n + 2)
= = = = ...
f (2) f (3) f ( n + 1)
f (1) + f (3)
t c = (1) suy ra f (n + 2) = c f (n + 1) f (n), n N (2)
f (2)
p
Ta chng minh c N . Tht vy, nu c = vi p, q N v ( p, q) = 1 th t (2) ta c
q

q( f (n) + f (n + 2)) = p f (n + 1), n N

Suy ra q| f (n + 1), n N hay q2 | f (n) f (n + 2), n N v n 2.


.
V 1997 = f (n) f (n + 2) ( f (n + 1))2 ..q2 .
M 1997 l s nguyn t nn q2 = 1 hay q = 1 suy ra c N
Gi f (2) = a, do (1) ta c ac = 1 + f (3) suy ra ac 1 = f (3) = f (1) f (3) = ( f (2))2 + 1997
ac 1 = a2 + 1997 a(c a) = 1998
Ta c a|1998, hay f (2) l mt c dng ca 1998.
Ngc li vi mi c dng a ca 1998 ta xy dng hm f : N N nh sau
f (1) = 1; f (2) = a
1998
f (n + 2) = ( a + b) f (n + 1) f (n), n N , trong b = N .
a
Ta chng minh f tha mn iu kin bi, ngha l f D. Tht vy,

f (n + 1) f (n + 3) ( f (n + 2))2 = f (n + 1)(( a + b) f (n + 2) f (n + 1)) ( f (n + 2))2


= ( a + b) f (n + 1) f (n + 2) ( f (n + 1))2 ( f (n + 2))2
= f (n + 2)(( a + b) f (n + 1) f (n + 2)) ( f (n + 1))2
= f (n + 2) f (n) ( f (n + 1))2 , n N

181
Hi tho khoa hc, Hng Yn 25-26/02/2017

Suy ra
f (n + 1) f (n + 3) ( f (n + 2))2 = f (n + 2) f (n) ( f (n + 1))2
...
= f (3) f (1) ( f (2))2 = f (3) ( f (2))2
T ta c
f (n) f (n + 2) ( f (n + 1))2 = f (3) ( f (2))2
= ( a + b) f (2) f (1) ( f (2))2 = ( a + b) a 1 a2
= ab 1 = 1998 1 = 1997
Vy ta c f (n) f (n + 2) = ( f (n + 1)2 + 1997 hay f D
Ta c tng ng mi f D vi mt gi tr f (2)|1998 l mt song nh gia D v tp cc
c dng ca 1998. Do s phn t ca D l: | D | = d(1998) = d(2.33 .37) = (1 + 1)(3 +
1)(1 + 1) = 16.
V vy c tt c 16 hm s tha mn bi.
V d 13. Xc nh tt c cc hm f : Z Z tha mn ng thi cc iu kin sau
a) f (1995) = 1996
b) Vi mi n Z nu f (n) = m th f (m) = n; f (m + 3) = n 3.
Li gii. Vit li iu kin b) ta c f ( f (n)) = n; f ( f (n) + 3) = n 2, n Z
Suy ra f (n 3) = f ( f ( f (n) + 3)) = f (n) + 3, n Z
Hay f (n) = f (n 3) 3, n Z. T ta c
f (3) = f (0) 3
f (6) = f (3) 3
. . . f (3t) = f (3t 3) 3
Suy ra f (3t) = f (0) 3t, t Z. Lm tng t nh trn ta c f (3t + 1) = f (1) 3t; f (3t +
2) = f (2) 3t.
f (0) 3t, n = 3t; t Z

V vy ta c f (n) = f (1) 3t, n = 3t + 1; t Z (1)
f (2) 3t, n = 3t + 2; t Z


Do vy, tnh f (n) ta tnh f (0), f (1) v f (2).
.
V 1995..3 theo (1) ta c f (1995) = f (0) 1995 = 1996 f (0) = 3991 (2)
T (2) suy ra f ( f (0)) = f (3991) f (3991) = 0.
M 3991 = 3.1330 + 1 do f (3991) = f (1) 3990 = 0 f (1) = 3990 (3)
+ Nu f (2) = 3k, k Z th f ( f (2)) = f (3k ) = f (0) 3k 2 = 3991 3k hay 3k = 3989
(v l), vy f (2) 6= 3k.
+ Nu f (2) = 3k + 1, k Z th 2 = f ( f (2)) = f (3k + 1) = f (1) 3k 3k = f (1) 2 =
3988 (v l), vy f (2) 6= 3k + 1.
Do f (2) = 3k + 2, k Z (4)
T (1), (2), (3), (4) ta c
(
3991 n, n 6= 2 (mod 3)
f (n) =
3k + 4 n, n 2 (mod 3); k Z

182
Hi tho khoa hc, Hng Yn 25-26/02/2017

Th li ta thy f (n) xc nh nh trn tha mn bi.

Bi tp tng t
Bi 1. Tm tt c cc hm f : N R tha mn cc iu kin sau
a) f (1) = 1; f (2) = 0
b) f (n + 1) 2 f (n) + f (n 1) = n + 1, n N
Bi 2 (Australia - 1989). Xc nh tt c cc hm f : Z Z tha mn cc iu kin sau
a) f (k + n) + f (k n) = 2 f (k ). f (n), k; n Z
b) Tn ti s nguyn N sao cho | f (n)| < N, n Z.
Bi 3 (Chn T La M - 1986). Gi s f , g : N N l cc hm tha mn f l ton nh, g l
n nh v f (n) g(n), n N. Chng minh rng f g.
Bi 4. Tm tt c cc hm f : N N tha mn iu kin
f (m f (n)) = n3 f (m); m, n N .
Bi 5. Cho hm f : N R tha mn cc iu kin sau
a) f (1) = 21998 .
b) (1 + ( f (n))2 ) f (n + 1) = ( f (n))2 , n N .
Chng minh rng f (n) 1, n > 1998.
Bi 6 (Brasil - 1988). Tm tt c cc hm f : N N tha mn cc iu kin sau
a) f (mn) = f (m) + f (n), m; n N ,
b) f (30) = 0,
c) f (n) = 0 nu n c ch s tn cng bng 7.
Bi 7. Cho hm f : N R tha mn
a) f (n) = 0 n = 0,
b) f (mn) = f (m) f (n), m; n N,
c) f (m + n) = f (m) + f (n), m; n N.
Gi n0 l s nguyn dng b nht trong cc s nguyn dng n tha mn f (n) > 1. Chng
1+[logn n]
f ( n0 ) 0
minh rng vi mi s nguyn dng n ta u c bt ng thc sau f (n) < .
f ( n0 ) 1

Ti liu
[1] Bi Huy Hin, Bi tp i s v s hc - T1, NXBGD, 1985.
[2] Nguyn Vn Mu, Phng trnh hm, NXBGD - 2003
[3] Nguyn Trng Tun, Bi ton hm s qua cc k thi Olympic, NXBGD - 2004.
[4] B Gio dc v o to, Tp ch Ton hc v tui tr, NXBGD

183
Hi tho khoa hc, Hng Yn 25-26/02/2017

CP CA MT S NGUYN V NG DNG
GII MT S BI TON S HC

ng Th Mn
THPT Chuyn Hng Yn

1 C s l thuyt
nh ngha 1. S nguyn dng k b nht tha mn ak 1 (mod n) c gi l cp ca a theo
mod n, k hiu ordn ( a) = k.

Nhn xt 1. Cho n N . Nu a Z v ( a, n) > 1 th khng tn ti s k N ak 1


(mod n), v nu ak 1 (mod n) th ( ak , n) = (1, n) = 1, m ( a, n) > 1 nn ( ak , n) > 1 (v
l).
Nu a Z v ( a, n) = 1 th lun tn ti s k N ak 1 (mod n), chng hn
k = ( n ).
T nh ngha trn ta c cc kt qu sau
ordn ( a) = 1 a 1 (mod n)
1, a, a2 , . . . , aordn (a)1 l i mt phn bit theo mod n. (Chng lp thnh cc s i mt
phn bit chia cho n c ordn ( a) s d khc nhau)

nh l 1. a) Gi s cp ca a theo mod n l k, khi ah 1 (mod n) k |h.


b) Nu ordn ( a) = k; ordn (b) = h m (h, k ) = 1 th ordn ( ab) = hk.
c) Cho cc s n1 , n2 , . . . , nk i mt nguyn t cng nhau v n = n1 n2 . . . nk v ordni ( a) = hi . Khi
ordn ( a) = [h1 , h2 , . . . , hk ].
h
d) Nu ordn ( a) = h v u N th ordn ( au ) = .
(h, u)
.
Chng minh. a) Nu h..k th h = kq (q N ) v ordn ( a) = k ak 1 (mod n) nn akq 1
(mod n).
Nu ah 1 (mod n) v h = kq + r vi 1 r < k th ah = ( ak )q ar ar (mod n)
nn ar 1 (mod n) m 1 r < k, mu thun vi nh ngha s k.
.
Vy r = 0 hay h..k.
b) ordn ( a) = k ak 1 (mod n) nn ahk 1 (mod n)
ordn (b) = h bh 1 (mod n) nn bhk 1 (mod n) suy ra ( ab)hk 1 (mod n).
.
Gi t = ordn ( ab) th hk..t (1)
Li c ( ab)t 1 (mod n) nn ( ab)th 1 (mod n) v ath .(bh )t 1 (mod n).

184
Hi tho khoa hc, Hng Yn 25-26/02/2017

. .
Mt khc (bh )t 1 (mod n) nn ath 1 (mod n) suy ra th..k do (h, k) = 1 nn t..k (*)
.
Tng t c ( ab)tk 1 (mod n) nn btk 1 (mod n) do ( ak )t 1 (mod n) nn tk..h m
.
(k, h) = 1 nn t..h (**).
.
T (*) v (**) suy ra t..hk (2)
T (1) v (2) suy ra hk = t (pcm).
.
c) Tng t c h = ordn ( a) ah 1 (mod n) nn ah 1 (mod n ) suy ra h..h , i = 1, ..., k
i i
Suy ra h l mt bi chung ca hi , i = 1, . . . , k.
.
Gi k l mt bi chung ca hi th ak 1 (mod ni ) nn ak 1 (mod n) suy ra k..h.
Vy h = [n1 , n2 , . . . , nk ].
d) Gi k = ordn ( au ), d = (h, u) th h = dx, u = dy vi ( x, y) = 1, x, y N .
+) ( au )k = aku 1 (mod n) nn h|ku nn h|dky suy ra dky = m.h = mdx, m N .
Suy ra ky = mx hay x |ky m ( x, y) = 1 nn x |k (1).
+) aux = adxy = ( adx )y = ( ah )y 1 (mod n) nn ( au ) x 1 (mod n) do k| x. (2)
h h
T (1) v (2) suy ra k = x = = .
d (h, u)

Nhn xt 2.
Nu n = p11 p22 . . . pk k l phn tch tiu chun ca n. tm ordn ( a) th ta tm ord pi ( a),
i
vi i {1, . . . , k }.
a (n) 1 (mod n); ( a, n) = 1 nn ordn ( a)| (n) (rng ra, ordn ( a) (n)), hay ordn ( a)
( n 1).
Nu p nguyn t th ord p ( a)| p 1; ( a, p) = 1
a x ay (mod n) ordn ( a)|( x y).
m|n th ordm ( a)| ordn ( a), ordn ( a) = h nn ah 1 (mod n)
.
m|n th ah 1 (mod m) nn h.. ordm ( a)
nh ngha 2. S nguyn g c gi l cn nguyn thy ca n, nu cp ca n theo mod n bng
( n ).
V d 1. Hy xc nh ord101 (2) v ord125 (12).
Li gii.
Gi d = ord101 (2), c 101 l s nguyn t nn (101) = 100.
2d 1 (mod 101) suy ra d|100, ta chng minh d 6= 50, d 6= 20.
210 = 1024 14 (mod 101) nn 250 145 1962 .14 (6)2 .14 1 (mod 101) suy ra
d 6= 50
220 = 10242 142 6 (mod 101) nn d6= 20 v d = 100 nn ord101 (2) = 100.
1
Gi d = ord125 (12); (125) = 53 . 1 = 100 nn d|100
5
ord5 (12)|d, m 12 2 (mod 5), 122 4 (mod 5) nn 123 3 (mod 5); 124 1 (mod 5)
do ord5 (12) = 4 nn 4|d.
M d|100 nn d {4; 20; 100}.
124 1442 192 361 14 (mod 125)
1220 = (124 )5 (14)5 71.71.(14) 41.14 74 (mod 125)
suy ra d = 100, vy ord125 (12) = 100.

185
Hi tho khoa hc, Hng Yn 25-26/02/2017

nh l 2. Cho a l s l v n = 2s , ( a, n) = 1.
( 2 .c vi u, v N ; b, c l.
Gi s a 1 = 2u .b; a2 1 = v

1, u s
Khi h = ordn ( a) th h =
2max{1;s+1;v} , u < s

Chng minh.
Nhn xt rng a2 1 = ( a 1)( a + 1) = 2u .b( a + 1) = 2u .b.(2u .b + 2) = 2u+1 .b(2u1 .b + 1)
nn v > u
s s (1 1 ) s 1
( a, n) = 1 nn a (n) 1 (mod n) m a (n) = a (2 ) = a2 2 = a2 suy ra h|2s1 nn
h = 2t vi t s 1 (1)
.
Nu u s th a 1 = 2u .b .. 2s nn n| a 1 suy ra a 1 (mod n) nn h = 1.
Nu u < s th h 2 nn t 1
t t 1 t 2
( ah 1) = ( a2 1) = ( a2 + 1)( a2 + 1) . . . ( a2 + 1)( a2 1) (*)
i
a 1 (mod 2) (a l) nn a2 1 (mod 2)
.
Nu v s th a2 1 .. n nn a2 1 (mod n) suy ra h = 2
Nu v < s th s 2.
i i
V a l s l th a2 1 (mod 4) nn a2 1 (mod 4) v a2 + 1 2 (mod 4)
(*) nn ah 1 = 2t1 .a.2v .c vi a, c l s l, nn
.
ah 1..2s t 1 + v s t s v + 1 2
s v +1 .
t nh nht (do h nh nht) nn t = s v + 1 hay h = 2
1; u s

Vy h = 2; u < s v
1+ s v

2 ; u<v<s

Nhn xt 3. Ta c th dng b nng ly tha chng minh nh l 2.


B : (Nng ly tha) Cho p l s nguyn t.
Nu p = 2; a 1 (mod p) th
n l ta c v2 ( an 1) = v2 ( a 1).
n chn ta c v2 ( an 1) = v2 ( a2 1) + v2 (n) 1
Nu p > 2; a 1 (mod p) th vi mi n nguyn dng ta c

v p ( a n 1) = v p ( a2 1) + v p ( n )

nh l 3. Cho p s nguyn t l v ( a, p)(= 1, n = ps , ord p ( a) = r v ar 1 = pu .q; ( p, q) =


r; us
1, u N , ordn ( a) = h, khi ta c h =
r.psu ; u < s

Chng minh.
Trng hp 1: r = 1
Nu u s th n| ar 1 = a 1 nn a 1 (mod n) nn h = 1.
Nu u < s; h = ordn ( a), ( a, n) = 1, a (n) 1 (mod n)
(n) = ( ps ) = ps1 ( p 1) nn h| (n).
Gi s h = pt k, (k, p) = 1.

186
Hi tho khoa hc, Hng Yn 25-26/02/2017

Nu k > 1
t t t t
a 1 (mod p); ah 1 = ( a p 1)( a p .(k1) + a p .(k2) + + a p + 1) k (mod p)
t
th ah 1 = ( a p 1).b; b k (mod p) (b, p) = 1.
. .
M ah 1..p nn a p 1..p v a p 1 (mod p).
t t

Suy ra pt |h nn pt < h, v l vi cch chn h, nn k = 1, do h = pt k; t > 0. p dng b


nng ly tha, ta c p l, p > 1

v p ( a h 1) = v p ( a 1) + v p ( h ) = u + t

hay ah 1 = pu+t .A; ( p, A) = 1


.
M ah 1..ps nn u + t s suy ra t s u 1.
Vy t b nht l t = s u nn h = psu .
Trng hp 2: r > 1
.
ah 1 (mod n) nn ah 1 (mod p) suy ra h..n.
Gi s h = r.k; ah = ( ar )k = bk (vi b = ar ) nn b 1 = pu q; b 1 (mod p); (b, n) = 1.
C bk ah 1 (mod n), suy ra k l cp ca b theo mod n.
p dng trng hp 1 cho b (r = 1) ta c
( (
1; us r; us
k= s u
h = rk = s u
p ; u<s r.p ; u < s

nh l 4. a) Gi s r, n l hai s nguyn dng nguyn t cng nhau vi r l cn nguyn thy


mod n. Khi tp X = {r1 , r2 , . . . , r (n) } lp thnh mt h thng d thu gn theo moun n.
b) Nu r l mt cn nguyn thy mod n vi n 2, n N, khi , r u l cn nguyn thy mod n khi
v ch khi (u, (n)) = 1.
c) Nu s nguyn dng n c cn nguyn thy th c ng ( (n)) cn nguyn thy.
d) Cho p l s nguyn t l th p2 c ng ( ( p2 )) cn nguyn thy. Hn na, nu r l mt cn
nguyn thy mod p th trong p s r + kp vi k {0, 1, . . . , p 1} c ng ( p 1) s l cn nguyn
thy mod p2 .
e) Nu p l s nguyn t l, khi pk c cn nguyn thy k N . Hn na, nu r l mt cn
nguyn thy mod p2 th r l cn nguyn thy mod pk k N .
i) Nu r, r 0 l hai cn nguyn thy mod p (p nguyn t) th r.r 0 khng l cn nguyn thy mod p.
h) Cho s nguyn dng n 2. Khi n c cn nguyn thy khi v ch khi n c mt trong 4 dng
sau: 2, 4, p , 2p vi p nguyn t l, N .

2 Mt s ng dng
Bi ton 1. Cho p l s nguyn t. Chng minh rng mi c nguyn t ca 2 p 1 u c
dng kp + 1, k N (ni ring lun ln hn p).

Li gii. Gi q l c nguyn t ca 2 p 1 (q > 1)


2 p 1 0 (mod q) nn 2 p 1 (mod q) suy ra ordq (2)| p.

187
Hi tho khoa hc, Hng Yn 25-26/02/2017
(
ordq (2) = 1
M p nguyn t nn (*)
ordq (2) = p
.
Nu ordq (2) = 1 th 2 1 (mod q) nn 1 .. q v q = 1, v l.
Nu ordq (2) = p th ordq (2)| (q) nn p|q 1 vy nn q = kp + 1 (k 1).

Bi ton 2. Nu p l c nguyn t l ca n2 + 1 th p 1 (mod 4); n N .


Li gii. Ta c p|n2 + 1 nn p|(n4 1) vy nn ord p (n)|4.
Nu ord p (n) = 1 th n 1 (mod p) nn p|n 1. Suy ra p|n2 1 m p|n2 + 1 nn p|2, v l
v p nguyn t l.
Nu ord p (n) = 2 th n2 1 (mod p) nn p|n2 1 m p|n2 + 1 do p|2, v l.
nn ord p (n) = 4 m ord p (n)| ( p) = p 1 nn 4| p 1 do p = 4k + 1 hay p 1 (mod 4).

Bi ton 3. Chng minh rng


a) Nu p l c nguyn t l ca n4 + 1 th p 1 (mod 8)
n
b) Nu p l c nguyn t l ca a2 + 1 ( a Z, n N ) th p 1 (mod 2n ), hay p c dng
p = 2n+1 .k + 1, k N .
Li gii. a) p l c nguyn t l ca n4 + 1 nn p|n4 + 1 suy ra p|n8 1
v ord p (n)|8 nn ord p (n) = 2t ; t N; t 3.
Nu(t < 3 thi t 2 nn 2t |22 = 4 do ord p (n)|4 nn n4 1 (mod p).
p | n4 1
M suy ra p|2 mu thun vi p nguyn t l.
p | n4 + 1
nn t = 3 suy ra ord p (n) = 8| ( p) = p 1 v p 1 (mod 8) hay p = 8k + 1 (k N ).
b) Chng minh tng t c ord p (n) = h ah 1 (mod p)
n +1 n +1
p| a2n + 1 v p|( a2n + 1)( a2n 1) = a2 1 do a2 1 (mod p)
nn h|2n+1 suy ra h = 2t (t n + 1).
n
Nu(t n th 2t |2n nn a2 1 (mod p).
n
p | a2 1
M n nn p|2, mu thun vi p nguyn t l nn t = n + 1.
p | a2 + 1
nn h = 2n+1 | ( p) = p 1 do p 1 (mod 2n+1 ) hay p = 2n+1 .k + 1.

n
Nhn xt 4. Mi c l ca a2 + 1 u c dng 2n+1 .k + 1; k N.
n
Khi a = 2, s Fecma th n l Fn = 22 + 1 c tnh cht: p l c nguyn t l ca Fn (n > 1)
th p 1 (mod 2n+2 ).

Bi ton 4. Cho p nguyn t, n N v q l c nguyn t l ca n p + 1 th q|n2 1 hoc


2p|(q 1).
Li gii. Gi s q|n p + 1 th n p 1 (mod q) nn n2p 1 (mod q)
h = ordq (n) nn h|2p, m h khng l c ca p v p nguyn t nn h = 2 hoc h = 2p.
Nu h = 2 th n2 1 (mod q) nn q|n2 1 (pcm).
Nu h = 2p th h| (q) = q 1 nn 2p|(q 1) (pcm).

Bi ton 5. Chng minh rng: n > 2 u khng l c ca 2n 1.

188
Hi tho khoa hc, Hng Yn 25-26/02/2017

Li gii. Gi s c n 2 v n|2n 1 m 2n 1 l s l, suy ra n l. Gi p l c nguyn t


nh nht ca n th p l, p 3.
2n 1 0 (mod p) ord p (2)|n v ord p (2)| p 1 = ( p).
1 < ord p (2)|( p 1, n), v l v p l c nguyn t nh nht ca n.
Suy ra ( p 1, n) = 1. Vy n khng th l c ca 2n 1.

Bi ton 6. Tm cc s nguyn dng a 6= b vi a + b nh nht sao cho 2012a v 2012b c ba


ch s tn cng ging nhau.
Li gii. Gi s a > b.
2012a 2012b (mod 1000) 12a 12b (mod 1000) nn 12b (12ab 1) 0 (mod 1000)
M 1000 = 23 .53 , m 12b l s chn, 12ab 1 l s l nn 23 |12b do b 2.
M (5; 12) = 1 nn 53 |(12ab 1)
.
Tm ord 3 (12), theo v d 1 ta c ord 3 (12) = 100 nn a b .. 100
5 5
vy a b 100 m b 2 nn 2b 4 suy ra a b + 2b 104; a + b 104.
Du "=" khi b = 2, a = 102.
M a 6= b; a + b nh nht suy ra a = 102, b = 2.
Kt lun: a = 102, b = 2 hoc a = 2, b = 102.
(
n | 2m 1 + 1
Bi ton 7. Tm tt c cc s nguyn dng (m, n) tha mn .
m | 2n 1 + 1
(
n | 20 + 1 n | 2

n=1
Li gii. Nu m = 1 th n 1
nn
m |2 +1 n=2
(
n | 2m 1 + 1

m=1
Nu n = 1 th 0
nn
m |2 + 1 m=2
(
n | 2m 1 + 1
Nu m > 1, n > 1 c suy ra m, n l.
m | 2n 1 + 1
Gi s m 1 = 2a .x ( a N , x l ) v n 1 = 2b .y (b N , y l )
a
Gi p l c nguyn t bt k ca n th p|2m1 + 1 nn p|(2x )2 + 1 theo bi tp 3.
nn p 1 (mod 2a+1 ) do p = a+1 k + 1.
Nh vy mi c ca n u c dng 2a+1 k + 1. Do tch ca cc s c dng 2a+1 k + 1 cng l
s c dng 2a+1 k + 1 nn n 1 (mod 2a+1 ).
n 1 = 2a+1 .y1 (y1 N ) m n 1 = 2b .y (y l) nn b a + 1 (1)
Gi q l c nguyn t bt k ca m.
b
q|m th a|2n1 + 1 nn q|(2x )2 + 1 theo bi tp 3 nn q 1 (mod 2b+1 )
q = 2b+1 .t + 1 nn m = 2b+1 .x1 + 1 ( x1 N )
m 1 = 2b+1 .x1 ( x1 N ), m m 1 = 2a .x (x l) nn a b + 1 (2)
T (1) v (2) v l. Vy khng c m > 1, n > 1 tha mn bi.
Kt lun: m = 1, n = 1; m = 1, n = 2; m = 2, n = 1.
n
Bi ton 8. Cho s Fecma Fn = 22 + 1 (c th l hp s, cng c th l s nguyn t).
a) Chng minh rng, nu Fn l hp s th Fn c t nht 2 c nguyn t phn bit.

189
Hi tho khoa hc, Hng Yn 25-26/02/2017

b) Chng minh rng, nu p| Fn , q| Fn (p, q l s nguyn t, p 6= q) th


( (
p | 2q 1 1 2q1 1 (mod p)
hay
q | 2 p 1 1 2 p1 1 (mod q)

Li gii. a) Gi s Fn l hp s v Fn c ng mt c nguyn t phn bit.


nn Fn = pk (k 2) v nu k = 1 th Fn = p l s nguyn t, v l.
Nu k chn: k = 2t (t 1).
n n n 1 n 1
Fn = pk th 22 + 1 = p2t 1 = p2t 22 = ( pt 22 )( pt + 22 ), v l (v n 5, Fn mi
l hp s).
Nu k l: k = 2t + 1 (t 1)
n n
22 + 1 = p2t+1 nn 22 = p2t+1 1 = ( p 1)( p2t + p2t1 + + p + 1)
n
suy ra 22 c c l ln hn 1, v l.
Vy Fn l hp s th phi c 2 c nguyn t phn bit tr ln.
b) Gi p, q l hai c nguyn t phn bit ca Fn th p, q l v Fn l.
Ta chng minh: ord p (2)|q 1 v ordq (2)| p 1.
n
Gi s ord p (2) = h th 2h 1 (mod p), c p| Fn nn 22 1 (mod p)
n +1
22 1 (mod p) nn h|2n+1 suy ra h = 2t vi t n + 1.
n n n
Nu t n th 2t |2n th 22 1 (mod p); p|22 1; p|22 + 1 nn p|2 mu thun p nguyn t
l.
nn t = n + 1 hay h = 2n+1 , chng minh tng t c ordq (2) = 2n+1 .
M ord p (2)| ( p) = p 1 nn 2n+1 | p 1; 2n+1 = ordq (2) do 2 p1 1 (mod q).
M ordq (2)| (q) = q 1 nn 2n+1 |q 1 m 2n+1 = ord p (2) suy ra 2q1 1 (mod p)
(pcm).

Bi ton 9. Cho n > 1, a N ; n| an + 1. Chng minh rng ( a + 1, n) = 1.


Li gii. Do n > 1 nn n c c nguyn t. Gi p l c nguyn t nh nht ca n nn
( p 1, n) = 1.
.
ord p ( a) = h nn ah 1 (mod p) suy ra ah 1 .. p
p|n m n| an + 1 nn p| an + 1
an 1 (mod p) do p| a2n 1 nn a2n 1 (mod p) suy ra h|2n.
M h| ( p) = p 1 nn h|(2n, p 1); (2n, p 1) = (2, p 1) suy ra h|(2, p 1).
Do (n, p 1) = 1 nn h|(2, p 1); (2, p) = 2
.
Nu h = 1 th a 1 .. p nn an 1 (mod p) nn an + 1 2 (mod p).
. . .
M an + 1 .. p th 2 .. p m p nguyn t nn p = 2, c ( a 1) .. 2 nn a l, p| a nn ( a + 1; n)
2 > 1.
.
Nu h = 2 th a2 1 .. p; 2|( p 1) nn p l m a 1 khng chia ht cho p
nn p| a + 1; p|n suy ra p|( a + 1, n) do ( a + 1, n) > 1 (pcm)

Bi ton 10. Gi s a, b l cc s nguyn dng, sao cho 2a 1; 2b 1; 2c 1 u l cc s


nguyn t. Chng minh rng a a + bb v ab + b a u khng chia ht cho a + b.
Li gii. C 2a 1; 2b 1 l s nguyn t nn a > 1, b > 1 v a + b > 2 m a + b nguyn
t, do a + b l s l, ( a + b) = a + b 1.

190
Hi tho khoa hc, Hng Yn 25-26/02/2017

.
Gi s a chn, b l. V b l nn ab + bb ..( a + b) (1)
.
Nu a a + bb ..( a + b) (2)
. .
T (1), (2) suy ra ( ab a a )..( a + b) v a a ( aba 1)..( a + b), nu a < b (*)
.
hoc ab (1 a ab )..( a + b) nu a > b.
. .
Nu a a ..( a + b) th a..( a + b) do ( a + b) nguyn t, v l v a < a + b.
. .
Nu ab ..( a + b) th a..( a + b) do ( a + b) nguyn t, v l v a < a + b.
T (*) c a|ba| 1 (mod ( a + b)). Gi( h = orda+b ( a) th h|(|b a|)
h|2a 1
M h| ( a + b) nn h| a + b 1 suy ra
h|2b 1
.
Do (2a 1), (2b 1) l s nguyn t nn h = 1 do a 1 (mod ( a + b)) nn ( a 1)..( a +
b), v l v 0 < a 1 < a + b.
. . .
Tng t, nu ab + b a ..( a + b) m ( ab + bb )..( a + b) nn (b a bb )..( a + b)
. .
b a (1 bba )..( a + b) nu a < b hoc bb (b ab 1)..( a + b) nu a > b (**)
. .
Nu b a ..( a + b) th b..( a + b) do ( a + b) nguyn t, v l v b < a + b
. .
Nu bb ..( a + b) th b..( a + b) do ( a + b) nguyn t, v l v b < a + b
.
T (**) c (b|ab| 1)..( a + b) nn b|ab| 1 (mod ( a + b))
Gi k = orda+b (b) th k|(| a b|)(.
k |2a 1
M k | a + b 1 = ( a + b) nn
k |2b 1
Do 2a 1; 2b 1 l s nguyn t nn k = 1 do b 1 (mod ( a + b))
.
suy ra b 1..( a + b), v l v 0 < b 1 < a + b.
Vy a a + bb ; ab + b a u khng chia ht cho 2a 1; 2b 1.

Bi ton 11. Cho p, q l hai s nguyn t l tha mn p = 2q + 1. Chng minh a2 lun l


cn nguyn thy mod p vi mi s nguyn a tha mn 1 < a q.
Li gii. Gi s q = 2k + 1, suy ra p = 4k + 3 nn 1 khng l s chnh phng mod p.
Gi g l mt cn nguyn thy mod p, khi tn ti cc s nguyn dng s, t tha mn
1 s, t p 1 sao cho 1 gs (mod p), a gt (mod p).
Do 1 khng l s chnh phng mod p nn s l, suy ra (s + 2t)q l s l v khng chia ht
cho p 1.
Ta c: ( a2 )q ( gs .g2t )q g(s+2t)q 6= 1 (mod p) (1)
Mt khc, v 1 < a q nn a2 6= 1 (mod p)
Do ( a2 )2 a4 6= 1 (mod p) (2)
Gi ord p ( a2 ) = h th h| p 1 nn h ch c th nhn gi tr 2, q, 2q.
T (1) v (2) suy ra h = 2q = p 1.
Vy a2 l cn nguyn thy mod p.

191
Hi tho khoa hc, Hng Yn 25-26/02/2017

3 Mt s bi tp tng t
.
Bi 1. Cho s nguyn dng n tha mn 3n 1..n. Chng minh rng n chn.
Bi 2. Tm s nguyn dng n nh nht sao cho 17n 1 chia ht cho 22011 .
Bi 3.
a) C bao nhiu s nguyn dng n l bi s ca 1001 v biu din c di dng n =
10 j 10i (i, j N, 0 i < j 99).
b) C bao nhiu s nguyn dng n l bi s ca 2011 v biu din c di dng n =
10 j 10i (i, j N, 0 i < j 99).
Bi 4. Chng minh rng vi mi s nguyn dng n th 3n 2n khng chia ht cho n.
Bi 5 (Bulgari OM 1995). Tm tt c cc s nguyn t p, q sao cho (5 p 2 p )(5q 2q ) chia ht
cho pq.
p s: ( p; q) {(3; 3); (3; 13); (13; 3)}.
n n
Bi 6. Cho hai s nguyn dng m v n (n > 1), sao cho 1 + m3 + m2.3 chia ht cho n.
Chng minh rng n = 3.
Bi 7. Tm tt c cc s nguyn t p, q phn bit sao cho 3 pq a (mod 3pq), vi mi s
nguyn dng a.
p s: ( p; q) {(17; 11); (11; 17)}.
Bi 8 (Russia OM 2000). C tn ti hay khng cc s nguyn dng a, b, c tha mn


( a, b) = (b, c) = (c, a) = 1
2a + 1 ... b



.


2b + 1 .. c
2c + 1 ... a

( a + 1) n a n
Bi 9 (China TST 2006). Tm tt c cc cp s nguyn dng ( a, n) sao cho Z.
n
Bi 10 (Turkey TST 2013). Tm tt c cc cp s nguyn dng (m, n) tha mn 2n + (n
(n) 1)! = nm + 1.

Ti liu
[1] Nguyn V Thanh, S hc, NXB Tng hp Tin Giang - 1992.
[2] ng Hng Thng, Nguyn Vn Ngc, V Kim Thy, Bi ging s hc, NXBGD - 1997.
[3] H Huy Khoi, S hc, NXBGD - 2004.
[4] B Gio dc v o to, Tp ch Ton hc v tui tr, NXBGD

192
Hi tho khoa hc, Hng Yn 25-26/02/2017

MT S M RNG V P DNG
CA BT NG THC K LAMKIN

Hong Minh Qun - Hi Ton hc H Ni


Ngy Phan Tin - Trng THPT Yn Dng 3, Bc Giang

Tm tt ni dung
Bt ng thc hnh hc th hin mi quan h mang tnh bn cht ca cc quan h
hnh hc v l mt chuyn th v trong ton s cp, thng xut hin trong cc k thi
hc sinh gii Quc gia v Quc t.
Nghin cu, tm ti v pht hin nhng iu mi t cc bt ng thc hnh hc lun
mang li nim vui cho gio vin v hc sinh trong qu trnh dy v hc, c bit l trong
cc trng chuyn, lp chn.
Bo co gii thiu bt ng thc Klamkin (1975) cng mt s ng dng v m rng ca
n, qua c th thy rng, nhiu iu th v vn cn n nu trong cc i tng c in
nh tam gic, t gic, hnh trn, cn c i su nghin cu, tm hiu v pht trin.

Trong bo co ny, cc k hiu sau y c s dng:


1) A, B, C l cc gc ca tam gic ABC.
2) a, b, c l di cc cnh ca tam gic ABC.
3) p = a+2b+c l na chu vi ca tam gic ABC.
4) S hay S ABC l din tch tam gic ABC.
5) r, R, r a tng ng l bn knh ng trn ni tip, ngoi tip v ng trn bng tip
cnh a ca tam gic ABC.
6) R1 , R2 , R3 ln lt l khong cch t im P bt k trong tam gic ABC n cc nh
A, B, C.
7) r1 , r2 , r3 ln lt l khong cch t im P bt k trong tam gic ABC n cc cnh
BC, CA, AB.
8) h a , m a , la tng ng l di ng cao, trung tuyn v phn gic trong h t A ca
tam gic ABC.

9)
a , AB l cc vect.


10) a . b , AB.CD l tch v hng ca hai vect.

1 Bt ng thc Klamkin
nh l 1 (Klamkin, [8], 1975). Cho tam gic ABC ty vi BC = a, CA = b, AB = c v P l
im bt k trong khng gian, khong cch t im P n cc nh A, B, C ln lt l R1 , R2 , R3 .
Khi vi mi s thc x, y, z ta c

( x + y + z)( xR21 + yR22 + zR23 ) yza2 + zxb2 + xyc2 . (1.1)

193
Hi tho khoa hc, Hng Yn 25-26/02/2017




Chng minh. T bt ng thc ( x PA + y PB + z PC )2 0 ta c




x2 PA2 + y2 PB2 + z2 PC2 + (2xy PA PB + 2yz PB PC + 2zx PC PA) 0. (1.2)

Ta li c


AB2 = ( PB PA)2 = PA2 + PB2 2 PB. PA.
Suy ra

2 PA. PB = PA2 + PB2 c2 .
Tng t


2 PB. PC = PB2 + PC2 a2 ,

2 PC. PA = PA2 + PC2 b2 .
Thay cc ng thc ny vo (1.2) v thu gn, ta c bt ng thc

( x + y + z)( xPA2 + yPB2 + zPC2 ) yza2 + zxb2 + xyc2


hay
( x + y + z)( xR21 + yR22 + zR23 ) yza2 + zxb2 + xyc2 .
Vy (1.1) c chng minh.
ng thc xy ra khi v ch khi P l tm t c ca h im ( A, B, C ).

2 ng dng
Bt ng thc Klamkin c rt nhiu ng dng, di y chng ti ch trnh by mt
s v d in hnh.
Bi ton 1. Cho P l im bt k thuc min trong ca tam gic ABC. Chng minh rng
1 2
PA2 + PB2 + PC2 ( a + b2 + c2 ).
3

Li gii. p dng bt ng thc (1.1) vi x = y = z = 1, ta c

3( PA2 + PB2 + PC2 ) a2 + b2 + c2 ,

hay
1 2
PA2 + PB2 + PC2 ( a + b2 + c2 ).
3
ng thc xy ra khi v ch khi P l trng tm tam gic ABC.
Bi ton 2. Cho G l trng tm tam gic ABC. Chng minh rng
9
m a 2 + mb 2 + mc 2 (m a + mb + mc ) m a a2 + m b b2 + m c c2 .
 
4

1 1 1
Li gii. p dng bt ng thc (1.1) vi x = ;y = ;z = v P G ta c
ma mb mc
GA2 GB2 GC2 a2 b2 c2
  
1 1 1
+ + + + + + .
ma mb mc ma mb mc mb mc mc m a m a mb

194
Hi tho khoa hc, Hng Yn 25-26/02/2017

Do GA = 32 m a , GB = 23 mb , GC = 32 mc nn bt ng thc trn tng ng vi

a2 b2 c2
 
4 1 1 1
+ + (m a + mb + mc ) + + .
9 ma mb mc mb mc mc m a m a mb
Ngha l,
4
( m a m b + m b m c + m c m a ) ( m a + m b + m c ) m a a2 + m b b2 + m c c2 ,
9
hay
9
m a a2 + m b b2 + m c c2 .

(m a mb + mb mc + mc m a ) (m a + mb + mc )
4
Kt hp vi bt ng thc

m a 2 + mb 2 + mc 2 m a mb + mb mc + mc m a

ta c
9
m a 2 + mb 2 + mc 2 (m a + mb + mc ) m a a2 + m b b2 + m c c2 .
 
4
Vy bt ng thc (2) c chng minh.
Bi ton 3 (Bt ng thc Weitzenbock). Cho tam gic ABC ni tip ng trn (O, R).
Chng minh rng
a2 + b2 + c2 4S 3. (2.1)

Li gii.
p dng bt ng thc (1.1) vi x = a2 , y = b2 , z = c2 v P O ta c

( a2 + b2 + c2 )( a2 R2 + b2 R2 + c2 R2 ) b2 c2 a2 + c2 a2 b2 + a2 b2 c2 ,

hay
R2 ( a2 + b2 + c2 )2 3a2 b2 c2 .
Ngha l,
2 2 2 3abc
a +b +c .
R
Mt khc, ta c
abc abc
S= = 4S.
4R R
Vy
a2 + b2 + c2 4S 3.
Bi ton 4 (Bt ng thc Euler). Cho R, r ln lt l bn knh ng trn ngoi tip, ni
tip tam gic ABC. Chng minh rng

R 2r. (2.2)

Li gii. p dng (1.1) vi x = a, y = b, z = c v P O ta c

( a + b + c)( aR2 + bR2 + cR2 ) bca2 + cab2 + abc2 ,

195
Hi tho khoa hc, Hng Yn 25-26/02/2017

hay
R2 ( a + b + c)2 abc( a + b + c).
Ngha l,
abc
R2 .
a+b+c
V
abc
S= = pr abc = 4pRr = 2( a + b + c) Rr
4R
nn ta c
abc 2( a + b + c) Rr
R2 = = 2Rr.
a+b+c a+b+c
Vy
R 2r.

Bt ng thc Klamkin khng ch c s dng trong chng minh cc bt ng thc


hnh hc, m cn c th c s dng trong chng minh bt ng thc lng gic. Di
y l mt trong nhiu v d.

Bi ton 5. Cho tam gic ABC. Chng minh rng

sin Acos2 A + sin Bcos2 B + sin Ccos2 C sin A sin B sin C.

Li gii. p dng (1.1) vi x = a, y = b, z = c v P H ta c

( a + b + c)( aH A2 + bHB2 + cHC2 ) bca2 + cab2 + abc2 ,

hay
aH A2 + bHB2 + cHC2 abc.
Do nh l hm s sin v

H A = 2R cos A, HB = 2R cos B, HC = 2R cos C,

ta c
sin Acos2 A + sin Bcos2 B + sin Ccos2 C sin A sin B sin C.
Vy Bt ng thc (5) c chng minh.

3 Mt s h qu quan trng ca bt ng thc


Klamkin
Jian Liu (2008, [9]) chng minh mt h qu sau ca bt ng thc Klamkin.

196
Hi tho khoa hc, Hng Yn 25-26/02/2017

H qu 1. Cho P l im ty trong mt phng tam gic ABC, x, y, z l cc s thc dng. Khi


ta c bt ng thc
R21 R2 R2 aR + bR2 + cR3
+ 2 + 3 1 . (3.1)
x y z xy + yz + zx
Chng minh.

Trng hp 1: Nu P l mt trong cc nh ca tam gic ABC, chng hn P C th


CA = R1 , CB = R2 , PC = 0. Bt ng thc cn chng minh tr thnh

CA2 CB2 aCA + bCB 2ba


+ = ,
x y yz + zx + xy yz + zx + xy

hay
b2 a2 2ba
+ .
x y yz + zx + xy
p dng bt ng thc Cauchy, ta c

b2 a2 2ba
+ .
x y xy

V x, y, z l cc s thc dng nn ta c

b2 a2 2ba 2ba
+ > . (3.2)
x y xy yz + zx + xy

Vy nu P trng vi mt trong cc nh ca tam gic ABC th nh l c chng minh,


trng hp ny khng xy ra du =.

Trng hp 2: P khng l mt trong cc nh ca tam gic ABC. Bt ng thc


Klamkin
( x + y + z)( xR21 + yR22 + zR23 ) yza2 + zxb2 + xyc2
c vit li thnh

a2 b2 c2
 
1 1 1
xR21 + yR22 + zR23

+ + + + .
yz zx xy x y z

p dng bt ng thc Cauchy-Schwarz, ta c

a2 b2 c2 ( a + b + c )2
+ + .
x y z x+y+z

Do
( a + b + c )2
 
1 1 1
xR21 + yR22 + zR23

+ + .
yz zx xy x+y+z
iu ny tng ng vi

( a + b + c )2
 
1 1 1
xR21 + yR22 + zR23

+ + . (3.3)
yz zx xy x+y+z

197
Hi tho khoa hc, Hng Yn 25-26/02/2017

ng thc xy ra khi v ch khi x : y : z = a : b : c v P l tm ng trn ngoi tip tam


gic ABC.
Thc hin php bin i R1 R2 R3 , R2 R3 R1 , R3 R1 R2 , a aR1 , b bR2 , c
cR3 trong (3.3) ta c (chi tit hn xem [4], trang 10-12)
i 1 ( aR1 + bR2 + cR3 )2

h 1 1
x ( R2 R3 )2 + y ( R3 R1 )2 + z ( R1 R2 )2 + + + .
yz zx xy x+y+z

Ngha l,
( R2 R3 )2 ( R3 R1 )2 ( R1 R2 )2
 2
aR1 + bR2 + cR3
+ + . (3.4)
yz zx xy x+y+z
Thc hin php bin i x xR1 2 , y yR2 2 , z zR3 2 trong (3.4) ta c
 2
1 1 1 aR1 + bR2 + cR3
+ + . (3.5)
yz zx xy xR1 2 + yR2 2 + zR3 2

Thc hin php bin i x 1x , y y1 , z 1


z trong (3.5) ta c (3.1).
ng thc xy ra khi v ch khi ABC l tam gic nhn c P l trc tm v

R1 R2 R3
= = . (3.6)
xa yb zc

Khi P l trc tm tam gic nhn ABC th ta c

R1 : R2 : R3 = cos A : cos B : cos C.

Do (3.6) tng ng vi

x : y : z = cot A : cot B : cot C.

Vi cc bt ng thc (3.1), (3.3),(3.4) v (3.5), ta c th xy dng c nhiu bt ng


thc th v khc, chng hn cc bt ng thc sau.

Bi ton 6.
R21 R2 R2 4S
+ 2+ 3 . (3.7)
x y z xy + yz + zx

Bi ton 7.
3
R21 + R22 + R23 S. (3.8)
3

Bi ton 8.
aR21 + bR22 + cR23
2Rr. (3.9)
aR1 + bR2 + cR3

198
Hi tho khoa hc, Hng Yn 25-26/02/2017

Bi ton 9 (Bt ng thc Hayashi).


R2 R3 R3 R1 R R2
+ + 1 1. (3.10)
bc ca ab

Bi ton 10.
 
2 1 1 1
( R2 R3 + R3 R1 + R1 R2 ) + + 4p2 . (3.11)
R2 R3 R3 R1 R1 R2

Bi ton 11.
( R2 R3 )2 ( R3 R1 )2 ( R1 R2 )2 16 2
+ + p . (3.12)
r2 r3 r3 r1 r1 r2 9

Bi ton 12.
( R1 + R2 + R3 )2 4
p. (3.13)
r1 + r2 + r3 3

Bi ton 13.
R1 2 R2 2 R3 2 4
+ + (r1 + r2 + r3 ) . (3.14)
ra rb rc 3

Bi ton 14.
R1 2 R2 2 R3 2
+ + r1 + r2 + r3 + r. (3.15)
ra rb rc

Bi ton 15.
R1 2 R2 2 R3 2
+ + 4r. (3.16)
ha hb hc

Nm 2011, Jian Liu li pht biu mt h qu sau y ca bt ng thc Klamkin, cng


c kh nhiu ng dng. Ta c

H qu 2 (xem [10). ]
R22 + R23 2 R23 + R21 2 R21 + R22 2 2
2
x + 2
y + 2
z (yz + zx + xy) . (3.17)
a b c 3
t
a b c
x= p ,y = p ,z = p
R2 2 + R3 2 R3 2 + R1 2 R1 2 + R2 2
trong (3.17), ta c

199
Hi tho khoa hc, Hng Yn 25-26/02/2017

Bi ton 16.
bc ca
q
2 2
 2 2
+q 2 2
 2 2
.
R3 + R1 R1 + R2 R1 + R2 R2 + R3

ab 9
+q . (3.18)
R2 2 + R3 2

R3 2 + R1 2
 2

T bt ng thc (16), ta c th d dng nhn c hai bt ng thc sau.


Bi ton 17.
8
R2 2 + R3 2 R3 2 + R1 2 R1 2 + R2 2 ( abc)2 .
  
(3.19)
27

Bi ton 18. p p p
R2 2 + R3 2 R3 2 + R1 2 R1 2 + R2 2
+ + 6. (3.20)
a b c

Mt kt qu tng qut hn (18) l


Bi ton 19.
p p p r
R2 2 + R3 2 2 R3 2 + R1 2 2 R1 2 + R2 2 2 2
x + y + z (yz + zx + xy) . (3.21)
a b c 3

Thay x = a, y = b, z = c vo (19) ta c
Bi ton 20.
2 
r 
q
2 2
q
2 2
q
2 2

R2 + R3 + R3 + R1 + R1 + R2 bc + ca + ab . (3.22)
3

4 Mt s gi thuyt
Bi ton 21.
r
2
q q q
R2 2 + R3 2 + R3 2 + R1 2 + R1 2 + R2 2 ( a + b + c) . (4.1)
3
Bi ton 22.
( R2 + R3 )2 R1 2 ( R3 + R1 )2 R2 2 ( R1 + R2 )2 R3 2 4
4
+ 4
+ 4
. (4.2)
a b c 3
Bi ton 23.
r2 2 + r3 2 r3 2 + r1 2 r1 2 + r2 2 2
2
+ 2
+ 2
( R1 + R2 + R3 )2 . (4.3)
sin A sin B sin C 9
Cc gi thuyt ny c Jian Liu kim tra trn my tnh.

200
Hi tho khoa hc, Hng Yn 25-26/02/2017

5 M rng ca bt ng thc Klamkin


Xiao-Guang Chu v Zhi-Hua Zhang, [6] m rng Bt ng thc Klamkin cho hai
im.
nh l 2. Nu P v Q l hai im trong ca tam gic ABC v ( x, y, z) l ta trng tm (the
barycentric coordinates) ca cc im P hoc Q i vi tam gic ABC th

( x + y + z) ( xPA.QA + yPB.QB + zPC.QC ) yza2 + zxb2 + xyc2 . (5.1)

Du bng xy ra khi v ch khi hai im P v Q trng nhau.


Chng minh. Gi s AQ BC = D, BQ CA = E, CQ AB = F; AD = q a , BE =
qb , CF = qc , AQ = R1 , BQ = R2 , CQ = R3 v r1 , r2 , r3 l khong cch t Q n cc cnh
ca ABC. Khi ta c
ar1 = x, br2 = y, cr3 = z. (5.2)
x+y+z x+y+z x+y+z
qa = R1 , q b = R2 , q c = R3 . (5.3)
y+z z+x x+y
Gi s x, y, z l cc ta trng tm ca im Q trong tam gic ABC v r1 , r2 , r3 l khong
cch t Q n cc cnh ca ABC. T nh l Stewart ta c

z 2 y 2 a2 yz
AD2 = q a 2 = b + c . (5.4)
y+z y+z ( y + z )2
v
z y a2 yz
PD2 = PC2 + PB2 . (5.5)
y+z y+z ( y + z )2
Xt APD, s dng nh l hm s csin, ta c

[ = PA2 + q a 2 PD2 .
2q a .PA 2q a PA cos PAD (5.6)

T (5.4), (5.5) v (5.6) ta c


2
2x (y + z)q a .PA x (y + z) PA2 + zxb2 + xyc zxPC2 xyPB2 . (5.7)

Du bng xy ra khi v ch khi im P nm trn on AD. Tng t vi BPE v CPF


ta c
2y (z + x ) qb .PB y (z + x ) PB2 + xyc2 + yza2 xyPA2 yzPC2 , (5.8)
v
2z ( x + y) qc .PC z ( x + y) PC2 + yza2 + zxb2 yzPB2 zxPA2 . (5.9)
Du bng trong (5.8) ( trong (5.9)) xy ra khi v ch khi im P nm trn on BE (trn
on CF).
Cng v vi v, t (5.7), (5.8) v (5.9) ta c

x (y + z) q a .PA + y (z + x ) qb .PB + z ( x + y) qc .PC yza2 + zxb2 + xyc2 . (5.10)

Theo (5.3), bt ng thc (5.10) tng ng vi (5.1). S dng cc ng thc

ar1 + br2 + cr3 = 2S; ar1 = x, br2 = y, cr3 = z, abc = 4RS

201
Hi tho khoa hc, Hng Yn 25-26/02/2017

v (5.1), ta d dng thu c


R1 r1 R2 r2 R3 r3 r r
2 3 r3 r r r2 
PA + PB + PC 2R + 1+ 1 . (5.11)
bc ca ab bc ca ab
Kt hp cc iu kin (5.7), (5.8)v (5.9) hoc bt ng thc (1.1) ta c

ar1 R1 2 + br2 R2 2 + cr3 R3 2 = 2R (ar2 r3 + br3 r1 + cr1 r2 ) . (5.12)

Du bng xy ra trong (5.1) khi v ch khi hai im P v Q trng nhau.

Nhn xt 1. Khi P v Q trng nhau th bt ng thc (5.1) tr thnh bt ng thc


Klamkin. T nh l trn ta c nhiu h qu p, th d,
H qu 3. Cho P1 , P2 , . . . , Pn v Q l n + 1 im bn trong ABC. Khi
R1 r1 R2 r2
( PA1 + PA2 + + PAn ) + ( PB1 + PB2 + + PBn )
bc ca
R3 r3 r r
2 3 r3 r r r2 
+ ( PC1 + PC2 + + PCn ) 2nR + 1+ 1 . (5.13)
ab bc ca ab
Du bng h qu (3) xy ra khi v ch khi n + 1 im gm P1 , P2 , . . . , Pn v Q trng
nhau.
H qu 4. Nu P v Q l hai im bn trong ABC v x, y, z l ta trng tm barycen-
tric ca Q i vi ABC khi

xPA2 + yPB2 + zPC2 yza2 + zxb2 + xyc2


 

( x + y + z) ( xPA.QA + yPB.QB + zPC.QC )2 . (5.14)


Chng minh h qu (4) bng cch kt hp nh l (1), nh l (2) v bt ng thc
Cauchy.
H qu 5. Cho P l mt im bn trong ABC. Khi
1 2
a + b2 + c2 .

m a PA + mb PB + mc PC (5.15)
2
Du = trong (24) xy ra khi v ch khi P l tm t c ca ABC.
Chng minh. Cho P v tm t c G trng nhau. Khi ta c
2 2 2
R1 = m a , R2 = m b , R3 = m c ,
3 3 3
1 1 1
r1 = h a , r2 = h b , r3 = h c ,
3 3 3
v
abc = 4RS, 2Rh a = bc, 2Rhb = ca, 2Rhc = ab.
T bt ng thc (5.13) suy ra (24).

p dng H qu (5) ta c bt ng thc sau.

202
Hi tho khoa hc, Hng Yn 25-26/02/2017

Bi ton 24.
a2 + b2 + c2
m a + mb + mc . (5.16)
2R

H qu 6. Cho P l im bt k trong tam gic ABC. Khi


A B C
PA cos + PB cos + PC cos p. (5.17)
2 2 2
ng thc xy ra khi v ch khi P l tm ng trn ni tip
p dng h qu (6) ta c bt ng thc sau
Bi ton 25.
A B C 3
m a cos + mb cos + mc cos ( a + b + c) . (5.18)
2 2 2 4

H qu 7.
ma m mc
PA + b PB + PC 3R. (5.19)
ha hb hc
ng thc xy ra khi v ch khi P l trng tm ABC.
p dng h qu (7) ta c bt ng thc sau.
Bi ton 26.
ma m mc
+ b+ 3. (5.20)
ha hb hc

H qu 8.
p
(cPA + aPB + bPC ) (bPA + cPB + aPC ) b2 c2 + c2 a2 + a2 b2 . (5.21)

ng thc (5.21) xy ra khi v ch khi P l im Brocard dng (m) ca tam gic

6 Thay li kt
Bt ng thc Klamkin l mt bt ng thc mi v hay, c nhiu m rng v h
qu. Vit Nam, bt ng thc ny c s lc nhc n trong [1] v [2], nhng
ni chung, cn t c quan tm. Ngoi cc h qu v cc bi ton gii thiu trn
cn c rt nhiu bi ton khc c xy dng t bt ng thc Klamkin cng nh cc
m rng ca n. C th xem thm [4] v cc ti liu trch dn.
Cc tc gi xin cm n GS.TSKH. Nguyn Vn Mu - ngi thy tuyt vi v truyn b
cc tng cng nh truyn cm hng ton hc cho cc th h hc tr. Xin cm n
PGS.TS. Nguyn Minh Tun v nhng n tng v nh hng v phong cch s phm
v khoa hc khi trnh by cc bo co. Xin cm n PGS. TS. T Duy Phng ng
vin v khuyn khch cc tc gi trong hc tp v vit bo co.

203
Hi tho khoa hc, Hng Yn 25-26/02/2017

Ti liu
[1] Trn Phng (2002), Tuyn tp cc chuyn luyn thi i hc mn Ton-H thc lng
gic, Nh xut bn H Ni, Bi s 5, trang 204.
[2] Hong Ngc Quang (2011), Mt s bt ng thc hnh hc (Lun vn Cao hc), i
hc Thi Nguyn, trang 112-121.
[3] Hong Minh Qun, Xy dng mt s dng ng thc v bt ng thc trong tam gic,Hi
tho ton hc, Tuyn Quang, 2012.
[4] Ngy Phan Tin (2016), Bt ng thc Klamkin: Mt s m rng v ng dng (Lun vn
Cao hc), i hc Thng Long.
[5] G. Bennett (1977), Multiple Triangle Inequalities, Univ. Beograd. Publ. Elektrrotehn.
Fak., Ser. Mat. Fiz., No 58, pp. 39-44.
[6] Xiao-Guang Chu and Zhi-Hua Zhang (2007), A Geometric Inequality for two Interior
Points of the Triangle, pp. 1-4.
[7] Dan Comnescu and S. S. Dragomir (2009), A Generalization of the Klamkin Inequality,
pp. 1-10
[8] M. S. Klamkin (1975), Geometric Inequalities via the Polar Moment of Inertia, Mathemat-
ics Magazine, Vol. 48, No 1, pp. 44-46.
[9] Jian Liu (2008), A Weighted Geometric Inequality and Its Applications,
Journal of inequalities in pure and applied mathematics, Vol 9, Issue 2, Article 58, 9
pages.
[10] Jian Liu (2011), A New Geometric Inequality and Its Applications,
Journal of Science and Arts, No 11 (14), pp. 5-12.

204
Hi tho khoa hc, Hng Yn 25-26/02/2017

NH L HELLY V NG DNG
Nguyn Th Tm - Hong Th Nhung - Trn Thi Hng
THPT Chuyn Hng Yn

Tm tt ni dung

nh l Helly l mt nh l rt quan trong phn hnh hc t hp. nh l ny cho ta


mt iu kin nhn bit khi no mt h cc hnh li c giao khc rng. i vi mt
lp tp hp c th nh hnh bnh hnh, on thng th c gim nh iu kin ca nh l
Helly. V y, ta cn xt s m rng ca nh l, c bit i vi tp hp c th khng
li nh cung ng trn chng hn.

1 C s l thuyt
1.1 Cc nh l Helly
nh l 1 (nh l Helly trong khng gian mt chiu). Trn ng thng cho n hnh li
(n 3). Bit rng giao ca hai hnh bt k trong chng khc rng. Khi , giao ca n hnh li
cho khc rng.

nh l 2 (nh l Helly trong khng gian hai chiu). Cho trc n hnh li trn mt phng
(n 4). Bit rng giao ca ba hnh li bt k trong chng khc rng. Khi , giao ca cc hnh li
cho khc rng.

nh l 3 (nh l Helly trong khng gian ba chiu). Cho trc n hnh li trong khng gian
(n 5).Bit rng giao ca bn hnh bt k trong chng khc rng. Khi giao ca cc hnh li
cho khc rng.

nh l 4 (nh l Helly trong khng gian n chiu). Cho trc n hnh li trong khng gian
k - chiu Rk (n k + 1, k N ). Bit rng (k + 1) hnh li bt k trong chng c giao khc rng.
Khi , giao ca cc hnh li cho khc rng.

1.2 Mt s h qu ca nh l Helly
H qu 1. Cho trc n on thng [ ai ; bi ] (i = 1,2,3,. . . ,n) trn cng mt ng thng. Khi
giao ca cc on thng ny khc rng khi ch v ch khi giao ca hai on thng bt
k trong chng khc rng.
Chng minh. Ta ch mi tp li A trn ng thng th hin hoc l on thng, hoc
l mt tia hoc l c ng thng. nh l Helly vi gi thit trong trng hp ny thc
s c ngha khi cc tp A1 , A2 , . . . , An l nhng on thng (cn nhng trng hp
khc nh trong s nhng tp hp ny tia v ng thng th ta cng a v trng hp

205
Hi tho khoa hc, Hng Yn 25-26/02/2017

tt c l on thng).
Mi on thng Ai trn ng thng s c xc nh bi hai s ti hai im cui ai , bi
v ai bi . S ai gi l u tri ca Ai , cn bi gi l u phi ca Ai . Ly b l s nh nht
ca bi , i = 1, 2, . . . , n; a l s ln nht ca ai , i = 1, 2, . . . , n. Ta chng minh rng a b.
Tht vy, gi s a > b. V a trng vi mt trong nhng s no trong ai , i = 1, 2, . . . , n,
cn b trng vi mt trong nhng s no trong bi , i = 1, 2, . . . , n, th ai1 = a > b = bi2 .
Khi ch ra rng nhng on thng Ai1 (vi u tri ai1 ) v Ai2 (vi u phi Bi2 ) khng
c im chung. Ta nhn c s v l.

H qu 2. Cho trc mt h I cc on thng [ ai ; bi ] (i I, I N) trn cng mt ng


thng. Khi giao ca cc on thng ny khc rng khi v ch khi giao ca hai on
bt k trong chng khc rng.
Hnh bnh hnh c cc tnh cht c bit hn nhng hnh li khc. c bit hn l
nhng hnh bnh hnh c cc cnh song song vi hai ng thng c nh. Khi , theo
nh l Helly ta cn c cc h qu sau:

H qu 3. Cho hnh bnh hnh F1 , F2 , F3 , cc cnh ca chng song song vi hai ng


thng c nh. Khi , nu mi cp hai hnh trong chng c im chung, th c ba hnh
bnh hnh cng c im chung.

Chng minh. Ta chn trong mt phng h ta vi cc trc nm trn hai ng thng


cho (khng nht thit h ta vung gc, ta ang xt h ta bt k.) K hiu A1 l
im chung ca F2 v F3 , A2 l im chung ca F1 v F3 , A3 l im chung ca ca F1 v
F2 . K hiu ( xi , yi ) l ta ca im Ai i vi h ta chn (i = 1; 2; 3). Khng mt
tnh tng qut, ta c th gi thit nhng hnh bnh hnh F1 , F2 , F3 c gn nhn sao cho
vi cc ta tng ng x1 , x2 , x3 tha mn bt ng thc x1 x2 x3 . Hnh bnh hnh
F1 s cha cc im A2 v A3 (hnh 3.7). Khi , n cha tt c nhng im P vi ta
( x, y) sao cho x2 x x3 , y2 y y3 vi y2 y3 hoc l y3 y y2 vi y3 y2 (hnh
1 l trng hp th hai).

Tng t, F2 cha tt c nhng im P c ta ( x, y) sao cho x1 x x3 , y1 y y3


hoc l y3 y y1 ph thuc vo y1 y3 hoc l y3 y1 . Ta lp li l lun trn vi hnh
bnh hnh F3 , ta s thui c s tn ti im P vi ta ( x2 , z), m n nm trn c ba
hnh bnh hnh. Tht vy, cho cc ch s i; j; k nhn nhng gi tr 1; 2; 3 v gi s rng bt
ng thc sau ng yi y j yk . Khi bng cch t yi = z, s nhn c kt qu cn
thit.
T kt qu trn v nh l Helly suy ra kt qu:

H qu 4. Trong mt phng cho n hnh bnh hnh c cc cnh song song vi hai ng
thng c nh. Nu mi cp hai hnh bnh hnh c im chung th tt c n hnh bnh
hnh c im chung.

206
Hi tho khoa hc, Hng Yn 25-26/02/2017

Chng minh. Nu mi hai bnh hnh c im chung th mi ba hnh bnh hnh cng c
im chung, t y p dng nh l Helly phn trc.
iu kin nhng cnh ca hnh bnh hnh song song vi hai ng thng c nh l bn
cht.
By gi ta xt tp hp tt c nhng cung trn ng trn.
D thy mt cung trn ng trn khng phi l tp li trong mt phng. Mt khc, mi
cung c trng bi hnh vin phn ca ng trn tng ng vi hnh trn cho. Mi
hnh vin phn ny l mt tp hp li v cho n hnh vin phn, ta c th p dng nh l
Helly. Hn na, nu hai cung c im chung th nhng hnh vin phn tng ng cng
c im chung. Tuy vy, nh l Helly khng ng na.
Nu ta xt nhng cung ca ng trn c di nh hn di na ng trn th khi
tnh th s khc i. Ta c kt qu sau:

H qu 5. Trn ng trn cho n > 3 cung, mi cung c di nh hn di ca na


ng trn. Khi , nu mi b ba cung c im chung th tt c cc cung c im chung.

Chng minh. Ta cho tng ng mi cung ng vi hnh vin phn ca ng trn xc


nh trn hnh trn. Trn mi hnh vin phn c dy cung khng phi l ng knh ca
hnh trn (do gi thit cc cung cho nh hn na di ng trn). T gi thit mi
b ba cung c im chung suy ra mi b ba hnh vin phn c im chung.
S dng nh l Helly cho tp hp cc hnh vin phn, ta nhn c tt c cc hnh vin
phn u c im chung. im ny khng th trng vi tm ca hnh trn. T im
chung ca cc hnh vin phn, k ng knh, mt trong cc im giao gia ng knh
va k v ng trn s l im chung cho n cung cho (d thy ng ni tm ng
trn vi mt im bt k trong hnh vin phn u ct cung ca hnh vin phn y).
B ba cung trong nh l trn l hon ton thc cht. Nu ta xt trn ng trn ba
cung AB,
c BC, d mi cung l 120 , ta nhn c mi cp hai cung ny c im chung,
c CA,
nhng c ba cung khng c im chung.

H qu 6. Trn mt ng trn cho n cung, mi cp cung c im chung. Khi , tn


ti ng thng i qua tm ng trn v ct tt c cc cung cho.

Chng minh. Ta k hiu l tp hp n cung. Ta chn trong chng cung ABc c di nh


nht (iu ny lm c v l tp hp hu hn), ngha l vi mi cung MN d AB.c
khng mt tnh tng qut, ta c th gi thit rng cung AB
c ng vi gc tm nh hn
180 , v trong trng hp ngc li mi ng knh s ct mi cung ca tp hp . Theo

iu kin ca nh l, mi cung ca cha hoc l im A, hoc l im B, hoc l c


hai im ng thi. Ta xt tp hp 1 gm nhng cung cha im A. Trn mi cung
d 1 , ta ly tng ng MN
MN d AB,
c n s khng phi l tp hp rng v c th ch
gm duy nht mt im. Cho CD d 1 tng ng c giao l mt im (i vi cung AB)
c
d AB, MN
hoc c di nh nht gia tt c cc cung MN c d 1 . Ta dng ng knh
DD 0 qua im D. Nu v D / , th cung cha nhng im B v C. T suy ra
n cha c im D 0 . Suy ra ng knh DD 0 ct mi cung ca , l iu phi chng
minh.

H qu 7. Cho n hnh li, mi cp hai hnh u c im chung. Khi qua mi im


bt k trong mt phng, c th k ng thng ct tt c n hnh.

207
Hi tho khoa hc, Hng Yn 25-26/02/2017

Chng minh. Ta xt ng trn k c tm ti mt im O. Trn mi hnh t nhng hnh


cho, ta cho tng ng vi mt cung ca ng trn ny theo cch sau: Mi im P
trn mt hnh tng ng ni vi tm O ca ng trn k v cho P tng ng vi im
ct ca tia OP vi ng trn k. Nu P trng vi O th cch t tng ng trn khng c
ngha, nn ta ch nh ngha cho php tng ng trn cho cc hnh khng cha im O,
vi cch cho tng ng trn s l mt cung no ca k. Nhng cung ny s tha mn
gi thit ca nh l va chng minh trn. iu ch ra rng tn ti ng thng i
qua o ct tt c cc cung v hn na ct tt c cc hnh cho (ng thng ny hin
nhin ct c nhng hnh, vi chng khng xc nh trn, v chng u cha O). l
iu ta cn phi chng minh.
Tn ti rt nhiu phng n ca nh l Helly, trong iu kin hai hoc mt s hnh li
c im chung thay bng iu kin nhng hnh ny c lt ct. Bn c c th suy ngh
v bin i cc tng trn.

H qu 8. Trong mt phng cho k im (k 3) im. Bit rng mi cp ba im trong


chng u nm trong ng trn bn knh bng 1. Khi tt c cc im cho u
cha trong ng trn bn knh bng 1.

Chng minh. Nhng im cho l x1 , x2 , . . . , xk . Cn phi chng minh rng tn ti


im x cch mi im cho mt khong cch khng vt qu 1. Ngha l phi tm mt
im nm trong tt c cc ngtrn Bi c tm xi v bn knh bng 1. Theo nh l Helly
ch cn chng minh mi b ba hnh trn t h B1 , B2 , . . . , Bk c im chung. Nhng iu
chnh l gi thit ca bi ton cho. Mi b ba im xi1 , xi2 , xi3 cha trong hnh trn
bn knh 1 v tm ca hnh trn ny s nm trong Bi1 , Bi2 , Bi3 .

H qu 9 (nh l Yong). Trong mt phng cho mt s hu hn im, khong cch gia


hai im bt k trong chng khng vt qu 1. Chng minh rng tn ti mt hnh trn
1
bn knh bng cha tt c cc im cho.
3
Chng minh. Nu mi b ba im ca nhng im cho cha trong ng trn bn
1
knh bng , th ta c th p dng l lun chng minh kt qu 8 trn v nhn c
3
1
tt c cc im nm trong ng trn bn knh bng .
3

Cho A, B, C l ba im trong cc im cho. Nu tam gic ABC l tam gic t hoc


vung, th hnh trn c tm ti trung im ca cnh di nht v bn knh bng na cnh

208
Hi tho khoa hc, Hng Yn 25-26/02/2017

ny s cha ton b tam gic. Bi v, khong cch gia hai im bt k t nhng im


cho khng qu 1, th hnh trn k trn ph nhng im A, B, C s c bn knh ln nht
1 1
l , m n nh hn .
2 3
Cn li, ta xt trng hp tam gic ABC l nhn. Ta xt ng trn ngoi tip tam gic
ABC. K hiu bn knh ca ng trn ny l r. C t nht mt trong nhng gc ca tam
gic khng nh hn 60 , gi s l A b (trng hp ngc li th tng ba gc ca tam
c nh nht l 120circ trn cung ny tng ng
gic nh hn 180 , v l.) Khi , cung BC
vi dy cung BC c di khng nh hn r 3 (r 3 l di ca dy cung tng ng

vi cung 120 ). Nhng t iu kin ca bi ton c 1 BC, ngha l 1 BC r 3. Suy
1
ra r v ba im A, B, C cng thuc ng trn vi bn knh bng 1.
3
1
Ch 1. S trong nh l Yong khng th gim hn c na. Nhng nh ca mt
3
tam gic u vi cnh bng 1 l v d ca tp hp tt c nhng im m khong cch
gia hai im trong chng bng 1, nhng ba im ny khng cha trong ng trn
bn knh nh hn 13 .

2 Mt s bi ton p dng
Bi ton 1 (Olympic Ton Canada 2009). Cho n 4 im trong mt phng, trong 3
im bt k lun c th ph kn bi mt hnh trn bn knh bng n v. Chng minh
rng c hnh trn bn knh bng n v ph kn c n im ny.
Li gii. Xt hnh trn bn knh 1 c tm l n im cho. iu kin " 3 im bt k
lun c th ph kn bi mt hnh trn bn knh 1" tng ng vi iu kin "3 ng
trn bt k k trn lun c giao khc rng" (chng c t nht 1 im chung l tm ca
hnh trn ph 3 im ). Theo nh l Helly n ng trn k trn c giao khc rng hay
c hnh trn bn knh 1 ph kn c n im ban u.
Bi ton 2 (Olympic Ton Canada 2003). Trn ng thng c 2k 1 on thng mu
trng v 2k 1 on thng mu en sao cho mi on thng ct ti t nht k on thng
khc mu. Chng minh rng c on thng mu trng ct tt c cc on thng mu
en v c on thng mu en ct tt c cc on thng mu trng.

Li gii. Ta chng minh c on thng c mu en ct tt c cc on thng mu trng.


Vi mi on thng mu trng cho trc ta xy dng mt tp li l on thng c to
thnh bng cch ly hp ca on thng vi tt c cc on thng mu en c giao
vi n. Mi tp li nh vy gm t nht k on thng mu en v do ta c tng cng
2k 1 on thng mu en nn hai tp li bt k nh vy chc chn c on thng mu
en chung. Do theo nh l Helly tt c cc tp li c on thng mu en chung.
y chnh l on thng mu en cn tm.
Hon ton tng t ta c on thng mu trng ct tt c cc on thng mu en.
Bi ton 3 (Olympic Ton Iran 2006). Trn bu tri m c hu hn ngi sao . Mi ngi
sao ta sng trong mt khong thi gian nht nh. Bit rng vi k > 1 ngi sao, c t
nht hai ngi sao ta sng cng mt thi im. Chng minh rng c th chp k 1 bc
nh v bu tri sao cho mi ngi sao to sng t nht mt bc nh.

209
Hi tho khoa hc, Hng Yn 25-26/02/2017

Li gii. Ta chng minh bng quy np theo k.


Vi k = 2, bi ton ng theo nh l Helly.
Gi s ta c khng nh vi k 2, ta s chng minh khng nh vi k + 1.
Tht vy trong tt c cc on k trn, chn on In = [ an , bn ] sao cho an ln nht. Chn
mt im l an . Khi tt c cc on c giao vi In u cha an .
B tt c cc on ny i v xt cc on cn li. Nu trong cc on cn li c k on
i mt khng giao nhau, mu thun. Do vi k on bt k trong cc on cn li
ny lun c hai on giao nhau. T theo quy np ta c th xc nh k 1 im sao
cho mi on trong cc on cn li ny cha t nht mt im trong . Cng thm
im an ta c k im cn tm.

Bi ton 4 (Chn i tuyn Iran thi Olympic Ton Quc t 2002). Cho mt ng cong
kn trn mt mt cu n v sao cho mi ng trn ln u c giao im vi ng
cong . Chng minh rng ng cong c di t nht l 2.
Li gii. Ta bit nu 4 im bt k ca ng cong lun nm trong mt na khng
gian to bi mt phng qua tm cu th ton b ng cong cng s nm trong mt na
khng gian to bi mt phng qua tm cu, mu thun. Do s tn ti 4 im A, B, C,
D trn ng cong sao cho tm cu O nm trong khi t din ABCD. Phn ng cong
ni 2 im A, B c di nh nht l cung trn AB. Khi ng cong kn ban u c
di nh nht l tng di cc cung trn AB, BC, CD, DA.
Gi D l im i xng tm ca D. Khi D s nm trong chm cu gii hn bi cc
ng trn ngoi tip tam gic ABC.
D dng chng minh c AB+ c BC c AD d0 + D
d 0C

Do AB+
c BC+c CD+
d DA d ADd0 + D d0 C + CD
d + DA
d = 2.

Bi ton 5. Trong mt phng cho k im x1 , x2 , . . . , xk . Chng minh rng tn ti im x0


vi tnh cht sau: T mi bn ca ng thng bt k l i qua x0 cha t nht mt phn
ba s im cho ( y ta tnh c nhng im nm trn ng thng l).

Li gii.

Ta xt tt c nhng na mt phng ng H, trong n cha hn hai phn ba s im


cho. Nhng mt phng nh vy tn ti, v d nh nhng mt phng cha ton b cc
im cho. Ta s chng minh rng mi cp ba na mt phng ny c im chung. ta
s dng nh l Helly cho v hn nhng tp li, ta tm c im x0 chung cho tt c cc
na mt phng. Sau , khng nh x0 c tnh cht c m t trong iu kin ca bi
ton.
Cho Hi , i = 1, 2, 3 l ba na mt phng trong mt phng m mi na mt phng cha s
2
im Si > k trong s im cho. K hiu s l s im trong H1 H2 . Khi , s lng
3
im nm trong H1 H2 l s1 + s2 s (v s lng im trong phn chung H1 H2 c
2 2 4
m hai ln). Ta c bt ng thc s1 > k, s2 > k. Ta nhn c k s1 + s2 s > k s,
3 3 3
1 2
ngha l s > k. Bi v mt phng th ba H3 cha s3 > k im, nn t nht mt trong
3 3
nhng im nm trong H1 H2 s nm trong H3 (nu ngc li th s lng im
cho thc s ln hn k). Suy ra x0 c th p dng nh Helly: Tn ti im x0 nm trn
2
tt c na mt phng m n cha hn s im cho. Cho l0 l ng thng bt k i
3

210
Hi tho khoa hc, Hng Yn 25-26/02/2017

k
qua x0 . Ta s chng minh rng mi na mt phng do l0 chia ra cha t nht s im.
3
Tht vy, gi s ngc li v ta k hiu H l na mt phng cng vi ng thng l0 m
k
n cha t hn s im.
3
Bi ton 6. Cho tp li A trong mt phng v h gm k > 3 nhng na mt phng ph
A. Chng minh rng tn ti mt h con gm ba na mt phng m chng cng ph A.
Li gii. Cho H1 , H2 , . . . , Hk l nhng na mt phng m chng ph A (ngha l mi
im ca A nm trong mt tp no trong Hi , i = 1, 2, . . . , k). Ta gi s rng khng
mt b ba na mt phng no ph c A v ta xt nhng tp hp A1 , A2 , . . . , Ak ,
y Ai , i = 1, 2, . . . , k nhn c t A v b i nhng im thuc mt phng Hi Nhng

tp hp Ai l li. Ta s chng minh rng mi b ba tp hp trong chng c im chung


v p dng nh l Helly. Ta khng nh A1 , A2 , A3 c im chung. Bi v, theo gi thit
phn chng, nhng na mt phng H1 , H2 , H3 khng ph A, s tm c mt im x0
thuc A khng cha trong bt c tp no ca H1 , H2 , H3 . Ngha l im x0 chung cho
A1 , A2 , A3 . Ta p dng nh l Helly v tm c im y0 chung cho tt c cc tp hp
Ai , i = 1, 2, . . . , k. Suy ra im y0 tuy nm trong A nhng khng nm trong bt c tp
no ca H1 , H2 , . . . , Hk . Ta nhn c s v l vi gi thit cho. Do , bi ton
c gii quyt.

Bi ton 7. Cho mt s hu hn cc ng thng (n 4). Bit rng vi ba ng thng


ty lun tn ti hnh trn c bn knh R v ct c ba ng thng. Chng minh rng
tn ti mt hnh trn c bn knh R ct c n ng thng .

Li gii.
Gi s d1 , d2 , . . . , dn l h hu hn cc ng thng (n 4). Vi mi ng thng di ta
xt Fi l hnh to bi hai ng thng song song vi di v cch di mt khong bng R.
Tm cc ng trn bn knh R m ct di th phi nm trong Fi . R rng Fi l hnh li vi
mi i = 1, n. Nh th, ta c mt h hu hn cc tp hp li F1 , F2 , . . . , Fn . Theo gi thit,

211
Hi tho khoa hc, Hng Yn 25-26/02/2017

vi mi i, j, k 1, 2, 3, . . . , n ta lun c Fi Fj Fk 6= (v vi mi i, j, k lun tn ti mt
hnh trn ct c di , d j , dk ). Theo nh l Helly suy ra:

F1 F2 Fn 6= .

Ly O F1 F2 Fn . Hnh trn tm O , bn knh R, (O , R) s ct tt c cc ng


thng d1 , d2 , . . . , dn . l iu phi chng minh.

Bi ton 8. Trn mt phng cho n hnh trn (n 4). Gi s c vi mi ba hnh trn u


c mt hnh trn bn knh r ct c ba hnh trn ny. Chng minh rng tn ti mt hnh
trn bn knh r ct c n hnh trn.

Li gii. Gi Si l hnh trn tm Ai , bn knh ri (i = 1, n, ) Si = ( Ai ; ri ).


Gi i l hnh trn tm Ai , bn knh ri + r (i = 1, n), i = ( Ai ; ri + r ).

Nh vy tm ca tt c cc hnh trn c bn knh r m ct Si u nm trong i .


Xt n tp hp li 1 , 2 , . . . , n . Vi i, j, k ty m i, j, k 1, 2, 3, . . . , n.
Theo gi thit tn ti hnh trn (Oi,j,k ; r ) ct c Si , S j , Sk , tc l Oi,j,k i j k . iu
chng t rng i j k 6= vi mi i, j, k 1, 2, 3, . . . , n.
Theo nh l Helly suy ra in=1 i 6= . Vy tn ti O in=1 i .
T T

Xt hnh trn tm O v bn knh r, (O , r ). Hnh trn ny r rng ct Si vi mi i = 1, n.


Bi ton c chng minh.

212
Hi tho khoa hc, Hng Yn 25-26/02/2017

3 Bi tp tng t
Bi 1. Trn mt bn hnh ch nht c cc hnh vung bng nhau v c cc cnh song
song vi cnh bn sao cho vi k hnh vung bt k lun tn ti hai hnh vung c th
ng cht vo bn ch bng mt chic inh. Chng minh rng c th ng cht tt c cc
hnh vung vo bn bng 2k 3 chic inh.
Bi 2. n na mt phng nm trn mt mt phng. Chng minh rng nu mi b ba na
mt phng c im chung, th tn ti mt im chung cho n na mt phng ny.
Bi 3. Chng minh nh l Helly cho n a gic li trn mt phng bng cch p dng bi
tp 2.
1
Bi 4. Trn ng trn cho n cung, mi cung c di nh hn cung ca ng trn.
3
Chng minh rng nu mi cp hai cung cho c im chung th tt c cc cung c im
chung.
Bi 5. khng gian cho n im A1 , A2 , A3 , . . . , An khong cch gia hai im khng vt
qu
1. Chng minh rng tt c nhng im ny c th ph bi hnh cu c bn knh bng
6
.
4
Bi 6. Cho hu hn im trn mt phng sao cho nu v mt hnh trn bn knh n v
c tm l mt im ty trong th hnh trn ny s cha nhiu hn mt na cc im
cn li. Hnh trn c bn knh nh nht l bao nhiu c th cha tt c cc im cho
(hnh trn ny khng nht thit phi c tm l mt trong s cc im k trn)?
Bi 7. Trn ng thng c 2k + 1 on thng sao cho mi on thng ct t nht k on
thng khc. Chng minh rng: Tn ti on thng ct tt c cc on thng cn li.
Bi 8. (nh l im trung tm) Mt im O c gi l im trung tm ca h n im
n
trong mt phng nu mi na mt phng ng cha O u cha t nht im ca h
3
cho (O c th khng thuc h im).
Chng minh rng mi h n im trong mt phng u c t nht mt im trung tm.
Bi 9. Trn ng trn cho hu hn cung trn c s o nh hn 180 sao cho hai cung
bt k lun c im chung. Chng minh rng: Tn ti ng thng i qua tm ng
trn ct tt c cc cung cho.
Bi 10. Chng minh rng: Vi mi a gic li cho trc lun tn ti mt im O trong
a gic sao cho vi mi ng thng ty qua O ct bin ca a gic ti A v B ta lun
1 OA
c 6 6 2.
2 OB

Ti liu
[1] Phan Huy Khi (2007), Cc bi ton hnh hc t hp,, NXB GD.
[2] Nguyn Hu in (2005), Mt s chuyn hnh hc t hp, NXB GD.
[3] G. Polya (1997), Sng to ton hc, NXB GD.
[4] V.N.Sachkov (2004), Nhp mn cc phng php t hp ca ton ri rc, NXB MCCME .
[5] on Qunh( ch bin) (2012), Ti liu chuyn ton hnh hc 12, NXB GD.

213
Hi tho khoa hc, Hng Yn 25-26/02/2017

S CHNH PHNG THEO MODUL BC TY


V Th Thun - Nguyn Th an Qu
THPT Chuyn Hng Yn

1 Kin thc c s
nh ngha 1. Cho s nguyn dng m. Nu hai s a v b c cng s d khi chia cho m th ta ni a
dng d vi b theo modul m. K hiu a b (mod m).

nh l 1 (Fecmat b). Gi s p l s nguyn t v mi a Z : a p a (mod p).

H qu 1. i) Vi p l s nguyn t, a l s nguyn tha mn ( a, p) = 1 th a p1 1 (mod p).


ii) Vi p l s nguyn t v a l s nguyn tha mn ( a, p) = 1 th a p( p1) 1 (mod p2 ).

nh l 2 (Wilson). Cho p l s nguyn t. Khi ( p 1)! 1 (mod p).

2 Mt s kt qu
2.1 S chnh phng theo mod p
nh ngha 2. Cho s nguyn t p. S nguyn a c gi l s chnh phng theo mod p nu tn
ti s t nhin x sao cho x2 a (mod p).

Hin nhin nu p l c ca a th a l s chnh phng theo mod p, do ta ch xem xt


trong trng hp ( a, p) = 1.

V d 1. 8 l s chnh phng theo mod 17 v 52 8 (mod 17).


Nhng 8 li khng phi l s chnh phng theo mod 11, v phng trnh x2 8 (mod 11)
khng c nghim.

V d 2. Cho p l s nguyn t l, a l s chnh phng theo mod p, a khng chia ht cho p.


Chng minh rng a l s chnh phng theo mod pk vi mi s t nhin k.
.
Li gii. Do a l s chnh phng theo mod p nn tn ti s t nhin x ( x2 a)..p.
.
Gi s ( x2 a)..pt , nhng khng chia ht cho pt+1 . Khi x2 a = pt .k vi (k, p) = 1.
Khi ta chng minh tn ti x 0 sao cho x 02 a chia ht cho pt+1 .
Xem xt x 0 dng x 0 = x + pt .r. Khi x 02 a = x2 + 2xpt r + p2t r2 a = pt k + 2xpt r

214
Hi tho khoa hc, Hng Yn 25-26/02/2017

.
Do , x 02 a chia ht cho pt+1 th k + 2xr..p. (1)
.
V p l, a khng chia ht cho p, ( x2 a)..p nn (2x, p) = 1. Do theo nh l Bezout, tn ti
r tha mn (1).
T , suy ra iu phi chng minh.
B 1. Cho p l s nguyn t l. Phng trnh x2 a (mod p) ch c 1 nghim x 0
(mod p) nu a 0 (mod p) v v nghim hoc c 2 nghim (mod p) nu p - a.
Chng minh.
Nu a 0 (mod p), ta c x2 0 (mod p), nn p | x2 , suy ra p | x, hay x 0 (mod p).
Nu p - a, v phng trnh x2 a (mod p) c t nht mt nghim l x b (mod p). Khi
b2 a (mod p) nn (b)2 a (mod p). Vy x b (mod p) cng l mt nghim ca
phng trnh. Ta s chng minh 2 nghim ny phn bit.
Tht vy, gi s b b (mod p), th p | 2b, nhng p l nn p | b, hay p 0 (mod p), suy
ra a 0 (mod p) (mu thun). Vy b v b l 2 nghim phn bit ca phng trnh.
Gi s ngoi nghim b v b, phng trnh cn nghim x c (mod p). Khi b2 c2
(mod p), hay p | (b2 c2 ), suy ra c b (mod p) hoc c b (mod p). Nh vy, nghim c
ny trng vi nghim b hoc b.
Do , phng trnh ban u hoc v nghim hoc ch c 2 nghim.
V d 3. x2 8(mod 17) c 2 nghim l 5 v 12.

Nhng ch rng, kt qu ny l sai khi p = 2, v x2 1 (mod 2) ch c mt nghim l


x 1(mod 2)
p1
H qu 2. Cho p l s nguyn t l. Khi trong tp 1; 2; . . . ; p 1 c s chnh phng
2
p1
theo mod p v s khng phi l chnh phng mod p.
2
Chng minh.
p1 p1
Ta c k2 ( p k)2 (mod p), vi mi k = 1, Nh vy, tp 1; 2; . . . ; p 1 c gi
2 2
tr bnh phng khc nhau theo mod p. Do tp 1; 2; . . . ; p 1 l h thng d thu gn mod p
p1 p1
nn c s chnh phng theo mod p v s khng phi l chnh phng mod p.
2 2

2.2 K hiu Legendre


 
a
nh ngha 3. Cho p l s nguyn t l v ( a, p) = 1. K hiu Legendre c nh ngha bi
p

  (
a 1 a l s chnh phng theo mod p
=
p 1 a khng phi l s chnh phng theo mod p
   
5 2 11
V d 4. (5, 11) = 1. = 1, v 4 5 (mod 11). Tng t nh vy, = 1, v 62 11
11 5
(mod 5).

215
Hi tho khoa hc, Hng Yn 25-26/02/2017

Trong trng hp p = 2, ta ch c 2 phng trnh sau x2 0 (mod 2) v x2 1 (mod 2)


v 2 nghim ln lt ca n l x 0 (mod 2) v x 1 (mod 2), tc l c 0 v 1 u l
thng d bc hai mod 2.
Trong trng hp p l hp s, ta xem xt v d sau.

V d 5. Gii phng trnh x2 79 (mod 91).


Li gii. V 91 = 7.13 nn ta xem xt 2 phng trnh ng d sau
x2 79 (mod 7) v x2 79 (mod 13) 
x 3 (mod 7)
x2 79 (mod 7) x2 2 (mod 7)
x 4 (mod 7)

2 2 x 1 (mod 13)
x 79 (mod 13) x 1 (mod 13)
x 12 (mod 13)
S dng nh l thng d Trung hoa,ta xem xt bi ton trong 4 trng hp, tuy nhin
v m2 = (m)2 , nn ta ch xem xt 2 trng hp sau
TH1. (
x 3 (mod 7)
x 1 (mod 13)

m 7 13
p 13 7
1
s p (mod m) 6 2
a 3 1

Vy x 13.6.3 + 7.2.1 248 66(mod 91). V x 66 25(mod 91) cng l mt


nghim khc ca phng trnh.
TH2. (
x 3 (mod 7)
x 12 (mod 13)

m 7 13
p 13 7
1
s p (mod m) 6 2
a 3 12

Vy x 13.6.3 + 7.2.12 402 38(mod 91). V x 38 53(mod 91) cng l mt nghim


khc ca phng trnh.
Di y l mt s cng c tnh ton k hiu Legendre.
 
a p 1
nh l 3 (Euler). Cho p l mt s nguyn t l, a > 0, (a, p) = 1. Khi = a 2 (mod p).
p

 Ta xt hai trng hp sau


Chngminh.
a
TH1. = 1.
p

216
Hi tho khoa hc, Hng Yn 25-26/02/2017

Khi , tn ti mt s t nhin b sao cho sao cho b2 a (mod p).


 p 1 p 1 p 1
Bi vy, b2 2 a 2 (mod p) b p1 a 2 (mod p) Nu p | b th p | b2 = a (mu
thun).
p 1
p1 1 (mod p ). Do a 2 1 (mod p ) v
 Nu p - a, theo nh l Fermat ta c b
a p 1
a 2 (mod p).
p  
a
TH2. = - 1.
p
Xt tp 1; 2; . . . ; p 1. Ta ch ra rng tp ny s c chia thnh nhng cp (s, t) sao cho
st a (mod p).
Gi s s 1, 2, . . . , p 1 th (s, p) = 1 nn tn ti nghch o mod p ca s, k hiu l s1 .
Khi , ta chn t 1, 2, . . . , p 1 sao cho t s1 a (mod p).
1 2
Ta chng minh rng, s v t l phn  Tht vy, gi s s s a (mod p), khi s a
 bit.
a
(mod p), mu thun vi gi thit = - 1.
p
p1
Nh vy tp 1, 2, . . . , p 1 c chia thnh cp (s, t) nh th.
p 1
2
Do 1.2 . . . ( p 1) a 2 (mod p).
Theo nh l Wilson, th ( p 1)! 1 (mod p).
p 1
Vy a 2 1 (mod p).
T , suy ra iu phi chng minh.

p 1
V d 6.Gis vi p = 13 v a = 10, th a 2 = 106 1(mod 13).
10
Do , = 1 v x2 10(mod 13) c nghim. Tht vy 72 = 49 10(mod 13).
13
   
a b
B 2. Nu a b (mod p) th = .
p p
Chng minh.
Nu a b (mod p) th x2 a (mod p) x2 b (mod p). Do a l  schnh phng
a b
theo mod p khi v ch khi b l s chnh phng theo mod p. Vy = .
p p  
a
Ch rng, ta c th s dng kt qu ny p dng cng thc Euler tnh trong
p
trng hp a < 0 bng vic thay a bi b > 0 sao cho a b (mod p).
B 3. Cho p l s nguyn t l, a, b > 0, ( a, p) = (b, p) = 1. Khi
    
a b ab
=
p p p
.
Chng minh.
Theo nh l Euler, th   
a b p 1 p 1
a 2 b 2 (modp)
p p

217
Hi tho khoa hc, Hng Yn 25-26/02/2017
 
ab p 1
( ab) 2 (modp)
p
    
a b ab
Do vy (mod p).
p p p
V 2 v ca ng thc trn ch nhn gi tr bng 1 hoc - 1 v p l s nguyn t l nn
    
a b ab
=
p p p
.
 2
a
H qu 3. Cho p l s nguyn t l, a > 0, ( a, p) = 1. Khi = 1.
  p
a
Chng ta c th s dng kt qu trn tnh cho trng hp a l s chnh phng.
p
B 4. (
1
 
1 nu p = 4k + 1
=
p 1 nu p = 4k +3
Chng minh.
Theo cng thc Euler, ta c
1 p1
   
p 1 p 1
= ( p 1) 2 (1) 2 (mod p)
p p
( (
(1)2k nu p = 4k + 1 1 nu p = 4k + 1
= =
(1)2k+1 nu p = 4k + 3 1 nu p = 4k + 3

S dng b Gauss, ta cng c th chng minh c rng


1
 
p2 1
= (1) 8
p
p2 1
(Ch rng vi p l s nguyn t l th p = 2k + 1, p2 1 = 4k (k + 1) nn l s t
8
nhin)

1
 
V d 7. = 1, v 13 = 4.3 + 1. Do , phng trnh x2 1(mod 13) c nghim. Tht
13
vy
52 = 25 12 1 (mod 13)
1
 
Tng t, = 1, v 23 = 4.5 + 3. Do , phng trnh x2 1(mod 23) v nghim.
23
 
2 72 1
= (1) 8 = 1
7
Do , phng trnh x2 2(mod 7) c nghim. Tht vy x 3 (mod 7) l mt nghim ca
phng trnh.

218
Hi tho khoa hc, Hng Yn 25-26/02/2017

V d 8. Cho x, y l cc s nguyn. Chng minh rng x2 2 khng chia ht cho 2y2 + 3.


Li gii. Gi s 2y2 + 3 | x2 2
v 2y2 + 3 l s l, nn mi c nguyn t ca n u l, gi s mt c nguyn t l l p.
Suy ra p cng l c ca x2 2, hay phng trnh x2 2 (mod p) c nghim. Vy

p2 1 2 .
(1) 8 = = 1 p2 1..16
p

p 1(mod 8)

p 1(mod 8)

Nh vy, mi c nguyn t ca 2y2 + 3 chia 8 d 1 hoc -1 nn 2y2 + 3 1(mod 8).


iu ny mu thun v y2 0, 1, 4(mod 8) nn 2y2 + 3 3, 5(mod 8). Vy iu gi s l sai,
suy ra iu phi chng minh.
V d 9 (VN TST 2004). Cho s nguyn dng n. Chng minh rng 2n + 1 khng c c
nguyn t dng 8k + 7.
Li gii.
Gi p l c nguyn t ca 2n + 1 v p 7(mod 8).
Nu n chn ta c
1 p1
 
p 1
n
2 1 (mod p) = 1 (1) 2 = 1 2 | p 1(mod 4)
p 2
iu ny mu thun vi p 7(mod 8)
Nu n l ta c

     
2 1 1 p 1 p2 1
2n+1 2 (mod p) =1 = 1 (1) 2 (1) 2 = 1 2 |
p p p
p1
p 1(mod p)(1)
2
p 1 p2 1 p 1 p2 1
V p 7(mod 8) nn (1) 2 = 1 v (1) 8 = 1 ko theo (1) 2 (1) 8 = 1 (mu
thun vi (1))
Nh vy, gi thit phn chng sai, suy ra iu phi chng minh.
V d 10. Cho n l s nguyn dng l v u l mt c nguyn dng l ca 3n + 1. Chng
minh rng u 1 chia ht cho 3 .
Li gii.
Gi p l mt c nguyn t l ca u, khi p cng l c nguyn t l ca 3n + 1.
Khi 3n + 1 0 (mod p) 3n+1 3 (mod  p ) V n l nn n + 1 l chn,
3
do -3 l s chnh phng theo mod p, hay = 1(*) Theo nh l Eu-
 p
3 1
    
3 p 1 3
ler, ta c = = (1) 2 . (1) Theo lut tng h Gauss
   p p p   p
3 p (31)( p1) p 1 3 p 1
 p
= (1) 4 = (1) 2 = (1) . 2 (2) T (1) v (2) suy
p 3 p 3
3  p  p  
2 31
ra = (1) p1 . Nu p 2(mod 3) = 2 2 1(mod 3).
p  3 3 3
3

Ko theo = 1,mu thun vi (*) Hin nhin p 6= 3, do p 1(mod 3).
p

219
Hi tho khoa hc, Hng Yn 25-26/02/2017

V u l s l nn mi c nguyn t ca u u l l v tha mn chia 3 d 1, do u chia 3


d 1, hay u 1 chia ht cho 3.

V d 11. Cho p l s nguyn t, p > 3, p c dng 3k + 1. Chng minh rng


p .
i=1 i2 + i + 1 ..p.
Li gii.
. .
Ta s chng minh tn ti i sao cho (i2 + i + 1)..p (2i + 1)2 + 3..p 3 l s chnh phng
p 1
theo mod p (3) 2 1 (mod p).

V d 12. Cho p l s nguyn t l. Chng minh rng tn ti s t nhin a sao cho a < p + 1
v a l s chnh phng theo mod p
h pi
Li gii. Gi s a l s chnh phng theo mod p dng b nht. t b = + 1.
a
Khi 0 < ab p < a nn ab p khng phi l s chnh phng theo mod p. Do
ab p
      
a b b
1 = = =
p p p p
p
Vy b l mt s chnh phng theo mod p. Suy ra a b, mt khc b < + 1 nn ta c
a
p
a < + 1 (1).
a
Nu a p th hin nhin a < p + 1.
p p
Nu a > p th < , kt hp vi (1) suy ra a < p + 1.
a p
p1
 
V d 13. Chng minh rng nu p l s nguyn t dng 4k + 1 th x = ! l mt
2
nghim ca phng trnh x2 + 1 0 (mod p).
p1
Li gii. Ta c i ( p i ) (mod p), vi mi i = 1, 2, . . . ,
2
Nhn v vi v ca cc ng thc ny ta c

p1 p1
   
p 1
! (1) 2 .( p 1)( p 2) . . . p (mod p)
2 2

p1
 
Vi x = ! th
2
p1 p1 p1
      
2
p 1 p+1
x = ! ! !.(1) 2 ( p 1).( p 2) . . .
2 2 2 2
p 1 p +1
(1) 2 .( p 1)! (1) 2 1 (mod p)(theo nh l Wilson)

nh l 4. Cho x, y l 2 s t nhin nguyn t cng nhau, a, b, c l cc s t nhin ty . Nu p l


c nguyn t l ca ax2 + bxy + cy2 m khng l c ca tch abc th D = b2 4ac l s chnh
phng theo mod p.
Trng hp c bit nu p | x2 Dy2 v ( x, y) = 1 th D l s chnh phng theo mod p.

220
Hi tho khoa hc, Hng Yn 25-26/02/2017

Chng minh.
t N = ax2 + bxy + cy2 . Khi 4aN = (2ax + b)2 Dy2 . V N 0 (mod p) nn (2ax +
b)2 Dy2 (mod p)(1).
. . .
Nu y..p th ax2 ..p, m a khng chia ht cho p nn x..p. Mu thun vi gi thit ( x, y) = 1.
Nh vy, tn ti y1 sao cho yy1 1 (mod p).
Nhn c 2 v ca (1) vi y21 ta c

(2axy1 + byy1 )2 D (yy1 )2 D (mod p)

Nh vy D l s chnh phng theo mod p.


p1
Vi mi s t nhin a m p khng l c ca a,v k = 1, 2,. . . , tn ti duy nht s
2
p1 p1
 
rk , . . . , 2, 1, 1, 2, . . . , sao cho ka rk (mod p). Hn na khng c hai
2 2


gi tr no ca rk c gi tr tuyt i bng nhau. Do tp |r1 | , |r2 | , . . . , r p 1 thc


2
p1
cht l mt hon v ca tp (1, 2, . . . , p0 ) (p0 = ).
2
Bi vy

0 a.2a.3a . . . p0 a r1 .r2 .r3 . . . r p0


ap = =
1.2.3 . . . p0 1.2.3 . . . p0

t rk = k . |rk | vi k = 1, 2,. . . ,p, k = 1 v p dng cng thc Euler ta c

 
a
nh l 5. = 1 2 . . . p0
p
   
ka 2ka
Nu k 1 (mod p)th ka = tp |rk |, khi = t 1, = 2t 1, hay
p p
2ka
" #
   
2ka ka
= + 1 V vy k = (1) p .
p p

2.3 Lut thun nghch Gauss


T kt qu ca 5 ta chng minh c b sau
   
a p 0 2ka
B 5 (B Gauss). = (1)S , vi S = k=1 .
p p
 
a
B Gauss gip ta d dng tnh c gi tr ca k hiu Legendre trong trng
p
hp a v p nh.    
2 p 0 4k
V d vi a = 2 th = (1)S , vi S = k=1 .
p p

221
Hi tho khoa hc, Hng Yn 25-26/02/2017

p0
   0
0 p
R rng, s hng u tin ca tng trn bng 0, v p s hng cn li l bng 1.
2  0   2
0 p p+1
Do S = p = v S chn khi p 1(mod 8), S l khi p 3(mod 8). T
2 4
iu ny, ta c c nh l sau
nh l 6.
p+1
" #
 
2
= (1) 4
p
Ni cch khc, 2 l s chnh phng theo mod p (p nguyn t, p>2) khi v ch khi p 1(mod 8).

Cc pht biu sau c ch ra mt cch tng t


nh l 7. a) -2 l s chnh phng theo mod p khi v ch khi p 1(mod 8) hoc p 3(mod 8).
b)-3 l s chnh phng theo mod p khi v ch khi p 1(mod 6).
c)3 l s chnh phng theo mod p khi v ch khi p 1(mod 12).
d)5 l s chnh phng theo mod p khi v ch khi p 1(mod 10).
V d 14. Chng minh rng c v s s nguyn t dng
a) 4k + 1
b) 10k + 9.
Li gii. a) Gi s ngc li, p1 , p2 , . . . , pn l tt c cc s nguyn t c dng 4k+1. Theo
nh l Euler, tt cc cc c nguyn t ca N = (2p1 p2 . . . pn )2 + 1 u c dng 4k + 1. Tuy
nhin, N khng chia ht cho s no trong cc s p1 , p2 , . . . , pn (mu thun).
Vy iu gi s l sai, do c v s s nguyn t dng 4k + 1.
b) Chng minh tng t a, y N = 5 (2p1 p2 . . . pn )2 1.
V d 15. Chng minh rng mi c nguyn t p ca s n4 n2 + 1 (n N) u c dng
12k + 1.
Li gii. Ta c n4 n2 + 1 = (n2 1)2 + n2 = (n2 + 1)2 3n2 Theo 4 th -1, 3 l cc s chnh
phng theo mod p.
T nh l Euler, -1 l s chnh phng theo mod p khi v ch khi p 1(mod 4)
T 7, 3 l s chnh phng theo mod p khi v ch khi p 1(mod 12).
Kt hp 2 iu trn suy ra p 1(mod 12).
     2015 
1 5 5
V d 16. Tnh S = + ++
2017 2017 2017
Li gii. Ta c 2017 l s nguyn t  v 2017 7(mod 10) nn theo 7 th 5 khng phi  l
5 5
s chnh phng theo mod 2017, hay = 1, li theo nh l Euler th
2017 2017
51008 (mod 2017). Vy 2017 | 51008 + 1 , ko theo 2017 | 5i 51008 + 1 = 5i+2008 + 5i . V 5i v
 

5(i+1008) u khng chia ht cho 2017 nn


 i   1008+i 
5 5 5i + 5i+1008
+ = 1
2017 2017 2017

222
Hi tho khoa hc, Hng Yn 25-26/02/2017

1 + 5 + 52 + + 52015
Cho i chy t 0 n 1007, sau cng v vi v, ta c S =
2017
52016 1
1008 = 1008
8068
nh l 8 (Lut thun nghch Gauss). Cho p, q l s nguyn t l phn bit. Khi
  
p q 0 0
= (1) p q
q p
p1 0 q1
Trong p0 = ,q =
2 2
 
q0 kp
Chng minh. K hiu S( p, q) = k =1
q
B 6. S( p, q) + S(q, p) = p0 q0 .  
kp
Chng minh. Vi mi k N th l s cc im c ta nguyn (k, l ) trong h trc
q
kp
ta tha mn 0 < l < hay 0 < ql < kp. Do S( p, q) l s cc im c ta
q
nguyn (k, l ) tha mn 0 < k < p0 v 0 < ql < kp. Bi vy S( p, q) l s cc im c ta
nguyn dng nm trn hoc trong hnh ch nht ABCD v pha di ng AE vi
A(0, 0), B( p0 , 0), C ( p0 , q0 ), D (0, q0 ), E( p, q).
Tng t, S(q, p) l s cc im c ta nguyn dng nm trn hoc trong hnh ch nht
ABCD v pha trn ng AE. Suy ra S( p, q) + S(q, p) chnh l s im c ta nguyn
dng trong hnh ch nht ABCD. Vy S( p, q) + S(q, p) = p0 q0 .(pcm) 

Ta tip tc i chng minh nh l.


Ta c
p2 1
S( p + q, q) S( p, q) = 1 + 2 + 3 + + p0 =
8
 
2 p2 1
T 6, ta c = (1) 8 .
p
p+q

      
2 p 2p 2( p + q )
Theo b Gauss = = = 2 = (1)S( p+q,q) =

q q q q q
 
2
(1)S( p,q)
q  
p
Do = (1)S( p,q) .
q 
q
Tng t = (1)S(q,p)
   p
p q 0 0
Vy = (1)S( p,q)+S(q,p) = (1) p q 
q p

223
Hi tho khoa hc, Hng Yn 25-26/02/2017
 
814
V d 17. Tnh
2017
        
814 2 11 37 11 37
Li gii. = =
2017 2017 2017 2017 2017 2017
Theo
 lut
  thun nghch
   Gauss
11 2017 4
= = =1
 2017   11   11    
1

37 2017 19 37
= = = = = 1
2017
  37 37 19 19
814
V = 1, hay 814 khng phi l s chnh phng theo mod 2017.
2017

2.4 Bi tp tng t
Bi 1. Chng minh rng s t nhin a l s chnh phng theo mod p vi mi s nguyn t
p khi v ch khi a l s chnh phng hon ho.
Bi 2 (Turkey TST 2013). Tm nghim nguyn dng ca phng trnh
m 6 = n n +1 + n 1
Bi 3. Cho p l s nguyn t l. Chng minh
2 p2 1
( ) = (1) 8
p
Bi 4. a) Tm cc s nguyn dng x, y sao cho y2 5| x2 + 1
b) Tm cc s nguyn dng x, y sao cho 2y2 + 3| x2 2
Bi 5. Tm tt c cc s nguyn t p sao cho 5 l s chnh phng theo modulo p
Bi 6 (Indonesia TST 2009). Chng minh rng tn ti v hn s nguyn dng n sao cho
n2 + 1 khng l c ca n!.
Bi 7 (Serbia TST 2008). Tm tt c cc nghim nguyn khng m ca phng trnh
12x + y4 = 2008z
n
Bi 8 (i Loan MO 1997). Cho k, n l cc s nguyn dng tha mn k = 22 + 1. Chng
k 1
minh rng k l mt s nguyn t khi v ch khi k l mt c ca 3 2 + 1.
Bi 9 (Serbia TST 2007). Tm tt c cc cp s nguyn dng ( x, n) tha mn phng trnh
x3 + 2x + 1 = 2n .
Bi 10. Cho m, n l cc s nguyn dng tha mn
(5m 1 ) = 5m 1
Chng minh rng gcd(m, n) > 1.
Bi 11. Cho m, n l hai s nguyn dng. Chng minh rng
6m|(2m + 3)n + 1 4m|3n + 1

224
Hi tho khoa hc, Hng Yn 25-26/02/2017

S DNG I LNG BT BIN V N BIN


TRONG TON T HP

V Th Thun
THPT Chuyn Hng Yn

1 Kin thc c bn
Trong mt lot bi ton ta thng gp tnh hung sau, mt h thng no thay i
lin tc trng thi ca mnh v cn phi ch ra mt iu g v trng thi cui cng ca
n. Kho st cc b sau tt c cc ln thay i nh vy l mt vic lm rt phc tp v
kh khn. Nhng ta li c th tr li cu hi m bi ton yu cu nh tnh mt i lng
c bit no c trng cho tt c cc trng thi ca h thng . Hai i lng thng
c s dng l bt bin v n bin. Bt bin l mt i lng (hay tnh cht) khng thay
i trong qu trnh chng ta thc hin cc php bin i. n bin l mt i lng (hay
tnh cht) thay i, nhng ch theo mt chiu (tc l tng ln hoc gim xung). Da vo
i lng bt bin hoc n bin, ta c th gii quyt c cc bi ton v thut ton.

Bi ton m rng 1. (Bi ton tm kim thut ton). Cho trng thi ban u 0 v trng thi kt
thc n . Hi c hay khng thut ton T trn A sao cho khi thc hin T hu hn ln ta thu c n ?
T T T T
o 1 2 . . . n

Bi ton m rng 2. Cho thut ton T trn A v trng thi ban u .


a) Xt trng thi A. Hi c th nhn c t sau hu hn ln thc hin thut ton T hay
khng?
b) Tm tp hp gm tt c cc trng thi c th nhn c t sau hu hn bc thc hin thut
ton T
= { A : = T n ()}

Cc bt bin thng c s dng l tnh chn l, s d trong mt php chia, mt tng,


mt tch, mt biu thc i s. i khi ngi ta cn s dng s t mu, tc l chia cc i
tng ang xt ra lm cc nhm (mi nhm gm cc i tng c nh du cng mt
mu).
Trn thc t phng php s dng i lng n bin hoc bt bin c tin hnh nh
sau: Tnh mt i lng no bng 2 cch, u tin n c tnh trng thi ban u v
trng thi cui cng, sau kho st s thay i ca n qua mt s ln thay i nh lin
tip.

226
Hi tho khoa hc, Hng Yn 25-26/02/2017

2 S dng i lng bt bin gii bi ton t hp


2.1 Bt bin lin quan n tnh chia ht (hoc s d trong mt php chia)
a) Bt bin l tnh chn, l.

V d 1. Cho mt bn c kch thc 8 8, t en mt bt k. Mi bc cho php i mu


tt c cc trn cng mt hng hoc mt ct ( en thay bng trng v ngc li). Hi c
khi no tt c cc trn bn c cng mu khng?

Nhn xt 1. + Vic kho st tt c cc phng n i mu trong bi ton l khng thc hin


c, ta th xem c quy lut no chi phi tt c cc phng n ny khng?
Ta xem xt s thay i s lng en v trng
Ban u, c 1 en, 63 trng
Kt thc c 64 en, 0 trng (hoc 64 trng, 0 en)
Mi bc thc hin s i mu 8 (1 hng hoc 1 ct), gi s trc bc i mu th k, c
xk en v 64 xk trng. Bc i mu k bin a en thnh a trng, 8 a trng thnh
8 a en. Nh vy, sau bc i mu th k, s en l xk a + 8 = xk + (8 2a), s
trng 64 xk + (2a 8).
T , ta c c nhn xt v s thay i s lng trng, en sau mi php i mu.
Li gii.
Mi bc thc hin s i mu 8 (1 hng hoc 1 ct), gi s trc bc i mu th k,
c xk en, v 64 xk trng. Bc i mu k bin a en thnh a trng, 8 a trng
thnh 8 a en. Nh vy, sau bc i mu th k, s en l xk a + 8 = xk + (8 2a),
s trng 64 xk + (2a 8).
Nh vy, sau mi ln thc hin, th s en tng hoc gim mt s chn. Ban u c 1
en, nh vy, sau mi bc i mu, th s lng en vn l mt s l. Nh vy, khng
th t c trng thi c 64 en, 0 trng hoc 0 en, 64 trng.

V d 2. Cho mt bn c kch thc 9 9, gm 1 en v 80 trng. Thc hin thut ton,


mi ln thay i mu tt c cc trn cng 1 hng hoc 1 ct ( en thay bng trng v
ngc li). Hi c khi no tt c cc trn bn c cng mu en khng?

Nhn xt 2. Thut ton tng t nh bi trc, tuy nhin, mi ln thc hin thut ton s
i mu 9 , nh vy tnh chn l ca s lng en khng bt bin. Tuy nhin, k mt
cht, ta hon ton c th a bi ton v bi ton 1: Xt hnh vung 8 8 1 gc m cha
en. Khi , vic thc hin thut ton ca bi s hoc khng thay i hnh vung 8 8
ny, hoc thay mu tt c cc trn cng mt hng hoc mt ct.
Li gii. Gi s mu en nm phn hnh vung 8 8 c t m trn hnh v. Khi
, mi ln thc hin thut ton th hoc khng lm thay i hnh vung 8 8 ny, hoc s
i mu 1 hng hoc 1 ct ca hnh vung 8 8 ny. Nh vy, theo bi ton 1 th s en

227
Hi tho khoa hc, Hng Yn 25-26/02/2017

trong hnh vung 8 8 ny lun l s l. Tc l khng xy ra trng hp c 64 u mu


en. Vy khng xy ra trng hp c bn c mu en.

V d 3. Hai ngi chi c. Sau mi vn ngi thng c 2 im, ngi thua c 0 im,
nu ha th mi ngi c 1 im. Hi sau mt s vn liu c th xy ra trng hp mt
ngi c 7 im v ngi kia c 10 im c khng?
Li gii.
Gi S(n) l tng s im ca c hai ngi sau vn th n.
Ta c S(n + 1) = S(n) + 2, Do , S(n) bt bin theo modun 2.
Suy ra S(n) S(0) 0( mod 2), n 0
Vy khng th xy ra trng hp mt ngi c 7 im, mt ngi c 10 im.

Nhn xt 3. Tnh bt bin y l tnh chn l ca tng s im ca hai ngi chi.

V d 4. Vit cc s 1, 2, 3, . . . , 2014 ln bng. Thc hin thut ton, mi ln xa i hai s a, b


bt k v vit thm s c = | a b| . Chng minh rng s cn li cui cng trn bng l mt
s l.
Li gii.
V a + b + ( a b) = 2a nn a + b v a b cng tnh chn, l. Gi S(n) l tng cc s trn
bng sau bc th n. V sau mi bc, tng a + b c thay bi c = | a b| nn S(n) gi
nguyn tnh chn, l, hay bt bin theo modun 2.
Mt khc S(0) = 1 + 2 + .. + 2014 = 1007.2015 l s l nn S(n) l l vi mi n.
Vy s cui cng cn li trn bng l mt s l.

228
Hi tho khoa hc, Hng Yn 25-26/02/2017

Nhn xt 4. Tnh bt bin y l tnh chn l ca tng cc s trn bng.

2.2 Bi tp tng t
Bi 1 (Trch thi V ch Lin X - 1975). Trn bng ta vit mt s ch s 0, mt s ch s
1 v mt s ch s 2. Sau ta c xa i mt cp hai ch s khc nhau v thay th vo
mt ch s khc vi hai ch s xa. Chng minh rng: Nu trong kt qu cui cng trn
bng ch cn mt ch s th kt qu khng ph thuc vo th t cc cp ch s c xa

Bi 2. Cho dy s 1, 2, 3, 4, . . . , 2013. Mi ln thay hai s a, b bi s a b. Hi c khi no thu


c ton s 0 hay khng? Thay 2013 bng s t nhin N, tm iu kin ca N kt qu
cui cng thu c ton s 0?

Bi 3. Cho mt bn c 7 7, t en 4 4 gc. Thc hin thut ton, mi ln thay i mu


1 hng hoc mt ct ( en thay bng trng v ngc li). Hi c khi no tt cc trn bn
c cng mu khng?
b) Bt bin lin quan n tnh chia ht cho mt s ln hn 2 hoc s d trong mt php chia
cho mt s ln hn 2.

V d 5. ( thi chn i tuyn d thi HSGQG tnh Bc Ninh, nm 2007)


Trn bn c 2007 vin bi gm 667 bi xanh, 669 bi , 671 bi vng. Thc hin thut ton nh
sau, mi ln ly i 2 vin bi khc mu v t thm 2 vin bi c mu cn li. Hi c th nhn
c trng thi m trn bn ch cn li cc vin bi cng mu c khng?
Li gii.
Gi X (n), D (n) v V (n) tng ng l s bi mu xanh, s bi mu v s bi mu vng sau
bc th n.
Xt cc i lng X (n) D (n), D (n) V (n), V (n) X (n).
V mi ln ly i 2 vin bi khc mu v t thm 2 vin bi c mu cn li nn cc i lng
X (n) D (n), D (n) V (n), V (n) X (n) bt bin theo modun 3.
Ta c X (0) D (0) = 2, D (0) V (0) = 2, V (0) X (0) = 4.
Nn s d ca cc i lng ny khi chia cho 3 sau mi ln thc hin thut ton l 1; 2; 1.
Ta li thy, s bi trn bn lun khng thay i (mi ln thc hin, ly i 2 vin
v t thm 2 vin khc), vy nu trn bn ch cn cc vin bi cng mu tc l s
vin bi , xanh, vng cn li l mt hon v ca tp 2007, 0, 0, vy cc i lng
X (n) D (n), D (n) V (n), V (n) X (n) u chia ht cho 3 (mu thun).
T , suy ra khng th nhn c trng thi m trn bn ch cn li cc vin bi cng mu.

Nhn xt 5. Vic ly thm i 2 vin bi khc nhau v t thm 2 vin bi c mu cn li to ra


bt bin v hiu s cc vin bi khi chia cho 3 bi ton.

229
Hi tho khoa hc, Hng Yn 25-26/02/2017

V d 6. Mi bc cho php chn 1 s a, phn tch a thnh tch hai s m, n v vit ln bng
m 2, n 2 ty (v d a = 99 = 9 11, 9 2 = 7, 11 + 2 = 13, nh vy c th vit ln
bng s 7 v s 13 thay cho s 99). Hi sau mt s bc nh vy,t s 99. . . ..99 (2012 ch s
9) c thu c trn bng mt dy gm ton cc s 9 khng?
Li gii.
Nu a l mt s chia cho 4 d 3 th trong 2 s m, n c mt s chia 4 d 1, mt s chia 4 d 3.
Mt s chia 4 d 1 th cng hay tr 2 u chia 4 d 3.
Vy, sau khi thc hin php thay i th t 1 s chia 4 d 3 lun tn ti 1 s chia 4 d 3 cn
li.
S 99. . . ..99 (2012 ch s 9) chia 4 d 3, nh vy, sau mt s bc bin i c ch ra
bi th cui cng, lun tn ti mt s chia 4 d 3.
Mt dy gm ton s 9, tc l dy gm ton s chia 4 d 1. Vy khng th thu c dy s
nh trn.

Nhn xt 6. Bt bin trong bi ton l s tn ti mt s chia 4 d 3 trong dy.

V d 7. Trn bng c hai s 1 v 2. Thc hin vic ghi s theo quy tc sau, nu trn bng c
hai s a, b th c php ghi thm s c = a + b + ab. Hi bng cch , c th ghi c cc
s 2010 v 11111 hay khng?
Li gii.
Dy cc s c vit l 1, 2, 5, 11, 17, . . .
D dng chng minh c cc s c vit thm trn bng u chia cho 3 d 2. Bt bin
trn cho php ta loi tr c s 2010 trong dy cc s c vit trn bng. Tuy nhin, bt
bin khng cho php ta loi tr s 11111. Ta i tm mt bt bin khc. Quan st cc s
c vit v quy tc vit thm s, ta c c = a + b + ab c + 1 = ( a + 1)(b + 1). V nu cng
thm 1 vo cc s thuc dy trn, ta c dy mi 2, 3, 6, 12, 18, . . .
Nh vy, nu cng thm 1 vo cc s vit thm th cc s ny u c dng 2n .3m vi m N.
Do 11111 + 1 = 11112 = 3.8.463 nn khng thuc dy cc s vit c.
Do khng th vit c cc s 2010 v 11111.
Nhn xt 7. Bi ton trn s dng 2 bt bin.

2.3 Bi tp tng t
Bi 4. Cc s t nhin 0, 1, 2, 3, . . . c vit trong cc ca mt bng vung kch thc
2003 2003 theo vng xoy trn c (xoy ngc chiu kim ng h) sao cho s 0 nm
trung tm (tm ca bng). Cc dng v ct ca bng c nh s tng dn t di ln trn
v t tri sang phi (bt u t s 1).
a) S 2004 nm dng no, ct no? Ti sao?
b) Thc hin thut ton sau, ln u tin, thay s 0 trung tm bi 1998; mi ln tip theo,
cho php ly ra 12 s trong 12 lin tip trong cng mt hng hoc trong cng mt ct hoc
trong cng mt hnh ch nht 3 4 ri tng mi s ln mt n v. Hi sau mt s ln
nh vy ta c th lm cho tt c cc s trong bng u l bi ca 2004 hay khng? Ti sao?

230
Hi tho khoa hc, Hng Yn 25-26/02/2017

Bi 5. x s n, nng cng cha b mt con rng hung hn 100 u bt i. Chng hong


t ln ng i cu cng cha, chng c 2 thanh kim, thanh 1 cht c 21 u rng,
thanh 2 cht c 3 u rng nhng rng li mc thm 2012 u. Nu hong t cht c
ht u ca rng th cu c cng cha. Hi hong t c cu c cng cha khng? Nu
s lng u rng ban u l N, N tha mn iu kin g th hong t cu c cng cha?

2.4 Bt bin l mt tng, mt tch, mt biu thc i s.


V d 8. Mt dy gm c 19 phng. Ban u mi phng c mt ngi. Sau , c mi ngy
c hai ngi no chuyn sang hai phng bn cnh nhng theo hai chiu ngc nhau,
Hi sau mt s ngy, c hay khng trng hp m
(a) Khng c ai phng c th t chn.
(b) C 10 ngi phng cui.
Li gii.
nh s cc phng theo th t t 1 n 19 Ta cho mi v khch mt th ghi s phng mnh

ang . Gi S(n) l tng cc s ghi trn th ca tt c cc v khch trong ngy th n. V mi


ngy c hai ngi no chuyn sang hai phng bn cnh nhng theo hai chiu ngc
nhau nn S(n) khng h thay i.
Vy S(n) = S(1) = 1 + 2 + 3 + + 19 = 190.
a) V c l ngi nn nu khng ai phng c th t chn th S(n) l tng ca 19 s l, tc
l S(n) l s l, mu thun. Vy trng hp ny khng xy ra.
b) Nu c 10 ngi phng cui (phng 19) th S(n) > 19.10 = 190, mu thun.Vy trng

231
Hi tho khoa hc, Hng Yn 25-26/02/2017

hp ny cng khng xy ra.

Nhn xt 8. Bt bin ca bi ton l tng cc s ghi trn th ca tt c cc v khch.

V d 9. Xt mt bng vung 4 x 4 . Ti mi ca bng vung c cha du + hoc du


- . Mi ln thc hin, cho php i du ca tt c cc trn cng mt hng hoc cng
mt ct. Gi s bng hnh vung ban u c 1 du + v 15 du -. Hi c th a bng
ban u v bng c ton du + c khng?
Li gii.
Thay tt c cc du + bng 1 v du - bng 1. Mi ln thc hin i du tt c cc
trn cng 1 hng hoc 1 ct, tc l i du ca 4 s nn tch ca 16 s trn bng l khng
thay i. Tch ca 16 s ban u l 1, sau mi ln bin i vn l 1.
Nu bng ton du + tc l tch ca 16 s trn bng l 1.
Nh vy, d c d c thc hin bao nhiu ln th t bng vung ban u khng th a v
bng vung ton du +.

Nhn xt 9. Bt bin trong bi ton l tch tt c cc s trn cc ca bng.

1 2 3 80
V d 10. Trn bng c cc s ; ; ;... .
80 80 80 80
Mi ln thc hin, cho php xa i hai s a, b bt k v thay bi a + b 2ab. Hi sau 1987 ln
thc hin php xa, s cn li trn bng l s no?
Li gii.
Gi s cc s trn l a1 , a2 , a3 , . . . , ak .
Xt tch P = (2a1 1)(2a2 1) . . . (2ak 1).
Khi , sau mi ln bin i, tch trn b mt i hai tha s (2a 1)(2b 1) v c thm
vo tha s 2( a + b 2ab) 1 = (2a 1)(2b 1)
Tc l sau mi ln bin i gi tr tuyt i ca tch P l khng thay i.
40 1
V tch ban u bng 0 (do bng ban u c cha s = ) nn sau mi ln bin i th
80 2
tch ny lun bng 0.
1
Vy s cn li cui cng trn bng l s tha mn 2s 1 = 0, hay s = .
2

Nhn xt 10. Bt bin trong bi ton l tch tt c cc gi tr ca hm s y = 2x 1ti cc s


trn bng.

V d 11. Cho a thc P( x ) = ax2 + bx + c, c th thc hin mt trong hai php bin i
a) i ch a v c
b) i bin x bi x + t vi t R.
Hi t x2 31x 3 c th thu c x2 20x 12 khng? Tm mi lin h ca hai a thc
P( x ) v Q( x ) sao cho t a thc ny c th thu c a thc kia bi hai php bin i ni

232
Hi tho khoa hc, Hng Yn 25-26/02/2017

trn.
Li gii.
Xt biu thc P( x + t) = a( x + t)2 + b( x + t) + c = ax2 + (2at + b) x + at2 + c c
= (2at + b)2 4a( at2 + c) = 4a2 t2 + 4abt + b2 4a2 t2 4ac = b2 4ac.
Xt biu thc P1 ( x ) = cx2 + bx + a c = b2 4ac.
Nh vy, c 2 php bin i trn khng lm thay i i lng ( bt bin vi hai php
bin i).
Xt P( x ) = x2 31x 3 c = 312 + 4.3 = 973
Q( x ) = x2 20x 12 c = 202 + 4.12 = 448
Vy t P( x ) ta khng th thu c Q( x ) thng qua hai php bin i trn.
Hai a thc P( x ) v Q( x ) m t a thc ny c th thu c a thc kia bi hai php bin
i ni trn khi chng c gi tr ca bit thc bng nhau.

Nhn xt 11. i lng bt bin l bit thc .

2.5 Bi tp tng t
Bi 6. Ti mi nh ca a gic li A1 A2 . . . A1993 ta ghi mt du + hoc mt du - sao
cho trong 1993 du c c du + v du -. Thc hin vic thay du nh sau, mi ln
thay du ng thi ti tt c cc nh ca a gic theo quy tc
- Nu du ti Ai v Ai+1 l nh nhau th du ti Ai c thay l du +
- Nu du ti Ai v Ai+1 l khc nhau th du ti Ai c thay l du +
(Quy c A1994 l A1 ).
Chng minh rng, tn ti s nguyn k sao cho khi thc hin lin tip k ln php thay du
ni trn, ta c a gic A1 A2 . . . A1993 m du ti mi nh Ai (i = 1, 1993) trng vi du
ca nh ngay sau ln thay du th nht.

Bi 7. Trn bng cho a thc f ( x ) = 3x2 + 4x + 3. Thc hin tr chi sau, nu trn
bng c a thc P( x ) th c php vit thm ln bng mt trong hai a thc sau
1 1
Q ( x ) = x 2 f ( + 1), R ( x ) = ( x 1)2 f ( ).
x x1
Hi sau mt s bc ta c th vit c a thc g( x ) = x2 + 10x + 9.

2.6 Bi ton t mu.


V d 12. Bn c vua 8 8 b mt hai hai gc i din. Hi c th lt phn cn li ca
bn c bi cc qun Domino 2 1 c khng?
Li gii.
Mi qun Domino lt vo bn c lun chim mt trng v mt en. Do , nu lt c
phn cn li ca bn c th s trng v s en bng nhau. Nhng do hai i din
ca bn c l hai cng mu nn s mu trng v s mu en trong phn cn li ca

233
Hi tho khoa hc, Hng Yn 25-26/02/2017

bn c khng bng nhau. Vy khng lt c phn cn li ca bn c bng cc qun Domino.

Nhn xt 12. T cc ca bn c an xen bng 2 mu, xt s lng cc ca tng mu.

V d 13. Cho mt bng vung kch thc kch thc 8 8 c in s nh sau

Cho php thc hin vic thay i cc s trong bng theo quy tc, mi ln ly tt c cc s
nm trong hnh vung kch thc 3 3 hoc 4 4 ri tng mi s ln 1 n v. Hi khng

234
Hi tho khoa hc, Hng Yn 25-26/02/2017

nh sau ng hay sai ? Vi mi cch in s ban u, nh vic thc hin lin tip php
thay s ni trn i vi bng s ban u ta s nhn c bng 8 8 m mi vung con
ca bng l mt s chia ht cho 3.
Li gii.
T mu bng nh hnh v.

Nhn xt 1. Bt k hnh vung con 3 3 no cng u cha ng 6 vung en hoc 9


vung en. Bt k hnh vung con 4 4 no cng u cha ng 12 en.
Do , sau mi ln thay s, ta khng lm thay i s d trong php chia cho 3 ca tng cc
s trong cc ca phn gch cho. Tng cc s trong cc phn bi en trn chia 3 d
2, nn tng cc s sau cc bc bin i cng chia 3 d 2, tc l khng th t c trng
thi m s trong mi vung con u chia ht cho 3.

V d 14. Hnh trn c chia thnh 2011 hnh d qut. Xp 2012 vin ko vo cc phn d
qut. Mi bc, cho php chuyn hai vin cng mt phn sang hai phn k khc hng.
Chng minh rng n lc no c t nht 1006 phn c cha ko.
Li gii.
Nhn xt 2. Qu trnh trn l khng dng li, v vic thc hin mi bc khng lm thay
i s vin ko ban u. C 2011 hnh d qut, 2012 vin ko, nn sau mi bc lun tn ti
mt hnh d qut c ln hn hoc bng 2 vin ko (nguyn l irichle), ni cch khc, sau
mi bc th lun c th thc hin c bc tip theo.
Nhn xt 3. n mt lc no , 2 phn cnh nhau bt k c ko (t nht l mt c ko).
Gi s iu ny khng ng, tc l tn ti hai hnh d qut k nhau khng bao gi c ko.
B 2 hnh d qut ny i, c th coi 2009 hnh d qut cn li l mt chui 2009 hnh ch
nht thng hng. Nh vy qu trnh ny dng li c. Vy iu gi s l sai. Tc l n
mt lc no 2 phn cnh nhau bt k th c ko.
Lc ny, chia 2011 thnh 1006 phn nh sau: 1 c ko, 1005 cp k nhau. Khi , mi
cp s c t nht 1 c ko, nh vy s c t nht 1006 c ko.
Nhn xt 4. 2 k nhau c ko sau bc nhy vn lun c ko. Xt 2 k nhau c ko, nu
ly ko t 2009 cn li thc hin php nhy th 2 ny vn lun c ko, nu ly ko t
1 trong 2 ny thc hin php nhy, v ly 2 vin ko v chuyn sang 2 phn bn n, nn
s chc chn s chuyn ko vo cn li. Vy trong mi tnh hung, 2 ny lun c ko.

235
Hi tho khoa hc, Hng Yn 25-26/02/2017

V d 15. in 29 s nguyn dng u tin vo cc vung con ca bng 6 5 nh sau


(bng 1 ).

Hnh 1: Bng 1

Cho php i v tr cc s trong bng theo quy tc,mi ln, ly 1 s nm k vi trng


ri chuyn s sang trng. Hi nh vic thc hin lin tip mt s hu hn ln php
chuyn s ni trn i vi bng ban u, ta c th nhn c bng s sau (bng 2) khng?

Hnh 2: Bng 2

Li gii.
Gi s nh php chuyn s theo quy tc ca bi, t bng 1 ta c th nhn c bng 2.
(1)
Ta coi trng ca mi bng l c in s 0.
Vi mi bng s nhn c trong qu trnh chuyn s, ta lit k tt c cc s trong bng
theo th t t tri qua phi, t trn xung di. Khi , ng vi mi bng s ta s c mt
hon v ca 30 s t nhin u tin. V do , t gi thit (1) cho thy, t hon v (1, 2, 3, 4,
5, 6, 7, 8, 9, 10, 11, 12, 0, 13, 14, 15, 16, 17, 18, 19, 20, 21, 22, 23, 24, 25, 26, 27, 28, 29 ) (gi l

236
Hi tho khoa hc, Hng Yn 25-26/02/2017

hon v I) ta c th nhn c hon v (29, 2, 3, 4, 5, 6, 7, 8, 9, 10, 11, 12, 0, 13, 14, 15, 16, 17,
18, 19, 20, 21, 22, 23, 24, 25, 26, 27, 28, 1 ) (gi l hon v 2) nh vic thc hin lin tip 1 s
hu hn ln php i ch cc s hng trong hon v theo quy tc, mi ln, ly mt s khc
0 ca hon v ri i v tr ca s hng v 0 cho nhau. (2)
Gi s ( a1 , a2 , . . . , a30 ) l mt hon v ca 30 s t nhin u tin. Ta gi cp s ( ai , a j ) l cp
s ngc ca hon v va nu nu ai > a j v i < j. D thy, sau mi ln thc hin php i
ch cc s hng theo quy tc (2) i vi hon v ( a1 , a2 , . . . , a30 ) th s cp s ngc ca hon
v s tng hoc gim mt s l n v. (3)
Ta c s cp s ngc ca hon v I l 12, s cp s ngc ca hon v II l 67. T kt hp
vi (3) suy ra t hon v I ta ch c th nhn c hon v II sau 1 s l ln thc hin php
i ch cc s hng. iu ny cho thy, nu t bng 1 ta nhn c bng 2 th s ln chuyn
s phi l s l. (4)
T mu tt c cc vung con ca bng 6 5 bi 2 mu xanh, sao cho 2 k nhau c 2
mu khc nhau. Th th, sau mi ln chuyn s, s 0 s c chuyn t c mu ny sang
c mu kia. V v th, do s 0 bng 1 v s 0 bng 2 nm hai c mu ging nhau nn
t bng 1 ch c th nhn c bng 2 sau mt s chn ln chuyn s. iu ny mu thun
vi (4). Vy t bng 1 ta khng th nhn c bng 2 nh php chuyn s theo quy tc ca
bi.

Nhn xt 13. Nh vic t mu cc vung con trong bng, ta tm c s ln chuyn s l


mt s chn. T cc suy lun v cp s ngc, ta tm c s ln chuyn s l mt s l. T
c c kt qu ca bi ton. Trong bi ton ny, pht hin v s dng cp s ngc c
vai tr quan trng. Cp s ngc cng c s dng trong bi ton sau

V d 16. cc v tr khc nhau ca mt ng ua t vng trn cng mt thi gian c


25 t xut pht theo cng mt hng. Theo th l cuc ua, cc t c th vt ln nhau,
nhng cm khng c vt ng thi hai xe cng lc. Cc t n ch l cc im m
chng xut pht ban u cng mt lc. Chng minh rng trong sut cuc ua c mt s
chn ln vt nhau ca cc t.
Li gii. Ta sn 1 trong s 25 t mu vng, cn cc t khc nh s th t l 1, 2, . . . , 24
theo th t m chng thi im ban u sau t mu vng (theo chiu chuyn ng ca
cc t). tm da ng ua ta s t mt ci bng ghi s th t ca cc t sp xp
sau t vng sau mi ln cc t vt nhau, tc l ta c mt hon v ca 1, 2, . . . , 24.
Trng hp 1: Mi ln 2 t trong cc t t 1 n 24 vt nhau th trn bng v c 2 s
lin nhau i ch cho nhau.
trng hp 2: Nu trc khi c ln vt ca mt t no vi t vng, cc s trn bng lp
thnh mt hon v a1 , a2 , . . . , a24 th sau ln vt s c hon v a2 , a3 , . . . , a24 , a1 . T
hon v trn c th chuyn xung hon v di bng 23 php chuyn v, tc l php i ch
2 s ng lin nhau.

237
Hi tho khoa hc, Hng Yn 25-26/02/2017

2.7 Bi tp tng t
Bi 8 (IMO - 2004). Ta nh ngha vin gch hnh mc cu l hnh gm 6 vung n v
nh hnh v di y, hoc hnh nhn c do lt hnh (sang tri, sang phi, ln trn,
xung di) hoc hnh nhn c do xoay hnh i mt gc.

Hy xc nh tt c cc hnh ch nht m n, trong m, n l cc s nguyn dng sao cho


c th lt hnh ch nht bng cc vin gch hnh mc cu.

Bi 9 (VMO - 1991). Cho bng 1991 1992. K hiu (m, n) l vung nm giao ca hng
th m v ct th n. T mu cc vung ca bng theo quy tc sau, ln th nht t 3
(r, s), (r + 1, s + 1), (r + 2, s + 2); 1 r 1989, 1 s 1990, t ln th hai, mi ln t ng
ba cha c mu nm cnh nhau trong cng mt hng hoc cng mt ct. Hi bng cch
no c th t mu c tt c cc ca bng c khng?

Bi 10 (VMO - 2006). Xt bng vung 4m n (m, n l cc s nguyn dng ln hn 3).


Thc hin tr chi sau, mi ln t 4 vin bi vo 4 ca bng (mi mt vin bi) m 4
to thnh mt trong nhng hnh di y.

Hi sau mt s ln ta c th nhn c bng m s bi trong cc bng nhau c khng


nu
a) m = 2004 v n = 2006?
b) m = 2005 v n = 2006 ?

238
Hi tho khoa hc, Hng Yn 25-26/02/2017

Bi 11. Cho n(n 2) hc sinh ng thnh hng dc. Sau mi ln c gio thi ci, c 2 em
i ch cho nhau. Hi sau mt s l ln thi ci, ta c th thy tt c cc em hc sinh u
ng v tr ban u ca mnh hay khng?

3 S dng i lng n bin gii bi ton t hp


V d 17. Xt bng vung m n(m, n 2). Trong mi ca bng ta in mt s thc.
Thc hin thut ton nh sau, mi ln ly ra mt hng hoc mt ct c tng cc s nh hn
0 v i du tt c cc s trong hng (hoc ct) . Chng minh rng sau hu hn bc ta
nhn c bng m tng cc s mi hng v tng cc s trong mi ct l s khng m.
Li gii.
Gi S(n) l tng tt c cc s trong bng sau bc th n. Ta c, S(n + 1) > S(n), n > 0. Do
, S(n) l mt hm n bin. Mt khc, s trng thi c th nhn c l hu hn nn ch
c th thc hin thut ton hu hn ln v ta nhn c bng tha mn yu cu bi ton.
V d 22.
Cho mt dy phng di v hn, c nh s 1, 2, 3, . . . C mt s hu hn ngi sng trong
dy phng. Mi ngy c hai ngi sng hai phng cnh nhau chuyn sang hai phng khc
nhau theo hai hng ngc nhau nhng khng c tro i v tr cho nhau. Chng minh
rng vic chuyn phng dng li sau hu hn ngy.
Li gii.
Ta a cho mi ngi mt cha kha, trn c ghi s phng ca mnh ang . Gi S(n) l
tch cc s vit trn cc cha kha ngy th n. Ta c

S ( n + 1) k ( k + 1)
= < 1, n 1
S(n) (k 1)(k + 2)

Trong k v k + 1 l cc phng c ngi chuyn.


Do , S(n) l mt n bin. Do S(n) gim v S(n) N nn vic chuyn phng phi dng
li sau hu hn ngy.

V d 18. T en 09 ca bn c 10 10. Mi ln t mu en mt cha t nu n k vi t


nht hai en (k c hiu l chung cnh). C th t mu ht bn c hay khng? Nu l
10 th sao? Nu l hnh vung n n th lc u cn t en t nht bao nhiu c th t
en c bn c.
Li gii. Chu vi nhng hnh en hoc khng gim, hoc gim 2, hoc gim 4. Tng chu
vi nhng hnh en ban u nh hn hoc bng 36. Chu vi bn c 10 10 l 40. Nh vy,
khng th t en c bn c.

V d 19 (VMO 2012). Cho s nguyn dng n. C n hc sinh nam v nnhc sinh n xp


thnh mt hng ngang, theo th t ty . Mi hc sinh (trong s 2n hc sinh va nu) c
cho mt s ko bng ng s cch chn ra hai hc sinh khc gii vi X v ng hai pha
ca X. Chng minh rng tng s ko m tt c 2n hc sinh nhn c khng vt qu
1 2
3 n ( n 1).

239
Hi tho khoa hc, Hng Yn 25-26/02/2017

Li gii. Gi cc bn nam ln lt l x1 , x2 , . . . , xn v cc bn n ln lt l y1 , y2 , . . . , yn .
Bng tnh ton trc tip ta thy rng nu cc bn nam n xp xen k x1 , y1 , x2 , y2 , . . . , xn , yn
1
th tng s ko ca 2n bn ng bng n(n2 1).
3
chng minh kt lun ca bi ton ta s chng t rng mt cch xp hng bt k u c
th chuyn dn v cch xp xen k nh trn m trong qu trnh chuyn tng s ko m cc
bn nhn c khng gim i. Gi s mt cch xp hng bt k c ng r bn nam v
ng r bn n cui c xp xen k (0 r < n). Khng mt tng qut, ta c th gi s
hng c mt trong hai dng sau

i) . . . , yr+1 , xr+k , . . . , xr+1 , xr , yr , xr1 , yr1 , . . . , x1 , y1


| {z } | {z }
k bn n lin tip k bn nam lin tip

ii) . . . , xr+1 , yr+k , . . . , yr+1 , xr , yr , xr1 , yr1 , . . . , x1 , y1 , khi k > 1.


| {z } | {z }
k bn n lin tip k bn nam lin tip

Trong trng hp i), ta chuyn bn yr+1 n v tr ngay trc bn xr . Khi ch c s ko


ca cc bn yr+1 , xr+k , . . . , xr+1 thay i. Bng tnh ton trc tip ta thy tng s ko tng
mt lng l (k2 k).
Trong trng hp ii), ta chuyn bn xr+1 n v tr ngay trc bn yr+1 . Khi ch c s ko
ca cc bn xr+1 , yr+k , . . . , yr+2 thay i. Ta tnh c tng s ko cng tng mt lng l
( k 2 k ).
Nh vy, sau khng qu n ln chuyn th hng c xp xen k nam, n. Do tng s ko
1
trong mt cch xp bt k lun nh hn hoc bng n(n2 1).
3

240
Hi tho khoa hc, Hng Yn 25-26/02/2017

MT S NG DNG CA S PHC TRONG I S


V TON T HP

ng Th Mn
THPT Chuyn Hng Yn

1 Dng lng gic ca s phc


Cho s phc z 6= 0. Gi M l im trong mt phng phc biu din s phc z.
Khi s o (raian) ca mi gc lng gic tia u Ox, tia cui OM c gi l mt
acgumen ca z.
Ch
+ Nu l acgumen ca z th mi acgumen ca z u c dng + k2, k Z.
+ Acgumen ca z 6= 0 xc nh sai khc k2, k Z.

Cho s phc z = a + bi ( a, b R), vi r = a2 + b2 l moun ca s phc z v l
acgumen ca s phc z. Dng z = r (cos + i sin ) c gi l dng lng gic ca s phc
z 6= 0, cn dng z = a + bi c gi l dng i s ca s phc z.
Nu z = r (cos + i sin ), z0 = r 0 (cos 0 + i sin 0 ) (r 0 v r 0 0) th
zz0 = rr 0 [cos( + 0 ) + i sin( + 0 )]
z r
= 0 cos( 0 ) + isin( 0 ) ( khi r 0 > 0).

z 0 r
[r (cos + i sin )]n = r n (cos n + i sin n), n N .
[cos + i sin ]n = cos n + i sin n, n N .
K hiu cos + i sin = ei , gi l ly tha ca e vi s m o.
Cho z = r (cos + i sin ), khi z cn c biu din di dng z = rei c gi l dng
m ca s phc z.
Cc php ton vit li
0 0 z r 0
z = rei ; z0 = r 0 ei suy ra z.z0 = r.r 0 .ei( + ) ; 0 = 0 .ei( ) (z0 6= 0).
z r
z = r.ei ; zn = r n ein .
ei + ei ei ei
Cng thc le (Euler): cos = ; sin = .
2 2i
Cho s phc z v s nguyn n 2, s phc w c gi l cn bn n ca z nu wn = z.
Nu z = r (cos + i sin ), r > 0 th cn bc n ca z gm n s phn bit xc nh bi


    
n + k2 + k2
wk = r cos + i sin ; k = 0; 1; . . . ; n 1.
n n

259
Hi tho khoa hc, Hng Yn 25-26/02/2017

+ Khi n = 2, c hai cn bc hai ca z l



        

r cos + i sin ; r cos + i sin = r cos + + i sin + .
2 2 2 2 2 2
+ Cn bc n ca n v
Cn bc n ca s phc z = 1 gi l cn bc n ca n v. T nh ngha ta c cc cn bc n
k2 k2
ca n v l wk = cos + i sin ; k = 0; 1; 2; . . . ; n 1.
n n
w l mt cn bc n ca n v v c gi l cn nguyn thy bc n ca n v nu mi s
nguyn dng m < n ta c wm 6= 1.
Tnh cht cn nguyn thy bc n ca n v
Nu w l mt cn nguyn thy bc n ca n v th 1 + wk + w2k + + wk(n1) = 0 vi
(k, n) = 1.
c bit, k = 1 ta c 1 + w + w2 + + wn1 = 0.

2 Cc bi ton v phng trnh, h phng trnh i s


Mt s phng trnh vi n phc f (z) = 0 v vi nghim z = x + yi ( x, y R), c th
gii bng cch tch phn thc v phn o, ta lun c th a v dng h phng trnh
(
h( x, y) = 0
g( x, y) = 0

Chng hn, tm cn bc ba ca s phc 1 + i, ta tm s phc z = x + yi sao cho z3 = 1 + i.


Bng cch tch phn thc v phn o trong ng thc ( x + yi )3 = 1 + i ta c h phng
trnh (
x3 3xy2 = 1
3x2 y y3 = 1
Gii h ny, ta tm c ( x; y), t ta s tm c z. Tuy nhin, r rng z c th tm c
bng cch tm cn bc ba ca 1 + i, c th l 

   
 6 k2 k2
1 + i = 2 cos + sin nn z = 2 cos + + i sin + ; k
4 4 12 3 12 3
{0; 1; 2}.
T , ngc li ta tm c nghim ca h phng trnh l
      
6 k2 6 k2
( x; y) 2 cos + ; 2 sin + ; k {0; 1; 2}
12 3 12 3
Nh th, mt s h phng trnh c th c "xut x" t cc phng trnh nghim phc.
Bng cch i ngc li qu trnh t phng trnh nghim phc v h phng trnh, t h
phng trnh cho ta thu c phng trnh nghim phc gc. Gii cc phng trnh
nghim phc ny, so snh phn thc v phn o, ta c nghim ca h phng trnh.

Ta xt v d sau

260
Hi tho khoa hc, Hng Yn 25-26/02/2017

V d 1. Gii cc h phng trnh sau


 
1
3x 1 + =2


x+y
a) p 

1
7y 1 =4 2


xy

3x y
x + 2 =3


x + y2
b) x + 3y
y 2 =0


x + y2
( p
4x y + 3 1 + y = 0
c) p
4 (1 + x )(1 + y) 6 1 + x + 1 = 0

Li gii.
a) iu kin x > 0; y > 0. t u = x; v = y (u > 0; v > 0).
 
1 2
u 1 + 2


2
=
u +v  3
H a v dng 
1 4 2
v 1 2 =


u +v 2
7
2 2
V u + v l bnh phng moun ca s phc z = u + iv, bng cch cng phng trnh
th nht vi phng trnh th hai (sau khi nhn vi i) ta c

u iv 2 4 2
u + iv + 2 = +i . (1)
u + v2 3 7

u iv z z 1
M 2 2
= 2 = = .
u +v z z.z z
Nn (1) c vit di dng
!
1 2 4 2 2 4 2
z + = + i z2 + i .z + 1 = 0.
z 3 7 3 7
!2 2


1 2 2 38 4 2 2
0 = + i 1 = + i = + i 2
3 7 21 7 21
!
1 2 2 2
z = +i 2 .
3 21 7
!
1 2 2 2
T suy ra (u, v) = + ; + 2 . Do , nghim ca h phng trnh
3 21 7
cho l

2 !2 !

1 2 2 2 11 + 4 7 22 + 8 7
( x; y) = + ; + 2 = ; .
3 21 7 21 7

261
Hi tho khoa hc, Hng Yn 25-26/02/2017

b) Nhn hai v ca phng trnh th hai vi i ri cng vi phng trnh th nht ta c

3x y xi 3yi 3( x yi ) i ( x yi )
x + yi + = 3 x + yi + = 3. (2)
x 2 + y2 x 2 + y2

Gi s z = x + yi th z = x yi; |z|2 = x2 + y2 . Khi (2) a v


3z iz 3i
z+ 2
= 3 z+ = 3 z2 3z + 3 i = 0
|z| z
3 + 1 + 2i 3 1 2i
= 3 + 4i = (1 + 2i )2 z1 = = 2 + i; z2 = = 1 i.
2 2
T suy ra nghim ca h ban u l ( x; y) {(2; 1); (1 1)}.
c) iu kin xc nh x 1; y 1.
(
p a2 + b2 + 3b 3 = 0
t a = 2 x + 1; b = 1 + y th h tr thnh
2ab 3a + 1 = 0
T h trn ta bin i v dng s phc nh sau

a2 b2 + 3b 3) + (2ab 3a + 1)i = 0
( a + bi )2 3i ( a + bi ) + i 3 = 0 z2 3iz + i 3 = 0. (3)
vi z = a + bi, z C.
Gii phng trnh (3) ta c nghim z = 1 + i hoc z = 1 + 2i.
Do a 0; b 0 nn 
a = 1; b= 1.
3
H c nghim ( x; y) = ; 0 .
4
Trn thc t, ta cng c th gii h trn bng cch dng bin i i s, nhn x v y thch
hp vo tng v ca cc phng trnh ri tr v vi v thu c quan h n gin hn gia
cc bin ny.

Mt s h sau cng c cch gii tng t



3x + 10y
x + 2 =1


x + y2
1. 10x 3y ( x, y R)
y + 2 =2


x + y2

x + 2y
x + 2 =2


x + y2
2. 2x y ( x, y R)
y + 2 = 0


x + y2

16x 11y
x + 2 =7


x + y2
3. 11x + 16y ( x, y R)

y = 1
x 2 + y2

262
Hi tho khoa hc, Hng Yn 25-26/02/2017


 
12
x 1 =2


3x + y 
4.  ( x, y R)
12
y 1+ =6


3x + y

 
3
10x 1 + =3


5x +y
5.  ( x, y R)
3
y 1 = 1


5x + y
(
x3 3xy2 = 1
6. ( x, y R).
3x2 y y3 = 3

3 Rt gn mt s tng t hp, chng minh cc ng thc t


hp
Gi w l mt cn nguyn thy bc n ca n v th ta c
1 + wk + w2k + + w(n1)k = 0; k m (k, n) = 1
Tnh cht trn c ng dng kh hiu qu trong vic rt gn cc tng t hp, ta xt v d sau
V d 2. Tnh tng S1 = Cn3k .
03k<n+1
n
Li gii. Xt a thc P( x ) = (1 + x )n = Cnk x k .
k =0
1 3
Gi w = + i l mt cn nguyn thy bc ba ca n v (c w2 + w + 1 = 0)th
2 2
w2k + wk + 1 bng 0 nu k khng chia ht cho 3, bng 3 nu k chia ht cho 3.
n
V th P(1) + P(w) + P(w2 ) = Cnk (1 + wk + w2k ) = 3 Cn3k .
k =0 03k<n+1
1
P(1) + P(w) + P(w2 ) m P(1) = (1 + 1)n = 2n .

Suy ra S1 =
3 !n !n 
1 3 1 3 n n n
P(w) = 1 + i = +i = cos + i sin = cos + i sin .
2 2 2 2 3 3 3 3
!2 n !n  n
1 3 1 3   
P ( w2 ) = 1 + + i = i = cos + i sin =
2 2 2 2 3 3
n n
cos + i sin .
3 3
1 n
 n 
Nn S1 = 2 + 2 cos .
3 3
Cng thc Euler ei = cos + i sin c th a cc tng lng gic thnh cc cp s nhn
hoc cng thc Nh thc Newton, c th xt v d sau
V d 3.
n
a) Tnh tng S2 = Cnk cos kx.
k =0

263
Hi tho khoa hc, Hng Yn 25-26/02/2017

m 1
b) Chng minh rng 22m1 cos2m x = C2mk cos(2m 2k ) x + 1 C m .
k =0 2 2m
Li gii.
n
a) Xt T2 = Cnk sin kx ta c
k =0

n n
S2 + iT2 = Cnk (cos kx + i sin kx ) = Cnk eikx
k =0 k =0
= (1 + e ) = (1 + cos x + i sin x )n
ix n
 x n  x x n
= 2 cos cos + i sin
2  2 2
n n x nx nx 
S2 + iT2 = 2 cos cos + i sin
2 2 2
x nx
So snh phn thc, phn o ta c S2 = 2n cosn cos .
2 2
eix + eix
b) Ta c eix = cos x + i sin x; eix = cos x i sin x suy ra cos x = . Do
2

22m cos2m x = (2 cos x )2m = (eix + eix )2m


2m
= C2m
k
(eix )k (eix )2mk
k =0
2m
= C2m
k 2(km)ix
e
k =0
m 1 2m
= k 2(km)ix
C2m e + k 2(km)ix
C2m e m
+ C2m
k =0 k = m +1
m 1 m 1
= k 2(mk)ix
C2m e + 2mt 2(mt)ix
C2m e m
+ C2m
k =0 t =0

m 1 m 1
= k 2(mk)ix
C2m e + 2mk 2(mk)ix
C2m e m
+ C2m
k =0 k =0
m 1
= k
C2m (e2(mk)ix + e2(mk)ix ) + C2m
m

k =0
m 1
=2 k
C2m m
cos(2m 2k ) x + C2m .
k =0

m 1
Suy ra 22m1 cos2m x = C2mk cos(2m 2k ) x + 1 C m .
k =0 2 2m

264
Hi tho khoa hc, Hng Yn 25-26/02/2017

Vi cch lm tng t nh trn, ta cng chng minh c ng thc


n
2 2n 2n+1
cos x= C2n
k
+1 cos(2n + 1 2k ) x
k =0

S dng cng thc 2i sin x = eix eix v


 bin i tng t nh trn, ta chng minh
 c
n 1 n
cc ng thc sau 22n1 sin2n x = (1)n (1)k C2n k cos(2n 2k ) x + (1) C n
2 2n .
k =0
n
22n sin2n+1 x = (1)n (1)k C2n
k
+1 sin(2n + 1 2k ) x.
k =0

Bi tp tng t
Bi 1. Tnh cc tng sau
S3 = (1)k Cn2k ; S4 = (1)k Cn2k+1
02k<n+1 02k+1<n+1
S5 = Cn4k ; S6 = Cn4k+1 .
04k<n+1 0<4k+1<n+1

Bi 2. Chng minh ng thc sau



a) (1)k (2k + 1)Cn2k+1 = n( 2)n1 cos(n 1) .
0<2k+1<n+1 4

b) (1)k 2kCn2k = n( 2)n3 sin(n 1) .
0<2k<n+1 4

4 Cc bi ton m
S phc c nhng ng dng rt hiu qu trong cc bi ton m v vai tr trung tm
trong k thut ng dng s phc vo cc bi ton m tip tc li l cn nguyn thy ca
n v. Vi tnh cht w l mt cn nguyn thy bc n ca n v th ta c

1 + w + w2 + + wn1 = 0,

1 + wk + w2k + + wk(n1) = 0 vi (k, n) = 1.

V d 4. Tm s tt c cc s c n ch s lp t cc ch s 3, 4, 5, 6 v chia ht cho 3.

Li gii. Gi Cn l s cc s c n ch s tha mn bi. Gi l mt nghim ca phng


trnh z2 + z + 1 = 0.
Khi 3 = 1 v 2k + k + 1 = 0 nu k khng chia ht cho 3 v 2k + k + 1 == 3 nu k
chia ht cho 3.
Xt a thc P( x ) = ( x3 + x4 + x5 + x6 )n d thy Cn chnh bng tng cc h s ca cc s m
6n 2n
chia ht cho 3 trong khai trin ca P( x ). Ni cch khc, nu P( x ) = ak x k th Cn = a3k .
k =0 k =0

265
Hi tho khoa hc, Hng Yn 25-26/02/2017

6n 2n
M P(1) + P() + P(2 ) = ak (1 + k + 2k ) = 3a3k
k =0 k =0
Do P(1) = (1 + 1 + 1 + 1)n = 4n
P ( ) = ( 3 + 4 + 5 + 6 ) n = (1 + + 2 + 3 ) n = (1 + + 2 + 1) n = 1
P(2 ) = (6 + 8 + 10 + 12 )n = (1 + 2 + + 1)n = 1n = 1
nn P(1) + P() + P(2 ) = 4n + 2,
2n 1 2 4n + 2
suy ra Cn = a3k =

P (1) + P ( ) + P ( ) = .
k =0 3 3

V d 5. (IMO1995) Cho p l mt s nguyn t l, tm s cc tp con A ca tp {1; 2; 3; . . . ; 2p}


bit rng
a) A cha ng p phn t.
b) Tng cc phn t ca A chia ht cho p.

Li gii. Xt a thc P( x ) = x p1 + x p2 + + x + 1. a thc ny c ( p 1) nghim phc


phn bit.
Gi l mt nghim bt k ca P( x ). Ch rng , 2 , . . . , p1 l p 1 nghim phn bit
ca P( x ) v p = 1.
Theo nh l Viet c ( x )( x 2 ) . . . ( x p1 ) = x p1 + x p2 + + x + 1.
Xt a thc Q( x ) = ( x )( x 2 ) . . . ( x 2p ).
Gi H = { A {1; 2; . . . ; 2p} : | A| = p}.
2p
Gi s Q( x ) = ak x k khi a p = S( A) vi S( A) = x.
k =0 A H x A
p 1
Nu S( A) j (mod p) th S( A) = j nn a p = n j j , trong n j l s cc A H sao
j =0
cho S( A) j (mod p).
Mt khc Q( x ) = ( x p 1)2 ta c a p = 2 nn

p 1
n j a j = 2. ()
j =0

p 1
Xt a thc R( x ) = n j x j + n0 2. Do (*) nn l mt nghim ca R( x ) m deg P( x ) =
j =0
deg R( x ) v l mt nghim bt k ca P( x ), nn P( x ) v R( x ) ch sai khc nhau hng s
nhn. T n p1 = n p2 = = n1 = n0 2.
p p
n p 1 + n p 2 + + n 1 + n 0 2 C2p 2 C2p 2
Suy ra n0 2 = = , do n0 = 2 + . S
p p 2
p
C2p 2
cc tp con A ca tp hp {1; 2; 3; . . . ; 2p} tha mn bi l n0 = 2 + .
2

266
Hi tho khoa hc, Hng Yn 25-26/02/2017

5 Cc bi ton v a thc
5.1 Xc nh a thc
Nghim ca a thc ng vai tr quan trng trong vic xc nh mt a thc. C th,
nu a thc P( x ) bc n c n nghim x1 , x2 , . . . , xn th P( x ) c dng P( x ) = c( x x1 )( x
x2 ) . . . ( x xn ), c l hng s khc 0.
Tuy nhhin, nu ch xt cc nghim thc ca a thc th trong nhiu trng hp s khng
s nghim, hn na trong cc bi ton phng trnh hm a thc, nu ch xt cc nghim
thc th li gii s khng hon chnh. nh l c bn ca i s v vy ng mt vai tr ht
sc quan trng trong dng ton ny l: Mt a thc vi h s phc (bao gm c s thc)
lun c t nht mt nghim phc (bao gm c nghim thc).

V d 6. Xc nh tt c cc a thc P( x ) khc a thc hng sao cho

P ( x ) P ( x + 1) = P ( x 2 + x + 1); x R (1)

Li gii. Gi s x0 l nghim ca P( x ) = 0 th P( x02 + x0 + 1) = 0. Khi x02 + x0 + 1 cng


l nghim ca P( x ). Thay x bi x 1 trong (1) ta c P( x 1) P( x ) = P( x2 x + 1). V
P( x0 ) = 0 nn x02 x0 + 1 cng l nghim ca P( x ).
Chn l nghim c moun ln nht (nu tn ti vi nghim vi moun ln nht, ta
chn mt trong s cc nghim ).
T cch chn suy ra |2 + + 1| || v |2 + 1| || v c 2 + + 1 v c 2 + 1
u l nghim ca P( x ).
Ta c 6= 0 v 2|| = |(2 + + 1) (2 + 1)| |2 + + 1| + |2 + 1|
| | + | | = 2| |.
Vy phi xy ra du ng thc nn 2 + + 1 = k (2 + 1) vi k l hng s dng.
M || l ln nht nn |2 + + 1| = |2 + 1| = ||
nn k = 1 suy ra 2 + + 1 = (2 + 1) v 2 + 1 = 0 = i nn x2 + 1 l tha s
ca P( x ).
Nh vy ta c th vit P( x ) = ( x2 + 1)m Q( x ); m N . Trong Q( x ) l a thc khng
chia ht cho x2 + 1. Th ngc tr li vo (1) ta thy Q( x ) tha mn phng trnh

Q ( x ) Q ( x + 1) = Q ( x 2 + x + 1); x R (2)

Nu phng trnh Q( x ) = 0 li c nghim th lp lun nh trn ta suy ra nghim c moun


ln nht ca n phi l i. iu ny khng th xy ra v x2 + 1 khng chia ht Q( x ).
Q( x ) l mt hng s, gi s Q( x ) = c; x R, thay vo (2) ta c c = 1. Vy cc a thc
tha mn bi l P( x ) = ( x2 + 1)m , n N .

V d 7. Tm tt c cc a thc P( x ) tha mn phng trnh P( x ) P( x + 1) = P( x2 ), x R.

Li gii. Gi s l nghim ca P( x ) = 0. Khi t phng trnh suy ra 2 , 4 , 8 , . . . cng


l nghim ca P( x ) = 0. T y suy ra || = 0 hoc || = 1, v nu ngc li ta s thu c

267
Hi tho khoa hc, Hng Yn 25-26/02/2017

dy v hn cc nghim phn bit ca P( x ). Tng t, 1 l nghim ca P( x ) v lp lun


tng t, ta cng c | 1| = 0 hoc | 1| = 1.
Gi s rng || = 1 v | 1| = 1. Ta vit = cos + i sin ; [0; 2 ], t y suy ra
1 5
cos = hay = hoc =
2 3 3

Gi s = , xt cng l nghim ca P( x ), nh vy 2 1 cng l nghim ca P( x ) v
2
3 2
2 2
|2 1| = cos 1 + sin2 = 3 mu thun v mi nghim ca P( x ) u c moun
3 3
bng 0 hoc 1.
5
Tng t trn vi trng hp = . Nh vy c th kt lun = 1 hoc 1 = 1. T y
3
P( x ) c dng P( x ) = cx m (1 x )n , c l hng s v m, n N, thay vo phng trnh cho ta
d dng kim tra c c = 1 v m = n. Vy cc a thc tha mn l P( x ) = x m (1 x )m , m
N.

5.2 Bi ton v s chia ht ca a thc


Ta bit rng, nu a thc P( x ) chia ht cho a thc Q( x ) th mi nghim ca Q( x ) u
l nghim ca P( x ). Tnh cht n gin ny l cha kha gii nghim bi ton v s chia
ht ca a thc.

V d 8. Vi gi tr no ca nth x2n + x n + 1 chia ht cho a thc x2 + x + 1


   
1 3 2 2
Li gii. Ta c w = i = cos + i sin l nghim ca Q( x ) = x2 +
2 2 3 3
x + 1 = 0.
a thc P( x ) = x2n + x n + 1 chia ht cho Q( x ) khi v ch khi P(w) = 0, iu ny tng
ng vi
       
4n 4n 2n 2n
cos + i sin + cos + i sin +1 = 0
3 3 3 3

cos 4n + cos 2n + 1 = 0

3 3
4n 2n
sin
+ sin =0
3 3
2n
2 cos +1 = 0
3
2n 1 2
cos = = cos
3 2 3
n = 3k + 1
suy ra ( k Z)
n = 3k + 2

Vy vi n = 3k + 1 hoc n = 3k + 2; (k Z) th P( x ) chia ht cho Q( x ).

V d di y, mt ln na, cn ca n v li ng vai tr then cht.

268
Hi tho khoa hc, Hng Yn 25-26/02/2017

V d 9. (USAMO 1976) Cho P( x ), Q( x ), R( x ), S( x ) l cc a thc sao cho

P( x5 ) + xQ( x5 ) + x2 R( x7 ) = ( x4 + x3 + x2 + x + 1).S( x ) (1)

Chng minh rng P( x ) chia ht cho x 1.


2i
Li gii. t w = e 5 th w5 = 1 v 1 + w + w2 + w3 + w4 = 0 (*)
Thay x ln lt bi w, w2 , w3 , w4 vo (1) ta c phng trnh

P(1) + wQ(1) + w2 R(1) = 0 P(1) + wQ(1) + w2 R(1) = 0, (2)

P(1) + w2 Q(1) + w4 R(1) = 0 P(1) + w2 Q(1) + w4 R(1) = 0, (3)


P(1) + w3 Q(1) + w6 R(1) = 0 P(1) + w3 Q(1) + wR(1) = 0, (4)
P(1) + w4 Q(1) + w8 R(1) = 0 P(1) + w4 Q(1) + w3 R(1) = 0. (5)
Nhn cc phng trnh t (2) n (5) ln lt vi w; w2 ; w3 ; w4 ta c
wP(1) w2 Q(1) w3 R(1) = 0
w2 P(1) w4 Q(1) wR(1) = 0
w3 P(1) wQ(1) w4 R(1) = 0
w4 P (1) w3 Q (1) w2 R (1) = 0
Cng v vi v cc ng thc trn v p dng (*) ta c P(1) + Q(1) + R(1) = 0 (6). Cng
v vi v ca (2), (3), (4), (5), (6) suy ra 5P(1) = 0 suy ra P( x ) chia ht cho x 1.

Bi tp tng t
Bi 3. Tm tt c cc a thc P( x ) tha mn phng trnh ( P( x ))2 P( x2 ) = 2x4 ; x R.

Bi 4. Tm tt c cc a thc P( x ) tha mn phng trnh P( x2 2) = ( P( x ))2 2; x R.

Bi 5 (VMO 2006). Tm tt c cc a thc P( x ) vi h s thc tha mn phng trnh

P( x2 ) + x (3P( x ) + P( x )) = ( P( x ))2 + 2x2 ; x R.

Bi 6. Tm tt c cc a thc P( x ) vi h s thc tha mn phng trnh 2x2 P( x ) = P(2x3 +


x ); x R.

Ti liu
[1] B Gio dc v o to, Gii tch 12 Nng cao (Ti bn ln th t), NXBGD - 2012.

[2] on Qunh, Trn Nam Dng, Nguyn V Lng, ng Hng Thng, Ti liu chuyn
ton - i s v Gii tch 11, NXBGD - 2010.

269
Hi tho khoa hc, Hng Yn 25-26/02/2017

[3] Nguyn Vn Mu, Trn Nam Dng, Nguyn ng Pht, Nguyn Thy Thanh, Chuyn
chn lc - S phc v p dng, NXBGD - 2009.

[4] B Gio dc v o to, Tp ch Ton hc v tui tr, NXBGD

270
Hi tho khoa hc, Hng Yn 25-26/02/2017

S DNG BT NG THC KARAMATA KHO ST


BT NG THC TRONG TAM GIC

V Vn Thng
THPT Yn Dng s 3, Bc Giang

Tm tt ni dung

Bo co gii thiu B tri v ng dng ca B tri trong chng minh cc bt


ng thc trong tam gic.

1 M u
Khi nim tri c a ra so snh hai phn t (hai vect) trong khng gian Rn .
Khi nim ny l c s ca l thuyt tri, c p dng rng ri trong nhiu lnh vc,
xem, th d, [9].
Khi nim tri c p dng kh thnh cng trong chng minh cc bt ng thc,
c bit l bt ng thc trong tam gic, xem [8], [9], [10]. C th ni, bt ng thc
Karamata (xem, th d, [1], [3], [5], [6], [7]) cng l bt ng thc tri. Khi nim tri
cng kh gn vi mt s tng v sp th t tam gic, xem, th d, [2].
Bt ng thc Karamata c cp n trong [1], [3], [5], [6], v c khai
thc trong [7] chng minh cc bt ng thc lng gic v hnh hc. Bt ng thc
tri c vn dng trong [8] chng minh cc bt ng thc trong tam gic trong
[8].
Bi vit S dng B tri chng minh cc bt ng thc trong tam gic c mc ch
minh ha kh nng s dng khi nim tri v bt ng thc tri (B tri) trong chng
minh, ci tin v lm mi cc bt ng thc trong tam gic. Theo chng ti, y l mt
k thut cn cha c quan tm nhiu nhng c ngha khoa hc v thc tin, c th
khai thc s dng trong ging dy ton s cp.
Bi vit gm phn M u (Mc 1), Kt lun v hai Mc chnh.
Mc 2 trnh by cc khi nim c bn: khi nim tri, hm li Shur, bt ng thc
tri.
Mc 3 trnh by ng dng ca b tri v h qu ca n trong chng minh cc
bt ng thc trong tam gic. Qua y c th thy c th mnh ca bt ng thc
tri trong chng minh nhiu bi ton v bt ng thc trong tam gic. Ngoi ra, Mc 3
cng nhc n ng dng ca bt ng thc tri trong chng minh cc bt ng thc ni
chung v so snh vi cc phng php chng minh bt ng thc khc.

270
Hi tho khoa hc, Hng Yn 25-26/02/2017

2 B tri
nh ngha 1. Cho a = ( a1 , . . . , an ) v b = (b1 , . . . , bn ) l hai vect trong khng gian
Rn . Cc ta ai v bi , i = 1, 2, . . . , n c sp th t nh sau: a1 a2 an ,
b1 b2 bn .
ni a tri hn b (a majorizes b) v vit a  b nu
Ta

a1 b1 ;
a1 + a2 b1 + b2 ;



.... Ta cng ni b b tri bi a
a + a + + a b + b + + b ;

1 2 n 1 1 2 n 1



a1 + a2 + + an1 + an = b1 + b2 + + bn1 + bn .

(b majorized by a) v vit b a.
1
Nhn xt 1. Gi a, b, c (a b c) l ba cnh ca tam gic ABC, s = ( a + b + c) l na
2
2s 1
chu vi, a = = ( a + b + c) l trung bnh cng ca ba cnh.
3 3
t s1 = a + b + c ; s2 = a b + c ; s3 = a + b c. Khi y:
1) ( a , a , a ) ( a, b, c) 
(s, s, 0) vi mi tam gic.
s s
2) ( a , a , a ) ( a, b, c) s, , vi mi tam gic cn (b = c).
2 2
Chng minh.
1 a+a+a 2a + 2b + 2c 2a + 2b + c + c
1) Ta c: a = ( a + b + c) = a; a + a = =
3 3 3 3
2a + 2b + a + b
= a + b; a + a + a = 3.a = a + b + c. Do : ( a , a , a ) ( a, b, c) .
3
a+b+c
Ta li c: a < b + c a + a < a + b + c 2a < a + b + c a < = s;
2
0 < c a + b < a + b + c = 2s = s + s;
a+b+c a+b+c
a+b+c = + = s + s + 0.
2 2
Do ( a, b, c) (s, s, 0) . Vy ( a , a , a ) ( a, b, c) (s, s, 0) .
2) Vi tam gic cn (b = c) ta c:
a < b + c a < 2c a + b < 2c + b a + b < 3c
4a 3a + 4b 3b < 3c 4a + 4b < 3a + 3b + 3c
3 3s s
a + b < ( a + b + c) a + b < a+b < s+ ;
4 2 2
a+b+c a+b+c a+b+c s s
a+b+c = + + = s+ + .
2 4 4 2 2
s s
Do ( a, b, c) s, , .
2 2  s s
V ( a , a , a ) ( a, b, c) nn ( a , a , a ) ( a, b, c) s, , .
2 2
Nhn xt 2. Cho 0 < 1 , 2 , 3 < ln lt l ba gc trong mt tam gic. Khi :

1) , , (1 , 2 , 3 ) (, 0, 0) vi mi tam gic.
 3 3 3
 
2) , , ( 1 , 2 , 3 ) , , 0 vi mi tam gic nhn.
 3 3 3 2 2

3) , , (1 , 2 , 3 ) (, 0, 0) vi mi tam gic khng nhn.
2 4 4
Chng minh.

1) Gi s 1 2 3 . Khi y = 1 + 2 + 3 1 + 1 + 1 = 31 , suy ra 1
3
;

271
Hi tho khoa hc, Hng Yn 25-26/02/2017

2 = 2 (1 + 2 + 3 ) 21 + 22 + 2 + 1 31 + 32 , suy ra
2
1 + 2 = + ; 1 + 2 + 3 = + + = .
 3 3 3 3 3 3
Do , , ( 1 , 2 , 3 ).
3 3 3
Ta cng c: 1 < ; 1 + 2 < + 0; 1 +2 + 3 = + 0 + 0.

Suy ra (1 , 2 , 3 ) (, 0, 0). Vy , , (1 , 2 , 3 ) (, 0, 0) . 2) Vi mi
3 3 3

tam gic c ba gc nhn, > 1 2 3 , ta c
2

1 < ; 1 + 2 < + ; 1 + 2 + 3 = + + 0 = .
2 2 2  2 2
Suy ra (1 , 2 , 3 ) , ,0 .
  2 2  
Vy , , ( 1 , 2 , 3 ) , , 0 cho tam gic nhn.
3 3 3 2 2

3) Vi mi tam gic khng nhn c 1 > 2 3 , ta c:
2

+ 1 + (1 + 2 + 3 ) = 21 + 2 + 3 21 + 2 + 2 = 2 (1 + 2 ) .
2

Suy ra: + 1 + 2 ; 1 + 2 + 3 = + + = ;
 2 4 2 4 4
Vy , , (1 , 2 , 3 ) (, 0, 0)vi mi tam gic khng nhn.
2 4 4
nh ngha 2. Tp X Rn c gi l tp li nu vi mi [0; 1] v x1 X, x2 X
ta c x = x1 + (1 ) x2 X.
Ngha l, tp li X cha mi on thng ni hai im ca n.

nh ngha 3. Hm f : X Rn R c gi l hm li nu X l tp li v vi mi
[0; 1] v x1 X, x2 X ta c
f ( x ) = f (x1 + (1 ) x2 ) f ( x1 ) + (1 ) f ( x2 ) .
Hm f c gi l hm lm nu f l hm li, hay ta c bt ng thc ngc li.
nh l 2.1 Cho hm s y = f ( x ) xc nh trn tp X v c o hm cp hai ti mi
x X.
Nu f 00 ( x ) 0 vi mi x X th f ( x ) l hm li trn X.
Nu f 00 ( x ) 0 vi mi x X th f ( x ) l hm lm trn X.
S dng tiu chun ny, ta c th chng minh tnh li (lm) ca hng lot cc hm s
sau y.

Nhn xt 3. Hm s y = sin x, y = sinx, y =ln sin x l nhng hm lm trn khong
x
(0; ) ; Hm s y = sin2 l hm li trn 0; ; Hm s y = sin kx (k > 0) l hm lm
h i 2 2
trn on 0; .
k
x  
Nhn xt 4. Cc hm s y = cos x, y = cos2 l cc hm lm trn khong 0; ; Hm
2 2
  x
s y = ln cos kx (k > 0) l hm lm trn khong 0; ; Hm s y = cos l hm lm
2k 2
trn khong(0; ) .
 x m
Nhn xt 5. Cc hm s y = (tan x )m v y = tan , m 1 l cc hm li trn khong
  2
0; ;
2
x  
Hm s y = f ( x ) = ln tan l hm lm trn khong 0; .
2 2

272
Hi tho khoa hc, Hng Yn 25-26/02/2017

nh ngha 4. Hm F : X Rn R c gi l hm li Shur (Shur-convex function)


nu x y trn X suy ra F ( x ) F (y) .
Mt bt ng thc cho hm li c s dng hiu qu trong chng minh cc bt
ng thc l Bt ng thc tri di y.
Bt ng thc tri (B tri, Shur, 1923)
K hiu [ a, b]n := { x = ( x1 , . . . , xn ) : xi [ a, b]} , trong [ a; b] R.
n
Nu f : [ a; b] R R l hm li th F : [ a; b]n R, vi F ( x ) := f ( xi ) l hm li
i =1
n n
n
Shur, tc l vi mi x, y [ a, b] , x y ta c f ( xi ) f (yi ), hay
i =1 i =1
f ( x1 ) + f ( x2 ) + + f ( x n ) f ( y1 ) + f ( y2 ) + + f ( y n ) . (1)
Nu f : [ a; b] R l hm lm th ta c bt ng thc ngc li, tc l vi mi
x, y [ a, b]n , x y ta c
f ( x1 ) + f ( x2 ) + + f ( x n ) f ( y1 ) + f ( y2 ) + + f ( y n ) . (2)
Du bng xy ra khi v ch khi xi = yi , i = 1, n.
Chng minh xem, v d, [8], [10].
Nhn xt 6. Nhiu ti liu (th d, [1], [3], [5], [6], [7]) gi bt ng thc (1) v (2) l bt
ng thc Karamata (Karamata Inequality). B tri c Shur chng minh nm 1923
v Karamata chng minh nm 1932. B tri cn c rt nhiu ng dng khc, khng
ch ton s cp (xem [10]), Do , chng ti gi (theo [10]), cc bt ng thc (1) v (2)
l bt ng thc Shur hay bt ng thc tri, mc d trong cc ti liu, t bt ng thc
Shur thng c dng cho mt bt ng thc hon ton khc.
H qu 1 (Bt ng thc Jensen). Vi mi hm li f ( x ) trn ( a; b) v vi mi xi ( a, b)
(i = 1, 2, . . . , n), ta lun c
f ( x1 ) + f ( x2 ) + + f ( x n ) x1 + x2 + + x n
 
f .
n n
Du bng xy ra khi v ch khi x1 = x2 = = xn .
H qu 2 (Bt ng thc Popoviciu). Vi mi hm li trn ( a; b) v vi mi x, y, z ( a; b) ,
ta lun c        
x+y+x x+y y+z z+x
f ( x ) + f (y) + f (z) + 3 f 2f + 2f + 2f .
3 2 2 2
Du bng xy ra khi v ch khi x = y = z.
H qu 3 (Bt ng thc Vasile Cirtoaje). Vi mi hm li f ( x ) trn ( a; b)
v a1 , a2 , . . . , an ( a; b), ta lun c f ( a1 ) + f ( a2 ) + + f ( an ) + n(n
a1 + a2 + + a n
2) f (n 1) ( f (b1 ) + f (b2 ) + + f (bn )) , trong bi =
n
1
a j , i = 1, 2, . . . , n.
n 1 j 6 =i
Du bng xy ra khi v ch khi a1 = a2 = = an .

3 p dng bt ng thc tri trong chng minh bt


ng thc
Khi nim tri, thm ch ch ring bt ng thc tri, c bit c li trong nh gi
(bt ng thc v tm cc tr) cc i lng. Di y l cc v d minh ha.

273
Hi tho khoa hc, Hng Yn 25-26/02/2017

3.1 V d minh ha
Bi ton 1 ([9). , 1996 Asian Pacific Mathematical Competition; Olympic chu
Thi Bnh
Dng
ln th 8, 1996]Cho a, b, cl ba cnh ca mt tam gic. Khi y a + b c +

b + c a + a + c b a + b + c. (3.1)
Chng minh. Khng hn ch tng qut, coi a b c > 0. Khi a + b c a + c b
b +c a v
a + b c a;
( a + b c) + ( a + c b) = 2a a + b;
( a + b c) + ( a + c b) + (b + c a) = a + b + c.

Do ( a + b c, a + c b, b + c a)  ( a, b, c) .
p
dng bt ng thc tri(2) cho hm lm y = x, ta c

a + b c + b + c a + a + c b a + b + c. Vy bt ng thc (3.1) c
chng minh.
Nhn xt 7. K hiu a = y + z, b = x + y, c = x + z. V mt hnh hc x, y, z chnh l
di ng tip tuyn ti ng trn ni tip tam gic k t cc nh A, B, C. t
a1 = a, a2 = b, a3 = c; b1 = z + y, b2 = y + x, b3 = z + x. Khi :
a1 = b1 ; a1 + a2 = b1 + b2 ; a1 + a2 + a3 = b1 + b2 + b3 .
p dng bt ng thc tri (1) ta c: f ( a, b, c) 0 f (y + z, y + x, x + z) 0.
a+b+c a+b+c
K hiu s = l na chu vi tam gic. Khi y ta c: s a = a =
2 2
b+ca (y + z) + (z + x ) + ( x + y)
= = x.
2 2
Tng t, s b = z; s c = y.
Khng hn ch tng qut, coi x y z. t c1 = x; c2 = y; c3 = z v d1 = s a =
x; d2 = s c = y; d3 = s b = z, ta c:
c1 d1 ; c1 + c2 d1 + d2 ;
c1 + c2 + c3 = x + y + z = d1 + d2 + d3 .
p dng bt ng thc tri (1) ta c:
f ( x, y, z) 0 f ((s a) , (s c) , (s b)) 0.
Nh vy, php bin i a = y + z, b = x + z, c = x + y cho mt tng ng gia
ba cnh a, b, c ca tam gic v ba s dng x, y, z. iu ny cho php to cc ng thc
v bt ng thc mi trong tam gic t cc cc ng thc v bt ng thc gia ba s
dng.
Bt ng thc tri c th c p dng chng minh cc bt ng thc hoc gii
cc bi ton cc tr. Di y l mt v d.
Bi ton 2 (Thi kt thc hc phn cao hc, chuyn bt ng thc, i hc Nng).
Choa, b, c l ba s thc tha mn iu kin
0 c b a 8;
a + b 13;
a + b + c = 15.

Tm gi tr ln nht v gi tr nh nht ca M = a2 + b2 + c2 . Chng minh. T gi
bi ton, ta c: a b c, 8 > 5 > 2 v
thit
0 a 8;
a + b 8 + 5;
a + b + c = 8 + 5 + 2.

Do ta c (8, 5, 2)  ( a, b, c) .
p dng bt ng thc tri (1) cho hm li y = x2 , ta c:

274
Hi tho khoa hc, Hng Yn 25-26/02/2017

f ( a ) + f ( b ) + f ( c ) f (8) + f (5) + f (2)


a2 + b2 + c2 82 + 52 + 22 = 93 M 93.
Vy max M = 93t c khi a = 8, b = 5, c = 2.
p dng bt ng thc Bunhiacopxki cho hai b s ( a, b, c) v (1,1,1) ta c:
(1.a + 1.b + 1.c)2 12 + 12 + 12 a2 + b2 + c2
 

152 3. a2 + b2 + c2


a2 + b2 + c2 75.
Vy min M = 75 t c khi a = b = c = 5.

3.2 S dng bt ng thc tri chng minh cc bt ng thc


trong tam gic
Nh bt ng thc tri v H qu ca n (bt ng thc Jensen, Popoviciu, Vasile
Cirtoaje,. . . ), kt hp cc Nhn xt 2.1, 2.2 vi tnh cht ca hm li (cc nhn xt 2.3, 2.4,
2.5,. . . ), ta c th d dng chng minh cc bt ng thc di y. Chng minh chi tit
v so snh tnh u vit ca k thut s dng bt ng thc tri so vi cc phng php
khc (bin i i s v lng gic, s dng cc nh gi bt ng thc, phng php
tam thc bc hai, phng php o hm, phng php vect,. . . ) c th xem trong [8].

Bi ton 3. Tam gic ABC bt k c cc cnh, na chu vi v ba gc tng ng c k


hiu l a, b, c, s, 1 , 2 , 3 . Khi y:
1 a2 + b2 + c2 1
1) 2
< ;
3 ( a + b + c) 2
9 1 1 1
2) + + ;
ps s a s
p b s cp
3) a (s a) + b (s b) + c (s c) 2s;

4) (Hc vin k thut Qun s, 1996-1997) s < s a + s b + s c 3s;
3 3
5)0 < sin 1 + sin 2 + sin 3 ;
2
3 3
6) 0 < sin 1 . sin 2 . sin 3 ;
8
 1
3 4
7) 0 < sin 1 + sin 2 + sin 3 3 ;
4
3  
1
 
2
 
3
8) sin2 + sin2 + sin2 < 1;
4 2 2 2
1 2 3 2 3 3
9) 0 < sin . sin + sin 1 . sin + sin . sin ;
2 2 2 2 2 2 4
1 + 32 2 + 33 3 + 31
10) sin 1 sin 2 sin 3 sin sin sin ;
4 4 4
1 1 1
11) 1
+ 2
+ 3 12;
sin2 sin2 sin2
2 2 2
1 2 3 3 3
12) 2 < cos + cos + cos ;
2 2 2 2
1 2 3 9
13) 2 < cos2 + cos2 + cos2 ;
2 2 2 2
1
14) cos 1 . cos 2 . cos 3 ;
8

275
Hi tho khoa hc, Hng Yn 25-26/02/2017

1 2 3 3 3
15) 0 < cos . cos . cos ;
2 2 2 8
3
16) cos 1 . cos 2 + cos 1 . cos 3 + cos 2 . cos 3 .
4
1 2 3 1 2 3 3
17) sin + sin + sin + tan + tan + tan 3+ .
2 2 2 2 2 2 2
Bi ton 4. Cho tam gic ABC c ba gc nhn 1 , 2 , 3 . Khi y:
3 3 1 2 3 1
1) 2 < sin 1 + sin 2 + sin 3 ; 2) 0 < sin sin sin ;
2 2 2 2 8
1

 
3 4
3) 2 < sin 1 + sin 2 + sin 3 3 ;
4
1 1 2 3 2 3 3
4) < sin . sin + sin 1 . sin + sin . sin ;
2 2 2 2 2 2 2 4
3
5) 1 cos 1 + cos 2 + cos 3 ;
2

1 1 2 3 3 3
6) < cos . cos . cos ;
2 2 2 2 8
1 1 1
7) + + 6.
cos 1 cos 2 cos 3
Chng minh rng vi mi m 1 v mi tam gic nhn ta c:
m+2
m m
8) tan 1 + tan 2 + tan 3 m 3 2 ;
8.1) tan 1 + tan 2 + tan 3 3 3; 8.2) tan2 1 + tan2 2 + tan2 3 9;
m2
 
1
 
2
 
3
9) 3 2 tanm + tanm + tanm ;
2 2 2
1 2 3 2 3
9.1) 3 tan + tan + tan ; 9.2) tan2 1 + tan2 + tan2 1;
2 2 2 2 2 2
1 2 3 3
10) 0 < tan . tan . tan .
2 2 2 9
Bi ton 5. Tam gic t ABC c ba gc l 1 , 2 , 3 . Khi y:
1) 0 < sin 1 + sin 2 + sin 3 < 1 + 2;
1
2) 0 < sin 1 sin 2 sin 3 < ;
2
3

3) 0 < sin 1 + sin 2 + sin 3 < 1 + (2) 4 ;
s
1 2 1 3 2 3 2 2 2 2
4) 0 < sin sin + sin sin + sin sin < + ;
2 2 2 2 2 2 4 2
1 1 1 2 3 1+ 2
5) + + 2 3; 6) 0 < cos 1 cos cos < ;
sin 1 sin 2 sin 3 2 2 2 4
1 2 3
7) tan + tan + tan 2 2 1.
2 2 2
1 a2 + b2 + c2 3
Bi ton 6. Vi tam gic ABC cn, ta c 2
< .
3 ( a + b + c) 8
ton 7. Gi a, b, c l ba cnh ca tam gic tha mn
Bi
2 a b c 1,
a + b + c = 4.
 
Khi y 4 + a2 + 4 + b2 + 4 + c2 2 5+ 2 .

276
Hi tho khoa hc, Hng Yn 25-26/02/2017

Bi ton 8. Gi s 1 2 3 . Khi y
2 4
cos 1 + cos 2 + cos 3 2.
Bt ng thc tri c th c p dng chng minh cc bt ng thc hoc gii
cc bi ton cc tr. Di y l mt s v d. C th xem thm [7].
 
1 1 1 1 1 1
Bi ton 9. Cho a, b, c > 0. Khi y + + 2. + + .
a b c a+b b+c a+c

Bi ton 10 ([8). , IMO 2000, Problem 2]Cho cc s dng a, b, c tha mn iu kin a.b.c =
1. Khi
 y   
1 1 1
a1+ b1+ c1+ 1.
b c a

Bi ton 11. Cho 2n s thc dng ai , bi i = 1, n tha mn iu kin
a1 a2 an ; b1 b2 bn ; a1 b1 ; a1 .a2 b1 .b2 ; . . . ; a1 .a2 . . . an
b1 .b2 . . . bn . Khi y
a1 + a2 + + an b1 + b2 + + bn .

Bi ton 12. Cho a, b l cc s thc dng. Khi y


p3
p
3
3
p
3
3
p
a + a + b + b a + b + 3 b + 3 a.
3

n
Bi ton 13 (xem [9). ]Cho n 4, 0 ai , i = 1, 2, . . . , n. Gi s S := ai 2.
2 i =1
n S
Khi y 4 sin ai n sin .
i =1 n

Bi ton 14 (xem [9). ]Cho ai , i = 1, 2, . . . , n l nhng s nguyn dng. K hiu S =


n
ai ( n) . Khi y
i =1
 2
S
(S n + 1)2 + n 1 a21 + a22 + + a2n n. .
n
1
Bi ton 15. Cho a, b, c l ba s thc tha mn: 1 a, b, c 1 v a + b + c = . Tm
2
gi tr ln nht ca biu thc A = a12 + b12 + c12 .
 a n
Bi ton 16 (V ch o, 2000). Cho a, b > 0 v s nguyn n. Chng minh 1 + +
 n b
b
1+ 2n+1 . ng thc xy ra khi no?
a

Bi ton 17 (IMO 1999). Cho n l mt s nguyn dng c nh, n 2. Hy tm hng s


C b nht sao cho  4
  n
2 + x2 C
x x
i j x i j i vi mi s thc khng m x1 , x2 , . . . , xn .
x
1 i < j n i =1

Bi
ton 18 (Iran 2008). Cho a, b, c > 0 v ab + bc + ca = 1. Khi y a3 + a + b3 + b +
c3 + c 2 a + b + c.

277
Hi tho khoa hc, Hng Yn 25-26/02/2017

4 Kt lun
chng minh cc bt ng thc trong tam gic, ta c th dng nhiu phng php:
bin i i s v lng gic, lng gic ha, phng php hm s, phng php nh
gi s hng, phng php tam thc bc hai, phng php vect,... Tuy nhin, thng
phi s dng nhiu php bin i tng i phc tp v cng knh.
S dng phng trnh bc ba (xem [4]) hoc B tri, ta thng c ngay kt qu cn
chng minh. B tri, theo mt ngha no , hay hn phng php phng trnh bc
ba, bi v phng php phng trnh bc ba thng ch p dng c cho cc tam gic
bt k (cc gc hoc cc cnh c vai tr nh nhau), cn phng php B tri c th p
dng cho c cc tam gic c bit (khng i xng: tam gic cn, tam gic t, tam gic
vung cn, tam gic tha mn mt tnh cht c bit no ,...).
Nh cch chn linh hot cc vect so snh v cc hm li, lm tng ng, ta c th
s dng bt ng thc tri chng minh cc bt ng thc. Ch cn mt bt ng thc
tri (hay bt ng thc Karamata) v h qu ca n, ta c th chng minh c RT
NHIU bt ng thc trong tam gic. Chng ti tin rng, cc bt ng thc nu trong
Mc 3 ch l mt s t cc v d minh ha tnh u vit ca B tri. Cn rt nhiu bt
ng thc c v mi ni chung, nhiu bt ng thc c v mi trong tam gic ni ring,
c th chng minh c nh bt ng thc tri hoc cc h qu.

Ti liu
[1] Phm Kim Hng (2006), Sng to bt ng thc, Nh xut bn Tri thc
[2] Nguyn Vn Mu (Ch bin, 2004), Mt s chuyn chn lc bi dng hc sinh
gii, Trng i hc Khoa hc T nhin, i hc Quc gia H Ni.
[3] Trn Phng (2009), Nhng vin kim cng trong bt ng thc ton hc, Nh xut
bn Tri thc.
[4] T Duy Phng (2004, 2006), Phng trnh bc ba v cc h thc trong tam gic,
Nh xut bn Gio dc.
[5] L H Qu (2011), Bt ng thc Karamata v mt vi ng dng, Hi tho Cc
chuyn Ton hc bi dng hc sinh gii cp Trung hc Ph thng, Ph Yn,
18-19 thng 4 nm 2011.
[6] Cao Minh Quang, Bt ng thc Karamata v mt s ng dng,
http://sachsangtao.com/bvct/sach-tham-khao/414/bdt-karamata-va-mot-so-
ung-dung-cao-minh-quang.html
[7] Mc Vn Th (2013), Mt s lp bt ng thc Karamata v p dng, Lun vn Thc
s, i hc Thi Nguyn.
[8] V Vn Thng (2016), S dng B tri chng minh cc bt ng thc trong tam
gic, Lun vn Thc s, i hc Thng Long.
[9] M. S. Klamkin (2002), On a Problem of the Month, Crux, Vol. 28, 86-90.
[10] A. W. Marshall, L. Olkin and B. C. Arnold (2011, Second Edition)), Inequalities:
Theory of Majorization and Its applications, in Springer Series in Statistics, N. Y.,
909 pages.

278
Hi tho khoa hc, Hng Yn 25-26/02/2017

MT S BI TON V DY S TRUY HI TUYN


TNH CP HAI

Quch Th Tuyt Nhung


THPT Chuyn Hng Yn

1 Dy s truy hi tuyn tnh cp hai


1.1 Cc nh ngha v v d
nh ngha 1. Dy s truy hi tuyn tnh cp hai l dy s cho bi cng thc

un+2 = aun+1 + bun (1.1)

vi mi n 0 v a, b l cc hng s thc.

V d 1. Dy s (un ) cho bi cng thc



u0 = 1

u1 = 2
1 2
u = u n +1 + u n , n 0

n +2
3 3
l mt dy s truy hi tuyn tnh cp hai.

1.2 Phng php xc nh s hng tng qut ca dy s truy hi tuyn


tnh cp hai
xc nh cng thc s hng tng qut ca dy s truy hi tuyn tnh cp hai ta thc
hin nh sau:
Xt phng trnh c trng ca (1.1)

t2 at b = 0. (1.2)

Phng trnh c bit thc = a2 + 4b. Ta xt cc trng hp:


Trng hp 1. > 0 khi (1.2) c hai nghim thc phn bit t1 , t2 . S hng tng qut
ca (1.1) c dng
un = x.t1n + y.t2n , n 0, x, y R.

278
Hi tho khoa hc, Hng Yn 25-26/02/2017

x, y s hon ton xc nh khi cho trc u0 , u1 .


Trng hp 2. = 0 khi (1.2) c mt nghim kp thc t. S hng tng qut ca (1.1)
c dng
un = x.tn + y.n.tn1 , n 0
(ta qui c 01 = 0) x, y R.
x, y s hon ton xc nh khi cho trc u0 , u1 .
Trng hp 3. < 0 khi (1.2) c hai nghim phc. Thut ton tm s hng tng qut
trong trng hp ny nh sau:
Bc 1. Gii phng trnh t2 at b = 0 ta nhn c hai nghim phc

a + i.
z= .
2

Bc 2. t r = |z| l module ca Z, = Argz, ta nhn c

un = r n .( p cos n q sin n)

vi mi p, q l cc s thc.
Bc 3. Xc nh p, q theo cc gi tr u0 , u1 cho trc.
Cch lm trn c chng minh bng kin thc i s tuyn tnh. y, ti s chng
minh trng hp 1 v trng hp 2 bng kin thc trung hc ph thng.
Trng hp 1. > 0 khi (1.2) c hai nghim thc phn bit t1 , t2 , theo nh l Vi-et
ta c t1 + t2 = a, t1 .t2 = b. Khi

un+1 = (t1 + t2 )un t1 .t2 .un1 .

Ta c

un+1 t1 un = t2 (un t1 un1 ) = t22 (un1 t1 un2 ) = = t2n (u1 t1 u0 ).

Nh vy

un+1 t1 .un = t2n (u1 t1 u0 ). (1.3)

Tng t

un+1 t2 .un = t1n (u1 t2 u0 ). (1.4)

Tr tng v (1.4) v (1.3) ta c

(t1 t2 )un = (u1 t2 u0 )t1n (u1 t1 u0 )t2n .

Do t1 6= t2 nn
u1 t2 u0 n u1 t1 u0 n
un = t t .
t1 t2 1 t1 t2 2

279
Hi tho khoa hc, Hng Yn 25-26/02/2017

Vy un c dng un = x.t1n + y.t2n , n 0, x, y R.


a2 a
Trng hp 2: = 0 khi b = , (1.2) c nghim kp t = . Ta c
4 2
un+1 = 2tun t2 un1 un+1 tun = t(un tun1 ) = = tn (u1 tu0 )

Nh vy,

un+1 tun = tn (u1 tu0 ) (1.5)


un tun1 = tn1 (u1 tu0 ) (1.6)
n 2
un1 tun2 = t (u1 tu0 ) (1.7)
.................................. (1.8)
u1 tu0 = u1 tu0 (1.9)

Nhn hai v ca (1.6) vi t, hai v ca (1.7) vi t2 , . . . , hai v ca (1.9) vi tn v cng li, ta


c un+1 = tn+1 .u0 + n.tn .(u1 tu0 ). Do un c dng x.tn + y.n.tn1 vi x, y l hai s
thc.

2 Mt s v d
V d 2. Xc nh s hng tng qut ca dy s tha mn
1 2
u0 = 11, u1 = 2, un+2 = un+1 + un , n = 0, 1, 2 . . .
3 3
Li gii.
1 2
Xt phng trnh c trng t2 + t = 0 ca dy c hai nghim thc phn bit l
3 3
2
t1 = , t2 = 1
3
Do ,
2
un = x.( )n + y.(1)n , x, y R.
3
Ta li c
x+y = 1 x=9
 (
u0 = 1 5
2 4
u1 = 2 xy = 2 y =

3 5
9 2 n 4
Vy un = ( ) (1)n , n 0.
5 3 5
V d 3. Xc nh s hng tng qut ca dy s tha mn
1 3vn vn+1
v0 = 1, v1 = , vn+2 = , n 0.
2 vn 2vn+1

280
Hi tho khoa hc, Hng Yn 25-26/02/2017

Li gii.
Ta c
3.vn .vn+1 1 1 2
v n +2 = = +
vn 2.vn+1 v n +2 3.vn+1 3.vn
1 1
Bng vic t un = ta c v0 = 1, v1 = suy ra u0 = 1, u1 = 2. Bi ton qui v vic tm
vn 2
s hng tng qut ca dy s trong v d 1.
Nhn xt. Bng cch bin i un , ta c nhng bi ton kh hay. Chng hn, trong v d 1
khi t un = lnvn ta c bi ton
Xc nh s hng tng qut ca dy s tha mn
s
v2n
v0 = e, v1 = e2 , vn+2 =
3
, n 0.
v n +1

3 Mt s dng ton v dy s truy hi tuyn tnh cp hai


Trc ht, ta xt mt s bi ton c gii quyt thng qua vic tm s hng tng qut
ca dy truy hi tuyn tnh cp hai.

V d 4. Cho dy s un tha mn

1
u0 = 0, u1 = 1, un+2 = un+1 un , n 0.
2
Tm lim un .
Li gii.
1
Phng trnh c trng ca dy l t2 t + = 0. Phng trnh c mt nghim phc
2
1+i 1
t= , |t| = , Argt = .
2 2 4

Do , s hng tng qut ca dy c dng


n
2 n n
un = ( ).( x cos + y sin ), n 0, x, y R.
2 4 4
T gi thit u0 = 0, u1 = 1 suy ra x = 0, y = 2.
Vy s hng tng qut ca dy l

n 2 2
un = 2sin ( )= .
4 2 ( 2) n
2
Khi lim un = 0 v |un | 0.
( 2) n

281
Hi tho khoa hc, Hng Yn 25-26/02/2017

V d 5. Cho dy ( xn ) tha mn
x0 = 1, x1 = 5, xn+1 = 6xn xn1 , n 1.

Tm lim( xn 2xn ).
Li gii.
Bng phng php xc nh s hng tng qut trn, ta xc nh c s hng tng qut
ca dy l
2+ 2 n 2 2
xn = ( )(3 + 2 2) + ( )(3 2 2)n .
4 4
Hay
1 1
xn = ( 2 + 1)2n+1 + ( 2 1)2n+1
2 2 2 2
1 1
2xn = ( 2 + 1)2n+1 + ( 2 1)2n+1
2 2
+ 1
2xn ( 2 1) 2n 1 = [( 2 + 1)2n+1 ( 2 1)2n+1 ]
2
n
+1 ( n t )
2xn ( 2 1)2n+1 = (2n 2t+1 )2 Z.
t =0
Li c 0 < 2 1 < 1 suy ra

0 < ( 2 1)2n+1 < 1
n 
2n + 1

[ 2xn ] = .2nt
t =0 2t + 1

2xn [ 2xn ] = ( 2 1)2n+1

2xn = ( 2 1)2n+1
1 1
Nn xn 2xn = [1 + ( 2 1)2(2n+1) = [1 + ( 2 1)4n+2 ]. Vy lim xn 2xn =
2 2 2 2
1 1 1
+ lim( 2 1)4n+2 = .
2 2 2 2 2 2
V d 6. Cho dy s (un ) c xc nh bi
u1 = 7, u2 = 50, un+1 = 4.un + 5.un1 1975
Chng minh rng u1996 chia ht cho 1997.
Li gii.
1975
Xt dy xn = un . Ta c xn+1 = 4xn + 5xn1 .
8
Phng trnh c trng x2 4x 5 = 0 c hai nghim x = 1, x = 5.
Ta xc nh c
1747 n 2005
xn = .5 + (1)n . .
30 3
Do
1747.(1)n + 2005.(1)n 1975
un = + .
120 8

282
Hi tho khoa hc, Hng Yn 25-26/02/2017

Suy ra
1747.51996 + 49675
u1996 = .
120
T
u1996 .120 = 1747.(51996 1) + 1997.24
V 51996 1 (mod 1997) nn u1996 chia ht cho 1997.

V d 7. Cho dy s ( an ) xc nh bi cng thc


q
a0 = 2, an+1 = 4.an + 15.a2n 60.

1
Chng minh rng s A = ( a2n + 12) c th biu din thnh tng ca ba s nguyn lin tip
5
vi mi n 1.
Li gii.
T gi thit p
an+1 4an = 15a2n 60
( an+1 4an )2 = 15a2n 60
a2n+1 8.an+1 .an + a2n + 60 = 0, n 1
Ta c a2n 8an1 an + a2n1 + 60 = 0.
T suy ra
a2n+1 a2n1 8( an+1 an1 ) an = 0
( an+1 an1 )( an+1 + an1 8an ) = 0.
T gi thit suy ra an 0, n. Nh vy, an+1 4an > an dy ( an ) l mt dy tng.
Suy ra, an+1 an1 6= 0. T an+1 + an1 8an = 0.
Gii phng trnh c trng
t2 8t + 1 = 0
n n
ta c t1,2 = 4 15. Ta tm c 2na n = ( 4 + 15 ) + ( 4 15) .
2n
Suy ra a2n = (4 + 15) + (4 15) .
Vi mi n 1, tn ti s k N

(4 + 15)n + (4 15)n = 15k

[(4 + 15)n + (4 15)n ]2 = 15k2

(4 + 15)2n + (4 15)2n + 2 = 15k2 .
Suy ra a2n = 15k2 2.
1 1
Ta c A = ( a2n + 12) = (15k2 + 10) = 3k2 + 2 = (k 1)2 + k2 + (k + 1)2 . Bi ton c
5 5
chng minh.
i khi, vic bt u t cng thc tng qut ca dy s tm h thc truy hi gia cc
s hng trong dy cng gip ta gii quyt c kh nhiu bi tp. Ta xt mt s bi ton
nh vy

283
Hi tho khoa hc, Hng Yn 25-26/02/2017

(2 + 3) n (2 3) n
V d 8. Cho dy (un ) tha mn un = , n = 0, 1, 2, . . . .
2 3
a. Chng minh rng un l s nguyn n = 0, 1, 2, . . . .
b. Tm tt c cc s hng ca dy chia ht cho 3.
Li gii.
a. Vi n =
0 th u0 =
1, n = 1 th u1 = 1. t
= 2 + 3, = 2 3.
Ta c: + = 4, . = 1.
D thy un l s hng tng qut ca dy s c cho bi cng thc
u0 = 0, u1 = 1, un+2 4un+1 + un = 0.
Do u0 , u1 Z, un+2 = 4un+1 un nn un Z, n = 0, 1, . . . .
b. Ta c un+2 = 3un+1 + (un+1 un ).
Do un+1 Z nn un+2 un+1 un (mod 3).
Bng php tnh trc tip ta thy 8 s hng u tin ca dy u0 , u1 , . . . , u7 khi chia cho 3 c
cc s d tng ng l 0, 1, 1, 0, 2, 2, 0, 1. Suy ra
u n +6 u n (mod 3).
T ta thy trong dy s ni trn mi s hng c dng u3k , k = 0, 1, 2, . . . chia ht cho 3 v
ch nhng s hng m thi.

V d 9. Chng minh rng vi mi n N th [( 3 + 2)2n ] khng chia ht cho 5 ( k hiu
[ x ] ch phn nguyn ca s x.
Li gii.
t A = ( 3 + 2)2n = (5 + 2 6)n v x1 = 5 + 2 6, x2 = 5 2 6. Khi x1 + x2 =
10, x1 .x2 = 1, hay x1 , x2 l nghim ca phng trnh
x2 10x + 1 = 0.

t Sn = (5 + 2 6)n + (5 2 6)n , n N. Ta c
Sn+2 10Sn+1 + Sn = 0 Sn+2 = 10Sn+1 Sn .
Mt khc S0 = 2, S1 = 10, bng phng php qui np ta suy ra Sn l s nguyn vi mi
n N. V 0 < (5 2 6)n < 1 nn
q
(5 + 2 6) n + (5 2 6) n 1 (5 + 2 6) n < (5 + 2 6) n + (5 2 6) n .
Suy ra
Sn 1 (5 + 2 6)n < Sn [(5 + 2 6)n ] = Sn 1, n N.
Li c Sn+2 Sn (mod 5), dn ti Sn Sn2 (mod 5). Vy

S2k (1)k S0 (mod 5) S2k (1)k .2 (mod 5)


 

S2k+1 (1)k S1 (mod 5) S2k+1 (1)k .10 0 (mod 5)

M [(5 + 2 6)n ] = Sn
1 nn [(5 + 2 6)n ] 2.(1)k 1 (mod 5) hoc [(5 + 2 6)n ] 1
(mod 5) suy ra [(5 + 2 6)n ] khng chia ht cho 5.

284
Hi tho khoa hc, Hng Yn 25-26/02/2017

V d 10. Chng minh rng biu thc Sn = (9 + 4 5)n + (9 4 5)n nhn gi tr nguyn
v khng chia ht cho 17 vi mi n N.
Li gii.
Ta chng minh c
S0 = 2, S1 = 18, Sn+2 = 18Sn+1 Sn .
Suy ra Sn Z. Li c

Sn+3 + Sn = 18Sn+2 Sn+1 + Sn = 17(Sn+2 + Sn+1 )

chia ht cho 17.


Ta c S0 = 2 khng chia ht cho 17 nn S3k cng khng chia ht cho 17. Ta c S1 = 18 khng
chia ht cho 17 nn S3k+1 cng khng chia ht cho 17. Ta c S2 = 322 khng chia ht cho 17
nn S3k+2 cng khng chia ht cho 17. Vy Sn khng chia ht cho 17 vi mi n N.

V d 11. Tm s nguyn d ln nht sao cho d|16n + 10n 1 vi mi n N .


Li gii.
Vi mi n N , t un = 16n + 10n 1, vn = un + 1 = 16n + 10n v d l c chung ln nht
ca un vi mi n N .
Ta kim tra c dy s (vn ) tha mn h thc truy hi

vn+2 (16 + 10)vn+1 + 16.10vn = 0, n N .

T
un+2 = 26un+1 160un 135, n N .
V d l c ca un , un+1 , un+2 nn d l c ca 135. Mt khc d l c ca u1 = 25, suy ra
d|(135, 25) = 5. Vy d = 5.
Cc bi ton lin quan n cng thc s hng tng qut ca dy truy hi tuyn tnh cp
hai cng xut hin nhiu trong cc thi trong nc v khu vc. Ta xt mt s bi ton nh
vy.

V d 12 (VMO 2011). Cho dy s nguyn ( an ) xc nh bi

a0 = 1, a1 = 1, an = 6an1 + 5an2 , n 2.

Chng minh rng a2012 2010 chia ht cho 2011.


Li gii.
Xt dy (bn ) c xc nh nh sau:

b0 = 1, b1 = 1, bn = 6bn1 + 2016bn2 , n 2

Dy ny c phng trnh c trng x2 6x 2016 = 0, phng trnh ny c hai nghim


x = 42, y = 48.
T y suy ra s hng tng qut ca dy l

41.48n + 49.(42)n
bn = , n N.
90

285
Hi tho khoa hc, Hng Yn 25-26/02/2017

Ngoi ra, ta cng d dng chng minh c bng qui np rng

a n bn (mod 2011), n N.
Do ta ch cn chng minh b2012 + 1 0 (mod 2011). Ta c

41.482012 + 49.(42)2012 + 90
b2012 + 1 = .
90
Do 2011 l s nguyn t v 2011, 90 l hai s nguyn t cng nhau nn ta ch cn chng
minh 41.482012 + 49.(42)2012 + 90 0 (mod 2011).

Tht vy, theo nh l Fecma nh ta c

41.482012 + 49.(42)2012 + 90 41.482 + 49.4262 + 90 (mod 2011).


Li c:
41.482 + 49.4262 + 90
= 90.b2 + 90
= 90[6.(1) + 2016.1] + 90
= 90.2010 + 90 = 90.2011 0 (mod 2011).

Bi ton c chng minh.


V d 13 (Chn i tuyn Vit Nam thi ton Quc t nm 2012). Cho dy s ( xn ) gm v
hn cc s nguyn dng c xc nh bi

x1 = 1, x2 = 2011, xn+2 = 4022xn+1 xn , n N


x2012 + 1
Chng minh rng l s chnh phng.
2012
Bi ton c th pht biu tng qut nh sau.

Bi ton m rng 1. Cho p l s nguyn dng l ln hn 1. Xt dy s ( xn ) gm v hn cc


s nguyn dng c xc nh bi

x1 = 1, x2 = p, xn+2 = 2pxn+1 xn , n N
x p +1 + 1
Chng minh rng l s chnh phng.
p+1
Li gii.
Phng trnh c trng ca dy s cho l
q q
2
t 2pt + 1 = 0 t1 = p + p2 1, t2 = p p2 1

Vy s hng tng qut ca dy ( xn ) c dng

xn = A.t1n + B.t2n , n = 1, 2, . . . .

286
Hi tho khoa hc, Hng Yn 25-26/02/2017

Do x1 = 1, x2 = p nn ta c
t2 t
A.t1 + B.t2 = 1, A.t21 + B.t22 = p ( A, B) = ( , 1 ).
2 2
Nh vy ta c
t1n1 + t2n1
xn = , n = 1, 2, . . . (dot1 t2 = 1).
2
Do p p
t1 + t2 p p
+1 p p 2 2
x p +1 + 1 2 t + t 2 + 2 ( t 2
+ t 2)
= = 1 = 1
p+1 p+1 2( p + 1) 2( p + 1)
Ch rng t1 + t2 = 2p, t1 t2 = 1 nn
q p q
t1 + t2 = t1 + t2 + 2 t1 t2 = 2p + 2 = 2( p + 1).

Nu t Sn = t1n + t2n , ta c:

Sn+2 = 2pSn+1 Sn , n N .

M S1 = 2p N, S2 = 4p2 2 N nn suy ra Sn N, n = 1, 2, . . . t t1 = a, t2 = b
p p
th a + b = 2( p + 1), ab = 1 v t12 + t22 = a p + b p . V p l s nguyn dng l ln hn 1
p

nn ta c
v
p 1 p 1
u
a + b = ( a + b) (1) a b = 2( p + 1) (1)i ai b pi1 .
u
p p i i p i 1 t
i =0 i =0

Xt bin i sau
p 1
a i b p i 1
i =0
p 3
2 p 1 p 1 p 1 p 1
= (1)i ai b pi1 + (1) 2 a 2 b 2 + (1)i ai b pi1
i =0 p +1
i= 2
p 3
2 p 1 p 1
= (1)i ( ab)i b p12i + (1)i ( ab)i b p12i + (1) 2
i =0 p +1
i= 2
p 3
2 p12i p 1 p12i p 1
= (1)i t2 2
+ (1)i t2 2
+ (1) 2
i =0 p +1
i= 2
p 3
p12i p12i
 
2 p 1
= (1)i t2 2
+ t1 2
+ (1) 2
i =0
p 3
2 p 1
= (1)i S p 1 2i + (1) 2 = N N.
i =0
2

287
Hi tho khoa hc, Hng Yn 25-26/02/2017
p p p
Nh vy, ta c t12 + t22 = N 2( p + 1). Ta thu c
 p p
2
2 2
t1 + t2
x p +1 + 1 N 2 2( p + 1))
= = = N2.
p+1 2( p + 1) 2( p + 1)

x p +1 + 1
Tm li = N 2 l s chnh phng. Bi ton chng minh xong.
p+1

4 Mt s bi tp tng t
Bi 1. Tm s hng tng qut ca dy s c xc nh bi u0 = 1, u1 = 1, un+2 = 6un+1
9un , n 0.
3+ 5 n 3 5 n
Bi 2. Cho dy s (un ) xc inh nh sau un = ( ) +( ) 2, n = 1, 2, . . .
2 2
a. Chng minh rng un l s t nhin n = 1, 2, . . .
b. Chng minh rng u2011 l mt s chnh phng.
Bi 3. Cho dy (un ) xc nh bi u1 = 0, u2 = 1, un+2 = un + un+1 + 1. Chng minh rng:
u p (u p+1 + 1) chia ht cho p vi p l mt s nguyn t ln hn 5.

Ti liu
[1] Phan Huy Khi (2000), Cc bi ton v dy s, NXB Gio dc.

[2] Nguyn Ti Chung (2006), Bi dng hc sinh gii chuyn kho dy s, NXB i Hc Quc
Gia.

[3] Cc ti liu khc trn Internet.

288
Hi tho khoa hc, Hng Yn 25-26/02/2017

MT S TNH CHT CA HM S HC C BN
V P DNG

Hong Tun Doanh


THPT Chuyn Hng Yn

1 Tnh cht ca cc hm s hc
nh ngha 1 (Hm s hc). Hm s hc l hm s c min xc nh l tp cc s nguyn
dng v min gi tr l tp cc s phc.

V d 1.
a) Hm d(n) m cc c khc nhau ca mt s t nhin n 1 l hm s hc.
b) Hm phi-Euler (n) l hm s hc.
(
1 nu n = 1
c) Hm : Z+ C, (n) = l hm s hc.
0 nu n 2
d) Hm O : Z+ C, O(n) = 0 l hm s hc.

nh ngha 2. Cho hai hm s hc f v g.


a) Ta nh ngha tng ca f v g l hm s hc c xc nh nh sau

( f + g)(n) = f (n) + g(n), n N .

b) Ta nh ngha tch ca f v g l hm s hc c xc nh nh sau

( f .g)(n) = f (n).g(n), n N .

c) Tch chp Dirichle ca f v g l hm s c xc nh nh sau

( f g)(n) = f (d).g(n/d) = f ( d ) g ( d 0 ), n N .
d|n 0
dd =n

Tnh cht 1. Cho f v g l cc hm s hc. Khi ( f g)(n) = 0 vi mi n N khi v ch


khi hoc f = 0 hoc g = 0.

Tnh cht 2. Hm s hc f l kh nghch trong A khi v ch khi f (1) 6= 0.

289
Hi tho khoa hc, Hng Yn 25-26/02/2017

2 Mt s hm s hc c bn
Tip theo, ta xt mt vi hm s hc c bn.
nh ngha 3 (Gi tr trung bnh ca hm s hc). Gi tr trung bnh F ( x ) ca mt hm s
hc f (n) c xc nh bi cng thc

F(x) = f ( n ), x R
n x

vi tng tt c cc s nguyn dng n x. c bit, F ( x ) = 0 vi x < 1. Hm s F ( x ) cn


c gi l hm tng ca f .

Phn nguyn ca s thc x c biu th bi [ x ] v c duy nht s nguyn n tha mn


n x n + 1. Phn thp phn ca x l s thc { x } = x [ x ] [0, 1).
nh ngha 4. Hm g(t) l hm n thc trn tp I nu tn ti mt s t0 I sao cho g(t) l
hm tng vi t t0 v gim vi t t0 .
5 5 1
V d 2. = 2 v { } = .

3 3 3
Mi s thc x u c th vit c duy nht di dng x = [ x ] + { x }.

logk t
V d 3. Hm f (t) = l n thc trn na khong [1, ) vi t0 = ek . Trong gii tch
t
thc c chng minh c mi hm l n iu hoc n thc trn on [ a, b] l kh
tch.
Tnh cht 3. Cho f (n) v g(n) l cc hm s hc. Xt hm tng

F(x) = f ( n ).
n x

Vi a v b l cc s nguyn khng m v a < b, ta lun c:


b b 1


f ( n ) g ( n ) = F ( b ) g ( b ) F ( a ) g ( a + 1) F ( n ) g ( n + 1) g ( n ) .
n = a +1 n = a +1

Chng minh. Bng cch tnh ton trc tip ta c


b b


f (n) g(n) = F ( n ) F ( n 1) g ( n )
n = a +1 n = a +1
b b 1
= F (n) g(n) F ( n ) g ( n + 1)
n = a +1 n= a
= F ( b ) g ( b ) F ( a ) g ( a + 1)
b 1
F (n)( g(n + 1) g(n)).
n = a +1

290
Hi tho khoa hc, Hng Yn 25-26/02/2017

nh ngha 5 (Hm s Mobius).



Hm s Mobius
c nh ngha nh sau:

1 nu n = 1

(n) = (1)k nu n l tch ca k s nguyn t phn bit

0 nu n chia ht cho bnh phng ca mt s nguyn t

Mt s nguyn c gi l s khng chnh phng nu n khng chia ht cho bnh


phng ca mt s nguyn t. Nh vy (n) 6= 0 nu v ch nu n l s khng chnh
phng.
nh ngha 6. Mt hm s hc f c gi l hm nhn tnh nu vi mi cp s m, n nguyn
t cng nhau, ta c f (n.m) = f (n). f (m). Trong tng trng hp ng thc ng vi mi
m, n (khng nht thit nguyn t cng nhau) hm f gi l hm nhn tnh mnh.
V d 4. Ta c
(1) = 1, (6) = 1, (2) = 1, (7) = 1, (3) = 1
(8) = 0, (4) = 0, (9) = 0, (5) = 1, (10) = 1
Tnh cht 4. Hm s Mobius
(n) l hm nhn tnh, v
(
1 nu n = 1
(d) = 0 nu n > 1
d|n

Tnh cht 5 (nh l Mobius


o). Nu f l hm s hc v g l hm s hc c nh ngha
n
bi g(n) = f (d) th f (n) = g ( d ).
d|n d|n
d
Tng
 n  t, nu g l hm s hc v f l hm s hc c nh ngha bi f (n) =
d g(d) th g(n) = f (d).
d|n d|n

Tnh cht 6. Cho f (n) l hm s hc v F ( x ) = f (n). Khi


n x
x hxi
F
m
= f (d)
d
= f ( d ).
m x d x n x d|n

Chng minh. Ta c
x
F
m
= f (d) = f (d)
m x m x d x/m dm x
hxi
= f (d) 1= f (d)
d
d x m x/d d x
= f ( d ).
n x d|n
x
V vy F
m
= f (d). Suy ra iu phi chng minh.
m x dm x

291
Hi tho khoa hc, Hng Yn 25-26/02/2017

Tnh cht 7. Nu f l hm nhn tnh th

f ([m, n]) f ((m, n)) = f (m) f (n)

vi mi s nguyn dng m v n.

Chng minh.
Cho p1 , p2 , . . . , pr l cc s nguyn t chia ht m hoc n. Khi
r
pi i
k
n=
i =1

v
r
pii
l
m=
i =1

vi k1 , . . . , kr , l1 , . . . , lr l cc s nguyn khng m. Suy ra


r
[m, n] = pi
max(k i ,li )

i =1

v
r
(m, n) = pi
min(k i ,li )
.
i =1

Do
{max(k i , li ), min(k i , li )} = {k i , li }
v v f l hm nhn tnh ta c
r r
f ( pi ) f ( pi
max(k i ,li ) min(k i ,li )
f ([m, n]) f ((m, n)) = )
i =1 i =1
r r
= f ( piki ) f ( pili )
i =1 i =1
= f (m) f (n)

Vy ta c iu phi chng minh.

Tnh cht 8. Gi s f l hm nhn tnh vi f (1) = 1 th

(d) f (d) = (1 f ( p)).


d|n p|n

V d 5. Dy ly tha cc s nguyn t l dy

2, 3, 4, 5, 7, 8, 9, 11, 13, 16, 17, 19, 23, 25, 27 . . . .pk , ..


vi p l s nguyn t v k nguyn dng.

292
Hi tho khoa hc, Hng Yn 25-26/02/2017

Tnh cht 9. Cho f (n) l hm nhn tnh. Nu

lim f ( pk ) = 0
pk

vi mi p P (tp tt c cc s nguyn t) th

lim f (n) = 0.
n

Tnh cht 10 (Hm tng cc ch s). Gi S(n) l hm tng cc ch s ca s nguyn dng n


c vit trong h thp phn. Gi s n c biu din n = ak ak1 . . . a1 vi ak , ak1 , . . . , a1 l cc ch
k
s, ak 6= 0 th S(n) = ai . Khi
i =1

1. 0 < S(n) n.
.
2. S(n) n..9.

3. S(m + n) S(m) + S(n) v S(m.n) S(m).S(n).

|{z} a) = 9k S( a) vi a l s c khng qu k ch s.
4. S(99..9
k

5. S(10k .m + n) = S(m) + S(n) vi n l s c khng qu k ch s.

V d 6. Cho m, n l cc s nguyn dng v m c d ch s vi d n. Tnh S((10n 1)m).


Li gii.
Ta c biu din:
(10n 1)m = (m 1).10n + 99 . . . 9} (m 1)
| {z
n

Do , S((10n 1)m) = S(m 1) + S(99 . . . 9} (m 1)).


| {z
n
M n d nn S(99 . . . 9} (m 1)) = 9n S(m 1).
| {z
n
Vy S((10n 1)m) = 9n.

V d 7. Tm tt c cc s nguyn dng n sao cho n = 2S3 (n) + 8.

Li gii. Gi d l s cc ch s ca n. Ta c 10d1 < n v S(n) 9n nn 10d1 2(9n)3 + 8.


Suy ra d 7
Khi , ta c S(n) 63. M S(n) n( mod9) nn:

2S3 (n) + 8 S (n) 0 ( mod9)


2S3 (n) S (n) 1 0 ( mod9) 
(S (n) 1) 2S2 (n) + 2S (n) + 1 0 ( mod9)
S (n) 1 ( mod9)

2S2 (n) + 2S (n) + 1 1 ( mod9)

293
Hi tho khoa hc, Hng Yn 25-26/02/2017

Phng trnh th hai v nghim do 2S2 (n) + 2S(n) + 1 = S2 (n) + (S(n) + 1)2 0( mod9)
m 9 c c nguyn t l 3 nn c hai s S(n) v S(n) + 1 u chia ht cho 3 v l. Do
S(n) 1( mod9).
Suy ra S(n) = 1, 10, 19, 28, 37, 46, 55
Th trc tip cc s ta c cc s cn tm l: 10, 2008, 13726.
V d 8. a) Tm s nguyn dng n nh nht tha mn:
S(n) = S(2n) = = S(2017n).
b) Tm gi tr nh nht ca S(2017n) vi n nguyn dng.

Li gii.
a) Nu n c 1 ch s th S(11n) = S(10n + n) = S(10n) + S(n) = 2S(n), khng tha mn.
Nu n c 2 ch s th S(101n) = S(100n + n) = S(100n) + S(n) = 2S(n), khng tha mn.
Nu n c 3 ch s th S(1001n) = S(1000n + n) = S(1000n) + S(n) = 2S(n), khng tha
mn.
Suy ra n c t nht 4 ch s. Gi s n = a4 a3 a2 a1
Khi , 1001n = a4 a3 a2 ( a1 + a4 ) a3 a2 a1
Nu a1 + a4 khng c nh th S(1001n) = 2( a4 + a3 + a2 + a1 ) > S(n), khng tha mn.
Nu a2 < 9 th S(1001n) > a4 + a3 + a2 + 1 + a3 + a2 + a1 > S(n), khng tha mn
Suy ra a2 = 9. Lp lun tng t, a4 = a3 = 9 nn n = 999a1
Ta c a1 6= 0 v nu a1 = 0 th 1001n = 9999990 nn S(1001n) = 54 = 2S(n), khng tha
mn. Li c n S(n) = S(2n) 2n( mod 9) nn n chia ht cho 9, suy ra a1 = 9 hay n = 9999.
Vi n = 9999, theo v d 1.1 ta c S(9999i ) = 36 vi mi 1 i 2017.
Vy n = 9999 tha mn bi ton.
b) Nhn xt 2017n khng th c dng 10n nn S(2017n) 2.
Do 2017 l s nguyn t nn theo nh l Fecmat ta c: 102016 1 chia ht cho 2017
Mt khc, 102016 1 = (1063 1)(1063 + 1)(10126 + 1)(10252 + 1)(10504 + 1)(101008 + 1)
Li c: 104 85( mod2017) 1064 121( mod2017). Do 1064 10 131( mod
2017) hay 1063 1 khng chia ht cho 2017.
Nh vy, tn ti 1 tha s khc 1063 1 trong tch trn chia ht cho 2017. M tng cc ch
s ca s bng 2. Vy gi tr nh nht ca S(2017n) = 2.
V d 9. Chng minh tn ti 2017 s nguyn dng phn bit n1 , n2 , . . . , n2017 tha mn
iu kin n1 + S(n1 ) = n2 + S(n2 ) = = n2017 + S(n2017 ).

Li gii.
Ta chng minh bi ton tng qut bng quy np: Vi mi m > 1, tn ti cc s nguyn dng
n1 < n2 < < nm sao cho tt c cc s ni + S(ni ) u bng nhau v s nm c dng 10. . . 08.
Tht vy, vi m = 2, chn n1 = 99, n2 = 108 ta c: n1 + S(n1 ) = n2 + S(n2 ) = 117, hay mnh
ng vi m = 2
Gi s mnh ng vi m = k vi nk = 10 . . . 08 (gm h ch s 0). Ta cn chng minh mnh
ng vi m = k + 1. Vi mi i = 1, 2, . . . , k ta xt s Ni = ni + C trong C = 99..900 . . . 0
(gm nk 8 ch s 9 v h + 2 ch s 0) v s Nk+1 = 100..8 (gm nk 7 + h ch s 0). Khi
, vi mi i = 1, 2, ..k ta c:
Ni + S( Ni ) = C + ni + S(ni ) + 9(nk 8)

294
Hi tho khoa hc, Hng Yn 25-26/02/2017

Cc s ny bng nhau theo gi thit quy np. Hn na:


Nk + S( Nk ) = C + nk + 9 + 9(nk 8) = 100..17
= 100..08 + 9 = Ni + S( Ni )
Bi ton c chng minh.

Bi tp tng t
Bi 1. Tm s t nhin n sao cho n + S(n) = 2003.
Bi 2. t a = S((29 )2017 ), b = S( a), c = S(b). Tnh c?
Bi 3 ( thi VMO 2004). Xt cc s nguyn dng m l bi ca 2003. Hy tm gi tr nh
nht ca S(m).
Bi 4. Tm s nguyn dng n nh nht sao cho trong n s t nhin lin tip ta lun chn
c s N sao cho S( N ) chia ht cho 13.
Bi 5. Chng minh rng vi hai s m, n lun tn ti s nguyn dng c sao cho cm, cn c
cng s lng xut hin ch s 0, 9.
nh ngha 7 (Hm Euler (-le)). Cho n l s nguyn dng. Hm -le ca s n, k hiu
(n), l s cc s nguyn dng khng vt qu n v nguyn t cng nhau vi n.

Tnh cht 11. 1. Cho m, a l cc s nguyn dng, nguyn t cng nhau. Khi , a (m)
1( modm).
2. Nu m, n l cc s nguyn dng nguyn t cng nhau th (mn) = (m).(n).

3. Gi s n = p11 p22 . . . pk k l phn tch s n ra tha s nguyn t. Khi :
1 1 1
( n ) = n (1 )(1 ) . . . (1 )
p1 p2 pk

4. Gi s n l s nguyn dng. Khi (d) = n


d|n

5. Nu p l s nguyn t th ( p) = p 1. Ngc li, nu ( p) = p 1 th p l s nguyn


t.
V d 10. Chng minh rng n = 2015 1 chia ht cho 20801.
Li gii. Ta c, 20801 = 11.31.61 nn ta ch ra n chia ht cho 11, 31, 61.
Do (20, 11) = 1 v (11) = 10 nn theo nh l -le ta c:

20 (11) 2010 1( mod11).


Do , 2015 205 (2)5 1( mod11), hay 2015 1 chia ht cho 11.
Chng minh tng t, 2015 1 chia ht cho 31 v 61.
M 11, 31, 61 l cc s nguyn t cng nhau nn 2015 1 chia ht cho 20801.

295
Hi tho khoa hc, Hng Yn 25-26/02/2017

V d 11. Tm tt c cc s t nhin n tha mn n chia ht cho (n).


Li gii. Nu n = 1, n = 2, ta c (n)|n.

Xt n > 2. Gi s n = p11 p22 . . . pk k l phn tch s n ra tha s nguyn t. Ta c: (n) =
1 1 1
n(1 )(1 ) . . . (1 ).
p1 p2 pk
T iu kin (n)|n suy ra n = x(n) hay p1 p2 . . . pk = x ( p1 1)( p2 1) . . . ( pk 1).
Nh vy phi c mt s pi bng 2 (v v phi l s chn). Gi s p1 = 2. Ta c:

2p2 . . . pk = xp2 . . . pk

Do cc s p2 , . . . , pk khc 2 nn t ng thc trn suy ra n c nhiu nht 1 c nguyn t l.


Gi s l p2 .
t p2 = 2y + 1 suy ra 2p2 = x (2y). Do p2 l s nguyn t nn x = p2 , y = 1. Khi p2 = 3
v n c dng n = 21 .32 vi 1 1, 2 0
D dng th li cc s n c dng trn tha mn bi ton.

V d 12. Chng minh rng vi mi s t nhin n 2, ta c:

(n) + (n) 2n

(vi (n) l tng cc c dng ca n).

Li gii. Gi s cc c dng ca n l 1 < d1 < d2 < < dk = n.


n
Trong cc s t nhin khng vt qu n, c s l bi ca di . Mi s khng vt qu n v
di
khng nguyn t cng nhau vi n phi l bi ca mt c no (ln hn 1) ca n. V th
ta c:
n n n
n (n) + ++ .
d2 d3 dk
n n n
Mt khc, + ++ = dk1 + dk2 + + d1 = (n) n.
d2 d3 dk
Vy n (n) (n) n. Tc l (n) + (n) 2n.
Khi n l s nguyn t, ta c ng thc (n) + (n) = 2n.

V d 13. Tn ti hay khng hai s nguyn dng m, n tha mn iu kin gcd(m, n) = 1 v


(5m 1) = 5n 1.

Li gii. Ta c gcd(5m 1; 5n 1) = 5gcd(m,n) 1 = 4, do khng tn ti s nguyn t l


p m p2 |5m 1. Tht vy, nu ngc li th p| (5m 1) suy ra p|5n 1, v l. Vy 5m 1 c
phn tch tiu chun dng:
5m 1 = 2s .p1 .p2 . . . pk
Tong , s 2, pi l cc s nguyn t l phn bit.
Khi , 5n 1 = (5m 1) = 2s1 .( p1 1).( p2 1) . . . ( pk 1).
Nu s > 3 th 8|5m 1 v 8|5n 1, v l. Suy ra s {2, 3}.
Nu s = 3, do 8 khng l c ca 5n 1 nn k = 0. Khi , 5m 1 = 8, 5n 1 = 4, v l.
Nu s = 2 th 5m 1 = 4.p1 .p2 . . . pk v 5n 1 = ( p1 1).( p2 1) . . . ( pk 1).

296
Hi tho khoa hc, Hng Yn 25-26/02/2017

Nu n l th v2 (5n 1) = 3, suy ra k = 1, tc l 5m 1 = 4p1 v 5n 1 = p1 1 hay


5m 1 = 2(5n 1), v l.
Nu n chn th n 2 v 8|(5n 1) m gcd(m, n) = 1 nn m l. Ta li c pi |5m+1 5 nm
5 l s chnh phng mod pi vi mi 1 i k, t pi 1( mod5) hoc pi 1( mod5).
R rng khng tn ti j p j 1( mod5) v p j 1|5n 1. Vy pi 1( mod5) vi mi
1 i k. Nu k chn th 5n 1 2.(2)k 2k+1 2; 3( mod5), v l. Nu k l th
5m 1 4.(1)k 1( mod5), v l.
Vy khng tn ti m, n tha mn bi ton.

Bi tp tng t
Bi 6. Cho p l s nguyn t, n l s nguyn dng. Chng minh rng n khng chia ht cho
p khi v ch khi (np) = ( p 1).( p).

Bi 7. Chng minh rng phng trnh (n) = k, vi k l s nguyn dng cho trc, c hu
hn nghim n.

Bi 8. Chng minh rng vi s nguyn dng n ta c: (2n) = (n) khi n l v (2n) =


2(n) khi n chn.

nh ngha 8 (Hm tng cc c dng v hm s cc c dng). Hm tng cc c s


dng ca n , k hiu l (n), c xc nh (n) = d.
d|n
Hm s cc c dng ca n, k hiu (n).
Mt s tnh cht c bn:

Tnh cht 12. 1. Hm s (n) v (n) c tnh cht nhn tnh.

2. Nu p l mt s nguyn t, ta c:

p +1
( p ) = ; ( p ) = + 1
p1


3. Gi s n = p11 p22 . . . pk k l phn tch s n ra tha s nguyn t. Khi :
+1
p11 +1 p 2 +1 p k
(n) = ( )( 2 )...( k )
p1 1 p2 1 pk 1

(n) = (1 + 1)(2 + 1) . . . (k + 1)

V d 14. Chng minh rng n l hp s khi v ch khi (n) > n + n.

Li gii. Nu n l hp s. Khi , ngoi c l 1 v n, n cn t nht mt c d (1 < d < n).


n n
Khi , cng l c ca n v 1 < < n.
d d

297
Hi tho khoa hc, Hng Yn 25-26/02/2017
n n
Nu 6= d. Khi n c t nht 4 c l 1, n, d, . V th
d d
n
(n) 1 + n + d + .
d
n
M ta c d + 2 n > n nn (n) > n + n.
d
n
Nu = d hay d = n. Khi n c t nht 3 c l 1, n, n. V th
d

(n) 1 + n + n > n + n.

Nu (n) > n + n. R rng n 6= 1 v (1) = 1 suy ra (1) <1 + 1. n cng khng th l
s nguyn t v nu n l s nguyn t th (n) = n + 1 < n + n. Do , n l hp s. Vy n
l hp s khi v ch khi (n) > n + n.

V d 15. Tm tt c cc s nguyn dng n sao cho n = 2 (n).

Li gii. K hiu cc s nguyn t l p1 = 2; p2 = 3 . . . Gi s, vi s chnh phng n ta c



2i
phn tch: n = pi v (n) = (2i + 1)
i =1 i =1
(n) 2 + 1
Suy ra, (n) l nn n l suy ra 1 = 0. T iu kin bi ton = 1 nn ta c: i i =
n i =1 pi
1.

Ta c, pi i ( pi 1)i + 1 > 2i + 1 vi mi s nguyn t pi > 3, l c ca n.
p dng bt ng thc 32 22 + 1 du bng xy ra khi 2 {0; 1}
Khi , bi ton xy ra khi n = 1 hoc n = 9. Th li thy tha mn.
Vy cc s cn tm l n = 1; 9.

V d 16. S nguyn n 2 c gi l s hon ho nu (n) = 2n. Chng minh rng s


nguyn dng l n l s hon ho khi n c phn tch thnh n = p a .q2b1 2b2 2bt
1 .q2 . . . qt trong
, a v p cng chia cho 4 d 1 v t 2.

a
Li gii. Gi s phn tch n thnh tha s nguyn t c dng n = p1a1 .p2a2 . . . pkk
k
a a
Do n l s hon ho nn (n) = 2n (1 + pi + p2i + + pi i = 2p1a1 .p2a2 . . . pkk l s chn
i =1
a
v khng chia ht cho 4. Suy ra tn ti ng 1 s i sao cho 1 + pi + p2i + + pi i 2( mod 4).
Suy ra ai l mt s l. t ai = 2x + 1 vi x l 1 s nguyn.
Do p2i 1( mod4) nn ( x + 1)( pi + 1) 2( mod4). Suy ra x chn hay ai 1( mod4)
aj
Vi i 6= j, 1 j k ta c, 1 + p j + p2j + + p j 1( mod2) suy ra a j chn.
Vit li n ta c: n = p a .q2b1 2b2 2bt
1 .q2 . . . qt . Ta chng minh t 2.

298
Hi tho khoa hc, Hng Yn 25-26/02/2017

Thy vy, Gi s t = 1, ta c:

1 + p + p2 + + p a )(1 + q + q2 + + q2b ) = 2p a .q2b


p a+1 1 q2b+1 1
. = 2p a .q2b
p1 q1
1 1
p a q 2b
p q p q 5 3 15
2 = . < . . = .
p1 q1 p1 q1 4 2 8

v l. Suy ra, t 2.
Vy bi ton c chng minh.

Bi tp tng t
Bi 9. Cho n l s hon ho l. Chng minh rng n c t nht 3 c nguyn t khc nhau.

Bi 10. Chng minh rng tn ti v hn s t nhin n tha mn (n) > 3n.

Bi 11. Chng minh rng vi mi s t nhin n 1 ta c: 2 (n) < 4n.

( n2 )
Bi 12. Tm tt c cc s nguyn dng k sao cho = k, vi n l s nguyn dng.
(n)

Bi 13. Cho n l s nguyn dng tha mn n + 1 chia ht cho 24. Chng minh rng (n)
cng chia ht cho 24.

Ti liu
[1] Cc bi thi Olympic ton trung hc ph thng Vit Nam (1990 - 2006), NXB Gio dc, 2007.

[2] Nathanson M.B (1999), Elementary methods in number theory, Springer.

[3] V Dng Thy (ch bin), Nguyn Vn Nho, 40 nm Olympic ton hc quc t, NXB
Gio dc, 2002.

299

You might also like